100% found this document useful (1 vote)
240 views566 pages

Sokolov - Quantum Mehcanics

Uploaded by

GianniNicheli
Copyright
© © All Rights Reserved
We take content rights seriously. If you suspect this is your content, claim it here.
Available Formats
Download as PDF, TXT or read online on Scribd
100% found this document useful (1 vote)
240 views566 pages

Sokolov - Quantum Mehcanics

Uploaded by

GianniNicheli
Copyright
© © All Rights Reserved
We take content rights seriously. If you suspect this is your content, claim it here.
Available Formats
Download as PDF, TXT or read online on Scribd
You are on page 1/ 566

A.A.

Sokolov
T. M. Loskutov
I. M. Ternov
Quantum meEhaniE5
• A. A. SOKOLOV

• Y. M. LOSKUTOV

• I. M. TERNOV
Moscow State University

Translated by Scripta Technica, Inc.

Problems prepared by Graham Frye


Physics Department
The City College of The City University of New York

HOLT, RINEHART AND WINSTON, INC.


NEW YORK - CHICAGO - SAN FRANCISCO - TORONTO - LONDON
E n g li s h t r a n s l a t i o n c o p y rig h t (c) 1966
by H o lt, R in e h a r t and W in ston, Inc.
All R ig h t s R e s e r v e d
Library of C o n g r e s s C a t a l o g C ard Number: 6 6 —17275
28036-0116
P r i n te d in th e U n ite d S t a t e s of A m erica

O rig in a lly p u b li s h e d in R u s s i a as
K v a n to v a y a M ekhanika
by A. A. S okolov, Y. M. L o s k u to v , and I. M. T e rn o v
T e x tb o o k P u b li s h i n g H o u se of th e Ministry
of E d u c a tio n of R S F S R , M oscow, 1962
Preface to the English Edition

This book is specifically a “ textbook” for learning the physical


content of quantum mechanics. There is a pleasing progression
from the gross quantum effects (blackbody radiation, photoelectric
effect, specific heats) to typical quantum mechanical behavior
(spreading of wave packets, barrier penetration, stationary states,
spin and angular momentum multiplets) to the more refined
quantum phenomenology (fine structure, effect of the nucleus on
atomic structure, quantum fluctuations of the electromagnetic
field, coupling of angular momentums in multielectron atoms and
molecules). At each stage of remoteness from everyday experi­
ence some of the conceptually and computationally abstruse parts
of the theory are dealt with in explicit detail that emphasizes the
real observability of the phenomenon. The mathematical form of
the theory is thereby dictated by the necessity of having a nota-
tional apparatus that is sufficiently rich and flexible to embrace
the scope of actual observable effects.
While it is an exposition of the principles of quantum mechanics,
the selection of material is unusual because the book includes
much that is ordinarily regarded as atomic structure and omits
any long excursion into the formal mathematical structure of the
theory. The formalism is a part of the practice of quantum
mechanics, however, so to be complete we ought to recognize
these tools and provide some guide to their practical utilization.
To accomplish this we added two appendices in which are collected
many definitions and formal statements and a few examples to
show how the notational apparatus is used. The material is pre­
sented in a way that is very abstract and condensed. It is not
intended as an expository treatment of the subject but rather as an
outline of useful reference material on the formal aspects of the
theory. The justification for this mode of presentation is that
textbook expositions are widely available and that once the material
is grasped a concise summary of definitions and results is often
all that is needed for reference.
The active participation of the student in solving problems is
an indispensable part of the discipline of quantum mechanics. We
have therefore included a relatively large number of problems to
supplement the text. The problems are coordinated with the organi­
zation of material in the text so that they serve to illustrate in
context the applications and principal ramifications of the theory.
GRAHAM F R Y E
N E W YORK, NOVEMBER 1965
Contents
PREFACE TO THE ENGLISH EDITION Mi

PREFACE xiii

INTRODUCTION xv

PART 1. NONRELATIVISTIC QUANTUM MECHANICS 1


1. The Quantum Theory of Light 3
The Maxwell-Lorentz equations. Radiation of electromagnetic
waves. Lorentz force. R elativistic Legrangian, Hamiltonian and
equations of motion for a particle in an external field. Scalar
and vector potentials. The Lorentz condition. Blackbody radi­
ation. Spectral density. Average energy of a harmonic oscillator.
Equipartition of energy. The Rayleigh-Jeans formula. Wien’s
thermodynamic law. The ultraviolet catastrophe. Planck’s
equation. Planck’s co n sta n ts. Stephan-Boltzmann law. Wien’s
displacement law. Einstein’s photon theory. Energy and mo­
mentum relations of a photon. Photoelectric effect. Transfer of
energy and momentum. Compton effect.
2. The Bohr Quantum Theory 20
Emission of light by atoms. Spectral terms. The classical model
of the atom. Continuous radiation of light. The Thompson model
of the atom. The Rutherford experiment. The planetary model
of the atom. The Coulomb potential. Differential scattering
cross section. The Rutherford formula. Applicability of Coulomb’s
law. The effect of nuclear size. The Bohr theory. The clas­
sical solution in terms of adiabatic invariants. Periodic and
quasi-periodic motion. Frequency and rate of cla ssica l radia­
tion. The postulate of stationary states. The frequency postu­
late. Franck and Hertz experiment. Ionization of atoms. Radius
of the first Bohr orbit. Balmer formula. The correspondence
principle. E lliptic orbits. Selection rules. Somerfeld formula for
relativistic Coulomb energy levels. Existence of stationary
states. R elativistic Coulomb scattering. (Quantization, selec­
tion rules and lifetimes in terms of the correspondence principle.
Universality of 7t. R elativistic harmonic oscillator.)
3. Wave Properties of Particles 41
De Broglie waves. Single valuedness. Davisson-Germer and
Tartakovskiy-Thompson diffraction experiment. Wave packets.
Group and phase velocities. The uncertainty principle. Local­
ization of particles. Spreading time of wave packets. Born’s
VI CONTENTS

statistical interpretation. The complimentarity principle. Heisen­


berg’s ‘‘ultramicroscope’’. Opinion of the Copenhagen school.
Gaussian wave packet. Fourth uncertainty relation. (Universal­
ity of ~h. Uncertainty principle for angular momentum. Zero
point energy. Use of the uncertainty principle.)
4. The Time-Independent Schrodinger Wave Equation 57
Monochromatic waves. Local wavelength of a de Broglie wave.
Probability density. Normalization. Continuity. Single valued­
ness. Boundedness. Boundary conditions. Eigenvalues and
eigenfunctions. Energy spectrum. Orthonormality. A particle in
a potential well, discrete spectrum. Motion of free particles.
Normalization of wave functions in the case of a continuous
spectrum. Born’s periodic boundary condition. Separation of
variables. Plane waves in three dimensions. The Dirac 5-function.
5-function normalization in the case of a continuous spectrum.
Cdthplete set of orthonormal functions. Completeness relation.
(The 5-function potential. Sturm-Liouville theorem. Comparison
potential for the existence and number of bound states.)
5. Ihc Iime-Dependenl Schrodinger Wave Equation 73
Time-dependent wave functions; expression in terms of energy
eigenfunctions. The energy, momentum and kinetic energy oper­
ators. The Hamiltonian operator. Operator form of the wave
equation. Charge density and current density. Equation of con­
tinuity. Conservation of charge. Probability amplitudes Cn.
Quantum ensembles. Pure and mixed states. Interference of
do Broglie waves. Connection with the cla ssica l Hamilton-
Jacobi equation. Action function. Q uasi-classical approximation.
The WKB method. Derivation of the Bohr quantization rule.
Zero point energy. Symmetric and antisymmetric wave functions.
Quasi-stalionary levels. (Motion of a wave packet. Dispersion-
free approximation. Green’s function solution of initial value
problem. Flux of plane and spherical waves.)
(>. Basic I ’ r i n c i p l c s o l i h c Quant um T h e o r y o l C o n d u c t i v i t y 97

Transmission of a particle through a potential barrier. Trans­


mission and reflection coefficients. The tunnel effect as a
manifestation of wave properties. Distribution of real momenta
in the cla ssica lly forbidden region. Motion of electrons in a
metal. Specific heat of free' electron gas. Fermi-Dirac quantum
statistics, distribution function. Pauli exclusion principle.
Density ol states. Electron density. Fermi energy. Average
energy in thermal (equilibrium. Degeneracy temperature. Removal
o( electrons from a metal. Cold emission. Contact potentials.
Viol nmol electrons in a periodic potential. The one-dimensional
Kronig-Peiiney model. Basic principles of the electron theory of
conductivity of crystals. Allowed and forbidden energy bands.
CONTENTS vii

(Gamow penetration factor. Pressure of an electron gas at 0°K.


Block wave functions.)
7. Statistical Interpretation of Quantum Mechanics 119
Elements of the theory of linear operators. Principle of super­
position. Differential and integral operators. The Laplacian
and inverse Laplacian operators. The coordinate variable and
potential function as linear operators. Elements of represen­
tation theory. Cononical commutation relations. Momentum rep­
resentation. Average values of operators. Reality and self-conju­
gate or Hermitian operators. Integration by parts, “transferring
a derivative.” Average in terms of eigenvalues and probabilities.
(Translation operator. Time evolution operator. Inner product.
Operator equations. Matrix elements of an operator. Hermitian
conjugate or adjoint operator. Unitary operators.)
8. Average Values of Operators. Change of Dynamic Variables
with Time 127
Derivation of the uncertainty principle. Schwartz inequality.
Condition of simultaneous measurement of two dynamic quanti­
ties. Poisson brackets in cla ssica l and quantum theory. Com­
mutator bracket. Constant of motion. Ehrenfest’s theorem.
Transition from quantum to cla ssica l equations of motion.
Conditions for validity of the classica l approximation. Hydrogen
atom. Motion in constant and homogeneous electric and magnetic
fields. (Time dependence of off-diagonal matrix elements.
Operator solution for time evolution operator. Schrbdinger picture
and Heisenberg picture. Virial theorem. Sum rules.)
9. Elemenlary Theory of Radiation 141
Spontaneous and induced transitions. Einstein coefficients.
Emission and absorption. Virtual photons. Vacuum fluctuations.
Quantum electrodynamics. Matrix elements of the position op­
erator. Allowed and forbidden transitions. Selection rules.
Electric dipole and quadrupole radiation. Charge to mass ratio.
Gravitational radiation. Nuclear transitions.
10. The Linear Harmonic Oscillalor 149
Description in the classical and Bohr theories. Energy eigen­
functions and eigenvalues. Asymptotic behavior. Hermite poly­
nomials. Normalization. Orthogonality. C lassical limit of spatial
probability distribution. Zero-point energy and the uncertainty
principle. Selection rules. Intensity of radiation. Matrix elements
of the position operator. Energy eigenvalues by the WKB method.
Theory in the momentum representation. Motion of a particle in
a uniform magnetic field. Diamagnetism of an electron gas.
Quadrupole radiation by a harmonic oscillator. Matrix elements
of x 2. Motion of a wave packet in a harmonic potential. Clas-
v iii CONTENTS

sical limit. (Gaussian wave packet. Generating function and


properties of Tchebycheff-Hermite polynomials. Fourier-type
transform. Heisenberg operators. The harmonic oscillator
Green’s function. Operator solution. Creation and annihilation
operators.)
11. General Theory of Motion of a Particle in a Centrally Symmetric
Field 168
SchrOdinger’s equation in spherical coordinates. Separation of
variables. Radial and angular Schrbdinger equations. Eigen­
functions for angular dependence. Eigenvalues. Associated
Legendre polynomials. Normalization. Orthogonality. Spherical
harmonics. Physical meaning of the quantum numbers I and m.
Angular momentum operators. Commutation relations. Eigen­
functions of operators L z and L2. Raising and lowering operators.
Connection with the Bohr theory. (Generating function for Le-
. gendre polynomials. Vector operators. Matrix representations of
the angular momentum commutation relations.)
12. The Rotator 185
Eigenfunctions of the rotator. Effect of non-commutativity of
angular momentum components. Energy lev els. Degeneracy.
Angular distribution and orientation. Selection rules. Matrix
elements of r. Spectra of diatomic molecules. Reduced mass.
Vibrational-rotational spectra. Motion of a free particle in
spherical coordinates. Quadrupole selection rules. Expansion
of the plane wave in terms of spherical waves. (Isotropic har­
monic oscillator. D iscrete and continuous spectra in an isotropic
potential. Phase shift. Spherical B essel functions.)
13. The Theory of the Hydrogen-like Atom (Kepler’s Problem) 203
Energy eigenfunctions and eigenvalues. Radial wave equation.
Centrifugal barrier. Effective potential. Behavior at r = 0 and
Associated Lagucrre polynomials. Normalization and expec­
tation values of r"y. Energy levels. Degeneracy with respect to
angular momentum. Scm iclassical interpretation. E lliptic orbits.
Radial probability density. Selection rules. Radial matrix ele­
ments. Emission spectra of hydrogen-like atoms. Continuous
spectrum of a particle in a Coulomb potential. WKB method.
Asymptotic form of the radial wave function. Phase shift. Scat­
tering problems. Ionization energy. Effects of motion of the
nucleus. Experimental values of the Rydberg constant for //,
I), T i//'"*, 2 //e 4 1. Average electrostatic potential of a hydrogen
atom. Discrete s|>ectrum by the WKB-mcthod. Magnetic field at
nucleus due to a 2p electron.
i 1. I i mc - l nd r p r i i d c n l 1'rrlurhnlioii ' t h e o r y 231
Basic principles and fundamental equations of perturbation
theory. Non-degenerate case. Degenerate case. Secular equation.
CONTENTS ix

Removal of degeneracy. The Stark effect. Splitting of spectral


terms. The Lorentz cla ssica l theory of dispersion. Index of
of refraction. Polarization. Radiation damping. Quantum theory
of dispersion. Oscillator strengths. Ramon effect. “Stokes" and
“anti-Stokes" lines. Energy correction in second order per­
turbation theory. A.iharmonic oscillator. Harmonic oscillator.
Matrix elements of x^. (Perturbation theory for the continuous
spectrum. Ingoing and outgoing wave boundary conditions.
Phase shift. Connection between partial wave and three di­
mensional formulations.)

PART II. RELATIVISTIC QUANTUM MECHANICS 257


15. The Klein-Gordon Scalar Relativistic Wave Equation 259
R elativistic invariance of the de Broglie relations. R elativistic
energy-momentum relation of a free particle. The Klein-Gordon
equation. Charge and current density. Nonrelativistic limit. The
initial data problem. Indefiniteness of the sign of charge. Inter­
action with an external electromagnetic field. R elativistic
energy levels of a spinless particle in a Coulomb field. Fine
structure constant. The case Za > V2 . Charge and current density
in the presence of an electromagnetic field.

16. Motion of an Electron in a Magnetic Field. Electron Spin 268


C lassical theory of the Zeeman effect. Interaction energy of a
magnetic dipole. Larmor precession. Magnetic moment of a
moving electron. Zeeman effect in nonrelativistic SchrOdinger
theory. Orbital magnetic moment. Bohr magneton. Normal and
anomalous Zeeman splitting. Einstein-de Haas experiment.
Landd g factor. Stern-Gerlach experiment. Uhlenbeck-Goudsmit
hypothesis of intrinsic angular momentum. Half-integral quantum
numbers for angular momentum. Electron spin. The Pauli equation.
Two-component wave functions. The operator for intrinsic mag­
netic moment. Pauli matricies. Coupled SchrOdinger equations.
Matrix elements. Spin operators. Commutation relations for spin
operators. Vectorial character of spin operators. Separation of
spin and space variables in a homogeneous magnetic field.
Eigenvalues of the spin operator along an arbitrary direction.
Probability distribution of spin directions.

17. The Dirac Wave Equation 285


Linearization of the energy operator. Dirac matricies and their
relation to Pauli matricies. The Dirac equation. Charge and
current density. External electromagnetic field. Velocity oper­
ator. Statistics in second quantization. Transformation proper­
ties of the spinor wave function under Lorentz transformations
and spatial rotations.
X CONTENTS

18. The Dirac Theory of the Motion of an Electron in a Central


Field of Force 293
Orbital, spin and total angular momenta. Conservation laws.
Properties of the total angular momentum operators. Quantization
of total angular momentum. Clebsch-Gordan coefficients. Spher­
ical spinors. The vector model of the addition of angular mo­
menta. Motion in a central field including spin effects. Theory
of the rotator. Selection rules. Parity of a state. Conservation
of parity. Solution of the Dirac equation for a free particle.
Negative energy states. Nonrelativistic limit. Four-vector trans­
formation law of the energy-momentum operators under Lorentz
transformations. R elativistic invariance of the scalar wave
equation. Vector model. Charge conjugation.
19. The Dirac Equation in Approximate Form 308
•Two component Pauli form. “Small” and “large” components.
Correction terms to order ( e /c ) 2. R elativistic increase of mass.
Interaction of the intrinsic magnetic moment. Spin-orbit inter­
action. Contact interaction. The velocity operator and Ehren-
fe st’s theorem in the Dirac theory.
20. The Fine Structure of the Spectra of Hydrogen-like Atoms 314
Advantages of the approximate method. R elativistic and spin
effects. Contact interaction. Stable motion for Z < 137. Fine
structure in the Dirac theory. Experimental verification of the
fine structure theory. Lamb-Rutherford experiment. Anomalous
Zeeman effect. Weak magnetic field. Landd g factor. Strong
magnetic fields. Paschen-Back effect. “Breaking” of spin-orbit
coupling. Paramagnetism and diamagnetism. Anomalous Zeeman
effect in the vector model. (Stark effect. Quenching of meta-
stable states. Intermediate field Paschen-Back effect.)
21. The Effect of Nuclear Si rue lure on Atomic Spectra 334
Reduced mass. Effect of finite nuclear size. Mcsic atoms. Ap­
proximate harmonic oscillator potential for large Z. Spin of the
muon. Application of the Dirac equation to the neutron and pro­
ton. Anomalous (Pauli) magnetic moment. Experimental deter­
mination of the magnetic moments of the neutron and proton.
Limitations on the measurement of angular momentum. Experi­
ments of Bloch and Alvarez and of Rabi. Nuclear magneton,
llyperfine structure of the hydrogen spectrum.
22. The Electron-Positron \ncuum and the Electromagnetic \ aciitim 347
A. Dirac theory of “h o les.” Negative energy stales. Discovery
of the positron. Pair creation and annihilation. Anliparticles.
Rigorous validity of conservation laws. Positronium. Inter-
convertibility of particles. B. Tile Lamb shift of energy levels
of atomic electrons. Fluctuations of the electromagnetic vacuum.
CONTENTS XI

Virtual particles. “Smearing out” of a point electron. C. Elec­


tron-Positron vacuum. Vacuum polarization. Anomalous mag­
netic moments of electron, proton, and neutron. D. Renormal­
ization. Quantum electrodynamics. Quantum theory of fields.
Cherenkov radiation.

23. Theory of llie Helium Atom Neglecting Spin Slates 358


Basic principles of the theory of multielectron atoms. Indis-
tinguishability of electrons. Exchange forces. Perturbation
theory solution of the helium atom. Permutation of electrons.
Exchange degeneracy. Exchange energy. Symmetric and anti­
symmetric wave functions. Coulomb interaction between elec­
trons. Ionization energy. The variational method. Derivation of
the SchrOdinger equation by the variational method. Hartree-
Fock method of self-consistent fields. Investigation of the
exchange energy. Exchange time.

24. Elementary Theory of Millticlcctron Atoms Including Spin States 378


Symmetric and antisymmetric states. Permutation operator.
Eermi-Dirac and Bose-Einstein statistics. The Pauli exclusion
principle. Fermions. Bosons. Determinental wave function.
Addition of angular momentum. Russell-Saunders coupling.
Clebsch-Gordan coefficients. LS coupling, jj coupling. Wave
function of the helium atom including spins. Triplet and singlet
states. Parahelium and orthohelium. Energy spectrum of the
helium atom. Variational wave function for a Yukawa potential.
Diamagnetic susceptibility of parahelium.

25. Optical Spectra of Alkali Metals 397


The structure of complex atoms. The Thomas-Fermi statistical
method. Boundary conditions for neutral and ionized atoms.
Solution of the Thomas-Fermi problem by the Ritz variational
method. Total ionization energies. Charge distribution in argon.
Energy levels of alkali atoms. Atomic core. “Penetrating”
orbits. Polarization of the atomic core. “Effective principle
quantum number.” Smearing of the atomic core. Fundamental
series. Multiplet structure of spectral lines. Spectral terms of
sodium. Sharp, principle and diffuse series.

26. Mendeleyev’s Periodic System of Elements 420


X-ray spectra of atoms. Continuous spectra. Bremsstrahlung.
Characteristic spectra of atoms and the structure of their inner
sh ells. Moseley’s law. Multiplet structure of x-ray spectra.
R elativistic and spin effects. Regular and irregular doublets.
The discovery of Mendeleyev’s periodic law. Filling of the
electron sh ells. Application of the Thomas-Fermi method. Peri­
odicity properties of the elements.
xii CONTENTS

27. Tin1 Theory of Simple Molecules 437


Chemical bond. Heteropolar molecules. Affinity. Valence. Kos-
sel. Molecular hydrogen ion. Exchange forces. Evaluation of
some integrals by Fourier transforms. Homopolar atomic mole­
cules. Hcitler-London theory. Spin and syinmetiy. Orthohydrogen
and parahydrogen. The valence theory. Spin valence. Masers
and lasers.

PART III. SOME APPLICATIONS TO NUCLEAR PHYSICS


28. Elastic Seatiering of Particles 465
Time-dependent perturbation theory. Golden rule. Cross section
for elastic scattering. Uncertainty of energy. Scattering ampli­
tude. Bom approximation. Scattering by a Yukawa center of
force. Range of nuclear force. Fast-electron scattering by neu­
tral atoms. Validity of Born approximation. Partial-wave cross
sections. Phase shift. Scattering from a spherical barrier and
spherical well. Resonant scattering. (Golden Rule #2, Density
of final states.)
29. Second Quantization 480
Second quantization of the SchrOdinger equation. The Heisen­
berg equation of motion, q numbers and c numbers. Commutation
relations for Boson field amplitudes. Creation and destruction
operators. Anti commutation relations describing particles obey­
ing Fermi sta tistics. Quantization of Maxwell’s field equations.
Spontaneous em ission. Dipole approximation. Beta decay.
Pauli’s hypothesis of the neutrino. The Fermi theory. Weak
and strong interactions. Fermi and Gamow-Teller selection
rules. Feynman and Gell-Mann theory. /3-decay spectrum. Non-
conservation of parity in weak interactions. Lee and Yang.
Ilelicity of the neutrino. Pion decay.

APPENDIX A. Hilbert Space and Transformation Theory 497

APPENDIX II. ’I lie Statistical Assertions of Quantum Mechanics 505

PROBLEMS 511
Preface

This textbook is based on my lectures to students at the Mos­


cow Regional Pedagogical Institute (1945 to 1948) and Moscow
University from 1945 on. In writing this book we set ourselves
the difficult task of treating in a single volume the fundamentals
of atomic theory, that is, Schrodinger’s nonrelativistic theory,
Dirac’s relativistic theory, the theory of multielectron atoms,
and the basic applications of quantum mechanics to solid state
physics. Our aim was to combine the exposition of general the­
oretical principles with examples of the application of quantum
mechanics to specific problems connected with atomic structure.
To avoid overloading this book, we have abridged the treatment of
certain specialized topics, but in such cases we have endeavored
to supply references to standard works on the subject.
In most textbooks the solution of specific problems with the
help of Schrodinger’s equation is handled in fairly elegant form.
The basic mathematical tools required for this purpose are a
knowledge of second-order differential equations and various spe­
cial functions (including the Hermite, Legendre and Laguerre
polynomials). However, applications of Dirac’s theory to specific
problems (such as the hydrogen atom) are on the whole handled
le ss satisfactorily. In some cases the calculations are so long
and cumbersome that it is difficult to perceive the physical mean­
ing of the solutions. In others there is no actual derivation of the
results or only a rough proof is given. In an attempt to avoid
these pitfalls, we have used an approximate form of Dirac’s equa­
tion for our treatment of the hydrogen atom (Chapter 19). This
approximation still enables us to obtain the formula for the fine
structure of the energy levels and the selection rules (Chapter 18
and 20). Our analysis of the Lamb shift due to the electron-
positron vacuum is also somewhat simplified (Chapter 22).
Several good problem books in quantum mechanics are avail­
able, and therefore we shall consider only a few problems chosen
with the aim of elucidating and supplementing the general discus­
sion.
The first part of this book was written jointly by me and Yu. M.
Loskutov, and the second part jointly by me and I. M. Ternov.
Great assistance was rendered by M. M. Kolesnikova in condens­
ing notes based on my lectures on quantum mechanics and in
preparing the manuscript for the press. Chapter 25 was carefully
xiv PREFACE

read by N. N. Kolesnikov, who made a number of valuable com­


ments. I would like to mention the great pains taken by S. I. Larin
in editing the whole manuscript.

A. A. Sokolov
Introduction

Quantum mechanics dates only from the 1920’s. This important


branch of theoretical physics deals with the fundamental problem
of the behavior of microparticles (for instance, the behavior of
electrons in an atom). As a theory, quantum mechanics represents
an extension of classical mechanics, electrodynamics (including
the theory of the electron and the theory of relativity), the kinetic
theory of matter, and other branches of theoretical physics.
Historically, the development of every branch of theoretical
physics involves two main stages. First comes the accumulation of
experimental facts, the discovery of semiempirical laws, and the
development of preliminary hypotheses and theories. This is fol­
lowed by the discovery of general laws, which provide a basis for
interpreting a large number of phenomena. For example, the first
or pre-Newtonian stage of mechanics consisted of the discovery of
a number of seemingly unrelated laws: the law of inertia, the law
of free fall under the action of a gravitational field and Kepler’s
laws of planetary motion. Most of these laws were discovered only
after years of painstaking work by many scientists. Thus, many
astronomical observations preceded the discovery of Kepler’s
laws. We may recall the great efforts of Copernicus, Bruno, Gal­
ileo, and others to establish that the Sun is the center of our
planetary system and that the Earth is only a planet like Mars,
Venus, or Jupiter. It was only after working for fifteen years on
Tycho Brahe’s extremely valuable observational data that Kepler
found the semiempirical laws describing planetary motion. After
these preliminary, seemingly independent laws had been estab­
lished, Newton was able to show that they all rested on the same
theoretical foundation. Newton’s three laws of motion and the law
of universal gravitation opened a new stage in the development of
theoretical mechanics. One of the great triumphs of Newtonian
mechanics was L everrier’s prediction of the existence of a new
planet, Neptune, from perturbations in the motion of Jupiter.
In a sim ilar fashion, Maxwell’s formulation of the laws of elec­
trodynamics was preceded by the discovery of empirical laws de­
scribing various electric and magnetic phenomena. Coulomb’s
law of interaction between electric charges and magnetic poles1
and the Biot-Savart law of interaction between an electric current
and a magnetic pole were found by analogy with Newton’s law of

' A s m a g n e t i c m o n o p o l e s do n o t e x i s t in n a t u r e , C o u l o m b ’s l a w in m a g n e t o s t a t i c s i s
v e r i f i e d b y m e a n s of m a g n e t i c d i p o l e s .
xvi INTRODUCTION

gravitation. All of these phenomena were explained on the basis of


the principle of “ action at a distance,” according to which one
charge acts directly on another through the intervening space.
After Newton, and independently of investigations of electric
and magnetic phenomena, considerable attention was devoted to
optics. At a relatively early stage, it was established that light
consists of transverse waves, propagating with a finite velocity
of c«=<3-10'0 cm /sec. The nature of these waves, however, re­
mained unknown.
All of these preliminary studies belonged to the first stage of
development of electrodynamics: they prepared the ground for
Maxwell’s theory, which had approximately the same unifying role
in electrodynamics as Newton’s laws in mechanics. Maxwell’s
equations provided a powerful tool for the investigation of electric,
magnetic and optical phenomena. Maxwell’s theory predicted the
existence of electromagnetic fields, which carry the interaction
continuously from point to point, and of electromagnetic waves,
which were later discovered by Hertz. The theory of propagation
of electromagnetic waves underlies all of modern radio engineering.
Another important result of Maxwell’s theory was aproof of the
wave nature of light.
The view that matter and electricity have an atomic structure
was of considerable importance in connection with the appearance
of quantum mechanics. This view had very ancient roots, but
remained without scientific foundation until the discovery of the
fundamental law of chemistry—the law of exact proportions. The
kinetic theory of matter and, in particular, the kinetic theory of
gases—based on the classical Maxwell-Boltzmann statistics—were
important steps in the development of atomic theory. It is worth
noting that the classical Maxwell-Boltzmann statistics, which rests
on probability theory, cannot be completely explained in terms of
Newtonian mechanics and contains certain features that are char­
acteristic only of large collections of particles (for example, the
irreversibility of certain p rocesses). Statistical methods made it
possible to explain a number of macroscopic properties of matter,
such as temperature and specific heat; this provided indirect
evidence of the atomic structure of matter.
One of the decisive proofs of the atomic theory of matter was
the discovery of fluctuations, that is, statistical fluctuations in the
behavior of individual m olecules. Brownian motion was particularly
important in this connection, as it provided the evidence of molec­
ular movement in a liquid. Even more suggestive proofs of the
atomic structure of matter were provided by Laue’s observation of
the diffraction of x-rays in crystals and Aston’s m ass-spectro-
graphic measurements of the atomic weights of individual isotopes
of various elements.
From an analysis of Faraday’s laws of electrolysis, Helmholtz
showed that there must be a fundamental quantity of electricity.
INTRODUCTION XVII

equal to 4.8-10 esu, such that any charge, positive or negative,


is an integral multiple of this charge. Studies of anode rays indi­
cated that positive charges always appear as ions; that is, a positive
charge is always associated with the basic mass of an atom. The
lightest positive ion is that of a hydrogen atom. It is known as a
proton and its mass is nearly the same as the mass of a neutral
hydrogen atom. The carrier of a negative charge can take the form
of a negative ion or of a much lighter particle known as an electron.
From measurements of the deflection of cathode rays (a beam of
electrons) in electric and magnetic fields, it was found that the
m ass of the electron was about 1/1836 of the mass of the proton.2
These discoveries led to Lorentz’s electron theory, which
represents an interesting synthesis of Maxwell’s electrodynamics
for a vacuum and the atomic view that matter consists of positive
and negative charges. In Lorentz’s theory the magnetic permea­
bility, dielectric constant, and conductivity of a medium were
obtained by averaging Maxwell’s equations for a vacuum over
charges and currents of particles of the medium. A conductor was
treated as a medium filled with free electrons or, in other words,
an “ electron g a s.” It followed from Lorentz’s theory that the
dielectric constant depends on the frequency of electromagnetic
waves, whereas in Maxwell’s theory it had been assumed that this
quantity is a constant. Lorentz’s theory provided an explanation of
the dispersion of light. The appearance of this theory was accom­
panied by the extension of electrodynamics to frames of reference
traveling with constant relative velocities; this culminated in the
special theory of relativity. It is well known that all laws of motion—
whether they be Newton’s laws or Maxwell’s equations for the motion
of an electromagnetic field—must be associated with a frame of
reference. Newton believed that his laws were related to an abso­
lute frame of reference. Even in his writings, however, this notion
remained purely metaphysical, and Newton him self discovered the
principle of relativity in mechanics according to which it is im­
possible to detect a uniform rectilinear motion of a body (or a
frame of reference) relative to this absolute system, because all
frames of reference moving linearly with constant relative velocities
with respect to each other are completely equivalent.
Consider the Galilean-Newtonian transformation from one in­
ertial system to another, moving along the x axis with relative
velocity v

x' — x — vt, y' = y, z' = z, t' — t.

2In 1932, a p a r t i c l e w ith a p o s i t i v e c h a r g e a n d m a s s e q u a l to t h a t of a n e l e c t r o n , kn o w n


a s t h e p o s i t r o n , w a s d i s c o v e r e d . P o s i t r o n s a r e fo rm ed in s m a l l q u a n t i t i e s w h e n c o s m i c
r a y s p a s s th r o u g h m a t t e r . U n d e r o r d i n a r y c o n d i t i o n s a p o s i t r o n c a n n o t e x i s t for a n y s i g ­
n i f i c a n t l e n g t h o f tim e b e c a u s e it c o m b i n e s w ith an e l e c t r o n a n d t h e two p a r t i c l e s are
c o n v e r t e d i n t o gam m a-ray p h o t o n s ( s e e C h a p t e r s 3 a n d 22).
x viii INTRODUCTION

where the primed coordinates refer to the moving system , and the
unprimed coordinates to the stationary system . We find that
accelerations and forces are identical in the two frames of refer­
ence, and therefore the equations of mechanics (in which the
velocity does not appear) are invariant under this transformation.
If the Galilean-Newtonian transformation is applied to the Maxwell-
Lorentz equations, they assume different forms in different inertial
system s, because the equations contain the velocity of propagation
of electromagnetic waves which, added vectorially, has different
values in different inertial system s. The original Michelson-Morley
and other numerous experiments showed, however, that the speed of
light is the same in any direction in all inertial coordinate system s.
As a result, Einstein generalized the Newtonian principle of rela­
tivity in a way that led directly to the so-called Lorentz trans­
formations
-vt
y'= y, z z,
v~f
t— X
C V
t'= c•
V I-?2
The classical laws of electrodynamics are invariant under this
transformation. Since the equations of Newtonian mechanics,
however, are not invariant under the Lorentz transformations,
they had to be replaced by relativistic equations in which the
m ass m of a moving particle was related to its velocity v and
its rest m ass m„by the relationship

At low velocities, where f)2«=0, the relativistic equations reduce


to the Newtonian formulations.
The Maxwell-Lorentz equations for an electromagnetic field
and the relativistic equations of motion of an electron constituted
the culminating stage of the classical electron theory. According
to this theory, light consists of electromagnetic waves and an
electron is a particle whose motion is described by relativistic
mechanics. The success of the Maxwell-Lorentz theory in account­
ing for certain microscopic phenomena (the propagation and dis­
persion of light, the motion of an electron In electric and magnetic
fields, and so forth) was accompanied by the discovery of exper­
imental facts that could not be explained with classical concepts.
Thi experiments will be described in Chapters 1—3, and there­
fore we shall mention them here only very briefly.
In the first place. It was found that black-body radiation, the
photoelectric effect, and the Compton effect could be explained only
INTRODUCTION xix

on the basis of corpuscular properties of light. This was the im­


plicit assumption of the Planck-Einstein photon theory, in which
the discrete structure of light was described in terms of Planck’s
constant h = 6.62 • 10“27 erg • sec. The photon theory was also suc­
cessfully used by Bohr in constructing the first quantum theory of
the atom, based on the planetary model suggested by Rutherford.
In the second place, a number of experimental facts, including the
electron diffraction, indicated that in addition to its corpuscular
properties, an electron has wave properties. De Broglie's def­
inition of the wavelength of an electron also included Planck’s con­
stant h. This led eventually to the development of a new s c ie n c e -
electron optics—which provides a theoretical basis for electron
microscopy.
The SchrSdinger wave equation (1926) was the first general
theoretical treatment that explained both of these cla sses of phe­
nomena and unified the preliminary theories of Planck, Einstein,
Bohr, and de Broglie. This equation made it possible to discover
the laws of behavior of electrons and other elementary particles
and to construct a relatively systematic theory of radiation that
took into account the quantum nature of light. For atomic physi­
cists, the Schrodinger equation was one of the most powerful tools.
Many phenomena associated with the behavior of an electron in an
atom and with the absorption and em ission of light by an atom were
provided with a theoretical explanation (see Chapters 4-14). The
later development of quantum theory showed that the Schrodinger
equation did not describe all the properties of atoms. In particu­
lar, it could not explain correctly the interaction of an atom with a
magnetic field (for instance, the anomalous Zeeman effect) and it
could not be used to construct a theory of multielectron atoms.
One of the main reasons for this was that the Schrbdinger theory
did not take into account the electron spin.
Dirac’s relativistic theory (see Chapters 15-17) was an exten­
sion of the Schrodinger theory that considered relativistic and spin
effects of moving electrons (see Chapters 18-20). It turned out that
the quantitative corrections due to relativistic effects were rela­
tively small, but that spin effects were of fundamental importance
in connection with the fine structure of multielectron atoms (see
Chapters 23 and 24). These effects explained the filling of electron
shells in an atom and gave a theoretical basis to Mendeleyev’s
periodic table of elements (see Chapters 25 and 26).
Although the fundamental problems related to the structure of
the atom were basically solved with the appearance of Dirac’s
equation, we are constantly adding further details to our knowl­
edge. At present a great deal of attention is being devoted to the
influence of the electron-positron vacuum and magnetic moments
on the energy levels of atoms (see Chapters 21 and 22). Quantum
mechanics has also been applied to simple m olecules (Chapter 27),
solid state physics (Chapter 6), and the atomic nucleus.
Part I

Nonrelativistic Quantum Mechanic


Chapter 1

The Quantum Theory of Light

Before the supremacy of classical physics was challenged by


the advent of quantum mechanics in the beginning of this century,
particle motion was sharply distinguished from wave motion.
According to the classical picture, the world consisted of particles
(for example, electrons and ions) and fields (for example, light).
This picture was completed by Maxwell’s theory (1873), which
appeared to have definitely established that fields had wave-like
properties.
Towards the end of the nineteenth century and in the first years
of this century, this state of affairs was disturbed by the discovery
of experimental facts that did not fit into the classical conceptual
framework. On the one hand, there were certain phenomena, such
as the radiation spectrum of an ideal black body, the photoelectric
effect, and the Compton effect, which could be understood only in
term s of particle-like properties of light. On the other hand, elec­
trons were observed to have wave-like properties, such as diffrac­
tion, which later served as a basis for the development of electron
optics.

A. PRINCIPLES OF THE ELECTRON THEORY

The behavior of the electromagnetic field produced by a given


distribution of charge and current is described by the well-known
Maxwell-Lorentz equations
u 1 OE
\7 X H --------
c at-j t = —c ,
r—
j r I 1 OH __
V x£+ r r 0' (i.i)
V • £ = 4-p,
V -H=0,

where £ and H are the electric and magnetic field intensities, re­
spectively, p is the charge density (for example, of the electron),
and v is its velocity.
4 NON R E L A T I V I S T I C QUANTUM MECHANICS

To start with, an electromagnetic field transmits interactions


between the charges. The interaction between stationary charges
e is transmitted by an electrostatic field which satisfies Coulomb’s
law [V —— 'j, whereas the interaction between moving charges
is transmitted by a system of electric and magnetic fields, since
a moving charge can be regarded as an electric current and it is
well known that a current interacts with a magnetic field. Elec­
tromagnetic fields are always associated with sources of the
appropriate type (for example, charges).
Secondly, electromagnetic fields may be regarded as electro­
magnetic waves, which propagate with the velocity of light c (radio
or light waves). As a particular example of a source of light waves,
we can take an accelerated charge. The latter em its radiant energy;
per unit time, this energy is

2 e-w'
3 c3
( 1. 2 )

where c is thecharge, and w its acceleration. Once electromagnetic


waves have been produced, they can exist independently of their
sources.
The equation describing the propagation of a light wave is
obtained from Eqs. (1.1) by setting the charge density p equal to
zero. We can then eliminate the vector H from the second Maxwell
equation by taking the curl of this equation and substituting into it
the first equation. Since ■E 0, we can use the vector relation

VxVx£= V ( V - E ) — V'2£ = — V2£ ,

to obtain the following wave equation, which holds for the compo­
nents of both vectors E and //:

(1.3)

where / is any component of the vectors E or H .


A more detailed analysis of the Maxwell-Lorentz equations
shows that electromagnetic waves are transverse. This means that
the electric field intensity (£) and magnetic field intensity (H) are
mutually orthogonal, and also orthogonal to the wave vector k,
which points in the direction of propagation of the electromagnetic
wave. The vectors form a triad such that when a right-handed
screw is turned from E to //, it moves along the direction of k\

H=k" x E, (1.4)
where k" — k k is a unit vector.
T H E QUANTUM THEORY OF LIGHT 5

A charge (say, an electron) moving in externally applied electric


and magnetic fields experiences a force

f= c(£ + |®x//), (1.5)

which is called the Lorentz force.


Taking into account the relativistic variation of m ass, the
equation of motion of an electron in an external field has the form

where

We can select a Lagrangian function X in such a manner that


the variational principle

8 J X ( x it xit t)dt = 0, (1.7)


or, in more explicit form,

2- 3 ). M

will yield the equation of motion of an electron (1.6). Here x-,


Xi—y, x 3= z are the spatial coordinates, and the x, denote the
corresponding velocities. To obtain (1.6), we must set

yB-i i , (1.9)
where A and ® are the vector and scalar potentials of the electro­
magnetic field. These potentials are related by the Lorentz condi­
tion
y . A -1 f ci ot
^ = 0. (1.10)

The electric and magnetic field intensities can be expressed in


terms of A and® by means of the relations

E = — V ® — 1c ^dt , '
( 1 . 11 )
H = VxA.

We find the following expression for the electron momentum:

Pi dX _ , _ e .
(1. 12 )
Y 1—p3T ( l'
6 NONRELATIVISTIC QUANTUM MECHANICS

Similarly, for the generalized force acting on the electron, we


obtain

(1.13)

Substituting (1.12) and (1.13) into (1.8) and taking into account (1.10)
and (1.11), we obtain Eq. (1.6) for the motion of an electron. Thus,
our choice of the Lagrangian is justified.
Since we know X, we may also determine the Hamiltonian H:

H = l p A - x = y = + ^ 1 ^ + ^ = ^ + *

It is well known that the Hamiltonian should not be expressed in


terms of the velocity c$, but in terms of the generalized momentum
P = p — y A, which, according to (1.12), is related to cp by the
equation

camav2
1—pa m:c* 1—pa *

Therefore, the relativistic form of the Hamiltonian is

H = ■/c*Pl -\- m\d -j- e<P. (1.14)

We note that if the potentials are time-independent, the Hamiltonian


is equal to the total energy(£ = /-/).
In the nonrelativistic approximation (P«^m0c), Eq. (1.14) can be
written in the form

// = //' -f- muc2,

where H’ is approximately equal to the nonrelativistic part of the


Hamiltonian

H’f ^nonrel. ■ P‘
- f e<l>= (1.15)
— 2m0

F r m this it can be seen that the relativistic equation for the


Hamiltonian (1.14) also includes the rest-m ass energy m#c* of the
electron. It is very important to take this rest-m ass energy into
account in studying transformations of elementary particles.
T H E QUANTUM THEORY OF LIGHT 7

B. THE CLASSICAL THEORY OF BLACK-BODY RADIATION

Among all the phenomena associated with electromagnetic fields,


special importance can be attached to the properties of cavity radia­
tion. This can be described as the radiation inside a cavity com­
pletely surrounded by opaque walls, which are heated to a certain
constant temperature T, or, alternatively, as radiation in equilib­
rium with an isothermal enclosure. A small hole made in the wall
of such an enclosure behaves like an ideal black body, because
practically no external rays incident on the opening are reflected.
In other words, essentially all rays entering the cavity through the
hole are absorbed, or, more precisely, the probability that they
reemerge from the hole is negligibly small. Consequently, the
cavity radiation escaping from the hole may be regarded as the
radiation which would be obtained from an ideal black body, and it
is generally referred to as black-body radiation.
The analysis of black-body radiation played a particularly im­
portant role in the foundation of quantum theory. Although a more
or le ss reasonable classical explanation could be found for many
other phenomena, every single theory of black-body radiation con­
structed on the basis of classical concepts failed to agree with the
experimental facts. A systematic theory of black-body radiation
was developed only in the beginning of this century, when Planck
introduced the concept of a quantum of energy. This concept
was later to play an important role in the development of the first
quantum theory of the atom, and afterwards in the development of
quantum mechanics.
We shall now consider the theory of black-body radiation, con­
fining ourselves for the time being to classical concepts. Let the
radiation be characterized by its spectral density pu, ’ which is
related to the ordinary electromagnetic energy density

(1.16)

by the relation

du
plu—'rfZT' (1.17)

In th e l i t e r a t u r e , t h e f u n c t i o n p ^ i s s o m e t i m e s u s e d for t h e s p e c t r a l d e n s i t y . The
f u n c t i o n p ^ i s r e l a t e d to p v by t h e e q u a t i o n

Pc0 - pv •
277

sin ce = 277 V.
8 NONRELATIVISTIC QUANTUM MECHANICS

where du is the energy density of the radiation in the frequency


range from to to lo-j-du). Obviously,

CO
( 1 . 18 )

On the basis of the second law of thermodynamics, Kirchhoff


showed that the density pa is determined only by the temperature
of the walls of the closed cavity and is entirely independent of the
material of which the walls are made; that is, pa) = /(u>, T),
Consequently, the walls of the cavity may be considered as a set of oscillators. The
average energy of these oscillators is completely determined by the spectral density of
the black-body radiation. We shall show this starting with the equation of motion for an
oscillator and taking into account Planck's radiation damping

+ E* (U9)

Here e and m0 are the charge and rest m ass of the oscillator, <a is its natural frequency
of oscillations, and Ex is the x component of the electric field intensity of the black-
body radiation.
Representing Ex in the form of a Fourier series
CD
£*= 2 e« e ,'n“0'. (L20)
n=—co

where Exn is the amplitude of an individual oscillation of the field with frequency

“n= n“o, (1.21)

we obtain the following equation for x (() from Eq. (1,19):

m0 ( 1. 22 )
x =
_1 „ 2 eHmc.)1
( " “ ») + ' 3 m 0c»

The average energy of the oscillator, which, according to the virial theorem, is twice
the average kinetic energy, is given by

e
m0 (1.23)
E = m0x 2 = m0
1

n= _oo
* / v.
CO ^ ( n c o o ) - +
. 1• 23 « m>f)C
“o)a
,

where the bar denotes averaging over time. Since

oo CO
{ y / / " - ‘y V f f
’ n* —r t' »
n =» — co n, n ' =j= — oo
TH E QUANTUM THEORY OF LIGHT 9

where /_„ = /* , and since, moreover.

2
gloj0<(«—n') __ J_ P e/<o0( («—.a') ^ _ 0 1or n = n,
T J \0 for n' y i n, (1.24)

—"2
where x = 2ji/ m0i Eq. (1.23) can be reduced to the form

(-o), ^ | £ , n|
m0 Jt2 = 2 m0 2 ^ra(noip)
i ««».)*- “T + { I w„cs

This equation has a very sharp maximum in the neighborhood of frequency a , and
hence the total energy of the oscillator will actually depend only on those terms of the
series for which na>0 «= <■>. Consequently, in the above equation, the square modulus of the
amplitude | Exn |2 can be replaced by| Ex |, where n0 — — , and at the same time the sum
can be changed to an integral. According to (1.21),2

d i a n - - ai 0d f l = a>0 = — , (1.25)
nQ
since d n = I. Therefore, we obtain

l* W !d<ji„
£= 2
m0 ( 2 e2 “
>;!l3 (1.26)
’ " 1 “ \3 m0 c3J

Replacing the frequency u„ by <■>everywhere except in the difference u>n — introduc­


ing the variable of integration £ = — w, and extending the limits of integration to
± oo, we find

ripe-1 ' A>1QI </£


£= 2
w0 2 r2 io2
4£2 + 3 w0c!

2 ua 1 ■*■'’ 0 1
(1.27)

On the other hand, the energy density u, which is related to the electromagnetic field
of the radiation by Eq. (1.16), can also be expressed in terms of | £ |2. Since the radia­
tion is isotropic we have, on the basis of Eq. (1.16),

“ = L (£2 + "*) = 4
“ + &y + Ei), (1.28)

^ T h e r e a d e r s h o u l d n o t f e e l u n e a s y a b o u t o ur e q u a t i n g t h e d i f f e r e n t i a l d<on to t h e f i n it e
q u a n t i t y <Oq = 2tt/ t, for i t i s a l w a y s p o s s i b l e to m ak e t h e p e r i o d T s u f f i c i e n t l y l a r g e s o t h a t
th e r e l a t i o n s h i p Cl)q oj w i l l b e s a t i s f i e d . M a t h e m a t i c a l l y , t h i s c o r r e s p o n d s to a t r a n s i ­
tion from F o u r i e r s e r i e s to F o u r i e r i n t e g r a l .
10 N O N R E I —A T I V I S T I C QUANTUM MECHANICS

Using the expansion (1.20) and the rule (1.24) for averaging over t , we obtain

! in .
u = l E* = k 2 (1.29)

Hence, taking into account Eq. (1.18) together with the relationship

, dm- dm-
in - — a = n0 — - , (1.30)
m0 m
foro>n = to (n = n0)t we obtain

3n» I Exn„ I’
P<0 --- 4
2 h ( U l)

Compairing Eqs. (1.31) and 0-27), we find the relationship between


the average energy £ = m 0 x i of the oscillator and the spectral
density of the radiation pm

d.32)

which forms the basis of the theory of black-body radiation.


In classical statistical physics, the energy distribution of
particles is given by the function

N {E) = Ae~aE, (1.33)

where « = 1'kT; k = 1.38 • 10" ’ 8 erg-deg ' 1 is Boltzmann’s constant,


and T is the temperature of the medium. The average energy of the
particles is
CO

(1.34)

= O
~n\ m = kT.

Substituting this value of E into Eq. (1.32), we obtain the Rayleigh-


Jearis formula
,9
P, kT. (1.35)
T.C
T H E QUANTUM THEORY O F LIGHT I

This equation satisfies the Wien’s thermodynamic law

P» = (1-36)

which was based on various results in thermodynamics and the


electromagnetic theory of light. In the region of long wavelengths
(low frequencies), the Rayleigh-Jeans formula is in good agree­
ment with experimental data. At short wavelengths, however, it
completely fails to agree with experiment (see Fig. 1.1).

F i g . l . l . R a d i a t i o n s p e c t r u m of a n i d e a l b l a c k body .
T he h e a v y d o tted line i n d ic a te s the R a y le ig h - J e a n s
2
c u r v e po) = pQX , a n d t h e s o l i d l i n e t h e P l a n c k c u r v e
Pa> —pox ^ / ( eX ~ 1)» w h i c h i s t h e s a m e a s t h e e x p e r i ­
m e n t a l c u r v e . H e r e p 0 = ( k T ) 3/ n 2h 2c 3, (o = oj0x, a n d
<u0 = kT/H.

In exactly the same way, the use of the Rayleigh-Jeans formula


for calculation of the radiation energy density [see Eq. (1.18)] re­
sults in a divergent integral, that is, we obtain the obviously absurd
relationship

« = j pu,dm= - ^ 5 - j* dm= oo. (1.37)


0 0

This was called the “ultraviolet catastrophe” by Ehrenfest. Thus,


the classical theory was completely unable to give a satisfactory
description of black-body radiation.

C. PLANCK’S EQUATION

In 1900, Planck put forward an important hypothesis which re­


moved the ultraviolet catastrophe and radically changed a number
of fundamental principles of classical physics. According to this
12 NONRELAT1VIST1C QUANTUM MECHANICS

hypothesis, the energy of microscopic system s (atoms, molecules,


and so forth) does not vary continuously and assum es only certain
specific discrete values. In particular, the energy of a harmonic
oscillator must be a multiple of a certain minimum value : e

£ = ns, (1.38)

where n — 0 , 1 , 2 __
In order to determine the average value of the energy, we must
replace the integral (1.34) by the sum
00

(1.39)

Substituting this value of E into Eq. (1.32), we obtain the spectral


density of the radiation
O)- £
Pa, = - ^ r — •(1.40)
ekT- l

We can bring this equation into agreement with Wien’s thermo­


dynamic law by letting e be proportional to u>:

t = fiu>. (1.41)

We then obtain Planck’s equation

na) 3
Pm fnu >(1.42)
7i’>c, (eW — I)

which was a brilliant achievement of quantum theory.


The quantity h = 1.05 • 1O' 27 erg.sec, which has the dimensions of
action. Is called Planck’s constant.3
At low frequencies 1 :, the exponential e ^ ,kTmay be expanded
in the form of a power series in too kT. Restricting ouselves to the
linear terms of the expansion, we have

1 kV

In Ihe 1i l e n i t ’iro, P l a n c k ’s c o n s t a n t is m ore o f t e n t a k e n a s t h e q u a n t i t y h 2 nfi =


6 . 6 2 4 9 • 10 erf» • s e c , w h i c h r e l a t e s th e e n e r g y ( to th e f r e q u e n c y v:

f his.
THE QUANTUM THEORY OF LIGHT 13

Thus, Planck’s equation (1.42) reduces to the Rayleigh-Jeans


formula (1.35).
In the case of high frequencies l), we may neglect the 1 in
the denominator of Eq. (1.42) and write pm in the form

a - 4 3 )

Planck’s equation (1.42), which describes the dependence of the


spectral density of thermal radiation on the frequency w, is in
excellent agreement with experiment (see Fig. 1.1).
From Eqs. (1.42) and (1.18), we can find the total radiation
density
oo
0)3dm
u= e 1lw/kT __J
(1.44)

Introducing the variable £= / ) i o a n d considering that

r e3 dk 7t4
] 7 -1 15’

we obtain the well-known Stefan-Boltzmann law 4

u = a7’4, (1.45)
15 cW

where

a = i h w ==7-56 • 10'" e rg -cm ' 3 -deg'4. (1.46)

From Eq. (1.42), it can be seen that the spectral density of


black-body radiation has a maximum at some value of i» and that
the position of this maximum changes with temperature. The
equation governing the position of this maximum is called Wien’s
displacement law. More often, Wien’s displacement law is expressed

^ U s u a l l y it i s n o t t h e d e n s i t y u w h i c h i s m e a s u r e d , b u t t h e e n e r g y a T 4 w h i c h i s r a d i a t e d
p e r s e c o n d p e r s q u a r e c e n t i m e t e r o f t h e b l a c k b o d y ’s s u r f a c e w i th in a s o l i d a n g l e o f 277
c C c
(U = 277). In t h i s c a s e , t h e Stefa n -B o ltzm a n n c onstant i s O ~ 277 ---- a = — a = 5.67 • 10
8 77 4
e r g •cm ^ •d e g 4
14 NONRELATIVISTIC QUANTUM MECHANICS

in term s of the spectral distribution with respect to the wavelengths


X. To determine px, we can use the expression for u

w = J P\dk

Since X— 2 kc / w , transforming to the spectral distribution over the


frequencies, we have
CO 00

«= jpo.du), (1.47)
o a

from which we find

2Ttc 16it3cft
Px-- X2 Pm-- 2 r.cH (1.48)
X5(eftrx._i)

To determine the wavelength Xmax at which px has its maximum,


we set ^
(M = 0 -

2Itch
*rxr
—5 2 rcrfl = 0.
h T l
e m ax .

Setting 2nchlkT\max = y, we obtain the equation

£/ = 5(1 — e~v),

whose solution can be given with good accuracy in the form

i / ^ 5 ( l — e-») = 4.965.

Thus is related to the temperature T by the equation

Xmax7' = 4?9 6 M = A= 0-29 cm -deg, (1.49)

which expresses Wien’s displacement law, and where b is the


Wien’s constant. According to this law, as the temperature of an
ideal black body increases, the maximum of the radiation intensity
is shifted towards shorter wavelengths (see Fig. 1.2).
TH E QUANTUM THEORY OF LIGHT 15

Equations (1.46) and (1.49) relate Planck’s constant /2 and Boltz­


mann’s constant k to the constants a and b.
Knowing the numerical values of a and b , we can determine
tl and k. This is the way in which a numerical value was first
obtained for U and a better value found for k.

F i g . 1.2. W i e n 's d i s p l a c e m e n t law.


C u r v e s of t h e sp e ctral energy distribution as
a function of t e m p e r a t u r e for an i d e a l b l a c k
body: Xm a x T = 0 . 2 9 cm . deg.

Recapitulating, it follows from Planck’s hypothesis that proc­


esses such as em ission and absorption involve discrete quanta. In
other words, the energy change of particles involved in these proc­
esses is discontinuous and not smooth as would follow from the
laws of classical physics.

D. EINSTEIN’S PHOTON THEORY

In deriving his equation, Planck assumed that the energy of the


oscillators is quantized. The original version of the theory, however,
does not provide any physical justification for this property. Indeed,
Planck himself chose to attribute the “ special properties” to the
heated body rather than to the electromagnetic radiation.
The second important step towards the development of quantum
theory consisted of Einstein’s hypothesis that oscillators absorb
and emit radiation in discrete amounts because electromagnetic
radiation itself consists of discrete particles, calledphotons, which
carry an amount of energy hw. In effect, Einstein interpreted
Planck’s equation as a description of the corpuscular properties
of light.
We shall now attempt to develop an elementary theory of
photons.
16 N O N R E L A T I V I S T 1C QUANTUM MECHANICS

According to classical theory, the energy of a light wave is

s= ( £ 2 + m d*x = <Px,(1.50)

where d*x is an element of volume, and the integration extends over


all space. The electromagnetic momentum n of the light wave in
classical theory is

"= i J E*Hd*x, (1.51)

According to Eq. (1.4), this can also be written in the form

«= j£ W x . (1.51a)

Comparing (1.51a) and (1.50), we find the relation between the


momentum jt and the energy t :

it = k" (1.52)

In the theory of relativity, a sim ilar relation between energy


and momentum holds for particles with zero rest m ass and it can
easily be obtained from Eq. (1.14) by substituting m„ = 0, <t>= 0
and >4= 0.
From these considerations, Einstein concluded that an electro­
magnetic field canbe considered as a set of particles called photons,
with zero rest mass and the energy

£= /ko). (1.53)

For the photon momentum the following equation is obtained:

n = k'>-
C
= k'>-K = flk, (1.54)

2~b° / 2 it
where h — 2-h, and k = — is the wave vector! k = — is the wave
number).
On the basis of these concepts, in 1905, Einstein constructed a
qunnfitative theory of the photoelectric effect, which had been dis­
covered by Hertz in 1887. What is observed in the photoelectric
effect is the following: the potential difference required for a
spark to jump between two small charged spheres is reduced if the
THE QUANTUM THEORY OF LIGHT 17

cathode is illuminated. To explain this phenomenon, Einstein


postulated the simple equation

= (1.55)

This is essentially the law of conservation of energy and indicates


that the kinetic energy of the ejected electron is equal to the
difference between the energy of the absorbed photon /Zoo and the
work function W of an electron in the metal. It is obvious that if
fiu><^W, electrons cannot be ejected from the metal. Only if the
energy of the incident photons exceeds W can electrons leave the
metal.
The experimental verification of Einstein’s theory of the photo­
electric effect provided striking confirmation of his basic conclusion
that the energy of the ejected electrons depends only on the fre­
quency of the incident light and not on its intensity, and that the
em ission of photoelectrons begins only when the frequency of
light u> exceeds a certain limit (so-called threshold frequency)

The implications of the photon theory were brought out and


verified in 1923 by experiments on the scattering of x-rays by
free electrons (the Compton effect).
The Compton effect was particularly
interesting because it confirmed not
only the law of conservation of energy
(which was already verified by the
photoelectric effect) but also the law
F i g . 1.3. S c a t t e r i n g o f l i g h t by a free
of conservation of momentum. e l e c t r o n ( t h e C o m p to n e f fe c t ).
It is well known that, in classical
theory, the frequency of light does not
change when it is scattered by a free electron (u/ = u)). By contrast,
in quantum theory, part of the photon’s energy e = /Zu> is transferred
to the electron (see Fig. 1.3). Consequently, the energy and
frequency of a scattered photon should generally be somewhat
sm aller (s'<^e, u/<^u>). To find the dependence of frequency on the
scattering angle, let us write the laws of conservation of energy
and momentum, treating the photons as particles:
/Zu> — h w ’ — c i ( m — m 0),
(1.56)
Uk — hk' — mv.

Here m0 and m = mnj Y 1 — represent the mass of the electron


before and after collision; v is its velocity; (3= u/c; Hk = Hu>lc and
18 NONRELATIVISTIC QUANTUM MECHANICS

Hk' — hm'lc represent the momentum of the photon before and after
scattering. We rewrite Eq. (1.56) in the form

c8
u> — = — m 0),
(1.56a)
* -* = " •

Taking the square of these equations and subtracting the first


equation from the second, we obtain

u)d/ (1 — cos 0) (^^ ■—


—rwf). (1.57)

Substituting X= 2rc/cu, \’— 2 tcc/u>', and dividing (1.57) by mu/, we find


an expression for the increase in wavelength of the scattered light

AX= X' — X= 2X0 sina y , (1.58)

where X0 is the Compton wavelength of the electron

2 rn
lo -IthC
- - = 2 .4 - 10"‘“ cm.
t l ’ oC

We therefore see that, according to quantum concepts, the wave­


length of the scattered light X'must be greater than the initial wave­
length X(X'^>X) since «/<u>. This difference increases with the
scattering angle < ‘t. Since the Compton wavelength X0 is relatively
small, Compton scattering is observed at relatively short wave­
lengths ( x-rays and gamma rays ). Indeed, for visible light
(X~ 10 5 cm )

I0~B== i ° ' 3 7*.

whereas for x-rays (X ~ 10 8— 1 0 9 cm)

-y- ~ 10 1 = lO°/0.

Therefore, the Compton shift can be observed experimentally only


in the second case.
In his experiments, Compton studied the scattering of radiation
from an x-ray tube by graphite and other substances (lithium,
beryllium, sodium, potassium, iron, nickel, copper, and so on) at
different angles S). The spectral distribution of the intensity of the
scattered radiation at different scattering angles was measured by
means of an ionization chamber.
TH E QUANTUM THEORY OF LIGHT 19

Figure 1.4 shows the spectral distribution of incident and


scattered waves. If the incident wave (upper curve) has one maxi­
mum, the scattered wave (lower curve) will have, in addition to
this maximum, a second maximum at a longer wavelength. The

Wavelength

F i g . 1.4. S p e c t r a l d i s t r i b u t i o n o f x - r a y s in
t h e C o m p to n e f f e c t b e f o r e (u p p e r c u r v e )
a n d a f t e r ( lo w e r c u r v e ) s c a t t e r i n g .

distance between the wavelengths of the two maxima must corre­


spond to the Compton shift; this is because the distance increases
with the scattering angle, and, in addition, it does not depend on
the type of scattering material [both these facts are in accord
with Eq. (1.58)5]. The unshifted maximum corresponds to scatter­
ing by electrons which are strongly bound to the nucleus (or
more precisely, electrons whose binding energy is greater than
the energy of the x-ray quanta). The shifted maximum corresponds
to scattering by electrons which are so weakly bound to the nucleus
that, in practice, they can be regarded as free.
Thus the results of Compton’s experiments completely confirm
the quantum nature of light (that is, the photon theory).

Only t h e i n t e n s i t y o f t h e m a x im a d e p e n d s on th e t y p e o f s c a t t e r i n g s u b s t a n c e . A s the
a to m ic w e i g h t o f t h e s c a t t e r i n g s u b s t a n c e i n c r e a s e s , t h e i n t e n s i t y o f t h e u n s h i f t e d m a x i ­
mum i n c r e a s e s , a n d t h a t of t h e s h i f t e d maxim um d e c r e a s e s .
C hapter 2

The Bohr Quantum Theory

A. BASIC INFORMATION ON PROPERTIES OF ATOMS

A theory of the atom was developed only after reliable experi­


mental data had been obtained from studies of the effects described
below.
1) Emission of light by atoms. From careful studies of the
radiation of atoms, it was established that they have bright-line
spectra and that the lines are arranged in certain definite series.
For example, all the lines of hydrogen are described by Balm er’s
formula 1

(2 . 1 )

where R is the Rydberg constant, and n'and n are integers. Setting


n' = 1 and n — 2, 3, 4, . . . , we obtain the Lyman series, which lies in
the ultraviolet part of the spectrum. F o rn '= 2 and n = 3, 4, 5 .......
we have the Balmer series, which is located in the visible part of
the spectrum and is, therefore, easiest to study.
Formula (2.1) can also be written in the form of a difference
between two quantities
(2 . 1 a)

In s p e c t r o s c o p y , i t is c u s t o m a r y to w r i t e B a l m e r ' s fo rm u la in t h e form

w h e r e t h e R y d b e rg c o n s t a n t for h y d r o g e n is R sp 1 0 9 , 6 7 7 . 6 c m T h e v a l u e o f th e R y d b e r g
c o n s t a n t in Kq. (2 1) is r e l a t e d to / f sp by t h e e q u a t i o n

It 2ncl< llp 277 ■ 3 29 ■ 1 0 1 5 s e c ' 1 2 0 .6 6 • 1 0 15 s e c 1 -

A 2 nc
THE BOHR QUANTUM THEORY 21

which are called spectral terms. For the hydrogen atom, these
terms are given by

This possibility of representing the radiation frequencies « as a


difference between two terms is a consequence of the Ritz combina­
tion principle, which has important spectroscopic applications in
regard to the hydrogen atom, as well as more complex atoms. For
example, hydrogen was initially found to have two series, corre­
sponding to n '= 1 (the Lyman series) and to n’= 2 (the Balmer
series). On the basis of the Ritz combination principle , 2 a third
series was predicted with n’= 3 and n = 4, 5, 6 , . . . . This series was
later discovered by Paschen in the infrared region of the spectrum.
2) The behavior of an atom in external electric and magnetic
fields and, in particular, the interaction of the atoms of a substance
with fast particles passing through it. The most important experi­
ments in this area were conducted by Rutherford, who succeeded
in finding the distribution of positive charges inside the atom from
the analysis of fast-a-particle scattering.
3) Finally, investigation of various properties of molecules
provided important data pertaining to the properties of atoms. For
example, the formation of simple homopolar molecules and the
valence theory found their explanation only on the basis of the
modern quantum theory of the atom.

B. THE CLASSICAL MODEL OF THE ATOM

Once it had been established that an atom consists of a positively


charged part associated with most of the m ass, and of light,
negatively charged electrons, attempts were made to construct a
static model. The reason this approach to the problem was adopted
is that, in classical electrodynamics, an accelerated electron emits
radiation, the amount of energy emitted per unit time being
dE_ 2 e2ws (2 . 2 )
dt ¥ •
T h e R i t z c o m b i n a t i o n p r i n c i p l e w a s f i rs t f o r m u l a t e d a s f o llo w s: if t h ere a r e tw o
d i f fe r e n t f r e q u e n c i e s b e l o n g i n g to th e s a m e s e r i e s , t h e d i f f e r e n c e b e t w e e n t h e s e fre ­
q u e n c i e s i s a l s o a f r e q u e n c y w h i c h c a n a l s o b e e m i t t e d by t h e atom , but b e l o n g s to a n o t h e r
s e r i e s . T h e c o n c e p t of “t e r m s '’, p e r m i t s a r e l a t i v e l y s i m p l e e x p l a n a t i o n o f t h i s . I n d e e d

Tv - T_
Hence

COn n - COn " n T " - T

a n d t h u s the R i t z c o m b i n a t i o n p r i n c i p l e l e a d s d i r e c t l y to E q . (2 .1 a).
22 N O N R EL A TIV ISTIC QUANTUM M E CH A N IC S

where e = — ea is the electron charge ( e0 = 4.80 • 10~ 10 esu is the


elementary charge), &yis the acceleration of the electron, and c is
the velocity of light in vacuum. The minus
sign in front of dE/dt shows that the energy
of the electron decreases as a result of
the em ission of radiant energy. Since an
atom does not radiate in the ground state,
it follows from the classical theory that
the charges in the atom should be at rest.
The most interesting classical model
was that of Thomson, according to which
the positive charge uniformly filled the
entire atomic volume, and the electronic,
that is, negative point charges were lo­
F i g . 2. 1. T h o m s o n ’s m o d el of cated inside the atom.
t h e h y d r o g e n ato m ( 2 = 1).
For example, in the hydrogen atom, the
T h e p o s i t i v e c h a r g e Z cq is
un ifo rm ly distributed over
positive charge was supposed to fill uni­
t h e v o lu m e of a s p h e r e of formly a sphere of radius R0 (see Fig.
r a d i u s /?q. T h e e l e c t r o n (with 2.1). The charge density inside the sphere
c h a r g e - c 0) i s l o c a t e d a t a was (for Z— I)
d i s t a n c e x from t h e c e n t e r o f
__
P= 4 ^ '
In the ground state, the electron was supposed to be located at the
center of the sphere, where the electric field is zero. At a distance
r = x<^Ra from the center, the electric field E is directed along
the radius and its intensity can be found from Gauss' law:

E's2= P
P 3- rr3= e-^ *
Hence

E = % r- <2-3)
Therefore, if an electron with charge e = — e0 and m ass m0 is
placed at a distance x from the center of the atom, it experiences
a quasi-elastic attractive force towards the center

F== — e0E = — ^ x = — m„%x.

With this force, the differential equation describing the motion of


the electron is
J?-)-<Dj;jc= 0 , (2.4)

I’he solution of this equation is

x — A cos K / + <p0),
THE BOHR QUANTUM THEORY 23

where

O>0
el
m0Rl’

Substituting for co0 the fundamental frequency observed in the Balmer


series, we obtain a very reasonable value for the radius of the
atom, namely, R o ~ \ 0“ 8 cm. This value is many times greater
than the classical radius of the electron

ro==^ 7 s = 2 -8 • 1 0 " 13 cm.

The Thomson model agreed completely with the classical


Lorentz theory, according to which atoms can be represented as
harmonic oscillators. Unfortunately, the Thomson model could not
explain the regularities of the line spectra of atoms and, in particu­
lar, the spectral series of hydrogen that are described by Eq. (2.1).
Indeed, from the standpoint of classical theory, the Thomson model
could emit radiant energy only at the fundamental frequency o0 or,
at best, at its harmonics
U>n=nu>o, (2.5)

where n = 1, 2, 3,...
The decisive blow to the Thomson model was dealt by the
experiments of Rutherford, who showed that the positive charge is
not distributed throughout the entire volume of the atom, but is
concentrated virtually at one point. Nevertheless, the Thomson
potential inside a nucleus of finite dimensions, with a charge Ze03
uniformly distributed through the volume, is

(2. 6 )
V2 2 Rl

If t h e c h a r g e of t h e n u c l e u s i s Z e g , t h e e l e c t r i c f i e l d i n t e n s i t y i n s i d e t h e n u c l e u s i s

Z e0 <9$ r
E = — r = ------------,
f?0 dr r

from w h ic h , u s i n g t h e b o u n d a r y c o n d i t i o n t h a t a t r = R q t h e p o t e n t i a l i s t h e s a m e a s for a
point charge

Z e0
$ 1"= R q
Ro
w e o b t a i n E q. (2.6).
24 NONRELATIVISTIC QUANTUM MECHANICS

does play an important role, especially when corrections for the


volume of the nucleus must be made. Moreover, when m esic atoms
with large Z are formed (in a mesic atom, an electron is replaced
by a negative n meson, whose m ass is 207 tim es larger than the
m ass of the electron), there may be states in which the negative /u
meson is always inside the nucleus. In this case, the motion of the
u meson will be determined mainly by the potential (2.6). The ap­
propriate equations of motion w ill, however, be quantum-mechanical,
rather than classical (see Chapter 21).

C. RUTHERFORD’S EXPERIMENTS AND CONCEPTS OF


ATOMIC STRUCTURE

Our present model of the atom is based on the famous experi­


ments conducted by Rutherford in 1911 on the passage of alpha
particles through matter. It is well known that alpha particles,
which are products of nuclear disintegration, possess a sufficient
energy to penetrate into an atom. At the time when Rutherford
conducted his experiments, there was no other source which could
produce sufficiently heavy particles (that is, particles with a mass
comparable to the nucleus) with an energy great enough to penetrate
the atom.
By passing alpha particles through thin sheets of metal (foil),
Rutherford showed 4 that most of the alpha particles which pass
through the foil are scattered through relatively small angles (2-3°)
from their initial direction of motion. Within the framework of
the Thomson model, these small deflections could be explained in
terms of the statistical theory of random processes, because of
the relatively weak interaction between the atoms and the alpha
particles. Rutherford and his co-w orkers, however, also detected
individual deflections of alpha particles through very large angles
of up to 180°. Although the number of these deflections was very
small (for example, when a beam of 8000 primary alpha particles
from RaC is passed through platinum foil, at most one particle is
deflected through an angle greater than 90°), it was, nevertheless,
much larger than the number which could be predicted on the basis
of superposition of a large number of small random deflections.
Large scattering angles were also observed when alpha particles
were passed through a gas. These could be easily seen in photo­
graphs taken in a Wilson cloud chamber.
From a general analysis of his experiments, Rutherford estab­
lished, first, that atoms are fairly transparent to alpha particles

In e a r l i e r e x p e r i m e n t s , R u th e r f o rd a n d h i s c o - w o r k e r s h a d e s t a b l i s h e d t h a t a l p h a
p a r t i c l e s h a v e th e s a m e m a s s a s t h e h e l i u m ato m , a n d a p o s i t i v e c h a r g e w h ic h i s t w i c e
the m a g n i t u d e of th e e l e c t r o n c h a r g e . It i s now known th at a l p h a p a r t i c l e s a r e th e n u c l e i
of he I ium atom s.
THE BOHR QUANTUM THEORY 25

(that is, their structure is relatively “ open” ); and, second, that


large deflections can take place only if a very strong electric field
exists inside the atom. This electric field must be produced by a
positive charge which is associated with a large mass and con­
centrated in a very small volume. We note in this connection that,
according to Eq. (2.6), the largest field produced by a nucleus of
radius R„ is
d> __Ze o
max~ Ro •
To explain these results, Rutherford proposed a planetary
model of the atom in which the structure of the atom resem bles
a planetary system. A positively charged nucleus constituting
almost the entire mass of the atom is concentrated at the center
in a very small volume of radius 1 0 ~ 13 — 1 0 1\ and charged
electrons move about this nucleus in closed orbits like planets
around the Sun. We note that the potential energy of the Newtonian
attraction between a planet of m ass m and the Sun (of m ass M)
y _ V.mM
Newt r ’

where 7. is the gravitational constant and has the same form as the
potential energy of the Coulomb attraction between an electron and
a nucleus

From this model, Rutherford developed a quantitative theory of


scattering. His calculations were based on the assumption of a
Coulomb interaction between the alpha particles and the nucleus.
The influence of the atomic electrons was neglected in the first
approximation, since their energy is considerably lower than the
energy of the bombarding particles.
Let us find, following Rutherford, the trajectory of an alpha
particle moving in the field of an infinitely heavy 5 point nucleus
having a charge Ze0. Our calculations will be carried out in a

5If t h e f i n i t e n e s s o f th e n u c l e a r m a s s Mnuc *s t a k e n i n t o a c c o u n t , th e n u c l e u s h a s a
c e r t a i n r e c o i l ( l ik e t h a t of th e a l p h a p a r t i c l e ) a s a r e s u l t of th e i n t e r a c t i o n . In t h i s c a s e ,
all th e c a l c u l a t i o n s m u s t b e p er fo r m ed in th e c e n t e r - o f - m a s s s y s t e m and, in t h e r e s u l t s
o b t a i n e d for t h e c a s e ,\7nuc it i s n e c e s s a r y to r e p l a c e th e m a s s of t h e a l p h a p a r t i c l e
Mg by t h e r e d u c e d m a s s
M0 »'nUc
re d
« 0 + Mnuc

( s e e C h a p t e r 12, S e c t i o n C for a d i s c u s s i o n o f the r e d u c e d mass).


26 NON R E L A T I V I S T 1C Q U ANT UM M E CH A N IC S

coordinate system whose origin coincides with the nucleus (see


Fig. 2.2). Since the field produced by the nucleus is centrally
symmetric, in determining the trajectory of the alpha particles we
can use both the law of conservation of energy

E = const, (2.7)

and the law of conservation of angular momentum

(r x v ) = const, (2 . 8 )

where M0 is the m ass of the alpha particle, r is its coordinate,


and v is its velocity.

F i g . 2.2 . D i a g r a m for t h e d e r i v a t i o n o f R u t h e r ­
f o r d ’s fo rm u la for t h e c r o s s s e c t i o n of e l a s t i c
s c a t t e r i n g o f a l p h a p a r t i c l e s by n u c l e i .

Let us introduce the polar coordinates r and 9 . The velocity of


the particle is given by

& = v\ +w*J. = ^ + rY , (2.9)

where and v i = r§ are the components of velocity parallel


and perpendicular to the radius vector r, respectively, andr=^T
and 'f = ‘-Jt . We then obtain, instead of Eqs. (2.7) and (2.8),

E ■= -| 1/ = (f2 + r'p) + — -5 = const, (2.10)

Lz = Mn(r >v)z = M0r't§ = const. (2 . 1 1 )

In the absence of interaction, the alpha particle would pass the


nucleus at a distance b (this distance b is called the impact para­
meter). Setting the initial velocity equal to u„ (that is, the velocity
THE BOHR QUANTUM THEORY 27

r -* — oo and as follows from Fig. 2.2), then (2.10) and


(2 . 1 1 ) can be reduced to the form

Y ('* + r y ) + <2-12)

i = M0 H . (2.13)

where the initial energy £ 0 is related to the initial velocity by


the equation
(2.14)

Introducing the new variable

(2.15)

and noting that then, according to (2.13),

Vob (2.16)
It 1= r2“
v0bu\
and
(2.17)

where t C = —t we transform Eq. (2.12) to

u'1+ u1+ M0v?,b* u — - =


ba
o (2.18)

Differentiating this equation with respect to 7 , we obtain

2Ze\ (2.19)
u M(,vlb2 = 0 .
Hence
u = A cos y-\-B sin 7 ------------ . (2.20)

The unknown constant coefficients A and B can be determined


from the initial conditions

( 2 . 21 )
lim r = lim —= 0 0 ,
■-* r. ^

and
( 2 . 22 )
limrsin 7 = lim =
28 NON R E L A T I V I S T 1C Q U A N T U M M E CH A N IC S

Setting <p— ~ and u - - 0 in Eq. (2.20), we obtain

A 2Zej (2.23)
MoVjb1 ’

and, consequently, applying the condition (2.22) to Eq. (2.20), we


have
(2.24)

Thus, we finally obtain

« = j s in c P- - ^ g | r (l + cos(p). (2.25)

This equation gives the relationship between the absolute value


of the radius vector r and the polar angle <p, and thus describes the
trajectory of the alpha particle in the Coulomb field of the nucleus.
It is an equation for a hyperbolic trajectory in polar coordinates.
By definition, the scattering angle & is equal to the angle
? (? ^ 'fo= " ) at which the length of the radius vector r becomes
infinite (that is, from Fig. 2.2, u = y = 0). Therefore, from (2.25),
we find
cot » = AWe! _ bE0 (2.26)
OOL 2 2 Zel — Ze\

It follows that the scattering angle increases as b decreases,


attaining 180° for 6 = 0 (see Fig. 2.3).

~C>-0) %
I‘ i^. 2. *1. D e p e n d e n c e of th e s c a t t e r i n g
a n g l e 0 on th e i m p a c t p a r a m e t e r b, w h e r e
b0 ^2 ^ 3 ^oo-
Equation (2.26) can be checked experimentally by photographing
the tracks of alpha particles in a Wilson cloud chamber. From the
maximum scattering angle, it is possible to compute the minimum
THE BOHR QUANTUM THEORY 29

value of b, which turns out to be of the order of the nuclear radius.


The actual form of interaction between an alpha particle and a
nucleus can, however, be determined more accurately from an
investigation of the cross section for the scattering of alpha particles
by nuclei. For this purpose, Rutherford calculated the relative
number of particles scattered at an angle & or, to be precise, the
number of scattered particles which we would expect to find within
the solid angle
d '2 = 2it sin f) cfft. (2.27)

Suppose N particles impinge per unit time on a unit surface


placed perpendicularly to the original velocity of the particles.
From Eq. (2.26), it follows that the scattering angle depends only
on the impact parameter b. For a particle to be scattered through
an angle ft, it must strike a ring formed by two circles with radii
b and b — db. The area of this ring is 2-bdb. Therefore, the number
of particles which hit this area and then, as a result of scattering,
are found within the solid angle dQ is

dN = N-2*\bdb\. (2.28)

From Eq. (2.26), we obtain for the relative number of particles


scattered through an angle ft

^ = *\db'\ = * ( * L ) ' \ d c W ± \ . (2.29)

The ratio dN/N has the dimensions of area and is called the differ­
ential cross section. It is usually denoted by dj. Taking the deriva-
tive of cot2y in Eq. (2.29), we obtain the Rutherford formula for
elastic scattering of alpha particles by a Coulomb center:

* = ( w ) ’- ^ r - (2'30)
sin 2

This formula no longer depends on parameter b.


If all the quantities in this equation are kept constant, with the
exception of 0 , we would expect the following equation to hold:

dasin4y = ^-^-j dQ = const. (2.31)

For ft — it, it is found, however, that the quantity sin4-^ dz ceases to


be constant and begins to decrease somewhat. This fact was
explained by Blackett, one of Rutherford’s students, who studied
30 NON R E L A T IVI S T 1C Q U A N T U M M E CH A N IC S

the lim its of applicability of the Coulomb law. Blackett took a large
number of photographs of tracks of particles in a Wilson cloud
chamber and calculated the frequencies with which the various
scattering angles occur. From analysis of the experimental data, he
established that the number of particles observed at large scatter­
ing angles [according to (2.26), large angles correspond to small
values of the parameter b ] is markedly sm aller than the number
yielded by the formula (2.30). On this basis, Blackett concluded
that, in air, for example. Coulomb’s law is valid down to dis­
tances of the order of 3-KP12 cm. At sm aller distances, there is a
deviation from Coulomb’s law. Indeed, at cm, the
interaction between the alpha particle and the nucleus appears to
take the form of a strong mutual attraction. Further experimental
investigations have confirmed the existence of a characteristic
attractive force at distances less than 10“ 12 cm. This attraction
drops off rapidly with increasing distance from the nucleus.
The Rutherford formula (2.30) can be used to find the number
Z from an experimental determination of dN and N. This was
undertaken by Chadwick, another of Rutherford’s students. Chadwick
showed that the value of Z is very clo se to the atomic number,
which gives the element’s position in Mendeleyev’s periodic table.
The existence of this phenomenon was rigorously proved at a later
date.
Thus the experiments of Rutherford and his colleagues definitely
established the planetary model of the atom. These experiments
proved that the positive charge of the electron is concentrated in
a nucleus with dimensions of 1 0 ' 1 3 — 1 0 ~ 12 cm, and that, inside the
atom. Coulomb forces keep the electrons moving in orbits with
a radius of the order of 1 0 ” 8 cm.

D. THE BOHR THEORY

First of all, let us attempt to develop Rutherford’s planetary


model of the atom on the basis of classical theory. We shall re­
strict ourselves to the case of a hydrogen-like atom, namely, an
atom with a nucleus of charge Ze0, with a single electron (of
charge e = — e0) moving around it. Particular examples of hydrogen-
like atoms include hydrogen (Z— 1), ionized helium (Z = 2), and
so forth.
Introducing the polar coordinates rand <f(x — rcos(f, y = r sin <p),
we obtain the following equations for the kinetic energy and the
Coulomb potential energy:

T = —- - 'r )>

V = — y.el
r
THE BOHR Q U ANT UM THEORY 31

Then the Lagrangian is

+ — , (2.32)

where m0 is the mass of the electron .6


From this Lagrangian, we obtain the following equations for
the motion of the electron:

d dX ,
dt dtp 0
(2.33)
d dX
dt Pr dr 0 .

Here
ujr, o,

and
pr= * X - = m0? (2.34)

represent the generalized momenta associated with the <p and r


coordinates, respectively. Since <p does not occur explicitly in
X (in other words, it is a cyclic coordinate), it follows that
:= 0. Therefore, the corresponding generalized momentum is a
constant of the motion
p?= /?V*$ = const. (2.35)

This is the law of conservation of angular momentum, which is well


known from classical mechanics. The second conservation law,
namely, the law of conservation of energy

E — T-\-V = const (2.36)

is obtained from the condition that the time t does not occur ex­
plicitly in the Lagrangian.
We shall consider only the sim plest case, that of circular orbits,
for which r —. 0. Accordingly, pr = m j vanishes, and from (2.33) we
have
dX
-F = m r. 2f - 7Ze\i = 0_ , (2.37)

6In t h e f o l lo w in g c h a p t e r s , t h e m a s s o f t h e e l e c t r o n in n o n r e l a t i v i s t i c t h e o r y s h a l l be
d e n o t e d b y mo, s i n c e m w i l l b e u s e d for t h e m a g n e t i c q u an tu m nu m b er.
32 N O N R ELA TIV ISTIC QUANTUM M E CH A N IC S

Hence

? = --> ■ (2.38)

Therefore, we obtain for the energy of the electron

£ = | , Y —' ^ = — (2. 39)

Let us now express the basic parameters of the atom in terms


of the adiabatic invariants, which were introduced by Ehrenfest.
According to Ehrenfest’s definition, in the case of periodic motion,
the quantities

(j) Pi dxi = /,- (2.40)

(pi is a generalized momentum, and xt a generalized coordinate)


remain constant during slow (adiabatic) changes of the parameters
of the system (for instance, the charge).
In our case, there is only one degree of freedom (*; = ?). The
conditions (2.35) and (2.40) yield the equation

pv = ml)r^ = -^ , (2.41)

or

? = 2nm0rs • (2.42)

From Eqs. (2.38) and (2.42), we can obtain expressions for r and
$ in terms of the adiabatic invariant:
_ l1 (2.43)
r 4K*m0Zel ’
(2.44)

According to Eq. (2.39), the energy of the electron is then

L — — 2^ . (2.45)

It follows that the frequency of mechanical oscillation v0 is given


by the derivative of the energy with respect to the adiabatic in-
vai iant:

dE W m o Z ’ei (2.46)
v°— dl — /»
THE BOHR QUANTUM THEORY 33

This relation holds not only for the case under consideration, but
also for any periodic or quasi-periodic motion . 7 Moreover, a
system performing any periodic motion can generally emit radia­
tion not only at the first harmonic k = 1 , but also at the harmonics
k = 2, 3, 4 , . . . . The following expression is, therefore, obtained for
the classical frequency of radiation:

v= A-v0 = *-^-. (2.46a)

The classical theory of the planetary model of the atom led to


a number of difficulties when it was used to explain the radiation
of atoms. Since this model is dynamic, it follows from classical
electrodynamics that the centripetal acceleration of the electron
(o>= ~ , where v is its velocity, and a is the radius of its orbit)
will cause the electron to lose energy at the rate

dE 2 e*w*
dt 3 c3

until it falls into the nucleus. This is not, however, what actually
takes place, and atoms can exist in a non-radiating state for an ar­
bitrarily long time. Moreover, according to the classical theory,
the frequency of the radiation should be the same as the frequency
of mechanical oscillation (the fundamental frequency u>= uj0 = 2 itv0)
or, alternatively, an integral multiple of this frequency (one of the
harmonics %= mu0, where n = 2 , 3 , 4 , . . . ) . Once again, this predic­
tion does not explain Balmer’s experimentally established formula
(2 . 1 ) for the lines in the spectrum of radiation.

7We d e f i n e p e r i o d i c m o tio n a s a m o t i o n in w h i c h a p o i n t r e t u r n s to i t s i n i t i a l p o s i t i o n
a f t e r a c e r t a i n p e r i o d o f tim e. A s e x a m p l e s o f t h i s , w e c a n t a k e h a r m o n i c m o tio n

x = a c o s (tit

or m otion a b o u t an e l l i p s e

x - a c o s cat and y = b s i n cot ,

a s p e c i a l c a s e o f w h i c h is m o tio n a b o u t a c i r c l e (a = b). Q u a s i - p e r i o d i c m o tio n i s m otion


in w h i c h a p o i n t d o e s n o t r etu rn , a s a ru le , to i t s i n i t i a l p o s i t i o n , b u t e a c h i n d i v i d u a l
c o o r d i n a t e r e a s s u m e s i t s i n i t i a l v a l u e a f t e r a c e r t a i n p e r i o d o f tim e ( w h i c h i s d i f f e r e n t
for e a c h o f t h e c o o r d i n a t e s ) . A s a n e x a m p l e of q u a s i - p e r i o d i c m o tio n , w e c a n t a k e

x = a c o s (i)\t ,

y = b c o s Cl)2 f »

w h e r e th e f r e q u e n c y c0 \ i s i n c o m m e n s u r a b l e w ith co2 -
34 N O N R EL A TIV ISTIC QUANTUM M E CH A N IC S

The solution to these difficulties was found in 1913 by Niels


Bohr, who added two postulates to the classical laws of motion.
First, Bohr assumed that each atom has a series of discrete
stationary states in which the electron does not emit radiation, even
though its motion is accelerated (the postulate of stationary states).
According to Bohr’s theory, these stationary states can be deter­
mined by quantizing the adiabatic invariants:

(j) Pidqi=nh, (2.47)

where the quantum number n can assume only integral values, that
is, n = 1, 2, 3 , . . . (in classical mechanics, the adiabatic invariant I
could assume any constant value).
Second, Bohr hypothesized that, when an electron p asses from
a stationary state with energy En (the initial state) to a state with
an energy £„-<£„ (the final state), the atom radiates a quantum of
energy h'< = hu> (the frequency postulate), whose angular frequency
COis

(2.48)

This equation can be written in a form sim ilar to the classical


expression (2.46a) for the frequency of radiation:

, = ( n - n ’) ^ - = k ^ - . (2.48a)

Here the quantum number k — ti — n' can be interpreted as the cor­


responding harmonic, and, moreover, the derivative of £ with
respect to /, which occurs in the classical expression, is replaced
by the ratio of the finite increments.
Bohr’s postulate that the stable energy states of atoms form a
discrete spectrum was confirmed in experiments conducted in 1919
by Franck and Hertz. Passing an electronbeam (a current) through
mercury vapor, they showed that, for electron energies le ss then
4.9 ev, the collisions of electrons with Hg atoms do not effect
the magnitude of the current. When the electron energy attains
4.9 ev (see Fig. 2.4), the current suddenly drops. With further in­
crease of the electron energy, periodic sharp dips of the current
are observed (approximately every 4.9 ev). This phenomenon can
be very simply interpreted from the point of view of the Bohr
theory. Let us suppose that the energy of an unexcited Hg atom
(that is, the atom before collision) is £n and, in conformity with the
first Bohr postulate, let us take the next possible energy value to be
Ei = En + 4.9 ev. It then follows that a beam of electrons of energy
E < 4.9 ev is not sufficient to raise the atoms to an excited state;
THE BOHR Q U ANT UM THEORY 35

therefore, the collisions are elastic and the current does not change.
If however, £ 3 s 4.9 ev, the electrons in the beam may give up a
part of their energy (namely, 4.9 ev) to the atoms, and therefore,
the current will change. If the electrons’ energy lies in the range
14.7 ev ^ > £> 9.8 ev, the transfer of energy to the atoms can occur
twice; 4.9 ev is given up in the first collision and 4.9 ev in the
second.

electron energy in th e F ran ck -H ertz


e x p e r i m e n t (1 e v = e 0 / 3 0 0 - 1.6 • 10 12
erg).

We shall now use the first and second Bohr postulates [Eqs.
(2.47) and (2.48)] to construct a theory of the hydrogen-like atom.
In the equation for the radius of the orbit (2.43) and the equation for
the energy (2.45), let us substitute for / its quantized value, which
from Eq. (2.47) is

/ = 2-xnh,

We thus have
n2 ft2 (2.49)
n m„Zel ’
m 0Z 2e \
2nJH2 (2.50)

For n = l , we obtain the energy of the lowest (ground) state of the


atom
c moZ'-e'o
£i — — 2V ’ (2.51)

and the corresponding radius


ft2 1
(2.52)
ri = m„Zel ~ Z G°’
36 NON R E L A T I V I S T I C QUANTUM M E CH A N IC S

where

g0 = —
TTIqBq «=>0.529-10"8*locm (2.53)

is the radius of the f i r s t Bohr orbit.


The second Bohr postulate (2.48), combined with Eq. (2.50) for
the frequency of radiation uw, gives

En — En, _ m0Z 2ei ( I 1 \


(2.54)
ft — 2 ft8 \ n'a n8 )•

Accordingly, for Z = 1, we obtain Balmer’s formula (2.1). The Bohr


theory also relates the Rydberg constant, established prior to this
on a purely empirical basis, with the Planck constant H:

m0eJ
R 2ft8
(2.55)

This derivation of Balm er’s formula, with a value for the


Rydberg constant that agreed with experiment, was one of the
greatest achievements of the Bohr theory. In spite of this success,
however, the Bohr theory has a number of inherent defects, which
became increasingly important in its further development.
In the first place, the Bohr theory was obviously sem iclassical
in nature. In addition, the Bohr theory could be used to compute
only the frequency of the spectral lines, and not their intensity. To
find the intensity, it was necessary to resort to classical electro­
dynamics, on the basis of the so-called correspondence principle , 8
Finally, the Bohr postulates could not be used to construct a satis­
factory theory of multielectron atoms, including that of helium,
which p ossesses only two electrons.

8 A c c o r d i n g lo t h e c o r r e s p o n d e n c e p r i n c i p l e , in t h e l i m i t , a l l t h e r e s u l t s o f a p r e v i o u s
t h e o r y ( h e r e , c l a s s i c a l e l e c t r o d y n a m i c s ) s h o u l d f o l l o w from a n e w t h e o r y ( h e r e , t h e D o h r
th eo ry ).
For in stan ce, fo r h ■* 0 (o r a t t h e lim it of la rg e q u an tu m n u m b e rs ), th e r e s u l t s of q u a n ­
tu m m ech an ics sh ould approach the c l a s s i c a l results. In e x a c t l y the s a m e w ay, w hen
fV > 0, th e r e s u l t s o f t h e r e l a t i v i s t i c t h e o r y s h o u l d a p p r o a c h t h e n o n r e l a l i v i s t i c r e s u l t s ,
a n d so on. I 'h u s , th e c o r r e s p o n d e n c e p r in c ip le e n a b l e s u s to c h e c k th e e x t e n s i o n o f a
t h e o r y b y r e q u i r i n g it l o r e d u c e to t h e c l a s s i c a l p i c t u r e a t t h e l i m i t .
In t h e i n i t i a l s t a g e s o f a n e w t h e o r y , w h e n it c a n n o t y e t b e u s e d Lo i n v e s t i g a t e c e r t a i n
phenom ena, th e c o r r e s p o n d e n c e p rin c ip le m ay be u s e d fo r r e a s o n a b l e e x t e n s i o n s o f th e
p r e d i c t i o n s o f t h e o l d t h e o r y lo the.’ n e w t h e o r y . T h u s , f o r e x a m p l e , t h e B o h r t h e o r y c o u l d
b e u s e d to c a l c u l a t e t h e f r e q u e n c y o f t h e r a d i a t i o n , b u t n o t i t s i n t e n s i t y . B o h r w a s a b l e
lo d e t e r m i n e t h e i n t e n s i t y fr o m t h e c o r r e s p o n d e n c e p r i n c i p l e , a c c o r d i n g to w h i c h t h e o n l y
c h a n g e s of the q u an tu m n u m b er n that w e re a llo w e d (th e s e l e c t i o n r u le s \ n n —n ) w ere
th o s e that c o i n c i d e d w ith th e c l a s s i c a l l y a llo w e d h a r m o n ic s of th e ra d ia tio n . T h e quantum
th eo ry of ra d ia tio n of light w a s c o m p le te d only w ith th e d e v e lo p m e n t of q uantum e le c tr o -
d y n am ic s.
THE BOHR Q U ANT UM THEORY 37

Accordingly, we shall not discuss in greater detail the subsequent


history of the Bohr theory. This theory represented only an inter­
mediate stage in the development of quantum mechanics, a theory
which can be used to determine both the frequency and intensity of
the radiation of atoms. However, we thought that at least a brief
discussion of the basic principles of the Bohr theory was advisable,
for this theory still retains considerable heuristic significance.
In particular, the Bohr theory often serves as the starting point in
the analysis of many results related to quantization.
The conclusions that follow from Eq. (2.54) will be discussed in
Chapter 13, where the problem of the hydrogen-like atom will be
solved quantum-mechanically.

Problem 2.1. Using the Bohr theory, quantize a hydrogen-like atom for the case of
elliptic orbits (that is, find the spectrum of the energy levels). Show that, In the non-
relativistic approximation, the coordinates r and f have the same frequency of variation;
that is, the motion is periodic. Show that the formula for the frequency of the radiation
of an atom remains the same as in the case of circular orbits.
What new feature is introduced in the theory by considering elliptic orbits?
Solution. Using condition (2.40) and noting that, in this problem, the generalized
momenta corresponding to the <pand r coordinates are

. •1 f n n , 2m0Zel P%
p f = m0r ‘<f = const and p r = m ar = \ 2m0E 4----------- p -,

we find
max _____
/» = j = /, = 2 J prdr = 2 * { - p v + Zel
0 'min

There the second integral was calculated from the formula


'max

(2.56)
min
Hence the energy E is
c- 4jc2m0Z2e$ _ n
E= ~ m r + L r < 0-

By direct differentiation of this equation with respect to the variables Ir and it is


easily shown that the frequencies with which the r and <p coordinates change

dE dE
a' = Wr and “?= c>7;
coincide with one another. Using the Bohr quantization rule (2.47), we replace the adiabatic
invariants lr and by nrh and/i^A, respectively, obtaining

Wo Z ‘ g}
Th 3 /ta ’
38 N O N R EL A TIV ISTIC QUANTUM M E CH A N IC S

where n = nr -\- nv, nr = 0 , 1 ,2 , 3 ,. .. and nv = 1 , 2 ,3 , .. . This expression is completely


identical with Eq. (2.50), which was derived for the case of circular orbits; therefore,
the radiation frequencies remain the same as before.
States with a given nr and different n9 have the same energy, but differ in eccentric­
ity e:

In particular, when n? = n , we have a circular trajectory. As n,f decreases, the


elliptic trajectory becomes more elongated.
The new feature to appear in the case of elliptic orbits is the selection rules 3/;^,= ±_ 1,
A« = ± I, ± 2 , ± 3 ......... These account for the appearance of series that can also be
found from the correspondence principle. In the case of circular orbits. An = An.f = rfc 1
(see also Chapter 13).

Problem 2.2. Find the classical equation of motion and the trajectory of an electron
about the nucleus according to relativistic theory. Show that, in this case, the r and tp
coordinates change with different frequencies; that is, the motion is quasi-periodic.
Determine the angle through which the perihelion of the electron Is shifted during "one”
revolution. Obtain a formula for the energy levels and find their splitting. Compare the
results with the corresponding nonrelativistic problem 2.1.
Solution. Using the relativistic Lagrangian function

X = - m0c2 V l - y + ,(2.57)
where

P2 = 5 = C
W 2+

we obtain the equation of motion


_d___Hhv__ _ Zel
dt Y | __ 32

From Eq. (2.57) we determine the generalized momenta /'9 =


dX and _ dX
Oi P r ~ c)F‘
dr
Since p r = yfr r', where r’ it follows, in accordance with the law of conservation
df
of energy, that

„ o , . Po Zel
/; = c \ rS----- / const,

which implies that

r v

Hence we obtain the equation of the trajectory

r ___ <1___ (2.58)


1 + E COS -J’f ’
THE BOHR Q U ANT UM THEORY 39

where

7 c‘p$ ’
f <?P\
9 Z e\E ’

£
It is apparent from Eq. (2.58) that the motion is quasi-periodic. For the shift i f of
the perihelion, we have from (2.58)

Atp =
2 it (1 —i) *-Z2el

With the help of (2.40) and (2.56) we get

/¥= 2 *pr

where

E2
A = ml c2 1----

B^ L ejJL I l el
c‘ c2

Then, for the energy E, we obtain the expression


Z 2e' 1 —Vi>
E = m0c2 1 (2.59)
Z 2e\

From this it is evident that the frequencies and cor = will be different.
'9 K
Using the Bohr quantization rule (2.47), we transform Eq. (2.59) to

, - - e - *.<■ = ».«■{. + K + ^ :w >),-Y (2.60)

where a =
€“ 1
is the fine-structure constant.
ch 137
Expanding the formula (2.60) into a series in a2 and restricting ourselves to quantities
of the order of a4, we have
,, _ R h Z J l. a2Z 2 ( n 3 VI
n,n? n2 [ n2 \n 4 )J‘ (2.61)

Since n9 varies from 1 to n, it follows from Eq. (2.61) that the energy levels, which
are determined by the principal quantum number n = n, + nv, are split into n closely
spaced sublevels (this close spacing is a consequence of the smallness of a-).
40 N O N R EL A TIV ISTIC QUANTUM M E CH A N IC S

The fine-structure splitting is


A h' = lFj n, n _F g2Z 4# ft
u n, n — n, n n3nf (nf — 1)'
f V V
The splitting, or fine structure, of the levels, is a characteristic result of relativistic
effects and essentially distinguishes the predictions of relativistic theory from those of
nonrelativistic theory (see Problem 2.1).

Problem 2.3. An electron is located in a central field

Determine the values of s at which stable states of the system are possible.
Answer, s < 2.
Hint. Use the expression for the effective potential energy

„ Pi -4
eff 2 m<,ra rs

and make use of the fact that stable motion is possible if Pgff has a minimum.

Problem 2.4. Find the scattering cross section for nonrelativistic electrons by nuclei
(Coulomb point charges). Compare the result with the scattering cross section for alpha
particles.
Answer.

di - Ze\ ,2 dQ
4i'0 1 .. »*
an 7

Problem 2.5. Show that in the relativistic scattering, unlike the nonrelativistic case,
electrons can be captured by nuclei. Obtain the total capture cross section.
Solution. According to nonrelativistic theory, the trajectory of electrons in the
Coulomb field of a nucleus is given by the equation

r ~ Tor ----<j>>’
1 + £*7 COS

where

_P\_ + DhZ-el
m„Zef, ’

At 0, this equation describes hyperbolic motion, and therefore the capture of elec­
trons Is impossible.
In relativistic mechanics, the equation of the trajectory has the form of Eq. (2.58).

If
Z-el > I in this equation, y becomes imaginary. Therefore, r —• Ofor <p— co; that is.

the particle falls to the center, which means it is captured.


Since the angular momentum /)? of the electron is related to the initial momentum p
and the impact parameter b by the expressionp v = p b , the condition for capture can be

written in the form V- , . Hence the total capture cross section is

__ _ T.Z-e},
“cap ~ r‘" m a x ~ cip-i ■
C hapter 3

Wave Properti es of Parti cl es

A. DE BROGLIE WAVES

As we mentioned in Chapter 1, the development of modern


quantum theory began with the discovery that light has particle
properties in addition to wave properties (characterized by the
wavelength X and the frequency «>). The energy e and momentum ji
of a quantum of light (photon) were established by Einstein as

E— — jt = M = yAr°. (3.1)

Analyzing these equations, the French physicist de Broglie sug­


gested that they could be generalized to apply to ordinary particles
and, in particular, electrons. Generally speaking, de Broglie as­
sumed that the wave-particle duality is not an exclusive property of
light, but is also a characteristic of electrons and all other parti­
cles . 1 Accordingly, a beam of free electrons, whose relative energy
£ and momentum p are related to the velocity v by the equation 2

_m„c-_ n _ iiiqV
(3.2)
vr^ w ’ p ~ ’
should also exhibit wave-like properties. The corresponding fre­
quency and wave number were defined by equations sim ilar to
Einstein’s:
£ = hu> and p = hk. (3.3)

T h i s h y p o t h e s i s w a s m a d e by d e B r o g l i e w i t h a tw o f o ld p u r p o s e : f i rs t , to p r o v i d e a
p h y s i c a l b a s i s for t h e B o h r q u a n t i z a t i o n ; s e c o n d , to e x p l a i n t h e f i r s t e x p e r i m e n t s on e l e c ­
tr o n d i f f r a c t i o n ( s e e below).
2
F ro m no w on, w e s h a l l l e a v e i t to t h e r e a d e r to d i s t i n g u i s h b e t w e e n t h e r e l a t i v i s t i c
e n e r g y (w h ich i n c l u d e s t h e r e s t m a s s e n e r g y ) a n d t h e n o n r e l a t i v i s t i c e n erg y . Only in
c a s e s in w h ic h th e two e n e r g i e s a p p e a r in t h e s a m e e q u a t i o n w ill t h e y b e d i s t i n g u i s h e d
by s o m e k i n d o f i n d e x , for e x a m p l e ,

E = E f m0 c 2 .
42 N O N R EL A TIV ISTIC QUANTUM M E C H A N IC S

Consequently, the de Broglie wavelength oi the moving particles is

X=■?“-=
k *p. ' (3.4)

The de Broglie relations (3.3) thus generalized Einstein equations


(3.1), derived from the photon theory. These now became equally
applicable to the analysis of light in terms of its particles and
of moving electrons in terms of their wave properties. It is worth
noting that the dual character of particles and light disappears
if Planck’s constant-ft is allowed to go to zero (the correspondence
principle).
Taking de Broglie’s equations (3.3) as the basis of discussion,
we may describe the motion of free particles (along, say, the x
axis) by the so-called wave function, which for this particular case
is analogous to that of light and represents a plane wave:

t) = Ae~2r,('‘l ~ ^ = A e - i <'a‘-* x>= A e~ *lEt~px). (3.5)

From the standpoint of Eq. (3.5), we can attempt to explain Bohr’s


postulate of stationary states [see Eq. (2.47)]. The physical inter­
pretation which we can offer is as follows: the only allowed circular
orbits are those which are divisible by an integral number of
de Broglie wavelengths, that is,

2f = n. (3.6)

Indeed, the wave function is single-valued only when this condition


is satisfied. Furthermore, since in the nonrelativistic case

m0v ’

Eq. (3.6) yields the Bohr condition for stationary states:

p^ — m^rv — hn. (3.7)

B. EXPERIMENTAL OBSERVATION OF THE


WAVE PROPERTIES OF PARTICLES

To investigate the wave properties of electrons and to prove


that electrons have a specific wavelength X, it is first of all neces­
sary to obtain a monochromatic electron beam (a beam of electrons
moving with the same velocity). One possible way of producing
such a beam is by means of an “ electron gun” (see Fig. 3.1),
WAVE PROPERTIES OF PARTICLES 43

which emits electrons with a certain definite velocity v. In the


nonrelativistic case (u<^c), this velocity is given by the equation

m0v3 _ <vt>
' 2 — 300’
(3.8)

where ® is the accelerating potential (in volts) of grid A relative


to the cathode C. Thus, after passage through grid A (at which
point no further acceleration is imparted), the electrons will have
a de Broglie wavelength of

, h h Y \ 50 1 - 2 ■1 0 “ 7 /Q
X= — = -v - — —TF7— cm. (3.9)
m°v Y m0e0<I> V 'l'

To exhibit the wave properties in the most pronounced form,


one must impart to them the longest possible wavelength X.
This can be done by decreasing <1>. Since, however, a certain
amount of energy, known as the work function, is expended in the
ejection of electrons from metal (this energy is of the order of
several ev and gives rise to a certain spread in the electron veloc­
ities), the sm allest potential ® at which the beam will be relatively
monochromatic is 15-20 volts. At these conditions, the de Broglie
wavelength of the electrons is approximately the same (X ~ 1 0 ~ 8 cm)
as the wavelength of soft x-rays.
Before we consider the physical nature of the wave-like charac­
ter of an electron beam, let us discuss several experiments that
have led to the direct detection
of de Broglie waves. To begin
T
with, we shall take the elec­ ~ n
tron diffraction experiments of —!» l>-
Davisson and Germer, who were
T T
the first to make experimental
observations of electron waves.
l|l|--------
Since the de Broglie wavelength
of an electron is of the order F i g . 3 .1 . An e l e c t r o n gun t h a t p r o d u c e s a
of 1 0 8 cm, they proceeded in m o n o c h r o m a t ic b e a m o f e l e c t r o n s .
the same way as for soft x-rays, C i s the cath o d e, and A is the anode.
using a crystal with a lattice
constant of the order of 10 8 cm for the diffraction grating. The
set-up of the Davisson-Germer experiments is depicted in Fig.
3.2. After coming out of the electron gun, the beam impinges
perpendicularly on the surface of the crystal, where the electrons
are scattered at various angles by the surface lattice. Since the
penetration of the electrons into the crystal can be neglected, the
diffraction grating can be regarded as two-dimensional. Accord­
ingly, the position of the diffraction maxima can be determined
from the condition that the path difference s = d s i n 8 (d is the
44 N O N R E L A T ! VI S T I C QUANTUM MECHANICS

two-dimensional lattice constant and D is the scattering angle) is


equal to an integral number of wavelengths (see Fig. 3.3):

d sin&„ = /ia = n 1 . 2 • I0 -' (3.10)


/*> '
where the integer n is the order of the given diffraction maximum.
The diffraction maxima were detected with a galvanometer, which
registered the intensity of the beam of scattered electrons at
different angles (see Fig. 3.2).
Electron
gun

Fig. 3 .2 . D i a g r a m o f t h e D a v i s s o n - G e r m e r
e x p e r i m e n t s on e l e c t r o n d i f f r a c t i o n .

A second prediction of the de Broglie theory was also verified:


namely, the prediction that as the potential & increases, the angle
^.corresponding to the nth diffraction maximum, will decrease in
accordance with Eq. (3.10). Thus the correctness of de Broglie’s
equations was completely confirmed by
these investigations.
It is well known that x-ray diffrac­
tion patterns are produced not only
by single crystals, but also by poly­
crystalline formations. This was shown,
for example, by Debye and Scherrer.
Tartakovskiy and Thomson obtained dif­
fraction patterns sim iliar to these x-ray
F ig . 3 .3 . E l e c t r o n s c a t t e r i n g diffraction patterns by extending this
by a t w o - d i m e n s i o n a l d i f f r a c ­ technique to electron waves and passing
tion g r a tin g . an electron beam through a foil . 3 The
following theoretical explanation can be
given for the diffraction pattern of electrons. The electron beam,
with an energy of several thousands or even tens of thousands of
electron volts, impinges upon a polycrystalline foil, where it*10

T o r e d u c e th e a b s o r p t i o n of e l e c t r o n s , a r e l a t i v e l y thin foil ( t h i c k n e s s of th e o rd e r of
10 cm) w a s u s e d .
WAVE PROPERTIES OF PARTICLES 45

encounters single crystals and is reflected from them. The path


difference s of two rays (see Fig. 3.4) is related to the lattice
constant d of the three-dimensional crystal by the relation

s = 2d sin 9 , (3.11)

where 9 is the angle between the ray and the lattice plane. Since
the single crystals of the foil are oriented at random, a ray can
leave the foil at any angle with
respect to the original direction
(see Fig. 3.5). Among the crystals,
there will be some that are oriented
at just the right angle to satisfy
Bragg’s law

2d sin ©= n\ = n - - - - - , (3.12)
y <i>
F i g . 3.4. R e f l e c t i o n from a th r e e -
dim ensional cry sta l lattice.
where n is an integer. Whenever
this is the case, a diffraction maxi­
mum occurs and a bright spot Q is found on the screen. Since the
experimental apparatus is cylindrically symmetric, the bright spots
form diffraction rings, whose radiuses Rn can be found from the
relation (see Fig. 3.5)

ta n 2 ? = f-, (3.13)
where L is the distance from the screen to the polycrystalline foil.
Since the angle 9 is very sm all in these experiments (9 ^ s i n © ^
tan 9 ), Eqs. (3.12) and (3.13) yield

L_
Rn= n d 1.2 • I 0 ~ 7 (3.14)

that i s , at constant L , d and n,

R . y <I>= const. (3.15)

These relations were completely confirmed by Thomson’s experi­


ments. Electron diffraction patterns and Debye-Scherrer x-ray
diffraction patterns are now widely used in studies of crystal
structure.
It is worth noting that de Broglie’s formula does not only apply
to electrons and other elementary particles such as protons and
neutrons, but also to complex nuclei, multielectron atoms, and
even to molecules. True, their de Broglie wavelength is very small
because of their relatively large m ass. Nevertheless, Stern and
Esterman have succeeded in observing the diffraction of beams of
46 NON R E L A T I V I S T I C QUANTUM MECHANICS

helium atoms and hydrogen molecules in reflections from LiF


crystals.
Polycrystalline

F i g . 3 .5 . D i f f r a c t i o n of e l e c t r o n w a v e s i n a
p o ly c ry sta llin e s u b s ta n c e (the T a rtak o v sk iy -
T hom son experim ents).

A method involving neutron diffraction has been found extremely


useful in analyzing the structure of substances. Neutrons have no
electric charge and, therefore, pass freely through matter even at
low energies (thermal neutrons), when their de Broglie wavelength
is relatively large. All these facts provide convincing evidence
that wave properties are displayed in some degree or other by all
particles and that the de Broglie formula for X is of universal
validity.
The analogy between light waves and electron waves has led to
the development of a new branch of physics, electron optics, which
is devoted to the study of wavelike processes in electron beams.
With the help of this new science, it has been possible to design
and construct electron m icroscopes, which have found wide appli­
cation in modern techniques. In ordinary optical m icroscopes, the
upper limit of the resolving power (and, consequently, the magnifi­
cation) is of the order of the wavelength of the light which is
utilized. To achieve the highest possible magnification, it is neces­
sary to reduce the wavelength of the light. The wavelength, however,
cannot be made arbitrarily small: it is impossible, for instance, to
construct an x-ray microscope, since no appropriate lens exists
for x-rays. On the other hand, satisfactory electric and magnetic
lenses can be produced for electron waves. Thus, the electric
lens consists of a capacitor which has an aperture in the middle
of the plate, while the magnetic lens consists of ordinary magnetic
co ils . 4

^M odern op tical m icro sco p es give a m agnification of a pproxim ately one or tw o


th o u san d . An e l e c t r o n m i c r o s c o p e c a n g i v e a m a g n i f i c a t i o n o f m o r e t h a n 1 0 0 , 0 0 0 . B esides
the e le c tr o n m ic ro s c o p e , the pro to n m i c r o s c o p e is a l s o w id e ly u s e d a t p r e s e n t. Its re­
s o lv in g p o w e r can b e m a d e e v e n g r e a te r th an th a t o f t h e e le c tr o n m ic r o s c o p e .
WAVE PROPERTIES OF PARTICLES 47

C. WAVE PACKETS. GROUP AND PHASE VELOCITIES

From de Broglie’s hypothesis it follows that the motion of a


free material particle having an energy E — mcl and a momentum
p = mv, can be described by means of the plane wave (3.5). The
velocity u of. the de Broglie wave can be found as the time rate of
displacement of a constant phase

Et — p* — const; (3.16)

that is, the phase velocity i s 5

d x E cs
dt p v
(3.17)

According to the theory of relativity, the velocity of the particle v


cannot exceed that of light in vacuum (c). However, the calculated
phase velocity of the wave appears larger than c. This would indi­
cate that it is theoretically impossible for a monochromatic wave
to transport a particle or carry energy, since all particles and
energy must travel with a velocity sm aller than that of light, in
accordance with the principle of relativity.
ST h i s e q u a t i o n c a n b e o b t a i n e d from th e f o l lo w in g s i m p l e c o n s i d e r a t i o n s . T he w ave

fu n c tio n d e p e n d s only on th e p h a s e <f> = (E t — p x ) —; t h e r e f o r e , a t t h e tim e fj = f + At,


H
t h i s p h a s e m o v e s to the p o i n t Xj = x + Ax, w h i c h c a n b e f o u n d from t h e e q u a t i o n

Eti —pxi = Et — px = co n stan t .


Thus
E A t - pAx = o.
H e n c e t h e r a t e of p r o p a g a t i o n o f t h e c o n s t a n t p h a s e (w h ich i s a l s o t h e v e l o c i t y of t h e
w a v e a s a w h o l e , a s c a n b e s e e n from F i g . 3 . 6 ) i s

Ax _ E
At p

F i g . 3 .6 . T h e p r o p a g a t i o n o f a m o n o c h r o m a t ic w a v e .
D u rin g t h e tim e A t, t h e w a v e a s a w h o l e m o v e s a d i s ­

t a n c e Ax. T h e p h a s e v e l o c i t y o f t h e w a v e i s ------.
At
48 NONRELATIVISTIC QUANTUM MECHANICS

An escape from this dilemma was found in the early stages of


the development of quantum mechanics. This solution retained the
wave properties of particles, which had received such striking
experimental confirmation. Thus, each particle was associated
with a group of waves of nearly equal frequencies, rather than with
a single monochromatic wave. This was further suggested by the
fact that the diffraction lines of electron waves were always ob­
served to have a certain definite width. Thus.it seemed that several
waves with very nearly equal frequencies were diffracted, rather
than an individual wave. Yet another basis for this solution was
provided by spectroscopic studies, which showed that all spectral
lines are also characterized by a certain definite width.
One advantage which results from using a set of waves with
nearly equal frequencies rather than an individual monochromatic
wave is that it is always possible to construct a wave packet whose
resultant amplitude is appreciably different from zero only in a
certain small region of space. This small region of space can be
associated with the position of the particle.
On the basis of these considerations, let us attempt to describe
the motion of a particle by constructing a wave packet out of a
continuous set of waves, assuming that the momentum p', which is
connected with the wave number k' by the de Broglie relationship
k' = p'/H, ranges from p — ^ to p ~ r ^ (Ap<Jp). We shall take the
amplitudes A (p') of the individual waves to be

0 , P' < P —
IV (3.18)
A^ = \ rA
^P' P ~ % =/; + 2 "
0 , ?>P+f-

The resultant wave function is

p+ 4 p
(£ 7 - p ’x )
6 dp’. (3.19)

In this equation, let us change from the variable //to the variable
p" r= // — / / — -fd cs p" j=s , dp’ = dp’ ^ and expand C' in a series
about the central point // = /;:

r f ■) i •/
Id = c /' -t ■wifi + !>■% + 7""‘ $ + ■ • • • <3-20>
WAVE P R O P E R T IE S OF P A R T IC L E S 49

Restricting ourselves for present purposes to the first-order terms


(terms proportional to p" ~ A/;) and integrating Eq. (3.19) with
respect to dp" we obtain

(|>= Be- n ( « - P-r)


(3.21)

where the amplitude of the wave packet is

(3.22)

From this equation, it follows that B does not remain constant


either in space or in time.
To determine the velocity of the wave packet as a whole, that is,
the group velocity I corresponding to the motion of some specific
amplitude, we shall use the same procedure as for the phase
velocity. Let us take a certain constant value of the amplitude such
that
(3.23)

We then obtain the following equation for u:

(3.24)

Since E = c Y for a free particle, and


d E __c-p
dp E ’ (3.24a)
we find
(3.24b)

which shows that the group velocity u of the wave packet as a whole
is exactly equal to the velocity v of the particle itself.
From the expression for the amplitude fl(£) = i4-^p of the wave
packet at time t — 0 (when the amplitude corresponds to ?= x; see
Fig. 3.7), it is readily seen that the maximum value of this
amplitude B(0) = A lies at the point x = Q. At all other points cor­
responding to the relative maxima the amplitude is sm aller. In
particular, considering different values of the argument we have
50 NON R E L A T I V I S T I C QUANTUM M E CH A N IC S

At the point £= -, the amplitude vanishes ( B ( k ) = 0 ) . Consequently,


at t = 0 the wave packet may be regarded as localized in the region
of the first maximum, that is, in the region AS A x~it. Hence,
we obtain the relation
ApAx^h, (3.25)

which is known as the uncertainty principle.

F i g . 3 .7 . F o rm of t h e w a v e p a c k e t a t t = 0.

Since the center of mass of the wave packet (that is, its principal
maximum) travels with the velocity of the particle (u = v), the wave
packet describes the localization of the particle. In particular,
it follows from other hypotheses that have been made that the
position of the particle is characterized by the square modulus of
the amplitude of the x wave, namely,

IB I4 = **>!'■

Consequently, the quantities Ax and Ap may be regarded as a m eas­


ure of the accuracy with which it is possible to compute the
momentum and position of a particle in space by means of the
wave theory.
It was also necessary to determine whether i|i waves could be
identified with the structure of particles, or whether these waves
characterize only their motion. The first interpretation of the
relationship between a particle and its associated wave was proposed
by Schrodinger. In terms of his hypothesis, a particle is a wave­
like formation and the density of its “ smearing out” over space is
given by
WAVE P R O P E R T IE S OF P A R T IC L E S 51

In theory, a group of waves can always be used to form a wave


packet whose size is of the order of the radius of any given particle
(for example, an electron). This representation of the particle is,
however, unstable. Indeed, as follows from Eq. (3.17), the phase
velocity of each of the monochromatic waves contained in the wave
packet depends on the corresponding wave number, or momentum
p. Accordingly, each of these monochromatic waves propagates
with its own phase 6

p p

As a result, the wave packet gradually spreads out with time. The
“ spreading” time is determined by the time interval in which the
initially disregarded part of the phase of <|> in (3.21) becomes com­
mensurate with it. According to (3.20), the neglected part of the
phase is proportional top' 4 <~(Ap)4. Thus, from (3.20) we can obtain
a measure of the time At which elapses between the initial forma­
tion of the wave packet and its distortion:

(V) 2 -d2E
T—— 7C.
2n dp2
Or, the time interval starting with the formation of the wave packet
and ending when the distortion of the latter can no longer be
neglected is
At
2th (3.26)

Let us replace Ap by its value from (3.25). Then, in the non-


relativistic case (p<Jm0 c), Eq. (3.24a) yields

d E _ _ p_
dp m0 ’
cEE_
dp1 m0 ‘

Thus we obtain for the “ spreading” time of the wave packet

A t~ ^ {A x )\ (3.27)

° It c a n b e s e e n from t h i s t h a t a s t a b l e w a v e p a c k e t c a n be form ed o n ly for a p a r t i c l e


w h i c h h a s z e r o r e s t m a s s (mo = 0) a n d w h i c h i s m o v in g in a v acu u m ( a s an e x a m p l e of
s u c h a p a r t i c l e , we m ay t a k e t h e p h o to n ). It i s o n ly in t h i s c a s e t h a t t h e p h a s e v e l o c i t i e s
of a l l t h e c o m p o n e n t s o f t h e w a v e p a c k a g e a r e t h e s a m e r e g a r d l e s s o f th e w a v e number.
52 N O N R E L ^T IV IST IC quantum m e c h a n ic s

For a particle with m „ ~ lg and Ax~0.1 cm, the “ spreading”


time is A /~10 '28 sec, and thus the wave packet does not actually
spread. In the case of electrons, however, m0 ~ 1 0 ~ 27 g and
A x~10 ~ 13 cm. Therefore, the wave packet of an electron begins
to spread out after A / ~ 1 0 - 6 sec, that is, almost instantaneously.
Thus, the electron is not a stable formation in the Schrodinger
theory of the “ sm eared” electron. This is inobvious contradiction
with the experimental facts.- Moreover, it becomes impossible to
explain the phenomenon of diffraction if a monochromatic wave,
which provides an appropriate description of the motion of several
electrons, is replaced by a set of several wave packets.
At present, another approach has been adopted, namely, Max
Born’s statistical interpretation of the wave function (the quantity
= as the probability density, or probability of finding the
electron at various points in space. The statistical interpretation
is not concerned with the structure of the electron, and the electron
can rdmain a point charge (or, to be more precise, a charge whose
radius r0 does not exceed 10 13 cm). Only the probability of finding
the electron at different points in space changes as the wave func­
tion changes with time, but the structure of the electron remains
completely unaffected.
From the standpoint of Born’s statistical interpretation, the
quantum-mechanical treatment of problems involving many elec­
trons in identical states presents certain methodological sim ilarities
with the treatment of various problems in the kinetic theory of
gases on the basis of Maxwell’s distribution function (it should be
noted, however, that the quantum-mechanical distribution function
/ = •!<*Ais devoid of the temperature term). For example, in Born’s
interpretation, the diffraction pattern of an electron beam may be
explained in the following manner. The bright spots correspond to
the maxima of the function / = | ^ |4, and thus the greatest numbers
of electrons travel towards bright spots. On the contrary, the
probability is sm allest for electron motion in the direction of dark
regions.
One attempt to explain a certain “ freedom of behavior” of an
individual electron is based upon the complementarity principle,
a solution adopted by Bohr, Heisenberg, and others (see also
Chapter 7). The complementarity principle asserts the theoret­
ical impossibility of extending our knowledge of the microscopic
world beyond a certain finite, even though sm all, limit of accuracy,
since our measuring apparatus necessarily must exert certain
indeterminable effects on the experimental object (an electron,
for example). In particular, for canonically conjugate quantities,
such as the position and momentum, the degree of accuracy
with which they can be measured is given by the uncertainty
principle.
Heisenberg attempted to provide a more rigorous foundation for
the complementarity principle by means of the following hypothetical
WAVE P R O P E R T IE S OF PA R T IC LE S 53

experiment. Suppose we wish to determine the position of an elec­


tron with the aid of an “ ultramicroscope,” that is, an instrument
that is designed for precisely this task and that utilizes a light
beam of appropriate wavelength X(it is not actually possible to
construct such a microscope). We shall assume that the electron
is located at the vertex of a cone of revolution with angle 2 ?, (the
objective being the base), where 9 —the angle between the incident
(wavelength X) and scattered light beams. Any light that enters
the objective after being scattered by the electron must have
traveled within this cone of directions. From the laws of optics,
it follows that the uncertainty in the determination of any of
the electron coordinates in the plane parallel to the plane of the
objective is
X
Ax ~ s in <p (3.28)

Moreover, since the light has a momentum p = y , part of this


momentum is imparted to the electron (the Compton effect). Ac­
cordingly, the momentum of the electron in the given direction can
only be determined with an uncertainty of

A p * ~ x s'nt?' (3.29)

The product of these two uncertainties yields the uncertainty


relation.
From (3.28), it is seen that to determine the position of the
electron with the maximum degree of accuracy, it is necessary to
use light of the sm allest possible wavelength X. Equation (3.29)
shows, however, that the sm aller the wavelength, the greater is the
momentum imparted to the electron (the position and momentum
are canonically conjugate quantities). Thus, according to the com­
plementarity principle, there would have to be two classes of
experimental aparatus. One of these would be designed to measure
the spatial coordinates with any desired degree of accuracy (in the
case of “ ultram icroscopes,” instruments with X->-0 ), the other, to
measure the corresponding momenta (instruments with X->oc).
Then the observer using an instrument of the first class imparts
all of the indeterminable effects to the momentum, whereas the
instrument of the second class imparts all of the indeterminable
effects to the position. Accordingly, in the opinion of the adherents
of the Copenhagen school, we cannot at any given time ascertain
both the position and the momentum of the object, although each of
these variables can be measured separately with any desired degree
of accuracy.
54 N O N R ELA TIV I S T I C QUANTUM M E CH A N IC S

Problem 3.1. Show that there is an integral number of de Broglie wavelengths in an


elliptic electron orbit.
Solution. According to Bohr, the set of stationary elliptic orbits may be found from
the two conditions
and A
where

, - d%- and p v
r ~ dr df *

Since 27"= ^ that ls »


k
* T
2 S Tdt= '2 .\p ^ K dt
- nh,
0 kb
where

n ,-\-n r, and 27" dt = mav ds = T ds,

we obtain the required result


ds
T = ft.

Problem 3.2. Show that the wave

pili„x
<f(x) =
0

is not monochromatic for finite values of /. Find the range of wave numbers Ak = k — ka
over which the amplitudes of the individual harmonics may be regarded as nonzero.
Solution. Let us represent <p(x) in the form of a Fourier integral
OO

where the amplitudes A (k) are given by the formula


4-00

J e - k*dx.
—CO
Substituting the expression for tp(x), we find

s i n - (k — ka) I
/I (k) = / jt (k — A’o) I

Ht-.ae it follows that the largest amplitude corresponding to k = k0 is A (k0) = l/2n.


Although all the other amplitudes are not equal to zero, in practice only those amplitudes
which are of the same order as A (k0) need be regarded as nonzero. These amplitudes
lie within the range Aft = | /e — ft0 1~ ~ . The quantity Aft 2tt// may be regarded as the
WAVE P R O P E R T IE S OF PA R T IC LE S 55

"spread” of the wave vector due to the finite width of the wave packet. Setting Aft = ,
and / — Ax, we again obtain the uncertainty relation:

Ap Ax ~ ft.

Problem 3.3. Find the mean velocity and the "spreading” time of the wave packet
m
? (T, t) = A (ft) e~ ‘ M - *x) dk (3.30)

If the amplitude A (ft) Is In the form of a Gaussian curve:


_ (ft - ftpp
A(k) = e qi •

Solution. Substituting the expression for A (ft) Into Eq. (3.30) and Integrating, we find
for t = 0

if (x, 0 ) = Y ntt e 4 e'*°x


2
It follows that the particle Is initially localized In the region Ax ~ — . This in effect is
<7
equivalent to the uncertainty relation (3.25).
To obtain the shape of the wave packet at any other instant of time, we take into account
the equation
f iV = m-c1+ c2ft2ft2.

In the nonrelatlvlstlc approximation (ft irioc/fi), we then have


m0c2 lift2
a)Rrs~1C + 2 w7 ‘
Substituting this expression Into Eq. (3.30), we obtain, after Integration

+ i (ftp*—a>0()
qYn +
?(■*. 0 =
2/flp /

where

C
D0
m0 c2 ftft2
n r + 2 w7 -
Hence we have for the probability density
<?a(jy—
71^ 2[l + (l £ ) 1
l?C*, 0 l2;
/■+ (£ )'
where t»0 = - — is the velocity of the center of mass of the wave packet. From this formula
mo
It is seen that the maximum of the wave packet, that is, the point x = v 0t, moves with the
velocity Vo of the particle.
56 N O N R EL A TIV ISTIC QUANTUM M E C H A N IC S

The effective width of the wave packet at time t is found to be

If we substitute q ~ into this formula, the "spreading” time of the wave packet is
expressed by a relationship which coincides with Eq, (3.27).

Problem 3.4. Show that a damped oscillatory motion

X (t) = COS 0>0t

results in broadening of the spectral line.


Solution. The damped oscillation can be represented in the form of a Fourier integral
00

x (t) = J a (to) cos oit du>,


o
where

a( cos (iit dt = — ___ 7____


7C + (“—“o) 1
12

For the largest amplitude, we obtain

The amplitudes with the same order of magnitude correspond to frequencies lying in the
range
d to = I CO W0 I ' •y.

It is this equation which represents the line broadening since, in effect, only the
amplitudes corresponding to these frequencies differ from zero.
Since the damping coefficient 7 is connected with the mean life of the damped oscilla­
tions by the relation Af ~ 1/ 7, the above equation yields the familiar optical relationship
connecting the broadening of the spectral lines and the mean life of the atom:
At Aw^ 1 .
Hence, since Am = AEjht we obtain what is known as the fourth uncertainty relation

AF.At ~ h.
C hapter 4

The Ti me-I ndependent Schrodi nger


Wave Equati on

Planck’s quantum theory, Bohr’s postulates, and de Broglie’s


hypothesis represented very important steps in the development of
the theoretical foundations of atomic physics. However, they were
overshadowed by the discovery of a fundamental differential equa­
tion describing the electron and accounting for its wave properties,
and the construction of a theory accounting for the quantum nature
of radiation. The crucial move in this connection was made by
Schrodinger in 1926, when he proposed a partial differential equa­
tion that turned out to be generally applicable to the motion of
charged particles in the nonrelativistic case (v ^ c ). This equation
represented a generalization of the classical Hamilton-Jacobi
equation to cases in which the de Broglie wavelength differs from
zero. The Schrodinger equation stands in approximately the same
relation to the Hamilton-Jacobi equation as does wave optics to
geometrical optics.

A. DERIVATION OF THE TIME-INDEPENDENT


SCHRODINGER EQUATION

We shall show how the Schrodinger equation can be obtained


most simply. We must insist that there can be no question of a
rigorous or general derivation of this equation, since it is not, in
general, possible to set up a new theory entirely on the basis of
old postulates. We shall adopt a mode of presentation which con­
sists essentially of a reasonable generalization of the wave equation
from classical electrodynamics or optics
!_ _ o (4.1)

to the case of de Broglie waves. Here <? is a function describing a


wave disturbance propagating with velocity u. If the wave is mono­
chromatic, a solution to Eq. (4.1) may be sought in the form

<?(r, t) = e-iwe$(r). (4.2)


58 N O N R EL A TIV ISTIC QUANTUM M E CH A N IC S

where u>= 2 :tv is the angular frequency, arid the spatial part (/*) of
the wave function satisfies the equation

V 'W + ^'M'') = 0. (4.3)

In this equation, we can use a single parameter in place of the two


parameters u> and u, namely, the wavelength

We then have

+ = (4.5)

From this general equation, we can obtain a wave equation describ­


ing the wave motion of electrons by substituting the de Broglie
wavelength
jt _ 2*h (4.6)
m0v p

Using the law of conservation of energy

we have

= (/*)]. (4.7)

Substituting this expression into (4.5), we obtain the time -independent


(or stationary) Schrodinger equation

*i’H r ) + 2-£ [ E - V ( r ) ] ^ ( r ) = 0. (4.8)

Once we have found the space-dependent part <Ji(r) of the wave func­
tion from (4.8), we can use Eq. (4.2), which is valid for all mono­
chromatic waves, to obtain the complete wave function, which
depends on both (spatial and time) coordinates. Substituting U>= Y ’
we have 1

' F r o m now on, we s h a l l w r i t e the w a v e f u n c t i o n s w h ic h d e p e n d on both p o s i t i o n a n d


t im e in th e form {//((), w h i l e t h e w a v e f u n c t i o n s w h o s e o n ly a r g u m e n t i s t h e p o s i t i o n w ill
b e d e n o t e d a s t//.
THE T IM E -IN D E PE N D E N T SCHRO D IN G ER WAVE E Q UATION 59

For the complex conjugate function, whose space-dependent


part also satisfies Eq. (4.8), we have

i ~ t
(4.9a)

B. RESTRICTIONS ON THE WAVE FUNCTIONS.


EIGENVALUES AND EIGENFUNCTIONS

The functions (t) which describe the behavior of a particle may


be statistically interpreted by means of the Schrodinger theory. In
particular, the quantity <J<*(/) (/) = <|>, which plays the role of a
distribution function, represents the probability density, or prob­
ability of finding the particle at any particular region in space. If
the probability density differs from zero only in some arbitrarily
large, but finite, region of space 2 , it is accurate enough to say
that the particle is located somewhere in this region. In other
words, the probability of detecting the particle in the region 2 is
unity. Mathematically, this can be expressed in the form

(4.10)

In quantum mechanics, relationship (4.10) is called the normaliza­


tion condition.
It is important to note that the region of nonzero probability
density is not always finite. There are cases where does not
go to zero over all of space (the sim plest of these cases is that of
the motion of a free particle, which we shall consider below). When
this happens, \ diverges, and a somewhat different formula­
tion of the normalization condition must be given.
We shall now give a general analysis of the Schrodinger wave
equation. The Schrodinger equation (4.8) is a second-order partial
differential equation. Its solution resembles the solution of certain
classical problems of mathematical physics, such as the problem
of a vibrating string. We note, first, that certain conditions
must be imposed on the wave function <]i, since it is a solution
satisfying a second-order Sturm-Liouville equation. It must be
continuous and have a continuous derivative. Moreover, it must be
single valued and finite over all of space. Finally, it must satisfy
certain boundary conditions. In general, solutions which satisfy
these requirements do not exist for all values of the parameters,
but only for certain specific values, which are known as eigen­
values. In the case of the Schrodinger wave equation, the energy
E is a parameter of this sort, its eigenvalues being

E'jt .. (4.11)
60 N O N R EL A TIV ISTIC QUANTUM M E C H A N IC S

The solutions of the wave equation corresponding to these eigen­


values
<W. fc. ^3 . ... (4.12)
are said to be eigenfunctions. The possible values of the energy
form the energy spectrum. We shall see below that, if the motion
of a particle is not bounded, its energy spectrum is continuous.
If, however, the position of the particle in space is bounded, the
energy spectrum is discrete.
Let us show that the eigenfunction satisfy the orthonormality
condition
(4.13)

where <w is the Kronecker-W eierstrass symbol, which is equal


to unity for n' = n (the normalization condition), and to zero for
n '^ n (the orthogonality condition). To prove Eq. (4.13), we write
the Schrodinger equations for <Ji„ and

V ^ n+ - ^ ( £ n- m n= 0, (4.8a)

V2^ + ^ °(C n. - m * . = 0. (4.8b)


If we multiply the first of these equations by and the second
by (— 6 „), and then add the two resulting equations, we obtain

+ 2-^ (En - En.) = 0. (4.14)

Since
— V-B,
where
«= — W * .,

integration of Eq. (4.14) over all of space yields

J V ■B d\x + ^ (En- £„.) J = 0. (4.15)

Since the ^ function tends to zero at infinity, we obtain2

^ V- B d 'x = J‘ BndS = 0,

^The surface S Aur2 t e n d s to i n f in i t y at r » ->j. T h e r e f o r e , t h e i n t e g r a l g o e s to z e r o at


r * ^ if t h e w a v e f u n c t i o n (// t e n d s to z e r o m o re r a p i d l y th an r T h is condition is alw ay s
s a t i s f i e d for a d i s c r e t e s p e c t r u m s i n c e th e w a v e f u n c t i o n , a s a r u l e , a p p r o a c h e s e x p o n e n ­
t i a l l y z e r o at in f in i t y . T h e c a s e of t h e c o n t i n u o u s s p e c t r u m w ill b e c o n s i d e r e d s e p a r a t e l y .
THE T IM E -IN D E P E N D E N T SCHRODINGER WAVE E Q UATION 61

Therefore, instead of Eq. (4.15), we have

(£» - £„■ ) j W-tntPx = 0. (4.16)

We shall now assume that E„ E„- (that is, n’ ^ n ). According to


Eq. (4.16), the following equation then holds (orthogonality condi­
tion):

j 6*.6,(rfb: = 0. (4.17)

If, however, n’— n (or £„ = £„■), this integral does not go to zero
and we may impose the requirement that it be equal to unity (the
normalization condition):

(4.18)

Thus the eigenfunctions 4 *1 , and which correspond to the eigen­


values Eu Ei and£a, do indeed possess the property of orthonormality
(4.13), This is one of the most important characteristics of eigen­
functions.

C. A PARTICLE IN A POTENTIAL WELL

As an example of the calculation of eigenvalues and eigenfunc­


tions, we shall consider the motion of a particle in a potential well.
Since the chief interest of this problem is simply that it provides
an illustration of methods used in the solution of this example, we
may assume a very simple dependence of the potential energy on
distance (see Fig. 4.1):

I
V'o for —oo<\\:<;0 (region I)
0 for 0 < . * < / (region II) (4.19)

Fofor / < * < ^ 0 0 (region III)

In the potential well (regionII), where £ > F = 0 , t h e Schrodinger


equation takes the form
^ii :0 , (4.20)

where
62 NON R E L A T I V I S T I C QUANTUM M E CH A N IC S

We note that the case E<C0has no physical 'meaning in this problem.


Since the general solution of Eq. (4. 20) is oscillatory, we have

= B\i cos kx + A Msin kx. (4.21)

In regions I and III, the Schrodinger equation has the form

(4.22)

Here two cases must be distinguished. In the first case (E^>V


the solution for these regions is also oscillatory in character (an
equation of the elliptic type). It is given by Eq. (4.21), the value
of k being

k = ± -V 2 m ,(E -V ,).

No restrictions need to be imposed on the wave functions at infinity.


Therefore, the energy E can assume any value in a continuous
spectrum of energies. It is better, however, not to investigate the
case of a continuous spectrum on the basis of this example, but
rather on the basis of the motion of a free particle (see below).
The potential well only adds to the mathematical difficulties of the
problem, without changing the general character of the solution.

F i g . 4 .1 . T h e m o tio n o f a p a r t i c l e in a
p o t e n t i a l w ell.

In the second case, namely, the case of a potential barrier


(£<^V0), the solution of Eq. (4.22) is exponential in character (an
equation of the hyperbolic type). The general solution can be
written in the form

'W, = A 1. me,JC-f- B\t me


hi (4.23)

where

= 2 m ,(K ,-£ ) = ^ r > 0 . (4.24)


THE T IM E -IN D E PE N D E N T SCHRO D IN G ER WAVE E Q UATION 63

If the energy can assume any value without restriction, the wave
function inside the potential barrier (0<^£<Fo) will contain both
an exponentially increasing part and an exponentially decreasing
part (see Fig. 4.2). Therefore, we must choose only those values
of £ for which exponentially increasing solutions do not exist inside
Exponentially

F i g . 4 .2 . Wave f u n c t i o n for a g i v e n v a l u e o f E .
T h e e n e r g y l e v e l i s t a k e n to b e t h e a b s c i s s a of
t h e w a v e fu n c tio n .

the potential barrier. Accordingly, we require that the coefficient


£i = 0in region I (x < 0), and the coefficient Am=0 in region III ( * > / ) .3
We then have
<h= Ale'x= Ae~'lx',
c?m= B me-"' = Be-'[x-i\ (4.25)

where, for the sake of simplicity, we have made

£ m= B e xl.

By joining the solutions4 at the boundary of regions I and II (x = 0),


and also at the boundary of regions Hand III (x = l), and making use
of the requirement that the exponentially increasing solution vanish,
we obtain the equation for the eigenvalues of the energy £.
We shall now further simplify our problem by requiring that
F0, together with x, go to infinity (see Fig. 4.3). It is apparent from
Eq. (4.25) that <^= i{j|„= 0, and therefore the boundary conditions
for the solution (4.21) inside the potential well (region II) take the
form
i^n= 0 for ,r= 0 (4.26)
and
<pn— 0 for x = l. (4.27)

when E < V q, t h e n u m b er of u n k n o w n c o e f f i c i e n t s in t h e w a v e f u n c t i o n i s s m a l l e r
t h a n t h e n u m b e r o f i m p o s e d c o n d i t i o n s . A c c o r d i n g l y , s o l u t i o n s are p o s s i b l e on ly for c e r t a i n
v a lu e s of E and a d is c r e te spectrum i s obtain ed .
4
In j o i n i n g t h e s o l u t i o n s , we m u s t m a t c h t h e a c t u a l w a v e f u n c t i o n s ( a n d a l s o t h e i r
firs t d e r i v a t i v e s ) a t t h e a p p r o p r i a t e p o in t.
64 NON R E L A T I V I S T I C QUANTUM M E CH A N IC S

Applying Eq. (4.26) and (4.27) to the general solution (4.21) in


region II, we find that Bn — 0, and the eigenvalue are described by
the equation

sin kl = 0 , (4.28)

from which

kl = -n, (4.29)

where n = 1, 2, 3 , 4 , . . . „ We exclude the value n = 0 from further


considerations, since the wave function in this case is identically
equal to zero. It is not necessary to consider separately the nega­
tive values of n, since the wave functions for negative n are equal

V=0 /=°°

F i g . 4 .3 . P a r t i c l e in a p o t e n t i a l w e l l with
in fin ite ly high w alls .

to the wave functions for positive n, taken with the opposite sign.
Since lA = E, we obtain the following equation for the energy
spectrum (the eigenvalues):

En (4.30)
'2 inj*

The wave functions corresponding to these values of energy (eigen­


functions) are

= A ;sin r.n * . (4.31)

The coefficient A„ can be found from the normalization condition


i i
^ <yidx — A \ sin'2r.n d x — -!T A ; t = 1,
f) i)
THE T IM E -IN D E P E N D E N T SCHRODINGER WAVE E Q UATION 65

which gives

Substituting the expression for A„ into Eq. (4.31), we finally obtain

'in= j . (4.32)

According to the general theorem of eigenfunctions [see Eq.


(4.17)] the eigenfunctions (4.32) of the Schrodinger equation satisfy
the orthogonality condition
i
= 0 for n' = n, (4.33)

as can be readily seen by performing the direct integration after


substituting Eq. (4.32) for
We shall now write down a few specific eigenvalues £„ and
eigenfunctions %, shown in Fig. 4.3:

F, — --r- , , / 2 . —.v
(4.34)
2 mj* ' ^ ) / -ln / ’
Ei = 4 £!, 0 o= ^ —sin 2 ~ (4.35)

E , = 9£,, ■y3= ] / -y sin 3~ y . (4.36)

These solutions are very sim ilar to the familiar standing-wave


solutions for a vibrating string with fixed ends. The ca sen = l
[see (4.34)] corresponds to the fundamental mode, th eca sen = 2
[see (4.35)], to the first harmonic, etc.

D. THE MOTION OF FREE PARTICLES.


NORMALIZATION OF WAVE FUNCTIONS IN THE CASE
OF A CONTINUOUS SPECTRUM

We shall consider the motion of a free particle (taking the one­


dimensional case first), when the Schrodinger equation (4.8) has
the same form in all of space (— cc < \v < c c ):

d ‘- l
k-'i = 0 , (4.37)
(lx-
66 NON R E L A T I V I S T I C QUANTUM M E CH A N IC S

where
. -i_ _ 2 maE _ _ p 2
ti‘ ~ li! ■
(4.38)

The solution of this equation is

<]>= Aeikx-\-B e ikx. (4.39)

To determine the physical meaning of each term in Eq. (4.39), we


write the complete, time-dependent function
<J>(t) = = Ae-iM-ox) Be-H*t+kx)'(4.40)

It is seen that the first term Ae~'(',“~kx) describes the motion of the
particle in the positive direction of the x axis, and the second term
Be the motion in the negative direction. If we restrict our­
selves to the traveling wave which is propagated in the direction of
positive x, we have
^ = Aeikx. (4.41)

It is readily seen that the integral ^ <\>*tydx diverges. Con-


— CO

sequently, we must revise the method of normalization [see Eq.


(4.10)]. There are two basic methods of normalizing ^functions of
this sort. We shall devote most of our attention to one of them,
which was proposed by Born. Thus, rather than imposing a bound­
ary condition, we shall subject the wave function to a periodicity
condition

ty(x) = ty{x-\-L), (4.42)

where the length L is said to be the period. It can be made


arbitrarily large. As a rule, L does not appear in the final result.
Rewriting (4.42) in the form
Aeikx = AeikxJrikL,

we obtain eikL= 1 , which implies

k=^. (4.43)

where /i= 0 , rh 1, zb2, zL3 ........ From Eqs. (4.38) and (4.43), the
e; crgy levels are

h2^ 2n2n2nt (4.44)


2 w0 m0L2
THE T IM E -IN D E PE N D E N T SCHRO DINGER WAVE E Q UATION 67

Since the ^ function is periodic in the interval L, the normalization


condition becomes
LI2

^ <p*(J>dx — 1 . (4.45)

Substituting the expression (4.41) for <ji, we have

(4.46)

Therefore, the normalized solutions are


_I_ 1_ . 2 nn
% = h 2elkx = L 2 e' ~ x . (4.47)

The direct integration shows that the functions (4.47) are


orthonormal. As can be readily shown by direct integration:

* , 1f ------ T ~ ^ dx =
^ nd x = T \ e
1/2
I -

sin * (n — ri) __ ^o r r i^ n , (4.48)


= it(n —ri) ~ [\ for ri = n.

Therefore, introducing the artificially defined period, we can


transform the continuous energy spectrum into a discrete one.
If, however, the length L is allowed to go to infinity, the discrete
spectrum again becomes continuous. Since k = ^ = — and An=l ,
the energy difference between two neighboring levels is

. » ___ 251 25c7I Ia . qv


A £-»= —
m —„ =L vL-r-. (4.49)
'

It immediately follows that A£ — 0 if Z.— oo; that is, the energy


spectrum becomes continuous.
We shall now generalize the free particle problem to the three-
dimensional case. The Schrodinger equation can be written in the
form

Just as in the one-dimensional case, the quantity k2 is given by


Eq. (4.20a). We shall solve Eq. (4.50) by the method of separation
of variables; that is, we assume a solution of the form:
(4.51)
68 NON R E L A T I V I S T 1C Q U ANT UM M E C H A N IC S

Substituting this expression into (4.50) and multiplying by

I + (*) ^ (y) I (*) ’

we find

(4.52)

where the primes denote the derivatives of the corresponding ^


function with respect to its argument. We note that an equation of
this type is satisfied only if each term (fraction) is independent of
the coordinates and is equal to a certain constant. We thus obtain
the following equations for the functions ij>(;c), <p(y) and^(z):

</' (*) + /ft (*) = 0 , if (y) + f t (y) = 0 ,


f ' («) + f t (z) = 0 , (4.53)

where
(4.54)

Taking a traveling wave propagating in some specific direction as


a solution for each of Eqs. (4.53), we get

^ {x) = Aeikix, if{y) = Beih*y, (z) = C e ik»*. (4.55)

The unknown coefficients A, B and C are determined from the


normalization condition, assuming that the functions <H*), 'My) and
•'f (z) are periodic in the interval L. We thus obtain

i]>(x) = L 2 if (y) = L 2 e,k-y * (4.56)

if(z) = L 2 e‘*a2,

where

(4.57)
and

In this case, the particle has energies of

(4.58)
where
= + + (4.59)
THE T IM E -IN D E PE N D E N T SCH RO D IN G ER WAVE E Q U A TIO N 69

Substituting the functions (4.56) into Eq. (4.51), we have

{i, = L ' Te“'r. (4.60)

These wave functions will satisfy the orthonormality condition

^ '^"'l n 's n ' ^ n i ni n ^ X = 8n2n^8/ian’. (4.6 1 )

The complete, or time-dependent, wave function can now be written


in the form

i/{t) = L ie-(nHEt-p-r)t (4 .6 2 )
where
p = hk, E = £ - o. (4 .6 3 )

We note that the energy spectrum is again continuous, just as


in the case of one-dimensional motion. This can readily be shown
in the same way as before by finding the energy difference between
two adjacent levels and then letting L go to infinity.

E. FUNDAMENTAL PROPERTIES OF 5 FUNCTIONS.


5-FUNCTION NORMALIZATION OF THE WAVE FUNCTION
IN THE CASE OF A CONTINUOUS SPECTRUM
As we have already mentioned, one of the fundamental properties of eigenfunctions Is
their orthonormallty [see Eq. (4.13)]. An arbitrary function F (x) with no ‘‘special’’
properties can thus be expanded In a series of orthonormal functions belonging to a
complete set:
F (x) = ^ (x). (4.64)
n
Multiplying this equation by (x) d x and integrating over the entire range of x, we
obtain

j w f w dx = y cn j i*, (.v) (x) dx.


n
Because of the orthonormallty condition, we have

C n ^ ^ K x ) F (x ) d x . (4.65)

In order to bring <]>„(x) under the Integral sign, we change here the variable of integra­
tion x to x' and substitute the expression for Cn into the expression (4.64):

= 2 J d x 'F ( x ') ^ n (x')i/n (x). (4.66)


70 N O N R ELA TIV ISTIC QUANTUM M E CH A N IC S

This expansion is a generalized Fourier series. We can obtain the ordinary Fourier
series from it by substituting the harmonic functions (4.47) for (*).
We note that the sum over n in Eq. (4.66) is a 8 function:

2 *5 M = 6(*—*'). (4*67)

since it transforms the function ^(a:1) into F (*).


In this respect, this 5 function resembles the Kronecker-Weierstrass 8 symbol, which
possesses a similar property with respect to the subscripts n and it':

^fn'Kn- = fn.
n'

It follows that the 8 function differs from zero only in a very narrow region ~ x , just
as the S symbol differs from zero only for n' = n. By setting F (x1) = 1 in (4.66), we obtain
one of the fundamental properties of the 6 function:

§ 6 (x — x')dx' = l. (4.68)

It is interesting to note that the relations (4.66) and (4.68) are completely independent of
the particular set of orthonormal functions used to construct the 8 function. In the simplest
case, the 8 function can be constructed from the
< 1 3(X-X'(X) orthonormal functions (4.47), which are used In
I expansion into a Fourier series:

t( x - x ')= j2 je L . (4.69)

Introducing a new variable ft = - j - and making


2n 2n
use of the relation Aft = -y An = — (since
w L
An = I), we can transform (4.69) to
X=X' X
Fig. 4.4. Graph of th e “ sm e a r e d * b(x — x') = — EAfte'* (* “ .
(S f u n ctio n .
If we now let L go to infinity (Aft — 0), the last sum is changed into an integral:

6 (x — x ’) = — ^ dkeik (x ~ *’>. (4-70)

The range of integration over ft should be taken from — oo to + oo. We note that the 8
function belongs to the class of functions that are known as "improper functions.” This is
manifested in the impossibility of direct evaluation of the Integral (4.70), since it diverges
at x = x'. Therefore, if we wish to obtain a representation of the 8 function, we have to
"smear out” somewhat the integrand of (4.70). For Instance, the 8 function may be
"smeared out" as follows:

»(*- dke~ “ I* I+ y <*-.*') * _


Ot
I
4 $
e a* cos ft (x — x ’) dk -■
re a' + ^ —jc')*'
(4.71)

A graph of the smeared 8 function is given in Fig. 4.4. At the limit a —. 0, the function
must have the following properties:
THE T IM E -IN D E P E N D E N T SCH R O D IN G ER WAVE E Q U A TIO N 71

t (x — x 1) = oo for x = x',

i ( x — jc') = 0 for x 96 x', (4.72)

As we approach the limit a -* 0, the area between the 8 function and the x axis remains
unchanged:

J " .•+(■,■-*)• =L
The three-dimensional 8 function can be similarly defined as

» ( r ) = » W 6 (jr) 8 (* ) = g^-a j d^ke'*' . (4. 7 4 )

It satisfies the following conditions:


8 (r) = 0 if r^ O ,

§ i(r)d * x = l. (4.75)

The integrals (4.74) and (4.75) are three-dimensional. The former extends over the
entire wave-number space (h), and the latter, over the entire coordinate space (r).
It is very convenient to express the density of a point charge with the help of a three-
dimensional 8function:
P (r) = e6 (r). (4.76)

Substituting this expression into Poisson’s equation

V2<I>= -4 n e 6 (r ) = - - — 2 ^ d 3ke‘*r ,

we can determine the potential of a point charge 1

sin kr
1 I" (4.77)

Thus, the three-dimensional 8 function enables us to describe the singularity at the point
r —>0 of the Laplacian operator applied to 1jr:

V2 -F = - 4*8 (r). (4.78)

In d e t e r m i n i n g th e p o t e n t i a l w e m a d e u s e of t h e r u l e for d i v i d i n g a n e x p o n e n t i a l
fu n c tio n by t h e L a p l a c i a n o p e r a t o r:

fV 2r v ftr = —
- k2 '
T he v alid ity of th is rule i s im m ediately obvious, sin c e

v 2( v 2r v ft' = v 2 — = cikr
-k 2
72 N O N RELA TIV I S T IC Q U ANT UM M E CH A N IC S

We shall not discuss the properties of the 6 function any further and shall proceed to
use it to normalize the continuous spectrum for one-dimensional free motion. From
(4.41), the wave functions for this case are of the form
p _ iS ix
p ) = A e ft , 't'* (P:) = Ae - s • (4.79)

The normalization of these functions is related to the 5 function in the following way:

— oo
= Ai2r.M (p —p') = h (p — p'). (4.80)

From this, we obtain

A = -^= r,

Therefore,

I ' j. *
• (4-81)
Let us rompare the ordinary normalization of wave functions with the Bfunction
normalization. We can write the orthonormality condition forfunctlons normalized by the
ordinary method as follows:

J l, if r. lies inside the range nt < t/j,


y $ (4.82)
|0, if n lies outside the range/tj < n*.
'• = n.
Similarly, replacing the sum in Eq. (4.82) by an integral, we can obtain the following
generalized form for wave functions which are normalized with the help of a 8 function:
I'i
dp' P)(lx 1, if p lies Inside the rangep i < p>, (4.83)
0, if p lies outside the range p t <
Pi

Analyzing the solutions that we have obtained for a particle in a potential well and for
a free particle, we arrive to the following conclusions. If the condition V E is satisfied
at all points of space at infinity, the energy spectrum Is discrete. If, on the other hand,
there are regions at infinity In which V <; /:, the energy spectrum is continuous. While
this conclusion was obtained on the basis of solutions for a rectangular potential well, it
is quite general as long as the potential energy is a continuous function of x, y and z.
C hapter 5

The Time-Dependent Schrodinger


Wave Equation

It has been shown that the solution of the time-independent or


stationary Schrodinger wave equation (4.8) amounts to a determina­
tion of eigenvalues En (the spectrum of energy levels) and eigen­
functions. The time-dependent wave function (i) of a given state
was found by multiplying by e~ u>n>En‘. The function >^n(/) obtained
in this way describes only strictly monochromatic processes (that
is, with only one value of energy). There is, however, a more
general form of the Schrodinger equation, which depends explicitly
on time and may be used in a much larger class of problems. This
equation is known as the time-dependent Schrodinger wave equation.

A. TRANSITION TO THE TIME-DEPENDENT SCHRODINGER


WAVE EQUATION

To obtain the time-dependent Schrodinger wave equation, it is


necessary to eliminate the energy E from the time-independent
equation, where it appears as a constant parameter. The tim e-
independent Schrodinger wave equation (4.8) can be written in the
form

(5.1)

The relation that is used to eliminate E from this equation is

(5.2)

Accordingly, the time-dependent Schrodinger wave equation is

(5.3)

This equation is more general than the previous one. In particular,


it can be used for the description of processes in which the poten­
tial energy V is a function of both position and time.
n o n r e l a t iv is t ic quantum m e c h a n ic s
74

If the potential energy V does not depend on time, it is necessary


to solve only the time- independent Schrodinger equation by finding
all the possible energy eigenvalues £„ and the corresponding
eigenfunctions The wave function which satisfies Eq. (5.3) is
related to these partial solutions by the linear equation

6 ( t ) = , ^ C ne - m)F^ n. (5.4)
n
To prove this, we substitute (5.4) into (5.3), remembering that Cn
are constant coefficients and that >]>, satisfies the equation

^ „ + ^ ( £ » - m » = 0, (5.5)

It is then readily seen that ^ (/) is a general solution of (5.3), since

The case of a monochromatic wave is a special case of the general


solution (5.4). The appropriate wave function can be obtained from
(5.4) by setting
C„0= l andC„ = 0 (if n ^ n„).

As we have presented it, transition from the time-independent


equation (5.1) to the time-dependent equation (5.3) is essentially
equivalent to a simple replacement of the energy £ by the expres­
sion ifi In quantum mechanics, this expression is known as the
energy operator1

E=m (5.7)

The effect of this operator on any function amounts to an ordinary


differentiation of the function with respect to time. Thus, E is a
linear differential operator. In the case of a monochromatic wave,
for which

$( t ) = e * En‘ t™.
w e have

E* (/) = En<|;(/).

( )p**rMors wi l l h e d e n o t e d by r o m a n c h a r a c t e r s .
T HE T I M E - D E P E N D E N T SCHRODINGER WAVE EQUATION 75

Hence, ii is seen that the energy En is an eigenvalue of the energy


operator E.
There are other operators besides the energy operator in
quantum mechanics. The most important of these is the momentum
operator

P= i i dr ’ (5.8)

which takes its name from the fact that in the case of a free particle,
its eigenvalue is identical to classical momentum. Indeed, operating
with p on the wave function of a free particle [see Eq. (4.62)], we
have

P<K0 = - f v { L -,/8e » lB pr)} = p H t).

Thus, in this particular case, the eigenvalue of the operator p is


the classical momentum p .
With this operator notation, which helps to bring out more
clearly the relationship between the quantum-mechanical and cla s­
sical laws of motion, the Schrodinger equation (5.3) has the form

(E- £ - i,b < '> - o . <5-9>

Thus, to carry out the formal transition from the classical theory
to quantum mechanics, it is necessary to replace the energy E and
momentum p in the classical equation for the law of conservation
of energy 2

by the corresponding operators and to operate with them on the


n2
wave function. The operator is known as the kinetic-energy
operator T, andT-|~K as the Hamiltonian operator H. For the sake
of brevity, we shall call the latter the Hamiltonian.

2If t h e e l e c t r o n i s in b o t h an e l e c t r i c f i e l d a n d a m a g n e t i c f i e l d , w h i c h i s c h a r a c t e r i z e d
by t h e v e c t o r p o t e n t i a l A, t h e n , u s i n g t h e c l a s s i c a l e x p r e s s i o n ( 1 . 1 5 ) for t h e H a m i l t o n i a n ,
t h e t i m e - i n d e p e n d e n t S c h r o d i n g e r e q u a t i o n c a n b e w r i t t e n i n t h e form

( P~ c ' 1)
Ip = o. (5 .9 a )
0
76 NONRELATIVISTIC QUANTUM MECHANICS

Using this operator notation, Eq. (5.9) can be rewritten in the


form
(E — H) <|»(0 = 0. (5.10)

For the time-independent potential V, the following relation is


obtained:
e<
mo = e „<MO (5.H)
and, therefore, the time-independent Schrodinger equation reduces
to the form

(E „ -H ) ^ = 0. (5.12)

It is apparent from this that, in stationary problems, the eigen­


values of the Hamiltonian are equal to the eigenvalues of the energy,
just as, in the classical case, a Hamiltonian function which does not
depend explicitly on time is equal to the energy of the system .

B. CHARGE DENSITY AND CURRENT DENSITY.


QUANTUM ENSEMBLES

In classical electrodynamics, an important role is played by


the equation of continuity

-a r + v -y = o , (5.13)
which involves the charge density p and the current density j. This
equation basically represents a general form of the law of con­
servation of charge. To show this, we multiply (5.13) by d'x and
integrate over all space

Jt d' x + $V. Jd*x = 0. (5.14)

Reversing the order of differentiation and integration in the first


integral (this is allowed since time here is only a parameter), and
changing the second from a volume to a surface integral, we obtain

at \ P<Px + J jndS = 0 . (5.15)

If there are no charges or currents at infinity, the surface integral


vanishes, and we obtain the law of conservation of charge

e = \ p d ' x = const. (5.16)


THE TIM E-DEPENDENT SCHRO DI NGER WAVE EQUATION 77

We shall now use the wave theory to find an expression for the
charge and current densities. For this purpose, we take the
Schrodinger equation (5.3), writing it in the form-

T l - - 2 'i- V * M O + ^ « O = 0. (5.17)

Similarly, for the complex conjugate equation, we have

t (o - - I r ( o = o . (5.i8)

Multiplying Eq. (5.17) by ^*(0> and Eq. (5.18) by '}(/), and adding
the two equations, we obtain

+ 2 ^ v { * (0 V r (l) - ** (0 V <*(0} = 0. (5.19)

Comparing (5.19) with the equation of continuity (5.13), and consider­


ing that the charge density is equal to the charge c of one particle
multiplied by the number of particles per unit volume (that is, in
this case the probability density), we have

P= e**(0'M0- (5-20)
From (5.19) and (5.20), we find the current density

/= { * (0 V ** (0 - r ( 0 V * (/)}. (5.21)

It should be noted that for a monochromatic wave

i( /) = e " » h‘^,

both the charge density

P= W (5-22)
and the current density

(5.23)

are independent of time.


In the case of real wave functions (<Ji* = (}i), the current density
is always identically equal to zero. For instance, the current
density of an electron in an infinitely deep, one-dimensional potential
78 NONRELATIVISTIC QUANTUM MECHANICS

well (see Fig. 4.3) is zero (/= 0 ). This is quite natural, since
oscillations described by real wave functions are actually standing
waves, and standing waves cannot give rise to a particle flux.
The case of motion of a free particle is somewhat different.
According to (4.60), the wave function for.this case describes a
traveling wave:
ty= L-*/*eiPr"1. (5.24)

Substituting this expression for <[i into (5.22) and (5.23), we obtain
the following expressions for the charge density and the current
density, respectively:

p— = L~3e,

j = 1 ^ j P = *v -
Hence, when the charge is distributed with uniform probability
over the entire volume, its density is equal to the charge divided
by the volume, as was to be expected. Moreover, the relationship
between the current density and the charge density remains the
same as in classical electrodynamics.
Let us now explain the physical meaning of the coefficients Cn
which appear in the solution ^(/) of the time-dependent wave equation
[see (5.4)]. For this purpose, we substitute the expression for <p(0
into Eq. (5.16), which serves to define the conservation of total
charge. Then, using (5.20), we obtain

Since the wave functions must satisfy the orthonormality condition

^ ^ nd*x = 8 „„.,

we have

1n C"C" n (5.25)

We can now give a physical interpretation to the coefficients C„ as


quantities characterizing the probability that a particle exists in
the quantum state n. Indeed, if we know with certainty that the
particle is in the state /i„ (£ = £„„), all of the coefficients C„, except
C„0, will have to be equal to zero; that is, we must set C„ = 8 „„0. If
the particle has a nonzero probability of occurring in two or more
states, then, accordingly, two or more coefficients will differ from
THE TIM E-DEPENDENT SCHRODINGER WAVE EQUATION 79

zero. The actual probability that a particular state is occupied is


given by|C „|2, while the probability density of the distribution of
the state over entire space is given by 1 |a.
When there is a large number N of particles in a region of
space, we have, instead of (5.25),

Here the coefficients Cn characterize the distribution of the total


number of particles among the different quantum states.
In this connection, we shall introduce the concept of quantum
ensembles: namely, a collection of identical quantum states that
are described by the same wave function A quantum ensemble
can be used to describe an electron beam, photon flux, and so on . 3
Two different cases are possible.
1) All of the particles are in the same quantum state jC„ 0 | 2 = N
and | Cn|4 = 0 for n ^ n 0.
2) The particles have a definite distribution among the various
quantum states:

|Cn i | 2 = yVni, \Cn, \ ^ N , , lt ICni\' = N„,...... (5.26)


where

2 i

In the second case, the motions of particles, say, in the states« = /i|
and n = n.it cannot be considered independently of one another,
because whenever two states i|»Bl and are both possible, the total
wave function is a superposition of the individual states, namely,

^(0 = CnAm (0 + c nt%a(0 + • •• •


This is important in determining the total probability of a state,
which is proportional to the product <|i*(f) <l (/). In addition to terms
such as 'I'n, (/) (/), this product will contain mixed terms of the
form <!<«, ( / ) < | I n other words, each particle will possess a
definite probability of being in both quantum states. The mixed
terms differ from zero for coherent u waves (a pure ensemble),
and this leads to interference of the de Broglie waves, which does
not happen with incoherent waves (a mixed ensemble). Thus, in a
pure ensemble, waves are added, whereas in a mixed ensemble,
intensities are added. Quantum ensembles are useful with regard

3S ee D. I. B l o k h i n t s e v , P rin c ip le s of Quantum M echanics ( t r a n s . ) , A lly n a n d B a c o n ,


1964.
NONREL-ATIViSTIC QUANTUM MECHANICS
80

to the statistical interpretation of the results of wave theory. For


a large number of particles or a large number of states occupied
by single particles, it follows from the law of large numbers that
the probability of some particular process, when calculated on the
quantum-mechanical basis, should agree with the distribution that
can be observed in experiment (see Chapter 7). Similarly, in
classical statistical physics, the probability distribution agrees with
experiment independently of the nature of the “ hidden” parameters.

C. CONNECTION BETWEEN THE SCHRODINGER THEORY


AND THE CLASSICAL HAMILTON-JACOBI EQUATION

The Hamilton-Jacobi equation, which is used in classical me­


chanics to describe the motion of a material particle in a field of
force, is a first-order nonlinear differential equation. We shall
briefly recapitulate its derivation, starting from the classical law
of conservation of energy

E P% V= T + V .
2 m0 (5.27)

Let us introduce the action function


t t
S ( t ) = ^ £ d t = (2T — E)dt — S — Et, (5.28)
0 u

where

S = <
\ 2Tdt. (5.29)
6

As we shall show, the dependence of S and t in (5.29) is only im­


plicit, namely, S = S(x(t), z(t)), and therefore we have written
it without including t as an argument. Thus, we can distinguish
between the stationary action function S and the time-dependent
action function S (/) in the same way as we distinguish between the
stationary wave function and the time-dependent wave function.
To show that S is not an explicit function of t in (5.29), we take
the total differential of S:

dS = 2Tdt.
From the equations

dS OS OS
d S ~ *<“ + % dx- 'r ^ d,J dz, (5.30)
27V// = //i„ (.U - j- ,jl -l- i '2) dt = pxdx - ■pydij + /y/z,
THE TIM E-DEPENDENT SCHRODINGER WAVE EQUATION 81

we find that

and thus 5 does not depend explicitly on L Moreover,

p=V S. (5.31)

From Eqs. (5.31) and (5.27), we obtain the stationary Hamilton-


Jacobi equation
- ± ( V S ) ' + V - E = 0. (5-32)

We can introduce the time-dependent action function

S (t) = S — Et

into this equation by making use of the relation p = V 5 = V S(t),


and eliminate the parameter E with the help of the relation
E—— We then obtain the time-dependent Hamilton-Jacobi
equation, which can be used in nonstationary problems:

— [v S (0 ] 2 + ^ + ^ = 0. (5.33)

The stationary and time-dependent Hamilton-Jacobi equations cor­


respond to the stationary and time-dependent Schrodinger equations,
respectively.
It can be readily shown that in the case of motion of a free
particle (F = 0 , p = const, /:=const), the action function is

S(t) = — E t + p r . (5.34)

To see this, it is sufficient to substitute the expression (5.34) for


S(0into (5.33). Let us take special note of the fact that the function
<|i(f) for the motion of a free particle is [see (4.60)]

<?(<<) = Aen' a+pr)= A e * Sm. (5.35)

The relationship between the wave function and the action func­
tion will hold in general form whenever we make the transition
from the Schrodinger equation to the Hamilton-Jacobi equation. We
shall consider further only the stationary case, for whichl

l. $
<b= Aeh * (5.36)
82
NON RELATIVISTIC QUANTUM MECHANICS

We start with the Schrodinger equation in operator form

( i + v' - E)<’= 0' <5-37>

Making use of the momentum operator p = — ih\j, we obtain from


(5.36)
p* = (VS)*, (5.38)

= (5’39)

We note from Eq. (5.38) that Eq. (5.36) leads to the same relation
between the momentum operator p and the action function 5 as in
the classical theory if we replace the momentum operator by the
classical momentum p. Substituting (5.39) into (5.37), we obtain

i ( VS)' + | / - £ - ^ = 0, (5.40)

which is simply the transformed Schrodinger equation.


To obtain the Hamilton-Jacobi equation, we must neglect the
last term in (5.40), that is, set h = 0. It is a well known fact that
the quantum-mechanical equations transform exactly into the
classical equations when H— 0. If, on the other hand, h 0, but the
condition

(VS)*>A|v*S|

is satisfied, the quantum-mechanical terms provide only sm all


corrections to the classical equations. The approximation corre­
sponding to this case is knownas the quasi-classical approximation.
Since p = v7 S, the above condition can be rewritten as

jr\V -p \< 1.

In particular, for the one-dimensional case, we have

I dp_ I \d(h<p) I I rfX (5.41)


P* I dx | | dx | |2ndx

Thus, the quasi-classical approximation turns out to be sufficiently


accurate in cases where the de Broglie wavelength is constant or
changes very little over distances of the order of the wavelength.
Since

p = V2mn(E— V),
THE T IM E -D E P E N D E N T SC H R O D IN G ER WAVE E Q U A TIO N 83

we can rewrite the condition (5.41) in the form

ft I dp I__| m0Fft (5.42)


P3 | d x I | p a

where F = — ^ i s the classical force acting on the particle. From


this, it follows that the quasi-classical approximation becomes
inapplicable at sm all values of the momentum and, in particular, at
the points where a particle would come to a stop in the classical
theory (E = V, p = 0). Such a state of affairs is obtained, for
example, in a potential well at the points where the direction of a
particle is reversed as a result of reflection from the potential
barrier (the turning points). A simple explanation can be given for
this conclusion: namely, the de Broglie wavelength tends to infinity
as p — 0 and, when this happens, the wave-like properties of a
particle become too important for the particle to be treated quasi-
classically.

D. THE WENTZEL-KRAMERS-BRILLOUIN APPROXIMATION


METHOD (WKB METHOD)

As mentioned above, Eq. (5.40) is completely equivalent to the


Schrodinger equation. Therefore, it would be possible to take
Eq. (5.40) as the basis of the wave theory by treating the term
which is proportional toft and which does not appear in the classical
equation as a new quantum-mechanical potential energy

yquan = — J lL V*S, ( 5 . 43 )

which has to be added to the Hamilton-Jacobi equation. The general


solution of the nonlinear Schrodinger equation (5.40) is, however,
much more complicated than the solution of the linear Schrodinger
equation, and therefore the many attempts to develop the quantum
theory by means of further analysis of Eq. (5.40) met with failure.
Fortunately, Wentzel, Kramers and Brillouin succeeded in finding
an approximate solution of the Schrodinger equation (5.40) by taking
only the terms of the order of ft. This solution was found to be
applicable to a number of problems in quantum mechanics. It is
referred to as the WKB approximation.
For the sake of sim plicity, we shall consider only the one­
dimensional case, assuming that the potential energy is a relatively
smooth function of x (see Fig. 5.1). For particles with energy E,
the range of variation of x can be divided into two regions. In the
first region (x<^x0), the energy E is greater than the potential
energy ( £ > V), and in the second region (x > x 0), E<^V. It is obvious
84 NONRELA TIV ISTIC Q U ANT UM M E CH A N IC S

that E — V(xlt) at the boundary (x = .v0) of the two regions. For the
one-dimensional case, the original equation (5.40) becomes

S’i -itiS" = 2m0(E— V) = p \ (5.44)

First, we shall find the solution of this equation for region I


(E > V'), where the quantity /;*> 0 can be interpreted as the square
of the classical momentum. The solution will besought in the form
of a series
5 — 5 q-j - - ) - S<i-|- . . . , ( 5. 4 5 )

where the quantity S„ is independent of H, S, is proportional to h, S.2


is proportional to //’, etc. Substituting the series (5.45) into (5.44)
and neglecting quantities proportional t o # 2 and to the higher powers
of li, we obtain4
S,',3 2 S„Sj — iflS^' = p 1. (5.46)

Taking both sides of the equation, we equate terms that are inde­
pendent of h and, sim ilarly, terms that are directly proportional
to h (here it is necessary to bear in mind that the quantity S, is
proportional to tl). We thus obtain
S i= p \ 2S0S'l = ihS'0\ (5.47)
Hence, it follows that

S0 = 7: ^ pdx, Sl = ih\nYp. (5.48)

J h i s a p p r o x i m a t e m eth o d of s o l u t i o n i s b a s i c a l l y th e s a m e a s th e p e r t u r b a t i o n m e th o d ,
w hich is a l s o s u c c e s s f u l l y u s e d in t h e s o l u t i o n of t h e S c h r o d i n g e r e q u a t i o n ( s e e C h a p ­
ter 1 1 below ;.
THE T IM E -D E P E N D E N T SCH R O D IN G ER WAVE E Q U A TIO N 85

Therefore, retaining only terms of the order of h, we have

s = S0 + S, = ± ^ pdx + ih In Yp. (5.49)


X

Substituting (5.49) into (5.36), we obtain the following expression


for the wave function in the first region (x<^x(l):
A'p *0

W , = J = ( ^ cos J ^ pdx-\-B sin ~ ^ pdx). (5.50)


' P X x

In exactly the same way, for the second region (x^>x0), in which
p1<^0, we obtain
.V X

^■>.v0 = y p (D e *<> -J- Ce *o ), (5.51)

where
\p\ = V2ml>(V - E ).

The wave functions (5.50) and (5.51) are the desired approximate
solutions. From these equations it is seen that, when £ > V, the
wave function is cosinusoidal or sinusoidal, as in the case of a
potential well [see Eq. (4.21)] or a free particle [see Eq. (4.47)],
whereas, w h e n F ^ E , it changes exponentially, as in the case of
transmission through a potential barrier [see Eq. (4.23)].
Comparing the solutions for V = const with the solutions for the
case where the potential energy is a function of .v, we see that the
transition from one case to the other is simply equivalent to re­
placing the area of the rectangular barrier contained between the x
axis and an axis indicating the constant quantity * = = LeJ
by the area enclosed between the x axis and the curve for V = V (x).
Schematically this transition can be represented in the following
manner:

\rlx ~ .ii\p \d x . (5.52)


o
A sim ilar transition is made in the case of a potential well.
Thus the specific form of the dependence of the potential energy
on x does not alter the character of the solution. Indeed the solution
is determined only by the sign of the difference between E and V,
as we indicated at the end of the preceding chapter.
The solutions (5.50) and (5.51) give a good approximation only
for regions that are relatively far from the special point (the
66 N O N R E L A T I V I S T 1C Q U ANT UM M E C H A N IC S

classical turning point) where the quantity |p| is relatively large.


Near the special point (x->-*„) the quantity p'--> 0; therefore, the
denominator in Eqs. (5.50) and (5.51) vanishes and the actual
solution diverges. If we could express the coefficients C and £>in
terms of A and B, the foregoing approximation would be entirely
adequate for many problems, since the region \x — *„ | — 0 is com­
paratively narrow. However, the relation between these coefficients
can be found only if we can connect the functions across the boundary
(.v= .?0) of the regions (by connection of the solutions we mean
matching the wave functions and also their first derivatives at the
boundary x-^-x0). The approximate expression for must, there­
fore, be represented in such a form that at large p1 Eq. (5.50) holds,
while for x = x0, when

P' = — (* — *0) 2m0V' (*0) —— ah2(x — *0),

the approximate solution satisfies the equation


ip' —a (* — *,) «p= 6 . (5.53)

At large z, the cosine function can be expressed asymptotically


in terms of an nth order B essel function:

cos{z - n
i (5-54)

Therefore, if we set
x„

z ~ ^ \ ' pdx, (5.55)


X

then
I A* "I/~^Z r ,
Vjf<jr0 — -y=^ y (5.56)

fo r large z (and any n), because of the asymptotic formula (5 . 5 4 ),


the solution (5.56) transforms into the solution (5.50) found by the
WKB method. Let us attempt to choose the order n of the B essel
function in such a way that the solution (5.56) satisfies the Schro-
dinger equation not only for large z, when x<*x0, but also near the
turning point * — that is, when

p ^ h yV(x„ —x) — 0 and z = ^4^ - ( * 0 — xf'*- 0 . (5 . 5 7 )

In this case (x-'X 0 — 0), the asymptotic expression for 6 becomes

r. (a 0 — x) j 2 Y a
V jc= .a-0 - o ---- A ' j / ^
' 'Ah ■ 3 (5.58)
THE T IM E -D E PE N D E N T SCH RO D IN G ER WAVE E Q U A T IO N 87

Substituting (5.58) into (5.53), we find that Jn must satisfy the


equation

dx* X q— x d x + (“ (*« x) 4(x0 — x y ) J n ~ ° -

Introducing the new variable z = ^O° (x0 — x)3/a (the argument of the
2

B essel function) into this equation, and denoting by primes the


derivatives with respect to this argument,
= (5.59)

If the B essel function is to satisfy Eq. (5.59) and if, at the same
time, the wave function is to obey the asymptotic equation (5.53),
we must put n = ± 4 u- . Thus, instead of the approximate solution
(5.50), we have

<Kv<.v„ == 1/ ~ j | P d x { A J x / ,{J ji j pdx} + B J - i /3( j X


^pdx)}. (5.60)

Similarly, in place of (5.51), we obtain for the second region (E <^V)


.1

|/> | c r . r { c / i /3 ( — J | / ? | d x ) +
X 0

( 7 ^ ^ IP Idx)}> (5.61)
Xq

where / i/a and /_ i/s are the B essel functions of an imaginary argu­
ment.
In order to connect the two solutions, we must find the asymptotic
forms of Eqs. (5.60) and (5.61) for the region x — x„. The appro­
priate values of z andp are determined from (5.57). For the B essel
function, it is enough to take only the first term of its expansion:

K (Z) = K (z) = T (^+l) ( 7 )"- (5-62)

The solutions (5.60) and (5.61) become, respectively,

A V « (3a)1/a __ B V *
ft1 /2 r(i/3) (* 0 — *) + r i ‘/ s ( 3 a ) 1/» r ( 2 / 3 ) ’
c /IT ( S a ) 1^ D VK (5.63)
tjf=.«-o+o— fi1 /2 r (i/3) (X— x„) -+ f t l / 2 ( 3 a ) l / “ 1’ ( 2 / 3 ) '
NON R E L A T I V IS T IC QUANTUM M E CH A N IC S
88

Connecting the solutions at the point x = xa, we find

D = B, C = — A.

Considering the asymptotic forms of the ordinary B essel function


[see (5.54)] and of the B essel function of imaginary argument5

/ n (2) ^ V 2r~ z [e* + C° S " ( " + J )) ’ (5 •64)

5T h e B e s s e l fu n c tio n of im a g i n a r y ar g u m e n t i s r e l a t e d to t h e B e s s e l f u n c t i o n o f the
firs t k in d by t h e e q u a t i o n
n7T j

/ „ ( * ) = r nJ n {iz) = e *2 J n ( x),
w here
77
/ —
2
x = ze

T h e a s y m p t o t i c form of t h e B e s s e l f u n c t i o n w i t h x = I x l e 1^ c a n b e c a l c u l a t e d e i t h e r for
, 77
d) • 4 0
2

■Jn (x ) = ^ 1 77 (n + V2 ) - / — c o s ( x + — 7T(n +- V4)J for — < 0 < — 77, (5.64a)


J 7 T X \ 2 / 2 2

. 77
or for o -----0
2

T~ / l \ 77 , „ 77

V — c o s t x — — 77(n h Vi) ) for


77 x \ 2 /
—— < 0 < —
2
( s e e , for e x a m p l e , P . Mor se, a n d H. F e s h b a c h , Methods o f Th e ore t i c al P h y s i c s , N e w York:
2
( 5 .6 4 b )

M cGraw-Hill, 1953, Vol. I, p. 622). T h e r e f o r e , t h e a s y m p t o t i c form o f In for r e a l z h a s a


d i s c o n t i n u i t y ( S t o k e s p h e n o m e n o n ) . I n d e e d , u s i n g ( 5 . 6 4 a ) a n d ( 5 .6 4 b ), w e f in d for t h e
two c a s e s

1
'O) ^ for 0 = (5.64c)
\/2nz

1 (' , . — ( - 4 )
- \ / for 0 - (5 .6 4 d )
J 2 ttz

U n f o r t u n a t e l y , many a u t h o r s , i n c l u d i n g M o rse a n d F e s h b a c h ( s e e Methods o f Th e ore t i c al


P h y s i c s , Vol. II, p, 1097), u s e ( 5 . 6 4 c ) in a n a l y z i n g t h e p a s s a g e o f p a r t i c l e s th r o u g h a p o ­
t e n t i a l h a r r i e r an d t h u s o b t a i n a co m p le x a s y m p t o t i c e x p r e s s i o n for t h e r e a l f u n c t i o n / n (z).
77
r h r c o r re c t p r o c e d u r e at <,> ( w h e re th e f u n c t i o n h a s a d i s c o n t i n u i t y ) i s to t a k e t h e h a l f -
2
.um of (S b 4 c) a n d (5 .6 4 d ) in o r d e r to o b t a i n t h e a s y m p t o t i c e x p r e s s i o n (5 .6 4 ), w h i c h , a s
t c d, i s r e a l .
THE T IM E -D E PE N D E N T SCH RO D IN G ER WAVE E Q U A T IO N 89

and also taking account of the relation between the coefficients, we


find that for large z, the formulas (5.60) and (5.61) take the form

^ = \h {A
' ^
cos (* Spdx -
X
)+
+ 5 co s(i J p d * --)} , (5.65)

- J \p\dx

- -7 ^ Ip Idx
-f- (B -)- A) cos 7- e •*« }.

Setting B = A = — ^= in the last equality, we find the first pair of


V3
connected solutions

^■*<*0 — sin j pdx + j j , (5.66)

— - 7 $ \p\<ix

• (5-67)

for which the exponentially decreasing solution (5.67) in the region


is the analytic extension of the sinusoidal solution (5.66) for
the region x<^x„.
To determine the analytic extension of the exponentially increas­
ing solution (x>x„), we must set B = —A = b . We then obtain the
second pair of connected solutions

^< x0= -—--= cos J p d x+ 7-), (5.68)


' ^ X

I J I p I dx

^ • = ? T p T e '• • (5-69)

According to Eqs. (5.66) and (5.68), the expression for tp in


region I (x<^x0) is of the form of standing waves. It can also
be written in the form of traveling waves. Indeed, setting

b = (g + h), a = i(g — h).


90 NON R E L A T I V I S T I C Q U ANT UM M E C H A N IC S

we have

*0
\ p rfx + -) (5.70)
V* < X o --------'
[ge * + he ■
Vp

The appearance of the factor VP in the denominator of (5.70) means


that the probability of finding a particle in a unit volume (that is, a
quantity proportional to |^|2) is sm aller, the greater the velocity.
Inside the barrier the exponental solution that is connected to (5.70)
takes the form

'n\
( g + / 0 e Xo +
V\P\

(5.71)

In casds where the potential barrier is located to the left of the


turning point, we must interchange the lim its of integration, so that
the lower limit will always be le ss than the upper limit.
On the basis of these results, we are able to quantize the problem of the potential well
(that is, find the energy levels of the particles) in the WKB approximation. Let us assume
that we have u potential well of arbitrary shape, as illustrated in Fig. 5.2. Obviously,

l‘ig. 5.2. Q u a n t i z a t i o n in a p o t e n t i a l w e l l a c c o r d i n g
to th e WKB m ethod.

the process of quantization in the WKB method will consist in finding the conditions under
which the exponentially increasing solutions on both sides of the potential barrier (x <C Xi
and .v > .v.) vanish.
According to (5.66), the wave function for this case has the following form in that
region of the potential well which is adjacent to the boundary of the barrier at x = x a
(there being only an exponentially decreasing solution inside the barrier);

a (5.72)
V p M " - V+ ■
THE T IM E -D E P E N D E N T SCH RO D IN G ER WAVE E Q U A T IO N 91

Because of the requirement of an exponentially decreasing solution Inside the potential


barrier in the region a- ^ a ,, the following solution is obtained for the potential well In the
region .v a t
x

a, = s’n a $ pi* + 1 )■ <5-73>


xi
Two solutions must be identical at any arbitrary point a: in the potential well (v, < x < x£,
as long as we do not take points too close to the boundary of the potential barrier. Joining
the solutions (5.72) and (5.73) at some point a:(that is, matching the two wave functions
and also their derivatives), we have

Xl x
a' sin ( 1 J p d x + - ) - « sin ( 1 $ / > ^ + - ) = 0,
* *1
* 2 X

a' c o s ( l J /» < * * + j ) + « cos ( 1 j p d x + - j ) = 0.


* Xl
If this system of homogeneous equations is to have a nontrivial solution for a and a1, the
determinant of the system must vanish. We then obtain the relation

sin ( I J p d x + ^ = 0.
*1

The integral ^ p d x cannot be negative, since p = y 2m0(E — V) ^ 0. Hence


■*1 xa
i J pd.r + - = ( n + 1)*, n = 0, 1, 2......... (5.74)
Xl
Thus the quantization rules obtained from the WKB approximation (that is, with an
accuracy up to the terms of the order of ft) have the form

j > p d x = 2*n ( « + - j. (5.75)

These quantization rules differ from the Bohr quantization rules by the presence of a
nonzero term - j 2 -ft, which corresponds to the lowest state (n = 0). A more exact solu­
tion of a similar problem in wave theory (for example, the harmonic oscillator problem)
shows that a zero-point energy necessarily exists, although it does not affect the radiation
spectrum.
Let us now find the normalization coefficient of the wave function for the case of a
potential well. In normalizing the function, we can restrict ourselves to integration over
the Interval X i ^ . x ^ x t (potential well), since '{'decreases exponentially everywhere
outside this region. The normalization coefficient can be then found from the equation

+ f |= ' - <5-76)
Xl Xl
Since the sinusoidal function oscillates rapidly, its square can be replaced by its average
value, 1/2, without significantly affecting the accuracy. We then have
92 NON R E L A T I V I S T I C QUANTUM M E CH A N IC S

O-
The oscillation period --(o> is the angular frequency) is
0)
t-., x«

*1 *1

where v = — is the velocity of the particle. Hence, for the normalization coefficient, we
rno
obtain the expression

Consequently, the eigenfunction (5.73) in the WKB approximation assumes the form

•*i
Problem 5. Lb Determine the eigenfunctions and eigenvalues of the energy of a particle
in a three-dimensional potential well bounded by an infinitely high potential barrier:
(0 for 0 < a: < h, 0 < y < /2l 0 < a<
\ oo outside the potential well.
Find the conditions under which different wave functions correspond to the same energy
value E, that is, the conditions under which the energy levels are degenerate.
Answer.
!n\ ni .

. nnix . nn2y . nn3z


th VV —hhh
sin—-— sin —— sin —j —
U la *a
(nit ns, n, = 1, 2, 3,...).

In particular, when /a = / 1, we have degeneracy for the case of the two different wave
functions and
E = En^nin3*
Problem 5.2. Find two classes of solutions (symmetric and antisymmetric) for a
particle in a one-dimensional symmetric potential well:
( l/ 0 f°r x < —/ (region 1)
F (.v )= 0 for — / < * < / (region II) (5.79)
I K0 for x > / (region III)
Show that, when V0 can have any arbitrary value, only the symmetric solution must
always have at least one energy level. Find the condition under which an antisymmetric
solution is possible. Show that the antisymmetric solutions corresponding to ,r;> 0 also
represent a complete solution of the system in the case where V (x) is described by
the function

co for X < 0,

I 0 for 0 < x < /,


Vo for x I.
(5.80)

111'-' ( p r o b l e m s a r c b a s e d >n t h e m a t e r i a l c o n t a i n e d in b o t h C h a p t e r s A a n d 5.
THE T IM E -D E P E N D E N T SC H R O D lN G ER WAVE E Q U A TIO N 93

Plot a graph of the potential energy and of the symmetric and antisymmetric wave
functions.
Solution. The wave functions in regions 1, II and III have the form

<K= Dte^x+l> ( * < -/),


<j>n = A2 sin k x + B a cos k x (— / < x < I),
(x>l),

where

k = 1yf 2m0E *= i y/ ~ 2 m Q —e).


The exponentially increasing solutions have been discarded because they diverge (indeed,
this is the factor that is ultimately responsible for the discrete values of the energy).

From the boundary conditions at x = -»-/, it follows that m (x ) = 'h iM “ d


^ I. HlW _ i From this, we find that it is possible to have either a symmetric
dx dx
solution, for which <}<(■*) = <)<(— x), or an antisymmetric solution, for which ^ (x) =
= — + (— *)•
For a symmetric solution Aa = 0, C, = Di , and the energy levels are found from
the equation ______
tM « = y r £ = £ , (5.8i)
where
1 f 2m0E 0 -j /~2ml>V(,
a=/ V p=/ V ~ w -

Since y p a— as/a >. 0, the minimum value of the angle a must lie in the first quandrant
and can be found from the condition
1 a
cos a = — -
V 1-)- tan2a

This equation will always have one root for any value of p in the region 0 < a < as is
easily shown graphically from the fact that P = const >- 0.
For the antisymmetric solution (x) = — ^ (— .*)], we h avefia = 0 and Ca = — Di.
The energy levels are determined from the equation
a
tan a = (5.82)
/ Pa—aa
Since the right-hand side of (5.82) has a negative sign, the minimum value of the angle a
must now lie In the second quadrant and can be found from the condition
tan a a
sin a =
y 1 -|-tana a P
Introducing the notation

a + y,
2

where the angle 7 lies In the first quandrant, we find


94 N O N R ELA TIV ISTIC Q U ANT UM M E C H A N IC S

It is evident that this equation has at least one root only when

t.e., for Vo
?>T 8 mot2'
Since the wave function of the antisymmetric solution vanishes at .v=0, the antisymmetric
solution for .v > 0 is also the solution for a particle in a potential field described by
<5. SO).

Problem 5.3. Find the transmission coefficient D of a particle through a potential


barrier of rectangular shape (see Fig. 5.3), if the particle energy E is less than the height
of the potential barrier K0.

F ig . 5.3. T r a n s m i s s i o n o f a p a r t i c l e th r o u g h a p o t e n t i a l
b a r ri e r .

Solution. The solution of the Schrodinger equation for the various regions has the form;
ty[ = Aiei k x Bie~ikx for x < 0 (region I)
in = Aze~*x + floe** for 0 < x < a (region 11)
= A i e ik'x - a' + D, e - ikix~a> for A r>a (region III)

Here As = ^,”“—, z- = (F0— E), A le‘kx and B\e~ikx characterize the incident and
reflected waves, respectively; A3e, klx~a> characterizes the transmitted wave; and
Ih? ,k v , characterizes the reflected wave coming from infinity. Since we have no
reflected wave from infinity in our case, we must set Z1„ = 0,
To determine the transmission coefficient we shall use the boundary conditions at
a = a and x = 0. We first express An and Bo in terms of Aa, making use of the fact that
' a ^ I:

A o = l 2 ‘n- A ae*a,

Do = — ■A,e~ 0,

and then e x p re s s Xl, in te rm s of /13

/l. e*aAa

Hie transmission (diffusion) coefficient D is then found to be

, i A,!: ^ 1 0 /t2 16n‘ \


“ M. r “- (1 + n - f e~ x0 = exp ^— 2xa -j- In
(1+«2)2J’
THE TIM E-DEPENDENT SCHRODINGER WAVE EQUATION 95

where
k
n= —
x E'

Neglecting the second term in the exponential for D (this is possible because the quantity
16n*
Is only slightly different from unity), we finally obtain

- 2± V 2m 0(Va - E )
D = <?

When h = 0, we get the classical result D = 0.

Problem 5.4. A particle is in a potential field of the form

I
oo, xcO,
0, 0 < xC I (region I)

V0, l c x < : l t (region II) (5.83)


0, h <
a) Show that the spectrum is continuous. : x (region III)
b) Show that for xl >> 1 and x (I, — /) = xaj> 1 (0 < E <Ko) there must exist quasi­
levels, that is, states such that there is only a transmitted wave in region III (see
Problem 5.3).
c) Construct a graph of the wave function corresponding to a quasi-level.
d) Find the squared modulus of the wave function inside the potential well and
explain why it decays exponentially with time.
Solution. In the various regions, the wave function has the following form (see the
notation of Problem 5.3):
'}[ = Ai sin kx,
<\,n = C>e-x 'x ~1' + D iexlx~l\
| 1U= Dte~ '*(■*—

From the boundary conditions, we find

O, } = {■<.(»"‘' * 7 - «»«).
These relations hold for any value of E, and therefore the spectrum is continuous.
We obtain quasi-levels by requiring that there should be only a transmitted wave in
the external region (region III), that is to say, only a wave moving in the positive a- direc­
tion. Setting D b= 0, we obtain the following equation for the energy of the quasi-levels:

_ 2-
tan a -f e X
+ / a (5.84)
X - - — — tana------_
Y P8 — a 2 — (a \ / P2 —a2

It should be noted that the amplitude Ct in this case is much smaller than the amplitude
A i:

ICa I~ A I ’
96 N ON R E L A T IVI S T I C Q U ANT UM M E C H A N IC S

For a = oo, Eq. (5.84) becomes exactly the same as Eq. (5.82) which gives the energy
levels of the potential well (5.80).
We let E0 denote the value of the energy of a particle in the potential well which we
obtain from (5.82) and, for present purposes, we disregard all real corrections to E q , as
they make no essential difference to the problem. We then obtain the following expression
for the energy:
(5.85)

where

- ^ V ^ - ^ q-E q) mgV-
X E„
2 '
and where
I6 n2 E
(1 4-n2)3’ - / Vo—£ ’
under the condition*/^ 1.
The presence of an Imaginary part In the expression (5.85) for the energy Indicates
that the wave function decays exponentially with time. The transmission coefficient
through the potential barrier for this case can also be found from (5.85). Indeed, the
squared modulus of the wave function inside the well is

| ij/ Is = const-e~u ,

and thus X, which Is called the decay constant, characterizes the decrease of the prob­
ability of finding the particle Inside the potential well. The quantity Xis related to D, the
transmission coefficient for a single collision of the particle with the potential barrier,
by the equation

V
where — is the number of collisions with the barrier per unit time. Hence we obtain an
expression for D:

The same expression for D was obtained in another way In Problem 5.3. In Chapter 6 we
shall obtain It for the general case In which the potential energy is an arbitrary function
of the coordinates.
The constant X also determines the duration of radiation t .= (see also Problem
rad x '
3.4). Therefore, the quantity A£ fix should characterize the width of an energy level.
Chapter 6

Basic Principles of the Quantum Theory


of Conductivity

A. TRANSMISSION OF A PARTICLE THROUGH A


POTENTIAL BARRIER (TUNNEL EFFECT)

According to the classical theory a particle can be located only


at those points in space where the potential energy V is less than
its total energy £. This follows because the kinetic energy of a
particle

must always remain positive. In a region where V E (a potential


barrier), the momentum has an imaginary value and, classically,
the particle cannot exist in such a region. Therefore, if two regions
of space in which E^>V are separated from one another by a
potential barrier inside which V > £, the classical theory does
not allow a particle to penetrate from one region into the other.
In the wave theory, however, an imaginary value of the momen­
tum (see the WKB approximation method. Chapter 5) simply
corresponds to an exponential dependence of the wave function on
the coordinates. Since the wave function does not vanish inside the
potential barrier, it is quite possible for a particle to leak through
it. This phenomenon is observed in the case of microparticles.
The penetration through a potential barrier is called the tunnel
effect. It is a specifically quantum-mechanical effect and has no
analog in classical mechanics.
With the WKB method, it is a relatively simple matter to deter­
mine the probability of penetration of a particle through a potential
barrier of an arbitrary but sufficiently smooth shape (see Fig. 6.1).
Let us assume that a particle is moving in the direction of
positive x in the region 1 (—co<^<^x,), where E^>V (x). It en­
counters a potential barrier (x, x x.,), where E<^V(x), at the
point x — xu and then falls into the region III (x>x.2), where again
E~f> V (x). The beginning and end points of the potential barrier can
be found from the condition
V(x) = E. ( 6 . 2)
98 N O N R EU A T IV IST 1C Q U ANT UM M E C H A N IC S

The de Broglie waves corresponding to the motion of this particle


will be partially reflected from the potential barrier and partially
transmitted through it. The transmitted waves will then propagate
in the region III (x >*.,). To determine the probability of penetra­
tion of a particle through the potential barrier, let us begin by

F ip . 6 .1 . S c h e m a t i c d i a g r a m o f a p o t e n t i a l b a r r i e r
of an a r b i t ra r y b u t s u f f i c i e n t l y s m o o th s h a p e .
The incident and tra n sm itted w a v e s are rep re­
s e n t e d by t h e s o l i d c u r v e , a n d t h e r e f l e c t e d w a v e
by t h e d a s h e d cu r v e .

analyzing the wave in region III, where the solution has the sim plest
form, since in this region there can be no wave moving in the
direction of negative x. According to Eq. (5.70), the solution in
region III has the form

( t I p^ + t )
'I'm= -f- /line (6.3)

where

p = V 2 n k ( C - V (x)). (6.4)

tVe define the transmission coefficient as the absolute value of the


ratio of the flux density of the particles transmitted through the
barrier to the flux density of the incident particles:

D= hr |
(6.4a)
■Zinc I

1° determine the particle flux, we shall make use of Eq. (5.23).


Setting the constants e and mu equal to unity (which we are allowed
to do because we are interested only in the ratio of the particle
fluxes), we obtain
THE QUANTUM THEORY OF CO N D U C TIV ITY 99

ih ( dtj/ (6.5)
/ 2 dx

Substituting (6.3) into (6.5), we find

/ill = !Sill I" f° r *1 11 = °* (6-6)

/ i n = — l/2 ml2 for ffm= 0 - (6.7)


It is seen from the above that the amplitude gm characterizes the
wave propagating in the direction of positive x, and the amplitude
hm the wave propagating in the direction of negative x. Since, as
has been already mentioned, there is no wave propagating in the
negative x direction in region III, we must set/zm = 0. Then

i t r — l ^ n i l 2* (6-8)

and the corresponding joined solution inside the potential barrier


(region II) can be found from Eq. (5.71). Setting h = 0 and g = gui in
Eq. (5.71), we find

*3
i
5 Ip I dx
X (6.9)
+ me

where
\P I= (V -E). ( 6 . 10 )

Using the equation


X'2 X

- J \p\dx = i — j J \p\dx, ( 6. 11 )
X X \

where
X> X-2

r = 4 $ \ p \ d x = j ^ V2nk ( V - E ) d x , (6 . 1 2 )
xi xi
the solution (6.9) can be put into a form such that it can be connected
to the solution in region I by means of Eq. (5.70). Thus we have

V
- T $ I P 1dx . T $ 1P 1
£i.,e'e
7e +-9ffiiic"1
2 M il '® ** (6.13)
V \ p \ ]
100 nonrelativistic quantum mechanics

According to Eq. (5.70), the solution in region I ( * O i ) has the


form

P rf-v + t ) pax + t )
+ hie (6.14)
]•

Substituting this solution into (6.5), the following expressions for


the incident and reflected waves are obtained:

/inc = l/h|®; (6-15)


/r e f = Ig) h (6.16)

Equations (5.70) and (5.71) can be used to relate the coefficients

gia.ndhl with g ^ a n d y <gme'T:

£i \ = ~2 Sme T> (6.17)


gl —hl = — 2tgnleh (6.18)

From this, we find

/b = /gni(^ + { ^ ) , (6.19)

g, = - '£ ,u ( e7 - T e~T)- (6-2°)

In accordance with the definition of the transmission coefficient


(6.4a), we have
D _ Ihr | _ | g ,„|« _ 1 ( 6 . 21 )
Unci I "1I’ f^+ je-T Y 1

In exactly the same way, we find the reflection coefficient

to , (‘' - W ( 6 . 22 )
, y 'i n c 1 + - « - ■ ') '

From these formulas, it follows that the sum of the transm ission
and reflection coefficients is equal to unity:
/? + D = l . (6.23)
THE Q U ANT UM THEORY OF C O N D U C TIV ITY 101

For cases where the quantity 7 is much larger than unity (these
are the only ones of practical interest), the transmission co­
efficient (6 . 2 1 ) is given by the expression 1

- T } V2m„ ( V - F . ) d x
Dg^e-v = e ** .(6.24)

In the classical limit (fl — 0), it is evident that the transmission


coefficient becomes zero, as We would expect, and thus the
penetration of particles through the potential barrier is impossible.

B. THE TUNNEL EFFECT AS A MANIFESTATION OF


WAVE PROPERTIES

The penetration of particles through a potential barrier is a


typical manifestation of their wave properties. Therefore, an
analog of this effect must occur in
every type of wave theory.
In optics, this analog is the w ell-
known phenomenon of total internal
reflection, which occurs when light is
reflected from an optically less dense
medium. Let us assume that a ray of
light propagating in glass strikes a
glass-air interface (air being the op­
tically less dense medium). (See Fig.
6.2.) Then the wave field, which can
be characterized by the electric or
magnetic field strength, is described F ig . 6 .2 . P r o p a g a t i o n o f l i g h t
in glass (where the index of refraction from a m ore d e n s e to a l e s s
d e n s e m edium .
1 ) and in air (n = l) , by the follow­

ing equations, respectively:


/id.n
9]=Aie- iai‘ + — (6.25)
/ID||
1(' 5in0ii+"cos0ii). (6 . 25a)

In s o l v i n g th e s i m i l a r p ro b lem o f p e n e t r a t i o n of a p a r t i c l e th rough a r e c t a n g u l a r p o t e n ­
t i a l b a r r i e r ( s e e P r o b l e m 5.3), w e o b t a i n e d th e s a m e e x p o n e n t a s in ( 6 .2 4 ), b u t in front of
16n*
the e x p o n e n t i a l t h e r e w a s a f a c t o r , w h i c h w a s of t h e o r d e r of
(1 +« 2) 2 V;
u n ity . In th e c a s e of a s m o o t h b a r r i e r ( t h a t i s , a b a r r i e r e x e r t i n g a f o rc e w h i c h is c o n t i n u -
o u s a t a l l p o i n t s ) , t h i s f a c t o r b e c o m e s e x a c t l y u n ity .
102 NONRELATIVIST1C QUANTUM MECHANICS

Equating these functions2 at the interface (at the planey = 0),


we obtain the familiar laws of refraction:

u)n = ioi; s i n On = n s i n 0i; co s0 u = [ 1 — ti1 s i n ‘2 0 i.

Substituting these values into (6.25a) we find for the refracted


wave
/oil ----------
— iW -1------ (a-n sin G» + y V I —«2 sin^ 0 )
<?ii = A i\e c '•

From this it is seen that, if n sin 0j_<[ 1, an ordinary wave will be


propagated in region II. In the case where nsinOi> 1, we have total
internal reflection, the physical analysis of which can not be given
on the basis of the laws of geometrical optics. From the standpoint
of physical optics, which accounts for the wave properties of light,
the electric and magnetic fields are exponentially decaying:
U)| __ /(!)[
--------Y n* sins Oi — 1 y — io>t/ -|----------- x n sin Or
?n = e c c

If in the case of total internal reflection the refracted wave en­


counters a second glass surface (region III) (that is, we have two
pieces of glass separated by an air layer), then in region III the wave
is again propagated according to Eq.
Incident (6.25). Its amplitude, however, will
be an exponentially decreasing func­
tion of the width of the air layer I.
COJ ______
, f t -------— F r t 2 sir.2 Oi — 1/ //>
Am = Aie c . (d.26)
A diagram of the tunnel effect in
optics is shown in Fig. 6.3.
Before concluding the physical
analysis of the penetration of par­
ticles through a potential barrier,
we should also consider the so-
I ' i p . 6 ..T S c h e m a t i c d i a g r a m o f t h e called “ tunnel effect paradox.” This
tu n n el e ff e c t in o p t i c s (to tal i n t e r ­ paradox lies in the fact that, at first
nal reflection). glance, it seem s as though the real
classical particles inside the poten­
tial barrier are in a peculiar state characteristized by an imaginary
momentum. However, it is important to remember that in this
purely quantum-mechanical phenomenon, it is only the probability
of a particle being somewhere inside the barrier which decreases
exponentially as we recede from the boundary into the potential
barrier. Inside the barrier, the momentum and position of a particle
are real, and are both given within the framework of the ordinary

W*’ s h a l l not w r i t e h e r o t h e e x p r e s s i o n for the r e f le c te d w av e.


THE Q U ANT UM THEORY OF C O N D U C TIV ITY 03

uncertainty relation. In order to show this, let us consider a


potential barrier with constant V. In the first approximation, the
wave function inside the barrier changes according to the equation

Vll = Ae-'*, (6.27)


where
x = ~ ] / 2 m 0(F — E) = const. (6.28)

The right-hand side of (6.27) can be represented in the form of a


Fourier integral

e~xx= I"/ (k) cos kx dk, (6.29)


55
where
2x
f(k) T. (x2+ k S) ’ (6.29a)

which means that it can be represented as a set of wave functions


having real momenta. Obviously, the amplitudes f(k) will effectively
differ from zero only when k varies in the range from 0 to x.
Thus the uncertainty in momentum is
Ap ~ fix.

According to wave mechanics, the position of a particle inside


the potential barrier can be determined only to an accuracy within
the order of the width of the barrier:
l = x.2— Xj, Ax ~ l .

Multiplying Ap and Ax we obtain


ApAx~hv.l.

Since our equations are valid only for the case /(;> 1 , the accuracy
in determining the momentum of the particle and its position inside
the potential barrier will not contradict the uncertainty relation.

C. MOTION OF ELECTRONS IN A METAL3

The theory of the tunnel effect has a number of very important


applications both in the theory of metals and in nuclear physics.
On the basis of this theory it is now possible to explain a number
of phenomena which could not be accounted for in classical physics,

3S e e H. B e t h e a n d A. S o m m erfeld , El ekt r ont hcori e dcr Metal l e, l l andbuch dcr P h y s i k ,


B e rl i n : S p rin g e r, 1933, Vol. 24, p a r t 2.
104 N O N R E L A T I V I S T 1C QUANTUM M E CH A N IC S

such as cold em ission (the em ission of electrons from a metal


under the action of an electric field), contact potentials, etc. Before
discussing these phenomena, we shall say a few words about the
theory of an electron gas, which underlies the electron theory of
the conductivity of metals.
The high conductivity of metals indicates that electrons are able
to move relatively freely inside the entire crystal lattice of a metal.
Their excape from the metal into vacuum is, however, hindered
because this requires the expenditure of a certain energy, the so -
called “ work function.” This suggests that as a first approx­
imation, we may simply consider the metal as a potential well,
inside which the potential energy of an electron can be taken as
equal to zero (V = 0), while outside the metal (that is, in a vacuum)
l/ = l/ o > 0 .
This simplified model enables us to explain several phenomena
occurring in metals. Some of its fundamental results, obtained for
the case of free electrons, can be extended (with the help of quantum
mechanics) to include the periodic field of a crystal (see below the
simplest one-dimensional Kronig-Penney model, which correctly
describes at least the qualitative aspects of many phenomena).
The electron gas model of a metal was first considered in
classical theory (the theories of Drude, Lorentz and others). In this
version of the model, the classical Maxwell-Boltzmann statistics,
which had successfully explained many phenomena in the kinetic
theory of gases, was now applied to electrons. However, the e lec ­
tron gas model encountered great difficulties in developing a
theory of specific heat. In accordance with the theorem of equipar-
tition of energy, well-known from classical statistical mechanics,
each degree of freedom must have, on the average, an energy 4

^Ew = \ k T , (6.30)

where h is Boltzmann’s constant. From this, it is evident that the


contribution of each free electron to the total specific heat will be
the same as that of a free atom:

This contradicts the experimental facts which indicate that the


specific heat of a monatomic metal is the same as that of the lattice
atoms; that is, in the first approximation, free electrons make no
contribution to the specific heat of a metal.

I h ‘- s p e c i f i c h e a l s o f m o n a t o m i c s u b s t a n c e s w i l l b e c o n s i d e r e d i n g r e a t e r d e t a i l in
f h a p i - T 12 | s e c f o r m u l a ( 12. 06) 1.
THE Q U ANT UM THEORY OF CO N D U C TIV ITY 105

This contradiction was resolved by Sommerfeld, who showed


that electrons in a metal do not obey the classical distribution
_R_
f = Ae kr.

Instead, the distribution is characterized by the Fermi-Dirac


distribution function
/ f - d = -j------ --------- .
Jr f E/*r+ I

The Fermi-Dirac quantum statistics is based on the Pauli


exclusion principle, according to which each energy level can be
occupied by at most two electrons (two quantum states which differ
only by the direction of spin).
If we are given a three-dimensional potential well of a cubic
shape, with side length equal to L, then, according to Eq. (4.57),
the components of momentum p = hk will be related to the integers
nt n.2 and n3 characterizing the energy level by the expressions

'Ir.h ili 'Ir.Tin-i ‘I r .tliu


Px £ > Py [ , Pz j •

We note that a unit interval of quantum numbers

( A iii - A n. = A ; i ;, — I )

Art, bz.Art.i — ^ h, d*p (6.30a)

is associated with only one level, occupied by two electrons. There­


fore, if there are p0 electrons per unit volume, the maximum
momentum of an electron at absolute zero(T = 0 )can be determined
from the relation

• 4k
pm a x •I
2 Art1 A/z.iArtJ
2
j p*dp
U
/>max
(6.31)

or

P m a x = = f l ( 3 ~ , Po)1/3• (6.32)

The corresponding maximum kinetic energy is

cm __ p max rr-
^max ■
~ 2mo (3"''>pu)‘/ (6.33)

This energy is called the Fermi energy.


106 N O N R E L A T IV IST IC QUANTUM M EC H A N IC S

As an example, let us compute the value of this energy for


silver. The density of silver is 10.5, and its atomis weight is 107.9.
Assuming that the number of free
electrons is equal to the number of
atoms per unit volume, we have

P° = T ^ J - 6.°2 - 10“ = 5.8- 10'22

where we have used the Avogadro


number (the number of atoms in one
gram-atom, equal to 6 . 0 2 * 1 0 2-1).
Hence, Eq. (6.33) gives
Fig. 6 .4 . Model of a p o t e n t i a l w e l l for
a m e ta l.
^max = 8.5 ■10 12 erg = 5.3 ev.
/Cmax is t h e u p p e r lim it o f t h e f i l l e d
Since the work function for silver
l e v e l s a t T = o (th e F erm i e n erg y ).
is W = 3,7 ev, the depth of the
potential well in silver is found to be equal to 9 ev. A schematic
diagram of the filling of energy levels in a metal is given in Fig. 6.4.
Thu average energy of an electron in a metal is given by the
equation

' ’max
p ,i _2 I P~_ <iaP_ pn (6.33a)
av— Vo 3 2«io Srj-'h3 — 5 L max •
u
In agreement with experiment, it follows that at relatively low
temperatures the electron gas makes no contribution to the specific
heat, since
dE'ky
cel
Tl
df
0 .

If the temperature differs from zero, some ofthe electrons will


jump into higher energy levels. The distribution of electrons in
the higher levels will not be characterized by a Maxwellian dis­
tribution, but by the Fermi distribution function
1
f= (6.34)
A «W + I

At 7 0 (the case considered above), this function equals unity if


/; " /:ma*' anc* zero if /; > For T V-0 the average energy can
be obtained from the expression

,:T n r.-’/e-T2
max 0 ( T ’‘).
THE Q U ANT UM THEORY OF CO N D U C TIV ITY 107

Hence, the contribution of each electron to the specific heat is


,:T
o u L , r .'- k n
,el _;av (6.35)
dT 2 /in

which vanishes as T -+ 0. At high temperatures, when the quantity


—ef i / * 7 becomes much greater than unity, the Fermi-Dirac distri­
bution function (6.34) approaches the classical Maxwell-Boltzmann
distribution
i = Ae~E/hTt (6.36)

which, as we know, implies the following express ion for the average
energy of a free electron:

Efy = ^kT. (6.37)

By comparing this expression with the average energy at low


temperatures [see Eq. (6.33a)] and using the condition

E * > E ° av (6.38)

it is possible to define the degeneracy temperature

< 6 ' 3 9 >

At temperatures higher than TdeR (T^>Tdeg),we can use classical


statistics to describe the behavior of electrons in the metal. If
we substitute the value of p0, say, for silver, we obtain Tdeg ^ 1 0 — 2 0
thousand degrees. Thus, at all temperatures at which a metal
exists in the solid state, the electron gas has a certain degree of
degeneracy. This means that in discussing the properties of elec­
trons, we must use only Fermi-Dirac statistics, and, moreover,
the principal term in the expression for the kinetic energy of free
electrons is independent of temperature.
This large value of the degeneracy temperature is associated
with the small mass of the electron m0. Ions and molecules have
a m ass thousands of times greater than the mass of an electron,
and therefore classical statistics applies to them at ordinary
temperatures.

D. REMOVAL OF ELECTRONS FROM A METAL.


COLD EMISSION

From the potential-well model of electrons in a metal (see


Fig. 6.4) we can see that to remove an electron from a metal
08 NON R E L A T I V I S T 1C Q U ANT UM M E C H A N IC S

it is necessary to impart to it an amount of energy no sm aller than


the work function
lF = y 0 ~ £ max- (6-40)

As we know, in the external photoelectric effect an electron


receives an energy fm from the absorbed photon. Thus, an electron
can leave the metal with a kinetic energy

= (6.41)

(Einstein’s equation). It follows that the work function represents


the minimum amount of energy that must be added to the electron
in order to make its energy greater than the height of the potential
barrier.

I'ip. 6 . 5 . P o t e n t i a l e n e r g y o f a n e l e c t r o n in a m e t a l
in t h e a b s e n c e a n d in t h e p r e s e n c e o f an e x t e r n a l
e l e c t r i c f ield .
The d a sh e d line show s the behav io r of the p o ten tia l
e n e r g y c u r v e w h e n t h e i m a g e f o r c e i s t a k e n in to
account.

In a metal at T — 0 some of the electrons occupy energy


levels lying above the Fermi level. If we increase the kinetic
energy of the electron gas by heating the metal, a certain fraction
of electi ons may acquire an energy exceeding the height of the
potential barrier, and thus a current will flow from the metal.
This phenomenon, which is known as thermionic em ission, is used
to obtain an electron beam in electron tubes. Under the action of
an external electrostatic field, this current may also arise even at
lower temperatures. Let us consider the influence of an external
electric field b applied to the surface of a conductor in the negative
a direction. The potential energy for this case is
V (-V) = V„ — c’/ox, (6.42)
where e — r„ is the electron charge and 8 is the electric field
intensity (sec Fig. 6.5).
THE QUANTUM THEORY OF C O N D U C TIV ITY 09

In addition to the external electric field, the electron experiences


an electric force called the image force. This force arises because
an electron with charge —er, induces an “ image’! charge <?0 at the
surface of the metal (see Fig. 6 . 6 ).

Metal Vacuum
x

F i g . 6 . 6 . Im ag e f o r c e s : a n e l e c t r o n o u t s i d e
th e m e t a l e x p e r i e n c e s t h e a t t r a c t i v e f o rc e
of the in d u c e d charge.

Thus the total force acting on the electron is

F = e& - & - (6.43)


The effective potential energy, taking into account the image force,
is of the form

^ eff — ^ — 4^. • (6.44)

The quantity V eff has a maximum at the point ,v0:

—dx— —
— _ e ft-L-il
4a.2 — v,
o-j- — 12 y g .
x0 — (6.45)

The maximum value of Feff is le ss than because

= — Vel&. (6.46)

Thus, taking into account the electric image force shows that when
an external field is applied, the work function decreases and be­
comes equal to
W ' = W — Vefa. (6.47)

The electric image forces, however, do not explain cold emission.


In fact, an estimate of the maximum current (with W' — O) for
tungsten, for instance, gives the value

§ = '7eor = 2 • 10s v/cm , (6.48)

whereas experimentally rather strong current is obtained with a


field as low as g ^ 4 • 106 v/cm (Millikan).
no NONRELATIVISTIC QUANTUM MECHANICS

Thus, within the framework of the classical theory it is


impossible to explain quantitatively the cold em ission of elec­
trons.
In the quantum theory of this phenomenon (essentially the trans­
mission of electrons through a potential barrier), we limit ourselves
to Eq. (6.42) for the potential energy and neglect the electric image
force since it does not significantly affect the final result. It can be
seen from the graph of potential energy (see Fig. 6.5) that the
external electric field produces a potential barrier of finite width.
Because of the tunnel effect an electron can penetrate this barrier,
the transmission coefficient being given by

- -- Yinu, J W ( x ) — E dx
D= e o (6.49)

The integral in the exponent must be taken over the entire width of
the barrier from x = 0 to the point x = xt given by the condition

V0 — e0$Xt = E, that i s , xt = V° E . (6.50)

Then
Xi X\

^ YV(x) — E d x = f \^V0— e0$ x — Edx =


0 0

= V~e& V x i —xdx = j y 7 ^ >x‘/ \


5 (6.51)

Finally, we obtain the following expression for the transm ission


coefficient D:

So
D —e (6.52)

where the quantity 8 0 depends on the nature of the metal and the
energy of the free electrons inside the metal. The cold em ission
current is proportional to the transm ission coefficient

y = y 0 D = y (Jg“ lK. (6.53)

It follows from the last equation that cold em ission should be


observed for an electron field of g ~ 1 0 Gv/cm . This result is in
good agreement with experimental data.
THE Q U ANT UM THEORY OF C O N D U C T IV IT Y

E. CONTACT POTENTIALS

Contact potentials, which were discovered by Volta, can also be


explained on the basis of the tunnel effect. Let us consider two
different metals with different work functions and different Fermi
energies (see Fig. 6.7). If these two metals are brought into contact,
they will still be separated by a potential barrier of finite width.

Vacuum MetaZ I Vacuum Metal II Vacuum

I *

F i g . 6 . 7 . T w o m e t a l s b e f o r e t h e y a r e p l a c e d in c o n t a c t w i t h e a c h
o ther.
Wj a n d W2 a r e w o r k f u n c t i o n s ; E q j a n d E 0 2 a r e ^ e u p p e r l i m i t s
of the fille d l e v e ls (the F erm i en erg ie s).

Since a certain number of filled energy levels in metal I lie above


the highest filled level of metal II, electrons can move from metal I
into the empty levels of metal II by the tunnel effect. From Fig. 6.7,
it is seen that no flow in the opposite direction is possible, since
electrons of metal II would then have energies corresponding to
filled levels of metal I. It is obvious that the electric current from
metal I to II ceases only when the uppermost filled levels of both
metals are of the same energy.

Fig. 6 . 8 . T w o m e t a l s a f t e r t h e y h a v e b e e n b r o u g h t into
c o n ta c t. Form ation of the c o n ta c t p o te n tia l.

As a result of the tunnel effect, metal II acquires an excess of


electrons and is charged negatively, whereas metal I is charged
positively. Thus, the energy levels of metal II are shifted upwards
relative to those of metal I (see Fig. 6 . 8 ). After the Fermi levels
2 NONRELA TIV ISTIC QUANTUM M E CH A N IC S

in both metals are equalized, the electric current ceases, but then
there arises a potential difference proportional to the difference
between the work functions of the metals:

= (6.54)
eo
This quantity is called the contact potential.

F. THE MOTION OF ELECTRONS IN A


PERIODIC ELECTRIC FIELD (THE ONE-DIMENSIONAL
KRONIG-PENNEY MODEL)

As has been previously mentioned, the representation of the


motion of an electron in a metal in terms of the potential well
model is an approximation in which we average out the periodic
potential of the lattice. A number of characteristic features of the
motion of electrons in a crystal appear only when the periodic
variation of the potential is taken into account. In the general case,
the solution of the problem is very complicated. In order to deter­
mine some of the qualitative features of this motion, we may con­
sider, however, a simplified model of a crystal.
In the one-dimensional Kronig-Penney model, the periodic
electric field produced by the positive ions of the crystal is approx­
imated by a periodic square-well potential of the form shown in
Fig. 6.9. The width of each well is denoted by a, and the width of
the barrier between two successive wells by b. Thus, the period of
the potential (the equivalent of the lattice constant) is c — a-\-b.
The barrier height is set equal to K0.

6 .9 . O n e - d i m e n s i o n a l K r o n i g - P e n n e y m o d e l o f a
crystal.

The solution of Schrodinger’s equation for the nth section of


the periodic potential has the following form:
for the potential well:
= /1„ sin kxn-}- Blt cos kxn, (6.55)

for the potential barrier:


''/k= A'n sinh (*., — c) -j- Bh cosh * (x;1— c). (6.56)
THE Q U ANT UM THEORY OF C O N D U C T IV IT Y 1 13

Here k = ^ / ~ x = j / ~ ) xn= x — crr, the coordinate


is measured from the originofthe/ithsection (that is , the nth well).
Similarly, for the (rc-j- l)st section, we can write

'tVi = ^,, i sin kxn, t - f cos kx,H,. (6.57)

We first join solutions (6.55) and (6.56) at the point x — cn -|-a (that
is, atx„ = a), obtaining

/ln3 in!?a-f- Bncoska = — A'„ sinh v.b -)- B,',cosh Ab, (6.57a)
Ancoska — Bnsinka = £ (/1,'jCosh %b — B.isinh %b).

Next, joining the solutions (6.56) and (6.57) at the pointx = c(«-f- 1)
and noting that xn— c and x,Iil = 0 , we obtain
B n = B . l+ l,
(6.58)
A 'n = ± A a+ i .

Let us substitute (6.58) into (6.57a) and simplify the problems by


considering the limiting Kronig-Penney case in which the width of
the barrier between two wells tends to zero (b-+ 0 ), the height
tends to infinity, and the width of the well remains constant:

t.-ba
T" P = const.

Then, since cosh xb-> 1 and sinh -/.b-^^b, we have

An sin ka + Ba cos ka = Bn,u ^ 5Q)


An cos ka — Bn sin ka = An., — -fa B,u

Equations (6.59) are linear difference equations; their solutions


should be sought in the form

An— Cirn, = (6.60)

Substituting (6.60) into (6.59) and dividing both equations by we


obtain an equation from which we can determine the quantity r and
the relationship between the coefficients Cjand C.,:

Ci sin ka — C.2 ( r — cos ka),


C., r — sin ka'. = (r — cos ka). (6.61)
114 NONRELATIVISTIC QUANTUM MECHANICS

Multiplying these equations together and dividing both sides of the


resulting equation by C,C2, we obtain an equation for r:

rl — 2r cos k'a-{-\ = 0 , (6.62)

where cos k'a is given by the equation


p
cos k'a = sin ka -j- cos ka. (6.63)

As we shall see below, (6.63) is the fundamental equation for the


energy levels in the periodic field of a crystal.
The solution of (6.62) has the form

r = cos k'a ± i sin k'a = e t ik'a. (6.64)

We note that if the right-hand side of (6.63) is greater than unity,


//w ill be imaginary, and in this case we get an exponential solution

r = e± ^’\a= e± x'a. (6.65)

Let us examine in greater detail the solution for the case of real
values of k'

r = eik'a.

From Eqs. (6.61) and (6.60), we have

r — r r - cos ka „ „ (eik'a —cos ka) eik'an


ka - A<‘— ------- .
C., = C0, B„ = Clie;k'nn.

Substituting the values of /l„and B„into (6.65) and using the fact that
x„ = x — an when b-> 0 , we find an expression for the wave function
in the crystal:

<!»= Cseik’xUn, (6 . 6 6 )

where U„ is a function with the same periodicity as the crystal


lattice
U"= ^ l w ,h { Xns i n kxn— c~‘k'xnsm k(xu— a)]. (6.67)

In particular, if P = 0 (that is, there is no barrier), we find from


(6.63) that// = /;; in this case the function U„ becomes unity.
It follows from (6 . 6 6 ) that an electron can move freely in the
crystal if /;' is a real quantity, that is, if the right-hand side of
(6.63) is less than unity.
THE Q U ANT UM THEORY OF C O N D U C TIV ITY 1 15

G. BASIC PRINCIPLES OF THE ELECTRON THEORY


OF CONDUCTIVITY OF CRYSTALS

The quantum theory of electron motion In a crystal lattice provides a key to distin­
guishing between conductors, dielectrics or insulators, and semiconductors (which In a
sense form an intermediate class of solids). We do not intend to treat this subject
quantitatively and shall confine ourselves to a few qualitative remarks based on the one-
dimensional Kronig-Penney model.
As the starting point of our analysis, we shall use Eq. (6.63) to determine the possible
values of the electron energy in a crystal lattice. This equation Is
p
cosk'a = — sin ka -f- cos ka, (6.63)

where a is the lattice constant, and the quantity k = - L _ — gives the electron energy.
The eigenvalues of the electron energy can be found from the condition that k' must be
real. This condition means that the right-hand side of Eq. (6.63) must be less than unity.
Setting P = co, we obtain the energy spectrum of isolated atoms (In this case the atoms
are separated from one another by an impenetrable barrier). The energy spectrum will
then consist of a separate set of levels for each well:

k' = k = Knta,
F iWW (6 .6 8 )
n 2m0aa •
where n = 1, 2, 3 . .. . We shall not consider the negative values of n, since they give
exactly the same values of energy, but correspond to the motion of electrons in the
negative x direction. The first two levels (n = 1, 2)
-Hi* E, = 4 Ei
Ei- (6.69)
2m0a2
of isolated atoms are shown in Fig. 6.10.

-El

e2

e !

Energy levels of -------------------------------


isolated atoms -
Energy levels of the 1
crystal lattice

F i g . 6 . 1 0 . T h e f o rm a tio n o f e n e r g y b a n d s in a c r y s t a l l a t t i c e .

If P is finite [see Eq. (6.63)], it is easiest to determine the energy levels graphically
(see Fig. 6.11). The allowed values of k (and therefore of the energy) correspond to the
values of the right-hand side of Eq. (6.63) lying between— land-]- 1. In Fig. 6.11, these
allowed values are denoted by a heavy line. Thus, In a crystal lattice containing N atoms
each energy level of an isolated atom is split into N levels. Each of these groups of
levels is called a band (see Fig. 6.10).
116 N O N R ELA TIV ISTIC QUANTUM M E C H A N IC S

The electrons in the crystal tend to occupy the lowest energy levels and, according to
the Pauli exclusion principle, each level is occupied by at most two electrons with
opposite spins.
For example, the crystal lattice of an alkali metal contains N valence electrons If
there are A' atoms. The electrons occupy only half of the lower band (since there can be at
most two electrons in each level). In
the ground state, half of the electrons
— ha + c o s ka move in one direction, and the other
ka
half In the opposite direction; con­
sequently, the average current is equal
to zero. When an electric field is
applied, more than half of the electrons
move in some preferred direction,
thus producing an electric current and
moving up into higher energy levels.
Therefore, In a metal, either the first
F ig . 6 .1 1 . D e t e r m i n a t i o n of p o s s i b l e e n e r g y
band must contain a sufficient number
l e v e l s in a c r y s t a l l a t t i c e . of unfilled levels, or it must come into
T h e h e a v y l i n e s on th e a b s c i s s a a x i s i n d i ­ contact with a second empty band, called
c a t e t h e a l l o w e d v a l u e s o f ka ( a l l o w e d the conduction band.
bands). On the contrary, in a perfect insula­
tor all levels in the first or valence
band are occupied and the levels of the second (conduction) band are all empty. The energy
spacing between these bands is usually several electron volts. It is easy to show that,
when a field is applied, the electrons cannot acquire a preferred direction of motion. The
direction of motion of an electron can be reversed only if the electron goes into a different
energy state. But since all the energy levels are occupied, this can only happen if the
electron ‘ occupying that other energy state makes the opposite transition to the state
originally occupied by the first electron. Therefore, on the average, there can be no
preferred direction of motion of the electrons even when a field is applied. A diagram
of the energy levels of a conductor and a dielectric is given in Fig. 6.12.
ao
S-a
*5
^ is
N1) o
§•3 U
■o u
•a id
'o ^ -*■
-9-
-9-
C
---- C----------- -------------------«--------
---- •----------- ----------- «-------- 4) 5
9
------------ 1
-------------------------------- ------------------------------------------------------ _______________ "3ua
---- e------------------------------ a-------- >
---- e------------------------------ c--------
---- •----------- ----------- e--------
Metal Perfect dielectric
1 ig. 6 .1 2 . E n e rg y l e v e l d ia g r a m o f a m e t a l a n d a d i e l e c t r i c .

Nevertheless, every dielectric p ossesses some (very small) conductivity. The con­
ductivity Is much greater in semiconductors, where the forbidden band is considerably
sm aller than In dielectrics; it Is of the order of 1 ev, and sometimes even le ss (for
example, in germanium, the width of the allowed band is 0.66ev at T = dOOcK). At absolute
zero, semiconductors behave like dielectrics; however, their conductivity increases with
rising temperature, particularly in the presence of impurities. At room temperature,
the resistivity of semiconductors is found to be of the order of I0~2— 102 ohm.cm. At the
same time, the resistivity of dielectrics lies within the range of 103— 1019ohm.cm, and
that of metals in the range of lO"6— 10~9 ohm.cm.
Ihe conductivity of dielectrics and semiconductors can be of two types. Let us assume
that under the influence of thermal excitation or the internal photoelectric effect (absorb-
tion of light by electrons), some of the electrons Jump from the valence Into the conduction
band, leaving vacant states (“holes” ) in the valence band. The electrons that have jumped
THE QUANTUM THEORY OF C O N D U C T IV IT Y 17

into the conduction band become carriers of electric charge, thus producing a current.
On the other hand, as soon as several states become empty in the valence band, an elec­
tric current can also be produced due to the motion of electrons in the valence band
itself. It can be shown that the motion of a system of electrons iiran almost completely
filled valence band can be treated in terms of the motion of a set of vacant states or
"holes." Obviously, the holes move in a direction opposite to the direction of electron
motion, so that they behave like positively charged particles. Thus, there are two ways
In which charge can be transported and, therefore, two types of electric current in solids:
n type ( due to the motion of electrons) and p type (due to the motion of holes).
With the help of the band theory, we can easily give a qualitative explanation of a
number of interesting phenomena.
For example, the conductivity of a metal increases with decreasing temperature
because its resistance is due to the interaction between free electrons and the vibra­
tions of the lattice. Since the vibrations of the lattice diminish as the temperature
decreases, the resistance will also decrease. On the contrary, the conductivity of semi­
conductors decreases as the temperature is lowered, because the number of electrons
in the conduction band becomes increasingly smaller. The existence of p type conductivity
in semiconductors has been demonstrated in investigations of the Hall effect and thermo­
electric emf. The sign of the potentials appearing in these phenomena is determined by
the sign of the current carriers. Experiment shows that in some semiconductors the
sign of the potentials corresponds to electron carriers, whereas in other semiconductors
the sign of the potentials is reversed; that is, the current carriers act as if they were
positively charged particles. A natural explanation for this is provided by the concept
of "holes.”
The above conclusions in regard to the conductivity due to transitions of electrons
from the valence band into the conduction band (the long arrow on the left-hand side of
Fig. 6.13) and due to the motion of holes In the valence band are based on the assumption
of a perfect crystal. This type of conductivity is known as intrinsic conductivity.
Another type of conductivity, known as impurity conductivity, also plays a significant
role in semiconductors. It is caused by the presence of foreign impurities or other
structural defects in the lattice. Such disruptions of the perfect periodicity of the lattice
lead to local deviations of the field from a perfectly
periodic one. As a result, discrete levels may Conduction
appear in the forbidden band of the energy spec­ band
trum of the electrons (see Fig. 6.13). In practice,
the wave functions associated with these levels
differ from zero only in a certain region near the Donor level
given defect. Thus, discrete levels are sometimes
called impurity levels. They do not themselves
contribute to the current (the electrons occupying Accepter
them are not free). However, they may affect the level
number of electrons contained in both the con­
duction and the valence bands. Valence
Impurity levels (denoted by short arrows on the band
right-hand side of Fig. 6.13) can be divided into two
types: donor levels and acceptor levels. A donor
is capable of supplying electrons to the conduction F i g . 6 . 1 3 . D ia g r a m o f e n e r g y b a n d s
band, so that free electrons will appear in the in a s e m i c o n d u c t o r .
conduction band. In contrast to a donor, an
a c c e p to r a b s o r b s e le c t r o n s ; a s a r e s u l t a " h o l e ” is f o r m e d in th e fille d b an d s o th at
e l e c t r o n s in th is b a n d a r e a b le to J u m p in to h i g h e r e n e r g y s t a t e s ( p t y p e c o n d u c tiv ity ) .
If the impurity levels are located sufficiently close to the edge of the corresponding
bands, ionization (or other effects) can arise fairly easily due to the energy of thermal
motion of the lattice. Therefore, an appreciable number of free electrons (or holes) can
exist in bands even under conditions when direct excitation of electrons from the valence
band into the conduction band is highly improbable.
In the case of impurity conduction, the magnitude and type of the conductivity are
determined mainly by the nature and concentration of the Impurities. By varying the
impurities, it is possible to control within a wide range the magnitude of the conductivity
as well as its type ( n type or p type).5 This fact is widely utilized in semiconductor

P u r e g erm an iu m h a s a h i g h r e s i s t a n c e b u t t h e a d d i t i o n o f a s m a l l a m o u n t o f i m p u r i t i e s
may g i v e it e i t h e r n or p t y p e c o n d u c t i v i t y .
118 N O N R EI—ATI VI S T I C Q U ANT UM M E C H A N IC S

electronics. It constitutes the basis of operation of modern high-quality crystal diodes


and transistor devices which are capable of rectifying AC currents, as well as of ampli­
fying and generating electrical oscillations.
In conclusion, we would like to draw attention to two important approximations which
we have implicitly assumed in the electron theory of solids. First, in developing the
theory of metals and semiconductors, we were actually concerned only with the problem
of single electrons. Accordingly, this theory is called the single-electron theory. The
collective properties of electrons in solids were taken into account only in connection with
the filling of energy levels by free electrons, when we calculated certain statistical
quantities (the Fermi energy, specific heat, etc.). Even in those cases we completely
neglected interactions between electrons, although they do In fact experience mutual
Coulomb repulsions. It is easy to show that, for the electron densities which we observed
in metals, the average energy of this repulsion Is not necessarily small in comparison
with the Fermi energy. There naturally arises the question of the extent to which this
crude single-electron model represents the properties of a real metal. The answer to
this question is given in the many-electron theory of solids, which we cannot consider
here since this recently developed theory Is a highly specialized subject. We may note
that the fundamental notion of the electron gas as a free carrier obeying Ferml-Dlrac
statistics is retained in the multielectron theory. Accordingly, the laws of purely
statistical character obtained in the single-electron theory still hold true, such as, for
example, the linear dependence of the electronic specific heat on temperature. At most,
certain numerical coefficients will have to be slightly changed because of the specific
form of the wave functions for a system of electrons. These refinements, however.
Involve exceptionally large computational difficulties.
On the other hand, in dynamical problems, where it is essential to take into account
the Interactions between electrons, the simplied approach used above (the single-electron
theory) is inadequate. One case of this type is the problem of the strength of a metal.
The second approximation which is implicit in the single-electron theory concerns the
crystal lattice, which Is regarded simply as the source of a certain statistical field. The
situation is actually more complicated because the lattice ions execute a vibrational
motion, which, as we know, persists even at absolute zero (zero-point vibrations). In
order to take this fact into account, it is necessary to consider the crystal lattice as a
quantum-mechanical system rather than simply as the source of a field (see Chapter 12, D).
The essentia 1 point in this connection is that the lattice Is not isolated from the system
of electrons, but coupled to It since the electrons Interact with the lattice ions. The energy
of this interaction can be uniquely represented by the sum of two terms. One of them
represents the potential energy of interaction between the electrons and the stationary
lattice. The motion of electrons in a field of this type was considered above in the simple
case of the Kronlg-Permey model. The second term is connected with the deviations of
the Ions from their equilibrium positions and represents the interaction energy between
the electrons and the lattice vibrations. The average value of this interaction energy is
extremely small in comparison with the energy of the electrons at the Fermi level,
because the amplitudes of the vibrations of the lattice ions are small (except at tempera­
tures near the melting point).
The vibrations of the lattice, however, mayplay an Important role at low temperatures,
when the Interaction of each electron with the vibrations of the lattice gives rise to an
additional interaction in a pair of electrons. If these two electrons have opposite spins,
the interaction results In an attraction (In contrast to electrostatic reptislon). As a
result, the pair of electrons begins to move with a certain degree of correlation. Such a
system Is found to have the property of superconductivity.6 The smallness of the inter­
action which causes the correlation explains why superconductivity Is observed only at
extremely low temperatures. 7

A th eo ry of s u p e r c o n d u c t i v i t y b a s e d o n t h e s e c o n c e p t s w a s d e v e l o p e d i n d e p e n d e n t l y
by the A m erican p h y s i c i s t s B a r d e e n , C o o p e r, a n d S c h r i e ff e r , a n d by t h e R u s s i a n p h y s i c i s t
Iiopo 1y u b o v . [ s e e N B o p o ly u b o v , V. T o l m a c h e v , D. S hirk ov, A N e w Me t hod in the Th e o r y
"f Superconduct ivi ty ( t r a n s . ) , New York: C o n s u l t a n t s B u r e a u , 1959.1
See R B e i e r l s , (Quantum r/ieory of Sol ids, Oxf ord: C l a r e n d o n P r e s s , 1955; W. S hock-
l e y , liU' et mns ami Hol es in Semi conductors, N ew York: V an N o s t r a n d , 1950.
C h a p Ic r 7

Statistical Interpretation of Quantum


Mechanics

A. ELEMENTS OF THE THEORY OF LINEAR


OPERATORS

In our general investigation of the Schrodinger wave equation


(Chapter 5), we saw that in quantum mechanics the momentum
operator p = — ihy is associated with the classical momentum p of
the particle, the energy operator E = i h with the energy E, the
Hamiltonian operator or simply Hamiltonian H with Hamilton’s
function H, etc. Before analyzing the physical meaning of the
quantities represented by operators in quantum theory, let us
consider certain general aspects of the theory of operators.
In the same way as a function relates a number x with another
number y = f(x), an operator M associates one function/(*) with
another function

nx ) = M.f(x), (7.1)

according to some given rule.


In order to satisfy the principle of superposition, only linear
operators with the two following fundamental properties are used in
quantum mechanics:

(7.2)
b\Cf — CMf,

where C is an arbitrary constant.


The linear operators most commonly encountered are the differ­
entiation sign (for example, the momentum operator M= y — and
the LaplacianM= V2) and the integration sign.
In Poisson’s equation

v7 (r) = ?(r) (7.3)


120 N O N RELA TIV IST1C QUANTUM M E C H A N IC S

the operator is the Laplacian which converts the function f(r) into
another function p(r). Conversely, we can solve Poisson’s equation
and find

/ (r) = V~?P(r) ■ = - J p(/") K(r, r') (7.3a)

Here the operator v~'2has the form of a definite integral, the kernel
of the operator being

l _________1____________
K(r.r') = 4r. | r —r’ | Y(x —x'Y + (y — y Y + (z — z')a *

The operators v2 and which have the same effect as multiplica­


tion by unity when they are applied in succession, are called inverse
operators.
The Hamiltonian operator

H = T -fF ,
ft 2
consists of the sum of the kinetic energy operator T = — v2»
which is directly porportional to the Laplacian, and the potential
energy V, which is simply a function of the coordinates.
We can regard the action of the potential energy V (x) on the
wave function as the action of a linear operator, because conditions
(7.2) are satisfied. Therefore, besides differential and integral
operators, the linear operators that may be used in quantum m e­
chanics include any function of coordinates whose action on the
wave function is simply to multiply it. For example, the coordinate
r is just as entitled to be considered an operator (the position
operator) 1 as the momentum operator p — — iUy, which is a differ­
ential operator. It is worth noting in this connection that in quantum
mechanics r does not represent the position of a particle, but is an
argument of the wave function and determines its value in the
coordinate space. The quantity which is equivalent to the position
can be found from the operator r and the function ip(r) in the same
way as the momentum of a particle is found from the momentum
operator, that is, by averaging (see the following).

T his will b e c o m e p a r t i c u l a r l y c l e a r w h e n w e w r i t e t h e S c h r o d i n g e r e q u a t i o n in m o m e n ­
tum spue v ( s e e b elo w ), w h e r e t h e w a v e f u n c t i o n d e p e n d s on p. In t h i s c a s e , t h e m o m entum
op' r.it tr p will c o r r e s p o n d to m u l t i p l i c a t i o n by a n o r d i n a r y f u n c t i o n , a n d t h e p o s i t i o n o p e r ­
ator x to d i f f e r e n t i a t i o n w ith r e s p e c t to p.
ST A T IS T IC A L IN T E R P R E T A T IO N 121

B. ELEMENTS OF REPRESENTATION THEORY

The position operators x, y and z obviously commute with each


other, since operating with them is equivalent to multiplying by
ordinary numbers. Accordingly, xy = yx, xz = zx , and so o n . The
operators p.v= — ,^ / v>Pj = — and Pz = — also commute
with each other, since the result of differentiation is independent
of the order in which it is performed

d- d2 d2 d■
axoy
i—= oyox ’
, -x-s-
oyoz
= ozoy
, and, so on.

Similarly, it can readily be seen that the pairs of operators px and


y, py and x, and so on, also commute with each other:

pxy*t= — ih ^ y ^ = — iny ^ = yp^ -


that is,
pxy = y p x-
An example of a pair of noncommuting operators is provided
by x and p*. Indeed,

xPx'!t = — i h x ^ , (7-4)
whereas

= = (7.5)

Consequently,
(PXx — xpx)ty = — ifiy, (7.6)
that is,
pxx — xpx = — ih. (7.7)

In a sim ilar fashion, it is readily shown that

Pyy — ypy = P z 2 - z p z = - ih. (7.8)

The noncommutativity of these operators, as expressed by Eqs.


(7.7) and (7.8), is a significant feature of quantum mechanics.
The specific choice of the form of position and momentum
operators x and px = — ift^— satisfying Eqs. (7.7) and (7.8) corre­
sponds to the so-called coordinate representation, in which the
wave function depends only on the spatial coordinate =
122 N O N R ELA TIV ISTIC QUANTUM M E C H A N IC S

The Schrodinger equation can also be written in the momentum


representation. For the sake of sim plicity, let us take the one­
dimensional case. Using the 8 functions [see (4.70)], we expand
'y(.v) into a Fourier integral

6 (a:) = ^ (xr) 8 (x — x') dx' = — j dkdx'(x') eik(x~x,).

Since P= wherep = pA, we can rewrite this equation in the form

J tp(/?) e' T %dp' (7,9)

<p(p) = y = ^ ' n *dx!- (7-10)

The Fourier transform of the function i{i (x) , namely, the function
y(/j)which depends on momentum, is called the wave function in the
momentum representation. Equations (7.9) and (7.10) relate the
wave functions in the coordinate and the momentum representa­
tions.
Let us find what operator in the momentum representation
corresponds to the position coordinate x. We note that in this
representation we need not write p as an operator (that is, in
roman type). If we substitute <]i(a:) for AAp(x) in Eq. (7.10), then

x? (P) = y = j xty(x)e ' n dx = in , (7.11)

and thus in the momentum representation the operator x has the


form

It is easily verified that Eq. (7.7) continues to hold true in the


momentum representation, since

(xp —px) <p(p) = in ( —p ) = iA? (p).

In exactly the same way, the fundamental relation (7.7) will be


satisfied if x and p are replaced by certain appropriate m atrices.
'I his form of representation is called matrix representation; it
was introduced by Heisenberg somewhat before the discovery of
Schrddinger’s equation.
'I he remainder of our discussion of operators will be conducted
in the coordinate representation.
S T A T IST IC A L IN TERPRETA TIO N 123

C. AVERAGE VALUES OF OPERATORS

In classical mechanics the motion of an individual point is


exactly specified by a function relating its position to time. This
dependence can be uniquely determined from the fundamental
differential equation of motion

m j r = — V I/ ( / ■ ) .

Once we have determined r as a function of time, we can also deter­


mine the momentum and energy of the particle.
The situation is somewhat different if there are many particles
involved, as, for example, in the kinetic theory of gases. In this
case statistical laws characteristic of a large collection of parti­
cles must be used. It turns out that the particles of such a collection
obey certain distribution laws, which, generally speaking, apply to
both coordinate space and momentum space (that is, a distribution
describing both the velocities and the energies). The function f that
characterizes this distribution is called the distribution function.
Thus, when we deal with a large collection of particles, we can
only consider a probability that a particle p o ssesses particular
coordinate and momentum values. From the distribution func­
tion we can fine that average values of the position and momen­
tum.

x = ^ xfd3xd3p, px= ^ p j d 3xd3p,

and the mean square of these quantities

^ x3fd3xd3p , and so on,

which in accordance with the law of large numbers should agree


with the corresponding experimental values.
We must mention one characteristic feature of these statistical
laws. In classical physics they are a result of averaging over the
so-called “ hidden” parameters, which determine the motion of each
particle in accordance with Newton’s equations. These hidden
parameters do not appear in the final results. In principle, however,
classical theory enables us to explain why, at any instant of time,
the coordinates and momenta of individual particles differ from the
average values, even though this explanation may be very com­
plicated mathematically.
In quantum mechanics the behavior of particles is described by
the wave function t), which is a probability function even when
the system it describes consists only of a single particle. Thus,
quantum mechanics allows us to determine only the average values
24 N O N R ELA TIV ISTIC QUANTUM M E C H A N IC S

of dynamic variables regardless of whether there is a large


number of microparticles or only one. It must be emphasized that
in quantum theory it is in principle impossible to explain the
deviations of observed variables from the average values . 2 The
method of calculating averages in quantum mechanics is sim ilar
to that used in statistical mechanics. The basic formula used for
this purpose is

M= J (7.12)

where M is an arbitrary operator (as a special case, it may be a


number), and the quantity ( t ) t y ( t ) plays the role of the distribution
function / provided that the wave functions are normalized:

jjVW'MO d3x = l .
The average values of the position and momentum, as has already
been mentioned, are in fact computed in basically the same way:

x = j" <)i* {t)xb (t)d''x,

P * = lr(t)* <7-13>
Here x is the coordinate of the center of m ass of the wave packet
associated with the function •]>((), and p^is the momentum of this
center of mass.
Since the outcome of a physical measurement is a real quantity,
the average values must be represented by real numbers. There­
fore, the following equation must hold:
= (7.14)
When this requirement is satisfied, the corresponding operators
are said to be self-conjugate (or Hermitian).
In particular, we shall show that the operator pA . satisfies the
condition (7.14), even though it appears to be purely imaginary. As
a preliminary, we must first prove an important theorem for
“ transferring” a derivative. This theorem is as follows. Suppose
we have an integral
oo
G = ^ uvln’dx, (7.15)
— 00

Von Neumann h a s d e m o n s t r a t e d t h a t h i d d e n p a r a m e t e r s c a n n o t b e t h e b a s i s for t h e


s t a t i s t s al law s of q u an tu m m e c h a n i c s . Von N e u m a n n ' s proof, h o w e v e r , is v a l i d o n l y w i t h ­
in th<- limits of th e a c t u a l fra m ew o rk o f q u a n t u m m e c h a n i c s i t s e l f , a n d i f q u a n t u m me-
clianii , is not t a k e n a s a n u l t i m a t e t h e o r y , Von N e u m a n n ' s th e o r e m c a n n o t b e r e g a r d e d a s
Kon' r a l l y v a l i d .
ST A T IS T IC A L IN T E R P R E T A T IO N 125

where vin) — . Then, if all terms of the type


CO

tw In- 1 )
v\ (7.16)
.1
— CO

vanish, the result of integration of G is not altered if we transfer


the nth derivative of the function v to the function u in (7.15) and
place the factor (—1 )" in front of the integral:
co co

^ uvin)d x = { —l)n ^ u,n)vdx. (7.17)


— CO — CO

Indeed, if we carry out an «-fold integration by parts in (7.15) and


assume that all terms in (7.16) vanish, we obtain the relationship
(7.17). In the case of a discrete spectrum, the conditions (7.16) are
always satisfied because the wave function decreases exponentially
at infinity. In the case of free motion (that is, a continuous spec­
trum), these expressions vanish as a consequence of the periodicity
condition. Physically, the fulfillment of condition (7.16) means that
no particles or currents exist at infinity.
Returning to the proof of the self-con jugate ness of the operator
p*, we substitute

u = ^*(t), v= — (/) and « = 1

into Eq. (7.17). From this it immediately follows that

Px = — $ (0 M (0 d x = ^ b(t)ih (t) dx = (px)*,

and thus the self-conjugateness condition (7.14) is satisfied for pv.


We note that unlike the operator p.t = — ih ~ , the real operator -^:
is not self-conjugate and its average value has no physical meaning.
If an operator M has only one eigenvalue ). (and one eigenfunc­
tion <^), it is readily seen that this eigenvalue is identical with the
average value of the operator. Indeed, using the general rule (7.12)
for determining the average value of an operator and substituting
the equation

M -M 0 = > 4 (0 . (7.18)

we obtain for M

M — ^ *(t)h\'b(t)d:'x = ). ^ 6 *(0 '} (t)d3x = k (7.19)


126 NONRELA TIV ISTIC Q U ANT UM M E C H A N IC S

On the other hand, suppose that the operator M [Eq. (7.18)] has
several eigenvalues . . . . X„,. . . , corresponding to the functions
c, (i), o.j(/)....... . . . (f°r example, this may be the case for
the energy operator E = //z~, for which l n= En). Then since the
general solution 6 (/)can be written in the form

*(0 = 2 W O . (7-20)
n

we find that the average value of the energy operator E = t7z-^- is

F = ^ \ Cn\*En. (7.21)

Here each | Cn | 2 is the probability that the particle is in the corre­


sponding quantum state. If all Cn except one,C„0, are equal to zero,
then £ = £„„, and thus the average energy corresponds to the eigen­
value E„a. Consequently, the eigenvalue E„0 corresponds to the
experimentally observed energy. In the case where several co­
efficients Cn differ from zero: C„lt C„.,,. . . , Cn., . . . . we can obtain
any of these values of energy in experimental measurements. If
the experiment is repeated many tim es, the number of measure­
ments which yield an energy En should be proportional to the
corresponding theoretical probability | Cn/|2.
C hapter 8

Average Values of Operators. Change of


Dynamic Variables with Time

A. DERIVATION OF THE UNCERTAINTY PRINCIPLE

As indicated in the preceding chapter, the observable dynamic


quantities associated with the operators must be regarded as
average values, given by Eq. (7.12). This is true regardless of
whether these operators commute with the Hamiltonian, that is,
regardless of whether the corresponding physical quantities are
constants of the motion.
We shall now show that if two dynamic quantities correspond
to noncommuting operators, they do not have simultaneous definite
values in quantum mechanics. Of greatest importance in this
respect is the calculation of the deviations from the average values
of two canonically conjugate quantities—the position x and momentum
px. Our discussion will be carried out in the coordinate represen­
tation and we shall restrict ourselves to the case in which the wave
function is independent of time (the stationary case). Then the
average values of the position and momentum can be found from
the relations

(8 . 1 )

(8. 2 )

First of all, we note that even though the average error or


average deviation from the mean, which is given by

(8.3)

is equal to zero, it in no way follows that the particle cannot occupy


positions other than x. The reason for this is that the deviations
have different signs relative to the mean X , and consequently they
cancel out on the average. Accordingly, the deviation from the
average value of the operator should be characterized by the
variance (mean-square deviation), which is positive for all deviations
[28 N O N RELA TIV 1STIC QUANTUM M E CH A N IC S

from x. The variance of the position can be calculated from the


formula

(Ax) 2 = ij* 6 * (x — x)~ ^ cPx = x2 — 2 ( a ) 1 - f ( a ) '2 — x~ — (A)2. (8.4)

We note, incidentally, that if the variance is (Ax) 2 = 0, it follows


that the probability of the electron occupying a position in space
differs from zero only at x = A. In this case the average value of
the position is equal to the exact value; that is, the corresponding
probability of the particle’s position can be described by a function
similar to the 8 function.
Similarly, the variance of the momentum is given by

(A~ p j 2 = ^ ^ * (p x — p,)2 ^tf'x==pl— (p,)2. (8.5)

In order to establish the relationship between (Ax) 2 and (ApxY,


let us take a coordinate system whose origin lies at the center of
mass of the wave packet (x = 0 ), and which moves with the same
velocity as the center of mass {px = 0). (The use of this coordinate
system does not involve any restriction on the generality of the
discussion.)
In this case

(Ax) 2 = x2= ^ <J»*x2 i)j d3x, (8 . 6 )

(W - f ** ( - ih ~ ) 2 * d*x.

Let us consider the integral

/ ( * ) = j ( a x ^ + Q ( a x ^ + ^)d3A-> (8.7)

where i is some arbitrary real quantity independent of x. Equation


(8.7) can be written in the form

/ ( ? ) = /I a 2 — fla-fC, (8.7a)
where

A = j •'!>*x‘^cPx = x2> 0 ,

b — J ( * T ' h -**, S )< i* -


(8 . 8 )

c — )r iii I
( Px =
<)x <)x h* ih L f
AVERAGE VALUES OF OPERATORS 129

Since the integrand in (8.7) is essentially a positive quantity

/(a)ssO, (8.9)

condition (8 . 9 ) imposes a certain definite restriction on the co­


efficients A, B and C. Indeed, if this relation is satisfied for a = a„
corresponding to the minimum of the function / (a), it will also be
satisfied for any arbitrary a. The value of a0 can be found from the
condition

/ ' (a0) = 271a0 — B = 0 , that is, a„= —,

and

/" (a) = 2/4 > 0.

Hence the minimum value of / (a) will be equal to

Anin— ^ (ao )= — 44 + C 3*0,

From this it follows that inequality (8.9) will hold for all real values
of a, provided the following condition is satisfied:

B* sg 4AC. (8.10)

Substituting the values for A, B and C from (8 . 8 ) and using (8 . 6 ),


we obtain the relationship between (Apx)* and (Ax)*

(Ax)* • (ApJ* ^ —. (8.11)

This inequality represents a rigorous formulation of the uncertainty


principle.
By using the relation — xpx — — ih [see Eq. (7.7)], we can
rewrite (8 . 1 1 ) in the form

( 8 . 12 )

Generalizing (8.12), we can say that whenever two operators Mi and


M.2 do not commute with each other, they satisfy the uncertainty
relation

(AM,)2 •(AM2)2Ss i |M,M2- M2M,I2. (8 .13 )


where

( AMj *= J'PM —M,)*yd*x, (i = 1,2). (8 .14 )


130 NON R E L A T I V I S T I C QUANTUM M E C H A N IC S

As we have already mentioned, the uncertainty principle is a


consequence of the wave-particle duality that underlies quantum
mechanics and is in no way connected with the experimental lim ita­
tions. Experiments may only prove the results which follow
from the uncertainty principle. The basic meaning of the uncertainty
principle consists in the following fact: the probability distributions
of variables whose operators do not commute cannot simultaneously
take the form of a 3 function (see Fig. 8.1). Moreover, if the prob­
ability distribution of one variable approaches a 8 function, the
probability distribution of the other variable will spread out. In
the limit, when, for instance, the probability function for x (that
is, 'i(x);1) takes the form of a 8 function [(Ax)i = 0], the probability
function for the momentum px (that is, | y(px)l2) becomes such
that it is constant for all values of px (Ap^ — oo.

I* ip. H. 1. T h e p r o b a b i l i t y d i s t r i b u t i o n f u n c t i o n in (a) c o o r d i n a t e
s p a c e a n d (b) mom entum s p a c e : [ ( A x ) 2 ( A p ) 2]^3 = fl/2.
If th e d i s t r i b u t i o n i n c o o r d i n a t e s p a c e (a) c o n t r a c t s , t h e d i s t r i b u ­
tion in m om entum s p a c e (b) s p r e a d s .

The necessary condition for simultaneous measurement of two


dynamic quantities is the condition of commutativity of their
corresponding operators.

B. POISSON BRACKETS IN CLASSICAL AND QUANTUM


THEORY

I he state of a system in classical mechanics is defined by its


dynamic variables. The quantities appearing in the canonical, or
Hamilton’s, equations of motion in classical mechanics depend on
the oordinates x,, momenta pi and time /, that is,

i). (8.15)
AVERAGE VALUES OF OPERATORS . 131

For example, in a one-dimensional time-independent problem,


the Hamiltonian depends only on x and px:

H = ^ + V (x). (8.16)

With the help of the canonical (Hamilton’s) equations of motion


we obtain
■ and dH _ _ d V
Px
(8.17)
dpx m0 dx dx'

or
dV(x)
m0x dx

If there are n degrees of freedom ( t = l , 2...... n), Eqs. (8.17)


take the form

. _dH _ . dH_
(8.17a)
X‘ dp i ‘ dxj '

Hence the time rate of change of the quantity f [see (8.15)] is given
by the equation

d f_ d f
dt dt + 2 (&%*'+
Using the canonical equations (8.17a), we obtain

dt dt ‘ 1 ’ /1 ’
(8.18)

where the expression

dji df_ dM d / \
[ H ,
dpi dxi dxi dp -,)
(8.19)

is called the classical Poisson bracket.


If f does not depend explicitly on t , then ^ = 0 , and consequently
the variation of / is completely determined by the Poisson bracket:

% = W , /]. (8 . 2 0 )

If the Poisson bracket vanishes ([//, /] = 0), the quantity f does not
depend on time, or it is conserved.
/ = const.
132 NON R E L A T I V I S T I C QUANTUM M E C H A N IC S

For example, if the energy does not depend.explicitly on time, then


dHdt = 0. Since obviously \H, H] = 0, it follows that Hamilton’s
function (the energy in this case) is a constant (// = const). Further­
more, substituting the coordinate xit and then the momentum p{ for
/ into (8.20), we obtain the relations (8.17a), that is, Hamilton’s
equations of motion.
We shall now generalize the classical Poisson brackets, which
can be used to find the time variation of any dynamic variable, to
the quantum case.
First of all, we recall that in quantum mechanics physical
meaning can be attached only to the average values of operators
(position, momentum, and so on). It is the time rate of change of
these average values that we must determine. The average value of
any operator f is given in quantum mechanics by Eq. (7.12), in which
the time t occurs as a parameter. From this equation, we can find
the total derivative of f with respect to time:

= J r (0% * (0#x + J f* (/) <Px+


( 8 . 21 )
+ J r (Of dW d'x-

Substituting for and —p the expressions H^*j and^- —


respectively, we can reduce (8 . 2 1 ) to the form

I f = J r (0 ^ ( 0 ^ +
t i J I(H r (/)) (f * (/)) - r ( 0 f (H <|. (/))] d3x, (8 . 2 2 )
where
H 2 m0 1

Using the theorem for transferring a derivative [Eq. (7.17)] and


keeping in mind that the potential energy is an ordinary function of
coordinates, we readily obtain

J (iir (0)a * (0) (Px= J y (t)m<j>(/) d3x.


Consec|uently, the change of / with time will be given by the equation

In = o, + !, 5 1* (0 (Hr - IH) (, m <fx =

= * TIH. H* (8.23)
AVERAGE VALUES OF OPERATORS - 133

The expression

{H, f} = - '-(H f -fH ) (8.24)

is the generalization of the Poisson bracket (8.19) to the quantum-


mecanical case and is called the quantum Poisson bracket.
Obviously, in the case where ^ = 0 (as a rule an operator f does
not contain the time explicitly), Eq. (8.23) becomes

^ = (H T fT = !- (8.25)

It follows that in this case the time change of f is completely deter­


mined by the quantum Poisson bracket. Furthermore, if the opera­
tor f commutes with the Hamiltonian operator H, the physical
quantity J corresponding to this operator is conserved, as can be
seen from (8.25).
With the help of (8.25) it is easy to prove that the energy of a
particle moving in a time-independent potential field V (r ) is con­
served. The expression {H, H} = — (HH — HH) vanishes in this
case and therefore from (8.25) we have

H = const. (8.26)

On the other hand, according to the time-independent


Schrodinger equation, and therefore, when>^ = '}(0, we have [see
Eq. (7.20)]

H= ^ J<d:,x = 2 C„ En= E:

that is, Eq. (8.26) is nothing but the law of conservation of energy
{E — const) for a particle moving in a time-independent field of
force.

C. EHRENFEST’S THEOREM

We shall now find the quantum analog of the classical equations


of motion (8.17). For this purpose we shall use the quantum Poisson
brackets. Noting that x and p* do not contain the time explicitly,
let us use Eq. (8.25) to determined and ~px, substituting into it either
f = d or f = p*, as the case may be. In the case of f = x , we find

x} = - '1 (Hx— xH), (8.27)


134 NONRELAT1VI S T I C QUANTUM M E CH A N IC S

where
H = 5 S + l'M - <8-28>

Since -v and f lx) commute, Eq. (8.27) can be reduced to the form

* = 2^hW*X- X^ - (8*29>

Adding the quantity (p^p* — Pxxpx) to the right-hand side of this


equation, we have

* = 2^n (P- (P**“ + (P** — P*) • (8'30)

Then, using Eq. (7.7), we obtain

X= &
IHq
. (8.31)

In order to determine the time rate of change of the momentum


we must substitute the momentum operator p* for the operator f
in (8.25). Then, since p^pj — p_ip.t = 0, we find for px

L = ir a = i = - tx • <8*32>

Hence, using (8.31), we obtain

mo* = - ® = / r (*)- (8*33)

Equations (8.31)-(8.33) constitute Ehrenfest’s theorem, according


to which the fundamental equations of classical mechanics can be
generalized to quantum mechanics by replacing the classical
variables by the average values of the corresponding operators.

D. TRANSITION FROM QUANTUM TO CLASSICAL


EQUATIONS OF MOTION

Let us compare the classical equation of motion

ni„x = F (x) (8.34)

w--!r the corresponding quantum-mechanical form (8.33). As was


previously stated, * is the quantity which corresponds to the
classical position coordinate in quantum theory. Accordingly, we
AVERAGE VALUES OF OPERATORS 35

could assume the quantum-mechanical equation to be identical with


the classical equation if we had
m0x — F(x), (8.35)

instead of (8.33). This would be equivalent to replacing x by its


average value x in the classical equation relating the force and the
position. Ehrenfest’s theorem asserts, however, that the equation
of motion for the quantum case contains the average value of the
actual force, that is, F (x). Therefore, in order to make a transition
from quantum equations of motion to c la s s ical equations, we must
first establish the relationship between F(x) andf(x).
Let us represent the force operator T (*) in the form

F(x) = F( x + Ax), (8.36)

where kx = x — x, and expand F (x) in a Taylor series about the


point x — x. Then we obtain

F { x ) = F ( x ) - \ - ( ± x ) F ' ( x ) + ^ F " ( x ) + ... . (8.37)

Taking the average of this expression in accordance with Eq. (7.12)


and considering that (Ax) = (x — x) = 0 , we obtain

F(X) = F ( x ) + ^ - V ( * ) + ... . (8.38)

The quantum-mechanical equation of motion (8.33),therefore,takes


the form

m„l: = F (x) -I- ^ F” (.v). (8.39)

Here the expression — F" {x) is the quantum-mechanical correction


to Newton’s classical equation. Clearly, the criterion which must
be satisfied in transition from quantum equations of motion to
classical equations is the inequality

( ^ < 2 |£ § |. (8.40)

It should be noted, however, that mere satisfaction of this inequal­


ity is still not sufficient to allow us to apply all classical concepts
to the description of the motion of a particle. Indeed, in quantum
mechanics the average value of the kinetic energy T is defined as

T{ P,) (pj- (8.41)


2 m0’
136 N O N RE L -A T IV IS T IC QUANTUM M EC H A N ICS

whereas the classical analog of the quantum-mechanical kinetic


energy should actually be taken as

T (~Px) = <8-42>

Let us now express the quantum-mechanical definition of the kinetic


energy 7'(pJ in terms of its classical analog T(px). For this pur­
pose, we shall use the equation

T(pJ = T(p, + Ap,) = ^ ± ^ 2( (8.43)

where S.px — px — px. Removing the parentheses in (8.43) and con­


sidering that after averaging

^PT= (P* — /3*) = 0 ,

we have
np7) = np*) + — ( W - (8.44)
From this we obtain the condition under which we can make a
transition from the quantum-mechanical expression for the kinetic
energy (8.41) to the classical expression

(Jp~Y < P x= 2 m0r (px). (8.45)

Multiplying (8.45) by (8.40), we obtain the general condition for


the validity of the classical approximation in the microscopic world:

( W •W J * < W (Px) [ P § ) I• (8.46)

If we take into account the uncertainty relation

condition (8.46) becomes

(8.47)

Let us apply this condition to the hydrogen atom, when

£5 6??,
r- ' r' '
AVERAGE VALUES OF OPERATORS 37

Substituting these values into (8.47), we obtain the inequality

3 n2
r
4 m<,el
(8.48)

Ti2
Since -— m0elr = a0l where a0 is the radius of the first Bohr orbit, and
r = n2 a0, we obtain instead of (8.48)

3_
n 4 ’
(8.49)

and therefore, in the lim it of large quantum numbers, the results


of quantum theory approach the classical results.
Problem 8. 1. 1 Determine the wave function of a freely moving electron in the
p representation. Write the normalization condition in the p representation.
Find the average values of the operators for the momentum and energy of a particle.
Solve the problem in the one-dimensional case, and then generalize it to the three-
dimensional case.
Solution. Let us choose the x axis along the direction of motion of the electron. The
wave function of a free electron in the x representation, normalized in terms of 8 (p'0 —pa\
will have the form [see (4.81)]

1 * PoA x
+ Oo, X)
Y~inh 6
To transform to the ^representation, we use Eq. (7.10) in the form

1 C —l Jr*
¥ (Po, p) = - y = J 4(Po, •*■') e n dx\ (8.50)

obtaining

¥ (Po, P) = 6 (P — Po)-
The normalization condition has the same form in both x and prepresentations:

J 4'* (Po. x) <|/ (p0, x) d x =

= ^ ¥* (Po, P) ¥ (Po, p) dp = 8(po — p0).

The average value of the operators should be calculated from the equations

Po + AP Po 4- AP
M= J dp'0<\>*(p'„, x) M i (j>o, x) d x = ¥* (Po, P) M ¥ (Po, P) dp.
S dp‘ l
Po — * P Po — * P

which gives us

E= Po
P=Po,
2 m0 ‘

^ T h e p r o b l e m s in t h i s c h a p t e r r e fe r to C h a p t e r s 6 and 7 a s w ell.
138 NON R E L A T IV IS T IC QUANTUM M E CH A N IC S

In the three-dimensional case, we have


f (Po, p) = l ( p — Po)-
Problem 8.2. Determine the probability of the various values of the momentum of a
particle in the ground state, the particle being in a one-dimensional square well with
infinitely high walls. Verify the normalization in the p presentation.
Solution. Taking the value of the wave function from Chapter 4

and using Eq. (8.50), we have


* - P
1 f r. - 1X x
^ p ) = y m ) * n- r xe dx-
Evaluating the integral and squaring its modulus, we obtain the required probability
distribution

4it/ft* 2pi
I <f(P) I2 (rfW — pH*Y C0S 2f t ’
which satisfies the normalization condition

j \<t(p) \"dp = l.
— CO

In evaluating the last integral we may use the relation


+ oo
cos ap dp sin | a \ b
—O
O
J bs —P2

which should then be differentiated with respect to the parameter b.


b

Problem 8.3. Investigate the motion of a charged particle in a constant and uniform
electric field g.

Solution. This problem is solved most simply in the momentum representation. Since,
according to Eq. (7.1 i), the potential energy in momentum space can be represented in
the form

V= -e% x = - F x = - F M ^ ,

the corresponding Schrodinger equation in momentum space becomes

The solution of this equation is

T (/-. P)
1 J r f a -&■;),
\ r 2 T .h l-'

w h e r e , because of the continuity of the spectrum, the normalization coefficient was found
f r o m the condition for S-function normalization:
CO
^ r* (/•’■P) r U-, /') dp = o (II' —
— fiO
AVERAGE VALUES OF OPERATORS 139

The wave function in the position space can be determined with the help of Eq. (7.9)

'K *)=— ( g $ r *(-w .


V*
where

cos (-------u; ] da

is the Airy function, which is proportional to the Bessel function of order 1/3 (see

Chapter 5), and 5 = (^x + j *%• Examining the asymptotic behavior of the Airy

function

1 c- a/31 £1 3 /2 for i < 0,


2 |$ I1'*
for 5> 0,

it is readily shown that the region of large negative values of jt,_where — Fx>- E, repre­
sents a potential barrier, whereas the region where E > — Fx is quasi-classical,
because

— S = — ^ p d x = -jp ^ Y 2n|o (E + Fx) dx + const =

= £3/s + const.

Problem 8.4. An electron moves in a constant and uniform magnetic field. Find the
time derivative of the average value of the position and momentum of the electron (in
other words, generalize the Ehrenfest theorem for the case of motion in a magnetic field).
Solution, According to Eq. (5.9a), the Hamiltonian of an electron In a magnetic field is

Choosing the direction of the uniform magnetic field to be along the z axis (Hz 0,Hx
= H y = 0), the field can be specified by the vector potential

AX = AZ = 0, Ay = x H z.

In order to determine the time rate of change of the electron’s position, let us use the
quantum-mechanical equation of motion

v = 7f {Hf-rH}’
Substituting H, we readily find

1 / e \ P
v= — ------ A = — . (8.51)
tn0 \ p c / m0
For the time derivative of the x component of the momentum operator, we obtain

T ? “ T <H P« - P' H > = i [ t * - 7 A> ) = f V '-


140 NON R E L A T IVI S T 1C Q U A N T U M M E C H A N IC S

Similarly, it can be shown that

Combining these equations, we have

(8.52)

Equations (8.51) and (8.52) constitute the required result.

Problem 8.5. As we know, the behavior of an electron in a metal (at < 0 ) can be
described with a sufficient degree of accuracy by the following potential energy function
(see Chapter 6):

Determine the coefficient of reflection from the surface of the metal for electrons located
inside the metal (x < 0) in the following cases: (a) E < V0 and (b) E > V0. Show that, even
though in case (a) the electrons do penetrate Into the region (x >. 0), ultimately they return
back into the metal. Construct a graph of the change of the potential energy and of the
wave function of the moving electrons.
Answer.

a) ForE < V0, ^ = 1, even though (x > 0) 0.

Hint. In choosing the solution for x > 0 (outside the metal), only the exponentially
decreasing solution should be retained In case (a), and only the solution corresponding to
a wave traveling along the x axis in case (b).
C hapter 9

Elementary Theory of Radiation

A. SPONTANEOUS AND INDUCED TRANSITIONS

According to classical electrodynamics, an accelerated charge


is a source of electromagnetic radiation. The amount of energy
radiated per unit time is given by the well-known equation 1

(9.1)

where r = w is the acceleration of the particle.


If the source of radiation is a one-dimensional harmonic o scil­
lator
X= a COSu)t, (9.2)

the frequency of the emitted radiation is the same as the mechanical


frequency of vibration of the oscillator, and its intensity is propor­
tional to a1.
In the case where the motion of a charge is governed by a more
complicated periodic function x = f(t) with a period x = ^ , we can
expand the function /(/) in a Fourier series:

(9.2a)

and treat the radiation as if it were generated by a set of oscillators


with frequencies <ak= ko>, where k = \ , 2, 3........ Radiation will be
emitted both at the fundamental frequency (k = 1 ) and at harmonics
kw of the fundamental frequency. The intensity corresponding to
the &th harmonic will be proportional to a*.
Thus, according to classical theory, the radiation of a system
is completely determined by its mechanical properties. Indeed, the

In t h i s c h a p t e r t h e q u a n t u m - m e c h a n i c a l a v e r a g e s w ill b e d i s t i n g u i s h e d from tim e


a v e r a g e s by w r i t i n g t h e l a t t e r w ith t h e s u b s c r i p t “ a v . ” In a c c o r d a n c e w ith th e p r e v i o u s
n o t a t i o n , q u a n t u m - m e c h a n i c a l a v e r a g e s w i l l b e i n d i c a t e d by a bar.
142 NON R E L A T I V I S T I C Q U ANT UM M E C H A N IC S

frequency of the radiation is either equal to or is a multiple of the


mechanical frequency of oscillation of the system , and the intensity
of the corresponding harmonic is proportional to the square of the
amplitude.
In quantum mechanics, the problem of radiation must be ap­
proached in a somewhat different manner. According to quantum
theory, radiation is emitted only when a particle (or a system)
makes a transition from one energy state to a lower energy state
(so-called “ downward” transition).
The first quantum treatment of the problem of radiation was
proposed in 1917 by Einstein. He introduced the coefficients A and
B (now called the Einstein coefficients) to characterize the spontane­
ous transitions and the induced transitions (that is, transitions due
to some external effects) of a system from one energy level to
another; Einstein also obtained an equation relating these two
coefficients.
The basic elements of the quantum theory of radiation are the
following. Suppose one of the electrons of an arbitrary atomic
system is in the excited state n with an energy En. Then there is
a definite probability An„' per unit time of a spontaneous transition
of this electron into a lower energy state n' with an energy £„,.
The transition is accompanied by the em ission of a photon with an
energy ho>— En— E„'. If the number of excited atoms is equal to N n,
the energy radiated per unit time during spontaneous transitions
only can be written as
U^P°n = N nAnn'H'». (9.3)
When the atoms are subjected to the influence of external electro­
magnetic radiation, the latter will cause both upward and downward
induced transitions. The upward transitions
■En will, of course, be associated with the ab­
sorption of photons.
Adopting the notation introduced by Ein­
Bp'n stein, we designate the probabilities of an
induced transition from level n to n’ by Bn„>
Ann' $nn'
and from level n’ to n by Since the
number of induced transitions should be
_____1
■En> proportional to the spectral energy density
F i g . *) 1. D o w n w a r d t m n - Po, of the external radiation, we obtain the
11i o n s ( s p o n l a n e o u s anti following equations for the energy radiated
induced) and upward and absorbed per unit time in induced
t r a n s i t i o n s (indue* eel). transitions:

C = NnBnn.Khu, (9.4)
r ind = N n .Bn.nKhv, (9.5)
where N„■ is the number of atoms in state n'.
ELEMENTARY THEORY OF RADIATION 143

Let us consider the case in which the number of upward and


downward transitions is the same (see Fig. 9.1):
N nA nn' = N n 'PmB n 'n , (9*6)

that is, when a state of thermodynamic equilibrium exist between


the heated atoms and the light radiated by them (black-body radi­
ation), which in turn interacts with the atoms. In this state, the
atoms and the radiated light form a closed system.
Since, in this case, the energy distribution of the electrons is
given by the Maxwell distribution

Nn= Ce-FnlkT, Nn' = Ce-En>/"T,


we obtain
A„n-c-WT+ ? 'P mi-e-E«'kT= ? aBn’ne-En'i". (9.7)

Dividing by the factor e -en/kT and noting that En— En>= fuo, we obtain
An n ’
~Bnn'
Pa, (9.8)
'* n ’« hm/kT

Since the spectral energy distribution of black-body radiation is


completely independent of the specific structure of the atoms or
molecules involved, Eq. (9.8) is essentially the same as Planck’s
formula [see (1.42)]
tioA 1
(9.9)
- 2 c 3 ena,/kT— \

Comparing (9.8) with (9.9) we find

Bnn' = Bain= ^— Ami. (9.10)

It is seen from Eq. (9.10) that the probability coefficients of upward


and downward induced transitions are equal to each other and
proportional to the coefficient of spontaneous transition Ann-. There­
fore, to describe the radiation of atoms or molecules, it is sufficient
to determine only one of these coefficients.

B. CALCULATION OF PROBABILITIES OF SPONTANEOUS


AND INDUCED TRANSITIONS
In quantum mechanics induced transitions are explained in terms
of an interaction between the electrons of an atom and external
electromagnetic radiation. The problem of determining the causes
of spontaneous transitions was left unexplained by the Schrodinger
theory.
144 N O N R ELA TIV ISTIC Q U ANT UM M E C H A N IC S

The answer was obtained only after the' development of a theory


of radiation in which quantization of the electromagnetic field
(second quantization) was used. The general features of the theory
are outlined below.
Electrons interact not only with real photons, but also with
virtual photons (photons which are in an unobservable state) or, as
they are called, vacuum fluctuations of the electromagnetic field
(for further details on vacuum fluctuations, see Chapter 22). This
interaction causes spontaneous transitions. The classical analog
of the interaction between the electrons and the field of virtual
photons is the effect of Planck’s radiation damping on a moving
electron
p ___± V
r rad 3 C3 x >

which represents the self-interaction of the electron with its own


electromagnetic field. Under certain conditions this electromagnetic
field may detach itself from the electron in the form of electro­
magnetic radiation. In the language of quantum electrodynamics
this amounts to a transition of photons from a virtual state into a
real state.
The exact expression for the coefficients A and B can be found
on the basis of quantum electrodynamics and, therefore, problems
of radiation can be completely solved .2
In the present discussion we shall obtain coefficient A by means
of an appropriate generalization of the results of classical radiation
theory to the quantum case. It should be emphasized that this
generalization leads to the same results as the rigorous method of
second quantization.
In our derivation we shall use the correspondence principle to
generalize the classical expression for the radiated energy [Eq.
(9.1)] to the quantum case. First, we replace the classical variable
r by the quantum-mechanical quantity

r = jj (9.11)

In addition, we use an expression for the radiated energy which is


consistent with the quantum theory:

r quant = g n g n 'I iu A (9 .1 2 )

where the coefficients g„ and characterize the occupancy of


states n and n by electrons, since according to the Pauli exclusion

S e e filso C h a p t e r 2 ° , w h e r e t h e c o e f f i c i e n t A i s o b t a i n e d b y t h e m e t h o d s o f q u a n tu m
e le < t rody n a m ic s.
ELEMENTARY THEORY OF R A D IA T IO N 145

principle it is impossible for two electrons to be in the same


quantum state (for more on the Pauli exclusion principle, see
Chapter 24).
Combining Eqs. (9.12) and (9.1) and substituting (9.11) we obtain

&&'A^M' = | £ ( r ) * v (9.13)

Let us note that Eq. (9.13) contains two averages. One is the
quantum-mechanical average, denoted by a bar, and the other is
the time average, denoted by the subscript “ av.”
We shall now assume that the electron has only two possible
states with energies En and En>. Then the wave function can be
written as
if(t)= C ne~ h C„.e <|v. (9.14)

The average (over the quantum-mechanical states) value of the


radius vector is

r = ICn|V„„ + 1Cn. | 2 rnW+ rnn> + (9.15)


where
(9.15a)

The matrix elements

(9.16)

form a certain infinite matrix

r m rol r02 ...


rw r n r n ... (9.17)
(r) = r20 ru rn ...

From (9.16), it follows that this matrix changes into its complex
conjugate when the rows are replaced by columns, and columns
by rows

rnn' = r„'n.

Matrices satisfying this condition are called Hermitian or self-


adjoint m atrices. Let us also emphasize that the matrix elements
(9.16) are independent of time, and therefore substitution of (9.15)
into (9.13) yields
146 NON R E L A T I V I S T 1C Q U A N T U M M E C H A N IC S

Here we have used the fact that the time average of a periodic
function is zero, since

(e+2 i<u/)av e±2iu>l dt - - 0 .

For further analysis of Eq. (9.18), we must introduce an additional


assumption, which can be rigorously justified only on the basis of
quantum electrodynamics. As we already know, quantum mechanics
deals with stationary processes and, therefore, there is no am­
biguity in interpreting the quantity |C„ | 2 = const as the probability
of finding an electron in the state n. When the em ission of radiation
is present, the coefficients C„ change discontinuously and their
physical meaning cannot be simply explained within the usual
formalism of quantum mechanics. We shall, therefore, base our
conclusions on simple physical considerations, which are rigor­
ously .proved only in quantum mechanics.
Let us substitute the initial values for the coefficients C°„ into
Eq. (9.18), bearing in mind that the Pauli exclusion principle allows
transition only in the case when the quantum state n is initially
occupied, while the quantum state n' is empty. Then setting

gngn' = | Cn I2 Ic n. I2 = IC%| 2 ( 1 - | C“I2), (9. 19)

we find that for C„ — 1 and C;i-= 0 the product gngn>= 1 . Hence

Ann- = y £ r |/V * |2, (9-20)

B„„' = Bn.n= ^ - |/v „ |2, (9 . 2 1 )

Wnn' = hwAnn’ = y — | |2. (9.22)

In these equations

If~n'n | 2 = | Xn'n | 2 -f- | gn'n | 2 -(- | Zn'n |2> (9. 23)


where

X n ' n ---- - ^ '^ n ' X t y n d ' X t

and so forth.
'Ihus, the energy eigenvalues can be used to find the frequency
of the radiation and the eigenfunctions to find its intensity. Thus,
al! basic classical radiation properties can be completely general-
i *1 to the quantum case by means of the Schrodinger equation.
f rom the last equation above it is evident that the intensity of
radiation will be dilferent from zero only for those transitions for
ELEMENTARY THEORY OF R A D IA T IO N " 147

which at least one of the matrix elements Xnn'y ynn' and Znn' is non.-
zero. These transitions are called in quantum mechanics the
allowed transitions.
It should be noted that in very many quantum-mechanical
problems it is sufficient to calculate the matrix elements alone and
thus to set up selection rules, that is, to find the changes in quantum
numbers that correspond to allowed transitions. From a knowledge
of the selection rules, one can answer the question of possible
frequencies of radiation. In the language of classical electro­
dynamics, the selection rules correspond to a specification of the
harmonics at which radiation can be emitted by a given system . If
the matrix elements for a given change (difference) in quantum
numbers are equal to zero, there will be no radiation at the corre­
sponding frequencies, and these transitions are said to be forbidden.
Here, in speaking of forbidden transitions, we are restricting the
use of this term to electric dipole transitions. By electric dipole
transitions we simply mean transitions whose probability depends
on matrix elements

In addition to the dipole transitions, there are also cases of quad­


r u p le transitions, multipole transitions of higher orders, and
magnetic dipole transitions. The intensity of these transitions turns
out to be much sm aller than that of the allowed dipole transitions.
As an example, if the intensity of an electric dipole transition is of
the order
2 p2 G>< .
dipole = -J ~ 73

where a is the linear dimension of the atom, then the intensity of


electric quadrupole radiation is of the order 3*

(9.24)

For an atom 10 8 cm and 10~Bcmand, therefore, the intensity


of dipole radiation is 1 0 times greater than that of quadrupole
radiation. Nevertheless, quadrupole radiation plays a very important
part in a number of phenomena. Indeed, if the electric dipole
transition is forbidden, it is still possible that a weak quadrupole
radiation will be emitted, which can be detected with a very sensitive
spectroscope. We note that no dipole radiation occurs in a system
consisting of particles having the same charge to mass ratio. The

3T h i s s u b j e c t i s t r e a t e d more fu lly for t h e c a s e o f a h a r m o n i c o s c i l l a t o r in C h a p t e r 10


[ s e e P r o b l e m (1 0.4 )].
40 N O N RELA TIV I ST1C QUANTUM M E C H A N IC S

electric dipole moment of such a system is proportional to the


coordinate of the center of mass rc-m- [p = e(rt + r2) = 2 e/£#mJ, and
therefore the derivative of the dipole moment with respect to time
vanishes. This is the situation that should hold for gravitational
radiation, since the gravitational charge, or rather the gravitational
m ass, is proportional to the inert mass m0. Therefore, if gravita­
tional radiation does exist at all, it can only be of quadrupole
character. Quadrupole radiation is also of importance in nuclear
physics since the charged particles of the nucleus (protons) have
the same charge and m ass . 4
Problem 9.1. Find the probability of quadrupole radiation in the quantum case as a
generalization of the classical formula by applying the corresponding principle.
Solution. In the classical case the intensity of quadrupole radiation is given by the
equation

1
\V = 180c6 (DabY, (9.25)

where the quadrupole moment is


Dab = e (3xax b — r2i ab) (a, b = 1,2,3).

To generalize (9.25) to the quantum case, it is necessary to consider that, in quantum


theory, radiation occurs as a result of a transition of the system from one quantum state
n to another, t i . Following the procedure similar to the derivation of Eqs. (9.13)-(9.18),
we first replace the classical expression for the quadrupole moment Dab by the matrix
element

(Dn'n)ab~ ^ Vn' Dab'^nd*X.


Next, using Eq. (9.3), which relates the intensity of radiation Wn,„ to the emission
probability A,, we obtain

A nn' 90 c6/t {Dn'n)ab n'n)abi (9.26)

where the frequency of radiation w is given by Eq. (9 .15a).

Problem 9.2. Find the selection rules for dipole and quadrupole radiation for a particle
in an infinitely deep potential well.
Answer. For dipole radiation An must be an odd number, and for quadrupole radia­
tion \u is an even number.
Hint. Using the wave function (4.32), it can be shown that the average value of the x
coordinate is

* = j 'I'n x ^ nd x = - .

I herefore, the matrix elements corresponding to dipole ( / = I) and quadrupole (y = 2)


radiation should be calculated from the equation

xi ’a = V„'(X~x)tyndx.

S. I- W l l c t l r r . Thr Quantum Theory of R adiation, 3 r(| E d . , N e w Y o r k : O x f o r d U n i v e r ­


sity P r e s s . l ' PSI .
150 NON R E L A T I V I S T 1C Q U ANT UM M E CH A N IC S

The solution of this equation has the form

A.' = acos«/, (10.3)

2“ t r k
where co= — = — is the angular frequency and a is the ampli-
tude of oscillation. From Eq. (10.3), it follows, in particular, that
the acceleration
w = x = — aw1cos u>t (10.4)

differs from zero and, consequently, that the oscillation of a charged


particle will be accompanied by radiation, the intensity of which
(that is, the radiant energy) is given by the following equation in
accordance with Eqs. (10.4) and (2.2):

^ c l = 2£ (l-)lv = — . (10.5)

In deriving (10.5), we calculated the average value of c o s f r o m


the equation

— JcosWf==y. (10.6)
6
We shall now express the intensity of radiation U^clin term s of the
total energy E = T -j-V of the harmonic oscillator. From the w ell-
known equations for the potential energy

V (x )= - ^ F(x)dx = ! ! ± ^ = ! ! ! ^ c o s iwt (10.7)


C
l
and kinetic energy
T m0X2 m0 u2 a2 . „ ,, 0 .
‘ — —7)— = ~ 2 — sinV (10.8)

of a harmonic oscillator, we find

E = V(x) + T = const. (10.9)

'Aith this equation, we can eliminate the quantity a'1 from (10.5),
obtaining

Wcl 2e2m2B (10 .10 )

Ihus, on the basis of classical theory one can determine both


the intensity and frequency of the radiation; it is also found that this
THE LINEAR HARMONIC OSCILLATOR 151

frequency is the same as the frequency of mechanical vibrations of


the harmonic oscillator. The energy of the harmonic oscillator,
according to the classical theory, can have any value in a continuous
range from zero to infinity.
Several new features were introduced in the problem of the
harmonic oscillator by the Bohr quantum theory. For example,
according to Bohr’s theory, the energy levels had to be discrete
and could be found from the quantization rule

:\^pxdx — 2-hn, ( 10 . 11 )

where
dT ( 10 . 12 )
p * = w = m»x-
. dx
Let us substitute pxdx = m„Z-^ dt = sinWcf/into Eq. (10.11).
Then, taking into account Eq. (10.9) and integrating over a complete
period, we find
:n/lm, (10.13)

where the quantum number n = 0, 1, 2, 3.........


We showed above that, according to Bohr’s theory, the energy
of a harmonic oscillator can take only discrete values, and radiation
will be emitted only when the oscillator makes a transition from
one energy level to another.
The discovery of a discrete spectrum of energy levels of a
harmonic oscillator played an important part in the theory of
black-body radiation. Planck’s law was first obtained under the
assumption that the harmonic oscillator could radiate and absorb
light only in the form of discrete quanta of energy /uu.

B. EIGENFUNCTIONS AND EIGENVALUES


OF THE ENERGY

In order to determine the behavior of the wave function in the


harmonic oscillator problem, let us first give a graphical repre­
sentation (Fig. 10.1) of the dependence of the potential energy V
on x

/ _ my**
1
V— ~ T ~ '
From the graph, it is seen that inside the potential well, where the
total energy E of the harmonic oscillator is greater than V (E^> V),
the solutions for & will take the form of harmonic functions. Inside
the potential barrier (E <^F), the solutions will contain two parts,
152 N O N R E L A T I V I S T 1C QUANTUM M E CH A N IC S

one exponentially decreasing and the other exponentially increasing


(see Fig. 10.1). It is clear that the solution of the problem reduces
to finding the conditions under which there is no exponentially
increasing solution. Just as in the case of a rectangular potential
well with infinitely high walls (Chapter 4), such levels exist only
at certain discrete values of the energy, which we must determine.

Since the potential energy V of a harmonic oscillator depends


only on the x coordinate, the Schrodinger equation can be written as

, 2 /;i0 f p \ _o (10.14)
dx"- T r- 2 ) r — w-
Setting
2mji n 1 /n„w a . 2E
h- ’ 1 -vj; ft ’ $ — K ~ tuo ’

and introducing a new variable

(10.15)
we obtain
9" + (>*-54)'> = 0 , (10.16)
where
6" = d~‘'y
y dv •
(10.17)

First, let us find the asymptotic behavior of the wave function


;|1 ; ■ 1 ■, that is, when the constant Xis negligible in comparison
w ith;2. Then

6 = o. (10.18)
THE LINEAR HARMONIC OSCILLATOR 153

We shall seek a solution of this equation in the form


(10.19)
Since
$" = (4e*s* + 2e) eEt-2^
we find
( 10 . 20 )

and, consequently,
<]»oo= + C2 e‘^ \ ( 10 . 21 )

Since the wave function must remain finite at S—> ± co , coefficient


C, must be set equal to zero. Coefficient C.2 can be taken to be equal
to unity, since the wave function has not yet been normalized. Thus,
the asymptotic behavior of the wave function <J> is described by the
function
^ = e -W . (1 0 . 2 1 a)
We shall seek a solution of the wave function in the general form
<|>= t^ocu = e- 1 / 2 ' 2 u, (1 0 . 2 2 )
which already takes in account the behavior at infinity. Substituting
(1 0 . 2 2 ) into (10.16) and considering that
u)' = [«" — 2 ;m' + ( ; 3 — 1 ) u] e~l^ \
we obtain the following equations for u:
u" — 2 ?«' + (X— 1 )« = 0 . (10.23)
Let us look for a solution of this equation in the form of a series

«= 2 b^ k- (io -24)
fc=0

Substituting this expression for u into Eq. (10.23) and collecting


terms with the same power of S, we find

2 5* [(ft + 2) (ft + 1) - b„ (2ft + 1 —X)] = 0.


*=o
Equating the coefficients of to zero, we obtain a recursion
formula for the coefficients bk
, _ h (2 6 + l - X ) (10.25)
*+s~ * (k + 2)(k+i y
This formula relates the coefficients bk to bM , and, therefore, the
series (10.24) will consist of even powers (if the minimum subscript
ft is even) or odd powers (if the minimum subscript ft is odd).
54 NONREUO lT I V I S T I C quantum m e c h a n ic s

If the series (10.24) does not terminate at a certain maximum


power, then beginning with , every term is positive and,
consequently, the series diverges for large values of < . This leads
to the second asymptotic solution ^asym ~ e at - — oo , which we
disregarded earlier because it diverges . *2 Therefore, in order for
the boundary conditions to be satisfied (<j>= 0 at ;^ c o ), we must
terminate the series (10.24) at a certain = n. We thus require

6 „ ^ 0 , 6 b+s = 0 . (10.26)

From (10.26) and (10.25), we find

X= 2 n + 1 , (10.27)
and, consequently,

(10.28)

where n can assume any positive integral value, including zero.


These are the only energy values for which the wave function
vanishes at infinity.
Comparing this expression with the one obtained from the Bohr
theory [see (10.13)], we note the appearance of a term called the
zero-point energy
£ 0 = - Urn. (10.29)

Later, we shall show that the existence of the zero-point energy


is related to the uncertainty principle and thus to the wave properties
of particles. The zero-point energy does not affect the frequency
of the radiation, however, since it cancels out in the expression
for the frequency wnn/ — .
Let us now find the wave function of the harmonic oscillator.
The recursion formula (10.25) for the coefficients bk when X= 2n -(- 1
takes the form
I t ( k — I)
bk — bk 2 (n —k + 2 )’

2T h i s f o l l o w s from t h e f ;i ct t h a t a t l a r g e k t h e r a t i o of t h e c o e f f i c i e n t s ( 6 ^ / 6 ^.+2 )fr —od~


h2 i s t h e s a m e a s for t h e s e r i e s e x p a n s i o n o f t h e f u n c t i o n

A2 ■e*.

T ' l 'i ■re f o r e ,

' usym Cl:


. , ‘/ ^ 2
THE LINEAR HARMONIC OSCILLATOR 155

where k < n Setting the coefficient of the highest power


equal to 3
bu n —
= 2 " * (10.30)
we obtain
, _ nrt -2 "(»—1 )
-- * 1| »
b
un -4 — ^
2 n ~4 ,|(”~ 1)(”~ 2)("~3)- and so forth
2! (10.31)

The power series with a finite number of terms obtained for the
function u is called the Hermite polynomial

u = Hn<$ = (2 ( ) * - ^ (2 ; r 2 +
I n(«—l)(n—2 )(n—3) ^g.y, _ 4 | | for odd « (10.32)
6 „for even n

In particular,
tfo(S)=l. /*.(*) = * , //*(;) = 4 ? - 2 ,
//,($) = 8 ?3 — 1 2 '. (iU.dd)
The Hermite polynomials //„(?) can be written in closed form 4

W„(S) = ( - l ) V a-— -. <10*34)

^ T h i s c o e f f i c i e n t c a n a l w a y s be c h o s e n a r b i t r a r i l y , s i n c e t h e n o r m a l i z a t i o n f a c t o r of
the w ave fu nction W i s s till undeterm ined.
a n —^ 2
T o s h o w t h i s , w e i n t r o d u c e t h e f u n c t i o n v = e * , w h ic h s a t i s f i e s t h e e q u a t i o n

v' + 2 £ v = 0 .
D i f f e r e n t i a t i n g t h i s e q u a t i o n n + 1 t i m e s , a n d u s i n g t h e L e i b n i t z form ula

n(n~ l) (n- 2 )
( y z ) (n) y ^ z + ny^ 1 'z ---------- y z + . . . ,
2 !
we o b t a i n

u (n + 2) + 2 f u (n + + 2(n+ l ) v (n) = 0.

M a k in g t h e s u b s t i t u t i o n

y (n) -<f2 W,
w e fin d t h a t t h e f u n c t i o n iv s a t i s f i e s E q. ( 1 0 .3 5 ), a n d t h u s it i s p r o p o r t i o n a l to t h e H e r m i t e
polynom ial

£2 dne~€2
w = es ------------ n •
,ifn
T h e p r o p o r t i o n a l i t y f a c t o r An c a n b e fou n d by e q u a t i n g t h e c o e f f i c i e n t s o f £ 2 n . A s a
r e s u l t , i t i s f o u n d t h a t A n = (—l ) n , from w h i c h w e o b t a i n Eq. ( 1 0 .3 4 ).
156 N O N R ELA TIV ISTIC QUANTUM M E C H A N IC S

From (10.32) it is clear that //„(;) satisfies Eq. (10.23) pro­


vided =
H'n- 2 - H ’nJr 2nHn= 0. (10.35)

According to (10.22) and (10.32), the solution of the Schrodinger


equation for a harmonic oscillator is

6 n= Cne -'^ H n(;), (10.36)

where - is related to the coordinate x by Eq. (10.15). The coef­


ficient Cn can be determined from the normalization condition
-[-CO -fo o

j ytt%dx = x9Ca„§ e~VHn (;) Hn( ? ) = 1. (10.37)

Substituting the closed form (10.34) for one of the polynomials


//„ (-), we obtain
-f-CO
dne- (10.38)
( - ] )nx„c% J / y „ ( ? ) d\n ■ d \= \.

Using the rule for transferring the derivative of one function to


another [see (7.17)] (that is, we integrate by parts n tim es), we
obtain

x0 C - J r ^ / i - = 1 . (10.39)

Noting that from (10.32)


dn
~‘W /y„(;) = 2 "/j! (10.40)
and
-fCO
(10.41)
—CO

we find
____ 1 ____
V ™ y rT ^ ’

that is,

>- 2' {—) X

Y'inn\ v~- H n
-Vo (10.42)
THE LINEAR HARMONIC OSCILLATOR 57

In a sim ilar manner, we can easily prove the orthogonality condition


for the wave functions. To do this, it is enough to represent one of
the Hermite polynomials, specifically, the one with larger tx, in
closed form (10.34). The orthogonality condition also follows from
the general investigation of the Schrodinger equation; it can be
proved that the eigenfunctions corresponding to the different eigen­
values are orthogonal.

- Jt

F i g . 10.2. T h e e n e r g y e i g e n v a l u e s a n d th e
b eh av io r of th e c o rre sp o n d in g e ig e n fu n c tio n s
o f t h e h a r m o n i c o s c i l l a t o r for s m a l l q u a n t u m
n u m b e r s (n = 0, 1, 2). F o r c o m p a r i s o n , t h e
c l a s s i c a l probability d istribution functions
p n a r e i n d i c a t e d by t h e d o t t e d l i n e s .

In the case of a harmonic oscillator, the orthonormality condi­


tion is

(10.43)
—00

For small quantum numbers n = 0, 1 , 2, . . . , when

(10.44)
158 N O N R ELA TIV ISTIC QUANTUM M E CH A N IC S

the probability distribution functions |b„|* (see Fig. 10.2) differ


considerably from the corresponding classical probability functions.
In the classical case the probability of a particle being at a certain
point is proportional to the amount of time the particle spends there,
and, consequently, is inversely pro­
portional to the particle velocity;
therefore, the cla ssica l probability
is proportional to 5 (a2— where
a is the maximum displacement of
a classical oscillator from the equi­
librium position. As we would ex­
pect, it is only for large quantum
numbers that there is a relatively
close agreement, on the average,
between the quantum and classical
probabilities (see Fig. 10.3).

C. ZERO-POINT ENERGY OF THE


HARMONIC OSCILLATOR AND
l ip. 10.3. C o m p a r i s o n o f q u a n tu m
an d c l a s s i c a l r e s u l t s for t h e o s c i l ­
THE UNCERTAINTY PRINCIPLE
la t o r in t h e re g io n o f l a r g e q u an tu m
We have seen that in quantum
n u m b e r s ( h e re n = 10).
mechanics the minimum energy of
the harmonic oscillator is given by Eq. (10.29) and cannot go to
zero, whereas in the classical theory or the Bohr theory, the
minimum energy is equal to zero.
We shall now show that the existence of the zero-point energy
(10.29) in the Schrodinger theory is, as mentioned above, very
closely related to the uncertainty principle (8.11). For the case
of a harmonic oscillator the uncertainty principle becomes

(10.45)

I'his can b e e a s i l y sh o w n on th e b a s i s of t h e f o l lo w in g s i m p l e q u a l i t a t i v e c o n s i d e r ­
a t i o n s . Hie p r o b a b i l i t y of fi n d in g a p a r t i c l e a t a p a r t i c u l a r p o i n t c a n b e r o u g h l y c h a r a c ­
t e r i z e d by th e a b s o l u t e v a l u e o f t h e r e c i p r o c a l o f i t s v e l o c i t y , s i n c e t h e t i m e a p a r t i c l e
s p e n d s in a reg io n w ill b e g r e a t e r in r e g i o n s w h e r e t h e v e l o c i t y i s s m a l l e r t h a n in r e g i o n s
w here it i s g r e a t e r . C o n s e q u e n t l y , Lhe p r o b a b i l i t y o f f i n d in g a p a r t i c l e in a r e g i o n w ith
la r g e r v e l o c i t i e s w ill b e s m a l l e r than t h a t for a r e g i o n with s m a l l e r v e l o c i t i e s . In th e
< a s e of a h a r m o n i c o s c i l l a t o r , w e h a v e from E q. ( 1 0 . 3 ) x / n ~ c o s cot a n d x/coa = —s i n cot.
Taking, th<* s q u a r e of bo th e q u a t i o n s a n d a d d i n g them , we get

(a
and. ‘ h'-P'fore,

1
2
P («2-* 2)
THE LINEAR HARMONIC OSCILLATOR 159

Here we have replaced (Ax)* by x2 and (A/ ; ) ' 2 by p'1. This is justified
by the fact that the wave functions are real and are either even or
odd. Indeed, since the expression <j*xty = xyl is odd, we have

x — ^ y*xtydx = 0 .

Hence

(Ax) 2 = x1— x1= x2.

Similarly, using the boundary conditions at infinity, we find

fi 1 2
P= T ^ S dx = 2 ^ | — 00

that is.

(Ap)* = p*-{p)* = p*.

Substituting the value of p2 from (10.45) into the equation for


the total energy

E = H = _£l-j.!2s^L f (10.46)

we obtain
P I WqM2(X2)
" 8 /;i„(P“) “r 2
(10.47)

From this it is seen that the energy E cannot vanish at any value
of(x2). Indeed, although the second term vanishes for (x2) — 0, the
first term becomes infinite. Conversely, when (x2) = oo, the first
term vanishes and the second becomes infinite.
Thus, the fact that £mindiffers from zero is directly connected
with the uncertainty relation (10.45) or, in other words, with the
fact that it is impossible to calculate exactly the position and
momentum simultaneously.
Let us find that value of (x2) at which Eq. (10.47) has a mini­
mum. Setting the derivative of this function with respect to (x2)
equal to zero, we obtain

m0ti>' ___ft2__
2~
0 ,
8 m0(x2)-

or

(x2) n _ i 3
2 m0co 2 x°'
160 n o n r e l a t iv is t ic quantum m e c h a n ic s

Substituting this into (10.47), we have

(10.48)

Hence, £ min=
min— ^2 , which is exactly the same as the value for £ 0 found
from the wave theory [see (10,29)].
The existence of a finite zero-point energy of the harmonic
oscillator is one of the most characteristic manifestations of the
wave properties of particles. Thus, the experimental verification
of the zero-point vibrations was of great significance for quantum
mechanics. The zero-point energy £ 0 was first observed experi­
mentally in the scattering of x-rays by crystals at low temperatures.
If there were no lattice vibrations at low temperatures ( £ 0 = 0), as
predicted, for example, by the Bohr theory, there would be no inter­
action oetween the x-rays and the crystal lattice, and consequently
no scattering would occur. If, on the other hand, the minimum
energy were different from zero (£„ 0) for T 0, the scattering
cross section at low temperatures should approach a finite limit.
Experiments have confirmed that the second situation corresponds
to the true state of affairs and, therefore, the conclusions of the
Schrodinger wave theory are justified.

D. SELECTION RULES. INTENSITY OF RADIATION

Let us consider the problem of radiation from the harmonic


oscillator on the basis of wave mechanics. For this purpose, as
was indicated in Chapter 9 [see (9.22)], we must calculate the
matrix elements

(10.49)

where is given by Eq. (10.36).


As a preliminary step, we shall derive a recurrence relation
for the Hermite polynomials, which will be necessary in our further
discussion. From the definition (10.32) of the Hermite polynomials
//„ (;), we find

//'„(:) = 2 n (2 ;)"-' (" —I)(« —2 )


II (2 ;)-* + . . . ] = 2 «//„_,(*).

from which it follows that

u; (;) = 2 nH\ _ , (5) = 2 n ■2 (n — 1 ) Ha.t (E). (10.50)


THE LINEAR HARMONIC OSCILLATOR 161

Next, substituting these equations for the derivatives into Eq.


(1 0 . 3 5 ) and replacing n->n'-{- 1 , we obtain a recurrence relation for
the Hermite polynomials

MM?) = / i 7 M , ( E ) + - ^ (?). (10-51)

By means of this formula, the matrix element (10.49) can be reduced


to the form
+ 0° +<»
*»■»= xlCnCn. { I J e-^Hn'+i (?) (?) d? + «' j (?)/U?)d?}.

In terms of the wave functions tp, we obtain

c +°° c +°°
AVn = *0 {y cT2-; 3 V + lM ^ + 'j'c T ^ J tu'-lM *}' (10.52)

Since the functions <J>„are orthonormal, we have

X n ' n — Xo q ~ 1 &n'+l, n ~)“ ( ^ -f- 1) 8 „ > _ | n j-. (10.53)

From this expression it follows that the only nonvanishing matrix


elements are those for which n' = n — 1 or ri = n~f- 1 ; therefore,
the selection rules for the quantum number n are expressed by the
equation
An = n — n' = ± 1, (10.54)

which indicates that only transitions between neighboring levels are


possible.
For the wave function as given by Eq. (10.42), we obtain from
Eq. (10.53)

1
x (10.55)
2
where a 0 is determined from Eq. (10.15). For the frequency of the
radiation, we obtain exactly the same expression as in the classical
theory:

U)7 *, n - 1 En En- 1 ID. (10.56)


%
162 NON R E L A T I V I S T 1C Q U ANT UM M E C H A N IC S

The energy levels and allowed transitions are shown in Fig.


10.4.
Since spontaneous em ission is possible only when transitions
occur from higher to lower energy levels (£„>£„_,), it follows from
(9.22) that the intensity of radiation
of the harmonic o sc illator W = Wn< ,
-Ejh is
■E3/ h
n' n 1 3 m0c3
-E,/h
<1 0 - 5 7 >
-E,/h
■EoA Comparing this equation with Eq.
(1 0 . 1 0 ), which was obtained from the
l ip. 10.4. A llo w ed t r a n s i t i o n s o f th e classical theory, we see that for
harm onic o sc illato r. large quantum numbers (n 1 ), when
£„ £„, both equations yield practic­
ally the same result. Transitions to higher energy levels /!->« + 1
are possible in the case of induced transitions. The occurrence of
spontaneous upward transitions is also possible under the condition
that the energy loss in the harmonic oscillator is compensated by
the simultaneous liberation of a large amount of energy, as for
example, in transitions of atomic electrons (see Chapter 12,
spectra of diatomic molecules).
Problem 10.1. Find the eigenvalues of the harmonic oscillator using the WKB method.
Solution. According to (10.14), the wave function for the harmonic oscillator is

»!'" + ( “ — P2* 2) <!< = o ,

where
_ 2m0E _ /« X
a~ ~W •
According to Eq. (5.75), the eigenvalues are determined from the equation

Y Oj?
— [iaA-2 dx = a +
-1

Evaluating this integral, we find that the energy eigenvalues are the same as in (10.28);
that Is, the zero-point energy is also present.

Problem 10.2. Construct the theory of the harmonic oscillator in the prepresentation
(for the one-dlrnenslonal case). Find the equation of motion, the eigenvalues and the
eigenfunctions.
Solution, Since X" — — h- in the p representation, we can write the Schrodinger
dp 3
equation as

p- muu>"h~ d - \
THE LIN E A R H A R M O N IC O SCILLA TO R 163

that is, transforming the wave equation for the harmonic oscillator from the x representa­
tion to the p representation and introducing the new parameters
, _ 2C _ p
fiu’ 11 pc’
where
Po = V m 0n u ,

we find that the wave equation changes identically into itself

(the prime indicates the derivative with respect to r,). Using the solutions (10.28) and
(10.42), we can write In the p representation

En=fla(n + ^j
and
1 (P y ,
ta(P) = :
V "2” j/npo

It is easily verified that this wave function satisfies the normalization condition

^ I (P) \‘dP = ••

Problem 10.3. Find the eigenfunctions and energy spectrum of an electron (c = — e0< 0)
moving In a constant, uniform magnetic field. Show that, according to the quantum theory,
the "electron gas” must be diamagnetic.
Solution. Let the magnetic field be directed along the z axis (/ft = Hy = 0 J I z =<□%''’).
We can then write for the components of the vector potential A v = xJ%",Ax = — 0.
The motion of an electron is described by the Schrodinger equation (see also Problem 8.4)

ft3 2 , I • p i|/ = 0 .
2 W V* + ‘ m0c dy ~‘ 2m„c2
Since the coordinates y and z do not appear explicitly in this equation, we shall look
for a solution in the form
4, = - L c «'l*ay+*a^) / ( . v ) .

For the fu n ction /^ ), we obtain the equation


^ / , 2 /n, £'
d x - ft2 2 m0c2
( t + 6 )s) / = 0 ,
where

2 /n0 ’ gooj^'
It Is easily seen that this equation has the same form as Eq. (10.14) for the harmonic
oscillator.
Consequently, we can use solutions (10.27) and (10.36) to determine the eigenfunctions
and eigenvalues. We thus find
1

V/l/f gMiy+i'az) e 2
IIn®,
(10.58)
L
164 N O N R ELA TIV ISTIC QUANTUM M E C H A N IC S

2/fl Q* "Jl/|
■7(fi23n(n|) a J ls the normallzation co~
P
[
efficient. 0 = _£Zll_is the frequency of Larmor precession, and
2m0c

For the eigenvalues, we have


go*3%r Wk\
£ 2/7JqC (2 « + 1 ) 2 m0 '

The last term in this equation ls simply the kinetic energy of a free electron moving
along the z axis, and is of no special interest.
The first term
En= ^JY-(2n+I), (10.59)

where (jl0 is the Bohr magneton, corresponds to the additional energy acquired by an
electron, in the magnetic field. This additional term represents the energy of electron
motion in the x y plane, which is perpendicular to the magnetic field.
This conclusion ls in agreement with classical theory, according to which an electron
placed in a magnetic field precesses with the Larmor frequency o in a plane perpen­
dicular to the magnetic field.
In the classical theory, however, the energy of an electron in a magnetic field is
determined entirely by Its unquantlzed kinetic energy. Therefore, according to the clas­
sical theory, an electron gas generally exhibits no diamagnetic properties.
In the quantum theory the energy (10.59) can be interpreted in terms of the appearance
of an additional magnetic moment ]i of an electron, which makes the following contribu­
tion to the energy:
£ m agn= _ (1^ . (10.60)

Comparing (10.60) with (10.59), we find

F.z — — H-o (2rt + 1).

Since the number 2n + I assumes only positive values ( n = 0 , 1, 2, 3, ...), the addi­
tional moment of an electron will be directed along the negative z axis. This naturally
leads to the diamagnetism of free electrons In a metal.
It should be noted that in quantum mechanics, solution (10.58) corresponds to harmonic
vibrations along the z axis along, whereas in classical theory the circular trajectory
means that there are harmonic vibrations along both x and y axes, with a phase difference
of—. The reason for this ls that the energy is Independent of the momentumhk». Con­
sequently, degeneracy occurs and the solution for a given energy has the form

^ n ,h ,= 0 = ^ C u fin k .2 , fcj=0 > (10.61)

where the coefficients Ck are arbitrary amplitudes satisfying the normalization condition

In r-'asslcal theory there is also an indeterminacy, since the center of the circular trajec­
tory Is not uniquely specified.
Ihe general solution (10.61) corresponds to a set of circular trajectories having
different centers located along they axis.
THE LINEAR HARMONIC OSCILLATOR 165

Concluding the above discussion, let us note that the solution (10.61; includes the
harmonic oscillations along both x and y axes. This can be seen by examining the expres­
sion for the energy [Eq. (10.59)] which represents a sum of the energies of two harmonic
oscillators (note that ^oa%^ = o/i)

En — 2fto ^n -f- y ) .

Problem 10.4. Show that the matrix element of the product of two operators M (x)and
N (x), which are independent of quantum numbers, is equal to the sum of the products of
the matrix elements of these operators, that is,

(10.62)
k
Solution. Writing

(MN)„,„ = j (•*) M (x) I (x — x') N (x1) (x’) d x dx'

and using the relation (see Chapter 4)

6 (x — x ' ) = ^ 4'* (•*’) h (•*).


k
together with the fact that the operators M (x) and N (x’)are independent of quantum num­
bers, we readily prove Eq. (10.62).

Problem 10.5. Find the selection rules for quadrupole radiation emitted by the
harmonic oscillator. Find the intensity of spontaneous quadrupole radiation and compare
it with the intensity of dipole radiation. Obtain Eq. (9.24), which relates the intensity
of quadrupole radiation to that of dipole radiation.
Solution. In Problem 9.1 we found that the quadrupole radiation is proportional
to the matrix element, which according the the preceding problem can be written as
CO

(•'-2 )n'n= ^ xn’kxknr

Substituting the values of x n,k from Eq. (10.55), we find the following nonvanishing matrix
elements of the quadrupole radiation:

(^2)n-8>rt= 4 Vn { n - \ ) ;
(**)»n,» = 4 / (n + 2) (n + F), (x % n = x*(n + ± ) . (10.63)

That is, the selection rules for the quadrupole radiation of the oscillator are
An= 0 , :t 2 .
The probability of spontaneous emission (n' = n — 2, An = 2) is calculated from Eq.
(9.26). For large quantum numbers when E — nflrn, we have

V^quad = J 6_ gSo>Sga
15 m^c5 ’

According to (10.57), the intensity of dipole radiation is

\yrtiipole_ 2l e2u>iE (eaf


(i>4
3 m0c8 "c1"*
166 N O N R EL -A T IV IS T IC Q U ANT UM M E C H A N IC S

Using the last two equations, we obtain


V^quad
W dipole

where aa = -----= Is the square of the classical amplitude of oscillations. This relation Is
m0 o>a
in agreement with Eq. (9.24).

Problem 10.6. Show that the center of a wave packet composed of the solutions for
the harmonic oscillator moves according to the laws of classical mechanics.
Show that this wave packet does not spread with time. Obtain the transition to the
quasi-classical case ( n ^ m 1, where 2m -|- lis the number of waves In the wave packet).
Solution. Let us assume, for simplicity, that the wave packet Is composed of 2m-)- 1
eigenfunctions of equal amplitudes

1 ("+'+ t )*
I {x, t)
/2 7 + T
J w *
i— — V

where yn+/ are the eigenfunctions of the harmonic oscillator, and u Is the mechanical
frequency of vibration.
The coordinate x of the center of the wave packet is given by

a -= j i/*(x, t)xty(x, t) d x =
V

= 2m+ 1 2 X n + i - n+ i ' ' e 1)1 = A - t o s a t ,

with

V—I
A= - v i/~2("+ y+i)
2m + 1 L \ 2n + 1 1

where a — 1/ is the classical amplitude of oscillations,


r
It follows from the last two equations that x obeys the classical equation of motion
for the harmonic oscillator

x -)- <d2x = 0.
Evaluating a 2 In a similar manner, we obtain (A.v)2= x 2— x 2 for the mean-square
deviation

(a7 )2= ( C2- " ' j + ( fl 2- * J cos 2at,

where

C* = rs2 = 1 V V(n + j + m > i + j + 2 )


2 2m f I
/= - "+ |

Consequently, (A.v)2oscillates about a certain mean value and, therefore, does not spread
with time.
THE LINEAR HARMONIC OSCILLATOR 167

In Che quasi-classical case, we have


__
A
a(1 _ 1
2 v+ 1 B 2f
40 2 v+l)>
and thus
a2
x ■=» a cos <i><, (Ax)2 const.
27+1
From this it follows that the larger the number of waves, the smaller will be the width
of the packet, and finally, for (2v + 1) >> 1 , the width tends to zero.
C.liapler 1 1

General Theory of Motion of a Particle in


a Centrally Symmetric Field

The problem of the motion of a particle in a central field of


force (a field in which the potential depends on the distance alone,
and not on the angles) is one of the standard problems of quantum
mechanics. This problem provides a basis for the quantum theory
of the rotator, which is of considerable importance in connection
with the spectra of diatomic m olecules, the theory of the hydrogen
atom, the nonrelativistic theory of the deuteron, and so on. It is
worth noting that in a central field of force the dependence of the
wave function on the angles 0 and 9 is completely unrelated to the
specific form of the potential energy. Accordingly, the spherical
harmonics are of general validity; they are applicable to any
centrally symmetric field. The classical analog of the quantum-
mechanical investigation of the angular parts of the wave function
is the derivation of the law of conservation of angular momentum
in a central field of force. This law is also independent of the
specific form of the potential energy.

A. SCHRODINGER’S EQUATION IN SPHERICAL


COORDINATES

The problem of the motion of a particle in a central field of


force
F=F{ r ) ~, (H .l)
is usually solved in the spherical coordinates r, 0 and 9 , which are
related to the Cartesian coordinates (see Fig. ll.l) b y the equations
x = p cos 9 , f / = |i s i i i 9 , z — rcosi), p = rsin{). (1 1 . 2 )
We shall now write the Schrodinger equation in spherical co­
ordinates.
I’jrst, using the general definition of the potential energy V as
a quantity whose negative gradient is equal to the force F, we have
dV = -(F-dr). (11.3)
M O T IO N OF A P A R T I C L E IN A C E N T R A L L Y S Y M M E Y R I C F I E L D 69

For the case of central forces (11.1), we-obtain

dV = — J y ( x d x -j- y d y -|- zd z) = — Fdr, (11.3a)


and hence
r
V{r) = - J F (r)d r, (11.4)
00

where the lower lim it of integration is chosen in accordance with


the convention that V (r) vanishes at infinity.
In particular, if the central forces are due to Coulomb inter­
action

where Ze0 is the nuclear charge (e = — e0isth e charge of an electron


moving around the nucleus), we obtain for the potential energy

Zel (11.5a)
r
co

Now let us find the expression


for the Laplacian y 2 in spherical
coordinates. Using the identity

( 11. 6 )

we shall first find the components


of a vector

B = V if (11.7)
in spherical coordinates.
Bearing in mind that a gradient
expresses the spatial rate of a
change of scalar field in a certain
di>
direction (Bt = V, <]>= ), we
obtain, in accordance with Fig. 11.1,
dty_
Br
W ’
( 11. 8 )
5, d Fig. l l . l . S p h e r i c a l c o o r d i n a t e s . T h e v o lu m e
pdy r sin Sd-p e l e m e n t in s p h e r i c a l c o o r d i n a t e s .

Let us use the definition of divergence

'.j) (BilS)
V •B = liin J l ----------- I ^ £ (BtdSt) d x u
S-.0 d3x d3x (H.9)
170 N ON R E L A T IVI S T 1C Q U A N T U M M E CH A N IC S

where d3.v is the volume element In spherical coordinates


d3x = r 2 sin 9 d r dftdip (11.10)

(Xi stands for the coordinates/-, Sand <f), and dS,-stands for elem entary areas perpendic­
ular to the directions dr, rdft, and pd<f respectively:

dSr = r 2 sin ft dft d<p,


dS8 = r sin 8 dr dcp, (11.11)
dSv = r d r dft.

With the help of Eq. (11.8), we obtain

VB = = r 2 sin 8 drdftdf { Tr { t r’ sin WW*) +


+ 55 ( i S r sin8 d rd * ) rff> + Ff { j W W ? rd rd *) rf<f} > (U ‘ 12)

from which we readily find the expression for the Laplace operator in spherical co­
ordinate:;'
_ i ___an
sin2 ft d^2.] ‘ (11.13)

Setting in Eq. (11.13)

_ L i ( r^ _ v 2 (11.14)
r2 dr\r dr) n
and
1 a d\ , 1 a2 (11.15)
sin ft aft \Sln &a»j + sin2 ft d<p2
we have
v*=v?+ (11.16)

so that the Schrodinger equation (4.8) takes the form

,)* + ^(r)<j/ = 0 , (11.17)


where
k*{r) = 2-^ [ E - V ( r ) 1 (11.18)

is, according to Eq. (11.4), a function of the radius r only.

B. SEPARATION OF VARIABLES. EIGENFUNCTIONS

We shall solve Eq. (11.17) by the method of separation of vari-


ab’ Let us represent the desired function as a product of the
radial and angular parts
* = * ( / ’)■ m ?). (11.19)
M O T I O N O F A P A R T I C L E IN A C E N T R A L L Y S Y M M E T R I C F I E L D 7

Multiplying the original equation by , We obtain

+ = (H.20)

Since the left-hand side depends only on r and the right-hand side
only on the angles ft and <p, this equation can be satisfied only if
both the left- and right-hand sides are separately equal to a con­
stant X, called the separation constant.
We, therefore, obtain the following equations for the radial and
angular parts, respectively:

V t f + (*' — £ ) /? = 0, (11.21)
Vi,,py + X7 = 0. (11.22)

The important point to note is that the angular part of the wave
function does not contain the variable r and is independent of the
specific form of the potential energy V. Consequently, as we men­
tioned at the beginning of this chapter, the angular solution will be
valid for any central force.
Using the method of separation of variables for the angular
part alone, we set
y = e (&) <d (<p), (ii.2 3 )

and thus obtain the following equations for the functions 9 and©:

(11.24)
Vi 0 + (X- l i ^ T ) e = 0’
V£©-}-m3© = 0. (11.25)

Here m5 is the separation constant and we have used the following


notation:

V§: I
sin 9 d% sin (H .2 6 )

V2 . - - (11.27)
f df’

where partial derivatives are replaced by total derivatives, since


each of the functions 0 and © depends only on a single variable.
Thus, we have obtained three equations—(11.21), (11.24) and
(11.25)—for the energy eigenvalues E, and the corresponding eigen­
functions <!>,-. The last equation contains only a single parameter
m'3, whereas the first and second contain two parameters each.
Since the solution of one equation yields the eigenvalues for only
one parameter, we must begin the solution of the entire problem by
172 NON R E L A T I V I S T I C QUA T UM MECHANICS

solving (11.25); then, knowing m.\ w proceed to solve (11.24) and


finally (1 1 . 2 1 ).
To find the normalization constant, we use the relation
CO « 2 it

f 'b*6cPx= j R*Rr°-dr ^ 0 * 0 sin m ^ O* <T'dtp,


^ 0 o o
which shows that each of the functions can be normalized separately:
ou
^ R*Rrad r = l , (11.28)
0
It
^0 * 0 sin WO = 1 , (11.29)
o
2 r.

[ 1 . (11.30)
o
The particular solution for the azimuthal function [see Eq.
(11.25)] can be written in two ways:

® = Ceimf (11.31)

or
(I) = A cos (mcp ?n)- (11.32)

Each of these solutions has a different physical interpretation.


The solution (11.31) represents a wave traveling around the cir­
cumference of a circle and corresponds, for example, to uniform
circular motion of an electron. On the other hand, the solution
(11.32) is associated with standing waves and corresponds, for
example, to oscillations of an electron along an arc. In order for
the function <I> to describe the motion of an electron around the
nucleus, it must have the form of traveling waves (11.31). More­
over, since a solution proportional to e 'mip can be obtained from
the first solution by replacing (m) by (■—m), we can take, without
any loss of generality,

H>= Ceimf, (11.33)

where the quantity m assumes both positive and negative values.


Since the wave function must be unique (see Chapter 4, Section
B), the function (l’(?) must be periodic with a period

<I>(?) = fT>(? + 2ir). (11.34)

If follows that
CIirm
M O T I O N OF A P A R T I C L E IN A C E N T R A L L Y S Y M M E T R I C F I E L D 173

and therefore the quantity m, which is called the magnetic quantum


number, assumes only integral values

m = 0, dr 1, ± 2 , i t 3......... (11.35)

l
From the normalization condition (11.30), we find C It
Vfc'
is readily shown by direct calculation that the functions

<I>tn 1 aim? (11.36)


V^

satisfy the condition of orthonormality

Since we now know the eigenvalues m and the wave function


associated with the azimuthal angle = , we can proceed to solve
Eq. (11.24). Introducing the new variable

x = cos 0 (11.37)

and denoting derivatives with respect to -v by prim es, Eq. (11.24)


becomes

[(1— — — ^-^9 = 0. (11.38)

It can be seen that (11.38) has singular points at x — t 1, that is,


points at which one of the coefficients of 9 becomes infinite. To
eliminate this divergence, we shall look for a solution 9 in the
form
0 = ( 1 — x Y 2u. (11.39)
Substituting (11.39) into (11.38) and dividing all the term s by
( 1 — x*Y/7, we obtain

(1 — A-?) « " _ 2a-(s-L 1)«' -p | \ — s- — s -j- -y 3 7 ^"] M= 0- (11.40)

We eliminate the singularity in the last term by setting

s = dLrn.

Since the fundamental equation for H depends only on nv, the


solutions corresponding to these two values of s both satisfy the
174 NONRELATIVI STIC QUANTUM MECHANICS

same equation, and, consequently, there must be a simple linear


relationship between them:
0 (m) = A 0 (— tri). (11.41)

It is, therefore, sufficient to solve Eq. (11.40) for

s = m ^ 0. (11.42)

With the help of Eq. (11.41), the solution can be automatically


extended to the negative values of in.
Under the condition (11.42), Eq. (11.40) becomes

(1 _**)/," — 2x(m + l)u' + (X— /n(/n + 1))« = 0. (11.43)

Since this equation has no singularities, its solution may be


represented as a polynomial

u= 2 a»xk- (11,44)
*=0
Substitution of this polynomial into Eq. (11.43) gives

^ { k (k — 1 ) a* * * - 2 + ak [X— (k -j- m) (k -j- m + 1 )] xb} = 0 .


*=o
Collecting the terms with the same powers of x, we obtain

^ {(& + 2 ) {k 1 ) ak + 2 + [1 — (k -)- m) (k + m -f- 1 )] ak } xk— 0 .


IcO
This yields a recurrence relation

(k + 2 )(* + 1) Qft+a= — — {k -f- m) (k -j- m -(- 1)] ak, (11.45)

which gives the relationship between the coefficients of the series


(11.44). Since the coefficients aft+2 are expressed in terms of ak and
thus only alternate terms are related, the function u will be either
even or odd depending on whether the main term is even or odd.
We require that the series (11.44) terminate at some maximum
power k q, so that
°7+« = 0, aQ=£ 0.
Then from (11.45) we obtain

x= (<7 + m) (q + m + I), (11.46)


where
<7 = 0, 1, 2, 3, ... (11.47)
MOTION OF A P A R T I C L E IN A C E N T R A L L Y S Y M M E T R I C F I E L D 175

(that is, q is equal to the power at which the series is terminated).


Introducing the orbital angular momentum quantum number I

l = q-\-m, (11.48)

we find that, just like the numbers q and m, this number can
assume only positive integral values (including zero), that is,

1= 0, 1, 2, 3......... (11.49)

and from (11.48) it follows that

l^m . (11.50)

According to (11.48) and (11.46), we have

X= i(Z + 1), (11.51)

and, therefore, Eq. (11.40) can be reduced to the form

(1—x1) u" — 2x(m-\- 1)«' + [/(/ + 1) — m (m + 1)] u = 0, (11.52)

where

u = at_mxl~m-\-ai-m- 2 xl 2 + ... + (11.53)

Instead of determining the relationship between the coefficients aq


and aql r3 by means of the recurrence relation (11.45), let us represent
the solution (11.53) in a closed form. For this, we introduce the
function

» = (** — 1)', (11.54)

satisfying the equation

(1 — * V + 2*it/ = 0, (11.55)
which is easily obtained by taking the first derivative of v with
respect to x. Differentiating Eq. (11.55) with the help of Leibnitz’s
rule [see (10.34a)] and setting

»< = = (11.56)

we obtain the following equation for the function

(1 — x^u'i — 2x(m + l)«l + (/ + /n + 1) ( * - « ) «i = 0. (11.57)


176 NON R E L A T I V I S T 1C Q U A N T U M M E C H A N IC S

We note that this equation is exactly the same as the differential


equation (11.52) for the function u. Consequently, functions u and
iii must be proportional to each other
« = const-«!. (11.58)
Since the normalization coefficient of the function 9 has not yet
been determined, we can set this proportionality constant equal
to - q in order to make the solution (11.58) for m = 0 identical with
the Legendre polynomial
i d‘(x2— \y
Pt(x) (11.59)
2 '/! dx‘
We thus obtain
1 dl+m
U
¥T\ d x
(x2 — 1 )'

from which, with the help of Eq. (11,39), we find the following
expression for the function 9:
$T = C?P?{x). (11.60)

Here P? is an associated Legendre polynomial defined by the


equation

P?{x) = { \ (11.61)

andC!" is the normalization coefficient.


Although (11.61) was obtained for positive values of m, it
can also be extended to include the negative values of m by using
the well-known relation 1

pr w = ( - i r f r f <n -62>

T o p r o v e Eq. (11 .6 2 ), we p u t it in t h e f o l l o w i n g form w i t h t h e h e l p o f (1 1 .6 1 ):

( l ~ \ r n |)' (x2 ~ l ) |m| - A . ---- ( i 2- l ) ' = ( I + | m |)! - ( x 2 - 1) . (11.63)


d x 1* lm ' d x /-lm l

S in c e an d /*/ m m u s t b e l i n e a r l y r e l a t e d to e a c h o t h e r [ s e e (11.41)1, it is s u f f i c i e n t for


u s to s h o w th at th e c o e f f i c i e n t s o f t h e l e a d i n g p o w e r o f * on b o th s i d e s o f E q. (1 1 .6 3 ) are
e q u a l to e a c h o th er, th at is,

</ -M )'* 2lml d l * ' mlx 2! (/ + m|)! 21


dxl * ' m 1 dx1'
Thi is e a s i l y s h o w n s h o w n s i n c e

d kx" xn~k for k ^ n


( («-*)! (1 1 .6 4 )
dxk
1 0 for k >n
M O T IO N O F A P A R T I C L E IN A C E N T R A L L Y S Y M M E T R I C F I E L D 177

From (11.61) and (11.62), it follows that the range of variation of


the quantum number in is

m = 0, ± 1 , dr 2 , . . ± /;

since for |m |> I the solution Pf vanishes . 2


The coefficient C(m in (11.60)canbefoundfrom the normalization
condition
r. 1

f e r e r Sin a ^ 0 ™(x) q ” (x) dx = 1 .


(f -1
Substituting the solution (11.60) and using (11.62), we obtain

(-D m(/+!»)! , Cm "— l *x - - 1 ) ' J dx = 1 .


(2l l\)2 (/—w)! ' l±m V

Transferring the derivative from the second factor in the integrand


to the first factor I -f- m times (that is, expanding the integral by
parts l-\-m tim es), we obtain

i q+w)i (\-x*)l ^ r ( x * - l ) ld x = \.
( 2 '/!)! (/ — ;n)l

Using the equation [see also (11.64)]


„ f 2/1 (n = 21),
dx2lX I0 (n < 21),
and

(l\f 22,+1
\ (1 — x*)l dx
(2 / + 1 )! ’

we find

-./ (2/+1) (l—m)1 (11.65)


V 2 +

2B e c a u s e o f t h e l i n e a r r e l a t i o n s h i p b e t w e e n P/m a n d P[ m, many a u t h o r s p r e s e n t t h e
s o l u t i o n for t h e f u n c t i o n 0 in t h e form

0 = c p j ml (x) .

We s h a l l n o t u s e t h i s form, s i n c e in t h i s c a s e t h e r e c u r r e n c e r e l a t i o n b e t w e e n t h e a s s o ­
c i a t e d L e g e n d r e p o l y n o m i a l s i s more c o m p l i c a t e d t h a n for t h e s o l u t i o n ( 1 1 .6 0 ) ( t h e r e c u r ­
r e n c e r e l a t i o n i s im p o rt a n t in c o n n e c t i o n w ith th e s e l e c t i o n r u l e s a n d t h e s o l u t i o n o f th e
Dirac equation).
178 N O N R ELA TIV ISTIC QUANTUM M E C H A N IC S

Then

9 ? = Y 0 V + ^ m)' K M - (11-99)

For the spherical harmonic Yr(&, <p), which satisfies Eq. (1.22),
the relations (11,23), (11,36) and (11.66) yield

W v) = S?*m= Y ^ i r - {( i ^ - $ p ?(cosb)elm\ (11.67)

The orthonormality condition for the spherical harmonics takes


the form 3
^ ( K(7y y m d2 = 8/i, 8mm% (11.68)

After having obtained the eigenvalues of theparameters m and


X, we,may proceed to the solution of Eq. (11.21) for the radial part
in which there remains only one unknown parameter. The radial
solution, however, can be obtained only if the form of the potential
energy V (r)is specified, and, therefore, we shall leave this question
aside until we come to consider various specific forms of V (r) in
the following chapters.

C. PHYSICAL MEANING OF THE QUANTUM NUMBERS


I AND m. ANGULAR MOMENTUM

We have found that the quantum number I characterizes the


eigenvalue X= /(/ + 1) of the operator ~ v L [see (11.22) and (11.51)],
which is a part of the Hamiltonian
»V + V(r) = - n"v? ft’vS. (11.69)
2 «i0 2 //i0 2m„r: f V(r).

Comparing this Hamiltonian with the cla ssica l Hamiltonian function

" = " f + ^ (0 = ^ + 2^ + V/W (11*70)

To p r o v e t h e o r t h o n o rm a lity c o n d i t i o n ( 1 1 .6 8 ), w e s u b s t i t u t e t h e e x p r e s s i o n ( 1 1 .6 7 ) for
th e s p h e r i c a l h a r m o n i c s i n t o ( 1 1 . 6 8 ). I n t e g r a t i n g o v e r t h e a n g l e 9 , w e c a n r e a d i l y s h o w t h a t

1 /r 27rcHm-m,)? d,. g ,
r / mm
27
%/o
I n t e g r a t i n g the L e g e n d r e p o l y n o m i a l s o v e r the a n g l e 5 , we c a n s e t tti1 ■ m. T h e n w i t h o u t
l o s f g e n e r a l i t y , we r u n l a k e / ' / . T h e c a s e I I w a s c o n s i d e r e d a b o v e in c o n n e c -
te- with t h e d e t e r m i n a t i o n o f t h e n o r m a l i z a t i o n c o e f f i c i e n t . In s i m i l a r f a s h i o n , it c a n b e
r e a d i l y sh o w n t h a t for I ^ I t h e i n t e g r a l ( 1 1 6 8 ) v a n i s h e s a s a r e s u l t o f t r a n s f e r r i n g t h e
d e r i v a t i v e from th e f u n c n o n with s u b s c r i p t I to t h e f u n c t i o n w ith s u b s c r i p t I*.
MO TION OF A P A R T IC LE IN A C E N T R A L L Y SY M M ET R IC F IE L D 179

where pr = m„r, and L = meri^, we see that the operator (—fi\l. T)


corresponds to the square of the angular momentum L1 in the
classical case, and the operator (—Aav?) to the square of the radial
momentum p‘-
Let us investigate these analogs in more detail. As we know
from classical mechanics, the angular momentum L is defined as
L = rxp. (11.71)

If external forces F exert a torque M = r x F, the time rate of


change of L is given by
dL = M, (11.72)
dt
In the case of central forces (/r||/')» no torque M is exerted and,
consequently,
L = const.
In classical mechanics this result is known as the law of conserva­
tion of angular momentum; it appears in Kepler’s theory of plane­
tary motion as the law of conservation of areal velocities (that is,
the law of areas).
To generalize the classical expression for the angular momen­
tum to the quantum case, we replace the classical momentum p in
(11.71) by the momentum operator p = —v- We then obtain
n
r xp = -j-r x v (11.73)

L*-- ypz zpyt


Ly — zpx -vp^j (11.74)
Lz = A^py ypx'
We note that the angular momentum operators Lx, Ly andL* do not
commute with each other. For example, by direct calculation of
the commutation relation between L*andLy, we find

LxLy — LyLx = (ypz — zpy) (zpx — xpz) — (zpx — xpz) (ypz — zpy).

Using the commutation relation between the momenta and the


corresponding coordinates [see (7.7) and (7.8)], we find

LXLy — LyL* = — ih (ypx — xpy) = ihhz. (11.75)


Similarly, it can be shown that
LyL, LzLy — IfiLx,
LJ.Li = iTiLy. (11.76)
180 NONRELATIVISTIC QUANTUM MECHANICS

To express the square of the angular momentum operator

L*= Li + m - L J (11.77)

in spherical coordinates, we must first determine the components


Lv, Lv and L, in these coordinates. Using the relations (1 1 . 2 ) between
Cartesian and spherical coordinates, we have

cty__ d'\i dx , ctjj dy , dty dz ___


~58 dx 58 ' dy (38 ' dz (38
= r cos &cos cp -j- r cos &sin cp — r sin & =
x z (3'} y z d(p <3^
p dx 1 T & — P dz (11.78)
dfy ___(3'|j dx , dty dy , <3^ dz __
<3<p dx d f ' dy d<? ' <3z <3(p
= — rsin&sincp -^ + r sin &cost? $ = — y j x + x ^ - (11.79)

Multiplying Eq. (11.78) by y and Eq. (11.79) by^— -rj, adding these
products, and remembering that Y = xi -\-y1, we obtain the relation

2 S - x S = c o s ? S - sin?cot&<|- (11-80)

Now let us multiply Eq. (11.78) by (— y j and Eq. (11.79) by ^f-j.


Then, proceeding in the same way as before, we obtain

y dd - 2 dy = - {sin 7 58 + cos ?cotft Hr}- (n -81)

Using Eqs. (11.79) and Eqs. (11.74), we find

= — 7 {sin ? cos (pcot&^j, (11.82)

L>= t {cos 'f’S - sin!?co ta ^}> (11.83)


= (H.84)

Introducing the variable x = cos D (which should not be confused with


the Cartesian coordinate x), Eqs. (11.82) and (11.83) can be written
in the form

± iL y = flex i f (11.85)
Y 1— x 1 df '
MO TION OF A PA R T IC LE IN A C E N T R A L L Y SY M M ETRIC F IE L D 161

To determine the effect of these operators on the spherical


harmonics, let us take advantage of the fact that a particular
spherical harmonic can be represented either as (11.67) or as

vr=(- ir "l/W ISli prm(cos (11*86)


Operating directly with Lz on the spherical harmonic, we find

LzYT = tlmYT. (11.87)

From this it follows that the quantum number m characterizes the


2 component of the angular momentum.
To determine the effect of the operator L^-t-iLy on the spherical
harmonic YT , we use the expression (11.67), and for the effect of
the operator Lx— (Lv we use the equivalent expression (11.86). Then
from the equation

7 1 = » r, + >/'ITrT’ s ) e" ,( l
1+m

— ei>(m+l)(l 2

it follows that

(L, ± lLy) YT = - h V{l + 1 ± m) (l + m) Y?±x- (11.88)

Equations (11.87) and (11.88) yield

L'Y? = [y (L * + * L v) — *L y) + i (Lj. — i L ,) X

x (L, + i Ly) + l;] Yf = - h \ l ,Y? =


= hH(l + \)YT■ (1 1 . 8 8 a)

The last equation shows that YT is an eigenfunction of the operators


L2 and L2. This follows from the fact that the operators L* and L4
commute not only with each other, but also with the Hamiltonian H.
Since the operators Lx and L„ do not commute with L2, it is impos­
sible to find a wave function that would be a simultaneous eigen­
function of the operator Lz and the operators Lx or Lr This does
not mean, however, that the direction of the z axis is a preferred
direction, since it can be chosen arbitrarily.
The spherical harmonic can be written in such a manner that it
will be an eigenfunction of the operators Lx and L4. In this case,
it will no longer be an eigenfunction of the operator (see Prob­
lem 1 2 . 2 ).
182 NON RE L A T I V I S T 1C Q U A N T U M M E C H A N IC S

D. ANALYSIS OF THE RESULTS

Quantum-mechanical results are generally analyzed either by


finding their classical analogs or by comparison with the results
of Bohr’s sem i-classical theory, which has a simple physical
interpretation. To apply Bohr’s theory to the motion of a central
field, we start from the classical law of conservation of angular
momentum; we then conclude that the motion takes place in a single
plane and that the angular momentum vector (which is perpendicular
to this plane) has the magnitude

k = p^= ^ = m0 r!<(>= const. (11.89)

Applying the quantization rules, we find the discrete values that the
angular momentum can assume

cj) pv d<p— 2nLB= 2 itftnr


Hence

L‘ti=n *h \ (11.90)

where

nv= 1, 2, 3........ (11.91)

If the z axis is not perpendicular to the plane of the orbit, then


the Bohr theory allows us to quantize the projection of the angular
momentum vector on the z axis. Tnis is known as space quantiza­
tion. Then
{LB)z = hnr (11.92)
where

— —'L + 1 .......... 0 ........nf — 1. nr (11.93)


It follows from this that the angle u between the direction of the
angular momentum L and the z axis is given by the equation

COS <7. = — ; (11.94)


n-t
that is, it can assume only certain discrete values.
Space quantization is illustrated in Fig. 11.2, which shows that
~'71? corresponds to the case when L is parallel to the z axis
(fig . 1 1 . 2 a), where tii)i= —n9 corresponds to the case of anti­
parallel L (iig . 11.2b). Finally, for /i+= 0, the vectors are mutually
MO TION OF A PA R T IC LE IN A C E N T R A L L Y ^ S Y M M E T R I C FIE L D 183

f
perpendicular (Fig. 11.2c). It is obvious that space quantization has
a meaning only when there is some preferred direction in space, for
example, the direction of the magnetic field intensity vector . 4 If
there is no preferred direction, the orientation of the z axis may
be taken as perpendicular to the plane of the orbit.

F i g . 1 1.2. S p a c e q u a n t i z a t i o n a c c o r d i n g to the Bohr


t h e o r y ( fo r L = 1; in u n i t s o f IT).

Now let us compare the quantum-mechanical results with those


of the Bohr theory for the square and the z components of the
angular momenta:

/ = o, 1 , 2 , 3........ n?= 1 , 2 , 3, 4.......


Lz = ftm, { L $ 2 = ?inv
— < n,,.

It is seen that Lqm is zero when 1= 0, whereas Lg can never be


zero. This means that the state with 1 = 0 has no classical analog.
It follows that the angular momentum of an atom in the lowest
state is equal to zero, contrary to the results of the Bohr theory.
The experimental data from atomic spectroscopy fully confirm
this quantum-mechanical result.

4It i s , o f c o u r s e , u n d e r s t o o d t h a t In th e p r e s e n c e o f a m a g n e t i c f i e l d t h e c e n t r a l
sym m etry i s d i s t u r b e d .
184 N O N R EL -A T IV IST IC Q U ANT UM M E C H A N IC S

In the Bohr theory the direction of the z axis can be taken to


coincide with that of the orbital angular momentum. In this case
nv = n .r and,' therefore,

(H.95)
In the wave theory this case corresponds to m = l, when

(n .9 6 )

whereas

Lqm= W + nn = Ljmax + m. (n .9 7 )
The appearance of the additional orbital angular momentum Kll is
related to the noncommutativity of the angular momentum opera­
tors L*, Ly , and L2, as a result of which the angular momentum
components cannot have simultaneous definite values. Therefore,
when I'z = k-smix=fil, the components Lx and Ly do not vanish but
have certain minimum values satisfying the relation
+ (‘^ Jmin4~(A^>’)mln. (11.98)
The minimum value of (ALxf and (ALyf may be obtained with the
help of the uncertainty principle [see (8.13)]:

(^xlrnln^^yJmln” "4 "ILxLy LyL^ 2 = K2LZmax = W 2. (11.99)

Since the problem is symmetric with respect to the x and y axes,


we may set (A^)smin=(ALy)^ln. Hence, we obtain

(H . 1 0 0 )

and the sum of (ALx)9mln and (ALv)amin is exactly equal to the addi­
tional angular momentum hH.. As a result we arrive at Eq. (11.97).
Thus the nature of this additional term is the same as that of the
zero-point energy of the harmonic oscillator. Both are related
to the uncertainty principle. For large values of the orbital angular
momentum quantum number I , we can neglect the term tl2l in (11.96)
in comparison with h'H2, so that in fact we have the Bohr sem i-
classical solution.
Chapter 12

The Rotator

Spherical harmonics, which are the eigenfunctions of the square


of the angular momentum, have their main application in the
quantum theory of the rotator, that is, in the quantum-mechanical
description of the free motion of a point over a sphere. The
results of the theory of the rotator can be used directly in connec­
tion with the spectra of diatomic m olecules. Since, however, the
angular part of the wave function in a central field is also described
by spherical harmonics, many predictions from the theory of the
rotator (for instance, the angular dependence of the wave function
<1>and the selection rules for the quantum numbers I and m) remain
unchanged in the theory of ap articlein a central field (for example,
a particle in a Coulomb field in the problem of the hydrogen atom).

A. EIGENFUNCTIONS OF THE ROTATOR

We shall first write the basic results of the quantization of the


rotator according to Bohr’s theory. These will be used as a
starting point in our further discussion.
Suppose a point is moving over a sphere of radius /- = a = const.
Let the origin of the coordinate system be at the center of the
sphere. The potential energy V (r) is then
V (r) = V (a) = const.
Since the reference level of the potential energy can be specified
in any desired manner by defining its value at some point as zero,
we set
V (a) — 0. (12.1a)
The total energy of the rotator is then equal to its kinetic energy
E==T==Jwl?_' (12.2)
The generalized momentum pf, which here has the meaning of the
angular momentum, is found to be
106 N O N R ELA TIV ISTIC QUANTUM M E CH A N IC S

Using Bohr’s quantization rule, we obtain

Py — (12.4)

and, consequently,
nW
Enf — -1— (12.5)
2J ’

where J — mad* is the moment of inertia.


The quantum-mechanical theory of the rotator is a special
case of the problem of motion of a point in a central field of
force. Consequently, we shall use Eq. (1.21) to determine the
radial function R(r):

V ? R ( r ) ^ [ ^ - -l ^ - ] R ( r ) = 0. (12.6)

We have here set the potential energy equal to zero and substituted
a = / (Z—
1—1) in accordance with Eq. (11.51). Since r = a = const for
the rotator, function R (r) = R (a) = const, that is, ^2
rR(a) = 0. The
energy E, is now easily found from Eq. (11.51)

«»/(/ + i) _ nH(i+ i) (12.7)


2 m 0a 2 2J

Comparing this equation with Eq. (12.5), we see that in the Bohr
theory £^wz;, whereas in quantum mechanics E ~ I ((-)- 1). As has
already been mentioned in Chapter 11 (Section D),this difference is
due to the noncommutativity of the components of Lx, Ly and Lz of
the angular momentum operator; it is one of the characteristic
features of quantum mechanics. Both equations become identical
only for large quantum numbers, that is, when /*;>/.
According to Eq. (12.7), the energy of the rotator depends
only on the orbital angular momentum quantum number I. The
magnetic quantum number m, which characterizes the projection
of the angular momentum L on the z axis (and, consequently, the
orientation of the angular momentum in space), does not appear in
the expression for £,. The eigenfunctions Yf corresponding to the
eigenvalue E, [sec (11.67)] do, however, depend on m. Since m can
vary from / to / [see (11.50)], each energy eigenvalue Et will
have 2/ ; ] corresponding mutually orthogonal eigenfunctions; they
describe the state of the rotator and differ only in the orientation
of ;heir angular momentum L relative to the z axis. In this case
tin1 energy level / ; is said to be (2/ -]- l)-fold degenerate.
in general, a state of a system (or a given level) is said to be
.‘.- .'old degenerate if N linearly independent eigenfunctions corre­
spond to the given energy eigenvalue.
THE ROTATOR 187

The physical explanation for the degeneracy of the energy levels


of the rotator is that the rotator forms a centrally symmetric
system, and consequently all directions passing theough the origin
of coordinates are equivalent. From these considerations it fol­
lows that degeneracy will occur in any centrally symmetric
system.
If there exists a preferred direction, for example, one deter­
mined by the direction of a magnetic field, the central symmetry is
disturbed and the possible directions of the angular momentum L
are no longer equivalent. As a result, the degree of degeneracy is
either reduced, or the degeneracy can be completely removed.
In spectroscopic notation the energy levels are called terms;
for example, the level corresponding to I — 0 is called the s term
and the level corresponding to / = 1, the p term. For the d term,
1 — 2; for the f term, / = 3; for the g term, 1= 4; and so on.
Correspondingly, the rotator is said to be in the s state when / = 0,
in the p state when / = 1, and so on.
Let us consider in more detail the s and p states of the rotator.
Since l = m = 0, in the s state, it follows from Eq. (11.67) that the
eigenfunction yj corresponding to the energy eigenvalue E0= 0 is

<12-8)

and the probability density | ygl2 is given by

|y
» si2=
ui 4J-
^ . (1 2 . 9 )

In the p state 1= 1, and the quantum number m can have any of


the three values — 1, 0 and -p i. Consequently, the energy eigen-
ft2
value Ei = -j- is associated with the three eigenfunctions:

r,“ = — sin», (12.10)

y; = j / | T c o s p (I2 .li)

Y[ = j / ~ ^ ei? sin d. (12.12)

The corresponding probability densities are

| yr11*= 1^1* = -^-sin* ft, (12.13)

| Y\ |2= -i-cos2&. (12.14)


188 N O N R ELA TIV ISTIC Q U ANT UM M E C H A N IC S

The probability distribution functions (12.9), (12.13) and (12.14)


are plotted in Fig. 12.1; they are shown only in the zy plane because
! Vj» | does not depend on the angle &. To obtain a complete picture,
it is necessary to rotate the graph about the z axis.

F ig. 12.1. P r o b a b i l i t y d e n s i t y d i s t r i b u t i o n f u n c t i o n s for t h e


rotator.

It can be seen from Eq. (12.9) and Fig. 12.1a that, for a rotator
in the s state, the angle?, which gives the direction of the angular
momentum L relative to the z axis, is arbitrary. This was to be
expected since the angular momentum L2= hH(l-\-1) is equal to zero
in this case. A material particle at rest has an equal probability
of being at any point on the spherical surface of radius a. In other
words, all positions of the rotator are possible. There is no cla s­
sical analog of this state.
From Eq. (12.13) and Fig. 12.1b it follows that in the p state,
when / = 1 and m = J 1 , the most probable of all the possible orbits
of the rotator is the one located in the xy plane. The states with
m— \ and ni — -1 will have different directions of rotation: for
m— 1 the rotator will rotate clockwise (the angular momentum L is
parallel to the z axis); for m = — 1 it will rotate counterclockwise
(the angular momentum L is antiparallel to the z axis). When
/ = ] and m = 0, the most probable orbit of the rotator lie s in a
plane passing through the z axis [see Eq. (12.14) and Fig. 12.1c].
In this case, the orientation of the angular momentum is perpen­
dicular to the z axis.
It is worth mentioning that a sim ilar analysis of the angular
part of the wave function applies to all system s characterized by
central symmetry.
THE ROTATOR 09

B. SELECTION RULES

As has already been shown, the selection rules indicating the


changes of the quantum numbers that correspond to allowed transi­
tions can be expressed in terms of the matrix elements

( r ) W = ?,&?')• rY?d<2. (12-15>

If the matrix element vanishes for some particular change of the


quantum numbers, the corresponding transition is forbidden (there
will be no radiation). Once we know the selection rules, we can
immediately find both the frequency and the intensity of the radia­
tion [see Eq. (9.22)].
Let us replace the coordinates x, y and z (that is, replace r) in
Eq. (12.15) by the following new variables:

z = a cos fr. (12.16)

£= x -|-iy = a sinde'L (12.17)

fj = x — /t/ = a sin 9e~,<p. (12.18)

From the physical point of view, this is equivalent to resolving


the motion of the rotator into three parts—an oscillation along the
z axis described by the z component; a clockwise rotation in the
xy plane (the ; component); and a counterclockwise rotation in the
xy plane (the n component). In combination, these three components
completely describe the motion of a point over the surface of a
sphere.
In terms of the new variables the determination of the selection
rules reduces to a calculation of the matrix elements:

(z)j^n' = (j) (Y'p')* cos 9 V™ dQ, (12.19)

(O j^ ' = (j) (Yf)* sin V 'f K™ dQ, (12.20)

Wim = (Yf) * sin 0 e-,'T Y[ndQ, (12.21)

where, for the sake of sim plicity, we have set a = 1.


190 N O N R ELA TIV ISTIC Q U ANT UM M E C H A N IC S

Using the recurrence relations for spherical harmonics'

( 12 . 22 )
cos&Kf = AY?+l + BY?_lt
sin = A± Y? ± + B±Y?±}, (12.23)

together with the orthonormality condition (11.68), we find

(zfim '= const 8m-, mbf, i± I, (12.24)


( ^ = const Bm\ m+ A'. <±i, (12.25)
h)im = const (12.26)

Therefore, we obtain the following selection rules:


(a) for vibration along the z axis

Am= m — m' = 0, A/ = Z— Z'= ± l ; (12.27)

(b) for clockwise rotation

Am= — 1, AZ= ± 1 ; (12.28)

(c) for counterclockwise rotation

Am= A/=±l. (12.29)


We have just shown that the only allowed transitions are those
for which the changes of the magnetic quantum number m and the
orbital quantum number Z are
Am= 0, ± 1, (12.30)
A/ = iL 1. (12.31)

1T h e c o e f f i c i e n t s A a n d B c a n b e fou n d in a fa i rl y s i m p l e w ay . We s u b s t i t u t e th e
e x p a n s i o n (1 1 .6 7 ) in to Eq. ( 1 2 .2 2 ), s e t t i n g
m
pm (2l y __ (1 _T 2 ) 2 j.r '~ m x i-m-2 ,
1 2‘I'O-my. } 2(21-1)
T h en , d i v i d i n g a ll t h e te r m s by r?,m<p(1 —x 2)m a nd e q u a tin g the c o e f f ic ie n ts of x * m+I
a n d the co effic i e n t s o f x l r n ” 1 on th e le f t- a n d r i g h t - h a n d s i d e s ( n o t h i n g f u r t h e r i s o b t a i n e d
by e q u a t i n g th e c o e f f i c i e n t s of t h e r e m a i n i n g p o w e r s o f x), w e fin d

(J f 1 - m ) ( m t m) J V*
A (I, m)
V (21 t 1) (2/ I 3)
li (I, m)
T (2i + l ) ( 2/ - l )
( 1 2 . 2 2 a)

S imilarly , we find

A ( (/, m) + J iilig l
T ( 2 / + 1) (2/ h 3)
(12.23a)
H±(/,m) f
f (2/( 1)(2/-1)
THE ROTATOR 191

We note that these selection rules for the quantum numbers m and
I will hold for any centrally symmetric system including, in
particular, the hydrogen atom.
From the selection rules, we can find the possible emission
(or absorption) frequencies of the rotator:

0>ir = 2 ™w = -E~ ^ . (12.32)

Substituting the expression for the energy £, [see Eq. (12.7)]


and considering that the moment of inertia of the rotator does not
change in this case, we can reduce Eq. (12.32) to the form

0 ,« '= = -5 7 [/(J+ ! ) _ / ' ( / ' + ! ) ] . (12.33)


From Eqs. (12.31) and (12.33), we obtain

U = (1.2.34)

u>(,;_).!= — y ( J + l ) , (12.35)
where the frequency corresponds to a transition from a higher
energy level to a lower one (a downward transition) and ^,;+i to an
upward transition.

C. SPECTRA OF DIATOMIC MOLECULES

There are three main types of spectra—the continuous spectrum


of radiation emitted by a heated body (for example, black-botfy
radiation, with a spectral distribution described by Planck’s
formula); line spectra (or atomic
spectra), caused by transitions of
atomic electrons between energy
levels (for example, the Balmer
series for the hydrogen atom); and
finally, band spectra character­
izing molecular radiation. A band
spectrum consists of bright bands
with a sharp edge on the low -fre­
quency side and a diffuse boundary F i g . 12.2. D ia g r a m o f a d i a t o m i c
on the high-frequency side. Only m olecule.
a high-resolution spectrograph can
show that the band actually consists of a series of individual
lines.
As we shall see below, band spectra are directly related to
the rotational motion of m olecules.
Let us consider a molecule consisting of two atoms with m asses
mi and ni.2 separated by a constant distance r (see Fig. 12.2). An
192 N ON R E l_ATIV I S T IC QUANTUM M E C H A N IC S

example of such a molecule is provided, in a first approximation,


by the diatomic HC1 molecule. It is well known that in the case of
two or more particles, the center of m ass moves as a single particle
whose mass is equal to the sum of the m asses of all the particles:

msum = -j m i- (12.36)

The relative motion of the particles ischaracterizedby the reduced


mass wired, the reciprocal of which is equal to the sum of the
reciprocals of the m asses of all the particles:

— = y — (12.37)
mred " ,nl

To prove this, let us write the Lagrangian if of a system consisting,


for example, of two mutually interacting particles with m asses
and nit

X = ~ y l + -^ r1 “ V (*■; - *i). (12.38)

where x, and x.2 are the coordinates of the first and second particles,
respectively, and x.2— x, is the distance between them. Introducing
the relative coordinate x = x2 — xt and the coordinate of the center
of mass

„ _ mtXj -\- m,x3 (12.39)


c*m« mi + "h

we obtain

if : fllsum-v c.m.
- ~ V (x)

from which, using the Lagrange equation

d OX d% n
dt dxi dx[ U’

we find that the motion of the center of mass is characterized by


the total mass
^sum •^c.m.=: const, (12.40)

and the relative motion of a particle by the reduced mass

(12.41)
THE ROTATOR 93

If the center of mass is at rest (*c.m=°), the coordinates x, and


are related to the relative coordinate x by the equations
tll>X . Ul [X
X i — — •------:------- . A-2 — ------ j------ •

Accordingly, the moment of inertia of a diatomic molecule is

J = tnKx\ -f- m.2xl = mred*' (12.42)

which is sim ilar to the expression for the moment of inertia of a


single material particle whose mass is equal to the reduced mass
and whose coordinate is equal to the relative coordinate. Therefore,
in all the results obtained for the rotator, we must substitute Eq.
(12.42) for the moment of inertia, setting x = u.
If the radiation is caused only by rotational transitions, it
follows from Eq. (12.34) that its frequency is

w/, i - 1== 2Bl, (12.43)


where
R -n n (12.44)
2J ^,nre d a~ '

From this it is seen that rotational spectra (molecular spectra


that result from transitions between rotational levels) consist of
sets of equally spaced lines (see
Fig. 12.3). The rotational spectrum,
however, lies in the far infrared
region (radiation wavelengths of the u>4 } - 4 co w

order of 100-300 n) and its investi­


gation is rather difficult. Absorption
to

lines of this type have been discov­


ii
.e
§

ered, for example, in the spectra of


HC1 molecules. A measurement of Cl>2i ~ 2 tO
the spacing between the lines enables
0
us to determinethe moment of inertia
of the molecule.
In addition to pure rotational spec­
tra, there are vibrational-rotational (‘J/O^2t 3Z^43
spectra, which result from the in­
ternal vibrations of a molecule in F i g . 12.3. S p e c tr u m o f a r o tato r.
conjunction with its rotation. These
spectra are not situated as far in the infrared as the rotational
spectra and are more easily studied.
Let us consider in general form the theory of a diatomic mole­
cule with varying interatomic distance. This diatomic molecule
represents an oscillating rotator. Without considering in detail
the atomic interactions in the molecule, and using only simple
194 N O N R ELA TIV ISTIC QUANTUM M E C H A N IC S

qualitative arguments, let us find the general form of the potential


energy curve V (r).
First, we must set V (r-*- 0) -> oo, since the atoms cannot be
arbitrarily close to one another. Second, the interaction between
the atoms must become negligibly small as r-+ oo, and hence
V (r -> co) -*■0. Moreover, since a molecule is a stable system, the
potential energy V must be negative at a certain finite interatomic
distance r = a, where it has a minimum. Otherwise, if 1 /^ 0 the
molecule would rapidly dissociate (see Chapter 27 for a more
detailed account of the bonding energy of atoms in m olecules). The
general form of the dependence of the potential energy of atoms in
a molecule on the interatomic distance is illustrated in Fig. 12.4.

V(r)

F i g . 12.4. P o t e n t i a l e n e r g y d i a g r a m for a d i a t o m i c
m olecule.

If the departures x = r —a of the molecule from the equilibrium


position (r — a ) are relatively small (x<^a), the potential energy
V(r) can be expanded in a series in the neighborhood of the equi­
librium point r — a
V (r) — V (a x) = V (a) 4- xV (a) + - V" (a) -f ... (12.45)

Keeping only the first three terms of this expansion and noting
that the function V has a minimum at the point r = a [F'(a) = 0and
F"(a)>0], Eq. (12,45) can be reduced to the form

1/ / r\ i ^ ^X*
2 (12.46)
THE ROTATOR 195

Here V"(a) = mTed(oa and V (a) = —D are the elastic constant


and the dissociation energy of the molecule, respectively.2
To find the energy levels of the molecule (and thus its energy
spectrum), we take the Schrodinger equation (11.21) for the radial
part of the wave function, since in our approximation the potential
energy (12.46) p ossesses a spherical symmetry.
Since we are interested only in the relative motion of the atoms,
we replace the mass m by mred in Eq. (11.21). The resulting equa­
tion is

V fR + d(E - V (/•)) - R = 0. (12.47)

Noting that
„ .n d‘R I 2 dR _ I d*(rR)
v ~ </r3 i~ r dr ~~ r dr2 ’
(12.48)

and introducing the function

rR = u (12.49)

we obtain after substituting (12.46) into (12.47)

S + ^ + D- - 5 5 ^ 1 " = »• (lM 9a)

Since x<^a, we may assume in this equation that —= , ,* ^


Then setting r (<2+'v) a

E + D — Btil(l + 1) = E', (12.50)

where 5 = ^ an^ J — mreda2» we reduce Eq. (12.49a) to the form

U" + ?|red ^ _ mred“- ^ ) « = 0 (1 2 .5 1 )

This equation is exactly the same as Eq. (10.14) for the harmonic
oscillator, and, therefore,

E’ = tu»(k-\- (12.52)

where k = 0, 1, 2, 3,...

2
T h e d i s s o c i a t i o n e n e r g y (~D) i s d e f i n e d a s t h e work r e q u i r e d to s p l i t t h e m o l e c u l e
in to a t o m s ( n e g l e c t i n g t h e v i b r a t i o n a l e n e r g y ) . T h i s e n e r g y i s u s u a l l y o f t h e o r d e r o f
se v e ra l electron volts.
196 NON R E L A T I V I S T 1C Q U ANT UM M E CH A N IC S

Thus, when we consider both the vibrational and the rotational


motion, the energy of the molecule is

E= — + l ) +/ t o( * + 7*). (12.53)

The first term here represents the dissociation energy, while the
second and third terms describe the rotation and vibration of the
molecule, respectively.
We note that the molecule has only a finite number of discrete
energy levels, since it is dissociated when

Qualitatively, the dissociation of a molecule at large quantum


numbers can be explained as follows. When k^> 1, the amplitude of
vibration may become so large that the atoms will in effect stop
interacting over this distance and the molecule will cease to exist
as a bound system. Moreover, if the orbital angular momentum
numbers I which characterize the rotational energy are too large,
centrifugal forces can also split the molecule.
We shall now proceed to investigate the vibrational-rotational
spectrum. We shall assume that the spectrum is determined
primarily by the vibrational energy of the oscillations because its
order of magnitude is larger than the rotational energy (Avib ~ 1Op,
and >rot 100|i). We must bear in mind that spontaneous transitions
can occur only from higher to lower energy levels, that is, with a
change from k to k — 1 [in accordance with the selection rules, the
quantum number I may change to either lower ( / - >/ — ]) or higher
(I -f I — 1) values]. Then for the frequency of the radiation

(,/ = ~ l,'±D

we obtain from (12.53)

o/==«)-!- m,r. (12.54)

Here, in accordance with (12.34) and (12.35), we have = 2Bl,


"‘i. i i — 2B (I - j- 1), and

Tnus, two branches are obtained (see Fig. 12.5):

'"v ib ' r 2/1/ and «> vib 2/1 (/ -!- 1) (12.55)


THE ROTATOR 197

Such vibrational-rotational spectra are observed, for instance,


in HC1 and CO m olecules, and the investigation of vibrational-
rotational spectra is of great importance for the study of molecular
structure. From these spectra it is possible to determine various
properties of molecules, such as their moments of inertia and the
isotopic composition (since the moments of inertia of two molecules
consisting of different isotopes of the same element will be some­
what different).
Negative branch Positive branch

F i g . 12.5. V ibrational-rotational spectrum of a


diatom ic m olecule.

To conclude this section, let us consider the spectrum of a


molecule when one of the atoms is in an excited state. In this case
the vibrational-rotational radiation is accompanied by the transi­
tion of one of the atomic electrons from an energy level n to a
lower level n'.
The energy of such a molecule can be written in the form

£„ = £ „ -|-£ A+ £„ (12.56)

where £„ is the energy of the excited atom. For hydrogen, £„ is


given by the Balmer formula (see Chapter 2)

£„= -§. (12.57)

The energies of the vibrational and rotational motions are, respec­


tively.

£* — — D ~j- ftu> (k -(- (12.58)


and ' '
E, — Bhl (/ - f - 1). (12.59)
As a result of the transition, the energy Ek of the molecule
changes and becomes equal to
£m' = En' -f- E/,’-f- £;>. (12.56a)
198 N O N R ELA TIV ISTIC QUANTUM M E CH A N IC S

Since the main part of the radiant energy is now due to the electronic
transition n-*n' in the atom, the quantum numbers k and / may
either increase or decrease

lt = k -± l, /' = /-!: 1 . (12,60)


Regardless of the changes in k and /, the overall change in energy
is a decrease because of the em ission of radiation due to the
electronic transition.
This case is characterized by a further important feature,
namely, the strong dependence of the energy bonding the atoms in
a molecule on the number of the particular shell in which the elec­
tron is located. Therefore, a transition causes a change in the
bonding energy, which in turn leads to a change in the interatomic
distance a. In transitions from an excited to the ground stated the
distance generally increases, together with the moment of inertia
J = m I^da1, whereas the quantity B = ^ decreases. Owing to the
change £-► 5', there is an additional slight change in the rotational
part of the energy, which now becomes

Er = B’hl'(l'+ 1), (12.59a)

where B' <^B in the present case.


Taking into account all possible vibrational and rotational
£ _£
transitions, we find that the frequency of radiation u>,n— - M M' is

^ n Ln'-±u> + u>1.1'- (12.61)

where
wlt c = Bl (I + 1) — BT (/' + 1). (12.62)

With the notation <o0= —— —“» we can write Eq. (12.61) as

= (12.61a)

Finally, we obtain three frequency branches for the molecular band


spectra

= lu0+ l-U (12.63)


= u>l) -f- “i, I i 1, (12.64)
>"= “o-f" *“/, I- (12.65)

In these formulas the first or positive branch (the R branch)


corresponds to downward transitions between the rotational levels.
THE ROTATOR 199

and the second or negative branch (the P branch) to upward transi­


tions. The thirdbranch (calledthe zero branch or Q branch) appears
when there are no transitions between rotational levels; it is due
entirely to changes in the moment of inertia caused by transitions
within the atom.
Using Eq. (12.62) we can represent w+, ur and <u° in the form

a>+= '*9+ (B — B')l*+ (B + B')l, (12.63a)

a" = o>o+ (B - B') (I + 1)* - (B + B') (I + 1), (12.64a)

id«= i»0 + (B — ZJ')(/a+ 0. (12.65a)

These branches are depicted in Fig. 12.6, where the frequency is


plotted along the abscissa and the orbital angular momentum
quantum number / along the ordinate. It can be seen from this

6 7 8 9 W it 12 13 « IS 16

U)
Si3 2 < 0 Q!23’4 S' 6 7 8
d)0
0 1 2 3 «
Id*

F i g . 12.6. M o l e c u l a r b a n d s p e c t r a : k>+ — t h e
p o s i t i v e b r a n c h : cj — t h e n e g a t i v e b r a n c h ;
— the zero branch.

diagram that the superposition of the rotational lines u>, v on the


electronic-vibrational line u>0 gives rise to a whole band of lines
with a sharp edge on the left and a diffuse edge on the right. This is
in complete agreement with the experimental facts.
200 N O N R EL A TIV ISTIC QUANTUM M E C H A N IC S

Problem 12.1. 3 Find the explicit form of the spherical harmonics for the cases when
the orbital angular-momentum quantum number is / = 2 and / = 3. Verify their ortho-
normality by direct calculation. Plot a graph of | Y™ |2 for m = 0, ± 1, ± 2 .
Answer.

^ = / - 4 7 ( t c0S,9- t )'
F f 1= -± ^ sin a cos ie ±

yt 2=Jr V ^ sinS9e±2,?-

Y»= Y i * { - T cos3» - 4 cos«).


y-r ' = ± 4 Y ^ sin 8 (5 cos88 — 1) e--t f t ,

Y ? 2= 4 Y ' 2 T sin”5 cos 8e± 3,> •

> ? ' - * -I-

Problem 12.2. Find the eigenfunction of the operator Lv, given that its eigenvalue
is zero and the orbital angular momentum quantum number is / = 1.
Solution. Let us look for a solution of the equation for/== 1 and

Ljf'l' = 0
in the form

<|» = C iK J + C _ 1K r ' + C a >'?.

Using (11.88), with / = 1 and m = + 1, — 1, 0, and applying the normalization condition

J sin i d M - f = 1,
we find

y = ( Yl —yT')= Y ~ h sin 3cns?-

Problem 12.3. Investigate the general form of the motion of a free particle In spherical
coordinates. Determine the normalized functions for / = 0(s states).
Solution. The Schrodinger equation for a free particle in spherical coordinates Is
written as
1 rf drR Id M)
r dr + *’ R = 0, ( 12. 66 )
dr

where

2
/I3 •

T h e p r o b l e m s in th is c h a p t e r a p p l y a l s o to C h a p t e r 11.
THE ROTATOR 201

Introducing the new function X = V~r R , we transform Eq. (12.79) to

The solution of this equation is a Bessel function of half-integral order. Since the
wave function must remain finite as r — 0, we retain only the Bessel function of the first
kind. Then
const
R 1 (Hr),
W r '+ 2
and for a free particle with a given energy the general solution of the wave equation in
spherical coordinates can be written as

n *.r)= y y c? yi (kr). (12.67)


/ —0 m = — /

In particular, fo r/ = 0 (s state), we have

( 12. 68 )
R (k, r) = — sin kr.
Since the spectrum is continuous, this expression must be normalized by a 8function
00

^ R* ( k \ r) R (k , r) rV r = 8 ( k - k ’).
o
Hence, we find:

Problem 12.4. Find the selection rules for quadrupole radiation from a rotator.
Solution. It Is necessary to find the transitions for which the matrix element of the
quadupole moment

Qss' = « J + |V (3V i ' - a\ s - ) *lm dQ =

= c(3 ^ x sx s.^*m^ lm(iQ — a°-hss, £

does not vanish. Here


s, s' = 1, 2, 3; x t = z = a cos 8,
,ri>3= x ± iy = a sin Sc— = | .

Since the matrix element of a bilinear combination can be expressed as the sum of the
products of linear matrix elements (see Problem 10.4),

m " , I"

and, moreover. It follows from Eqs. (12.24), (12.25)and (12.26) that the only nonvanishing
linear matrix elements are of the form

^ ± 1 . ^ + 1 J - ± 1, m — 1 J -± m
m * V, m » m »

we obtain the following selection rules for the quadrupole radiation from a rotator:
lm = 0, ± 1, ±2; A/ = 0, -±2.
202 N O N R ELA TIV ISTIC QUANTUM M E CH A N IC S

As we shall see later, the parity of a spherical harmonic is determined only by the
value of I and Is Independent of m (see Chapter 17). Consequently, for dipole radiation,
the only allowed transitions are from an odd state to an even state or vice versa (A/ = i 1),
whereas for quadrupole radiation, the transitions can occur only between even states
or between odd states.

Problem 12.5. Expand the plane wave


| = e ikz = <■"" cos0 (12.69)

In terms of spherical waves.


Solution. Introducing the notation k r = y and cos 0 = x, let us look for a solution in the
form of an expansion in Legendre polynomials
OO

eh’x=JL Bi(y) Pt (x)-


1= 0
Using the orthonormality condition for Legendre polynomials [see (11.68)], we find

£, = ( / + — ) \ e‘y*P,(x)dx.
- I
Substituting the expression for the Lengendre polynomials as given by Eq. (11.59) and
transferring the derivative from the function ^ 2 _ |j( to the function e1?*. I times (that
is , integrating by parts I times), we obtain

1
5/ = 277 (/ + -Jrj ily l ^ (1 — x2)lc'vx dx.

Using the well-known relation from the theory of Bessel functions4

J (i - * 2)V>rfx= jAT/i + ,/V, + l/s(y).

we determine the coefficients /!,. The desired expansion of a plane wave in terms of the
spherical waves Is now written as

^ i'( 2 / + 1 ) ./ 3(ftr) Pi (cos 0). (12.70)

Let us emphasize that the plane wave e,kz satisfies the Schrodinger equation for a free
particle

VJI]; + *,| = 0
and, therefore, the right-hand side of Eq. (12.83) also represents a linear combination
of particular solutions of the above equation written in spherical coordinates. This
explains the expected proportionality between the coefficients B1 and the Bessel functions
Jl 4_ i/, (&r)-

4S ee, for e x a m p l e , P . M. M orse a n d H. F e s h b a c h , Methods of Theoretical Physics,


p a r t I, p. 572, M c G raw -H ill Book Co.
C hap ter 13

The Theory of the Hydrogen-like Atom


(Kepler's Problem)

The Bohr theory of the hydrogen-like atom (see Chapter 3) is


sem iclassical in nature and can only provide a very incomplete
explanation of some of the basic properties of the atom. It does not
enable us to calculate the intensity of radiation emitted by an atom
or to construct a theory of an atom with more than one electron.
The wave-mechanical theory of the atom is able to deal with these
problems without any fundamental difficulties. The problem of the
hydrogen-like atom presents in addition a certain methodological
interest since it can be solved exactly, like the harmonic oscillator
and rotator problems. Mathematically this problem can be regarded
as a generalization of the classical problem of the motion of a
planet around the sun (Kepler’s problem).

A. ENERGY EIGENFUNCTIONS AND EIGENVALUES

The energy of interaction between an electron and a nucleus

y —_ (13.1)
r

depends only on the distance r between them. The problem of the


hydrogen-like atom—a single electron moving about a nucleus—is,
therefore, a typical example of motion in a central field of force.
If the origin of the coordinate system is at the center of the nucleus,
we may regard the angular part Y? of the wave function as known
[see Eq. (11.67)] and find the energy levels and the radial part
£(/-) from Eq. (11.21), which for the present case is written
as

(13.2)
2 04 NONRELATIVIST1C QUANTUM MECHANICS

Let us introduce the effective potential energy of the electron1


_ ?gl _ i_ »**(/+!) (13.3)
eff — — r ' 2m0rs ’
where the first term represents the Coulomb interaction and the
second the centrifugal forces.

F i g . 13. 1. T h e effe ctiv e p o ten tial energy ( so lid line) as a


function of d is ta n c e .

Z e 0 . h 2 l (I + l)
2m0rz

T h e d a s h e d lin e s h o w s th e b e h a v io r o f th e w a v e function.

A graph of Ve({ is given in Fig. 13.1. This graph shows that if


the total energy of an electron is negative ( £ < 0 ) , it moves within
We s h a l l a t t e m p t to i n t e r p r e t E q . ( 1 3 . 3 ) fr om t h e s t a n d p o i n t o f t h e c l a s s i c a l t h e o r y ,
u s in g th e c la s s i c a l re la tio n s h ip

p; ,, ( Z el , p$ (13.3a)
'' I ' 2mnr2

Since p c o n s t a n t fo r c e n t r a l f o r c e s , w e m a y w r i t e

v Zel Pi
‘'off
2 m 0r

To g e n e r a l i z e t h i s e x p r e s s i o n fo r t h e q u a n t u m c a s e , we m ust rep lace p by it s q u a n tu m -


\2
net'hanic a l v a l u e £ ^ f (I
p,/f l/ 1). In the same w ay w e c a n regard the expression —-—( —
(/1^ t Tn ih/i c am a u*a ir lira /. an rarrarrl tk a a irn.a on i/>rt 1 / [A.\ /
2m
in Eq. (13 2) a s c o r r e s p o n d i n g to the term p f '2m 0 in the c l a s s i c a l theor y
THE THEORY OF THE H Y D R O G E N -L I KE ATOM 205

a region bounded on both sides by potential barriers (the case of


elliptic orbits in the classical analog), so that the energy spec­
trum will be discrete.
For ll > 0, there is no barrier on the right (r —co) and the posi­
tion of an electron can range to infinitely large R (the case of
hyperbolic orbits in the classical analog). Since the electron’s
position in the atom must be bounded by a certain rmax (elliptic
orbits), it is necessary to assume that E<^0 in order to develop
a theory of the atom. Accordingly, Eq. (13.2) becomes

™ + t v + { - a + t - —, < 13-4 >

where

= £ > 0 and — 2'^- = A > 0. (13.5)

Introducing the new variable

P = 2 l/A r , (13.6)

we obtain the equation

^ + + + = (13.7)

where R' — {dRjdp) .


From the graph of Feff. we can see the general nature of the
solution. Clearly, inside the well (''m in < X /'max) the solution will
be oscillatory, whereas outside the well ( r —■0 and r — c o )
there are two solutions (on each side): one that tends to zero and
a second that tends to infinity. To make the solution an acceptable
wave function, we must impose on it restrictions that will elim i­
nate the solution that tends to infinity. Just as in the problem of
the harmonic oscillator, it turns out that this requirement can be
satisfied only for certain discrete values of the energy of the
electron.
Since the potential well is not sym m etric, we shall look for
asymptotic solutions for p— 0 and for p— oo separately. The
asymptotic solution for p—>oo may be found, according to Eq. (13.7),
from

(13.8)

which gives

R ea = -f- C4e‘/sP. (13.9)


206 N O N R ELA TIV ISTIC QUANTUM M E CH A N IC S

To eliminate the exponentially increasing solution, we must


set Cj = 0. Coefficient C, can be included in the normalization
coefficient of the wave equation and can, therefore, be set equal to
unity. We then have

p. (13.10)

The asymptotic solution for p— 0 can be obtained from Eq.


(13.7), which in this case reduces to the form

r: + j Ro- ‘alP 1 Ro= o. (i3.ii)

Setting R0= pi, we find <7(<7+ 0 — ^ + 1) = °, and thus qi = l,


<7i = — (/-}- 1)• Consequently,

f?0= ClP, + C4p-i- 1. (13.12)

Setting C.2= 0 (to exclude the solution increasing to infinity at


p= 0 ) and Ci = 1, we obtain
tfo = p'- (13.13)

Equation (13.7) can be rewritten as

and we assume its general solution to be of the form

R = R J t vu. (13.14)
In this case

pR = pt+'e-'/-fu — vu, (13.15)

and for an unknown function u we obtain the equation

According to (13.15),

Inu — — V«P "t- 0 OInp,

and, therefore,

= (lny)- = — - -1 (.+ 1 that is, v = ! j v.


t
Furthermore, we have

v•t i+ i /+ ! v, and 1 /+ ! I /(/+ !)


pa P V 4 p ' p*

Using these formulas, we transform (13.16) to

P«" + 12 (/ —
J—1) — p] [-~ f= — I — l] « = 0, (13.17)

If the behavior of the solution for R is to be determined at the


origin and at infinity, by the asymptotic formulas (13.10) and
(13.13), we must find the conditions under which the function u
will be a finite polynomial of degree k without negative powers:
k
« = 2 a«pv- (i 3 -i8 >
v=0

Substituting (13.18) into (13.17) and collecting powers of p, we


have
k

+ a,til* ( * + l) + 2 ( * + l)(* + l)]} = 0. (13.19)


Since ak+i = 0 and ak ^ 0, we obtain

J ^ = A+ Z+ 1 = «. (1 3 . 2 O)

Here the quantum number n, which is equal to the sum of the


orbital angular momentum quantum number

1= 0, 1, 2, 3,...
and the radial quantum number

k = 0, 1, 2, 3,... (13.21)

plus unity, is called the principal quantum number n. It may assume


the values
n = 1, 2, 3...... (13.22)

To determine the unknown coefficients av of the series (13.18), we


derive a recurrence relation with the aid of Eqs. (13.19) and (13.20):

av(£ — v) = — av+i (v — 1) (v —
J—2/ —
j—2). (13.23)
208 NON R E L A T IV IS T IC QUANTUM M ECH AN ICS

Setting the coefficient of the highest power in (13.18) a * = ( — l)" and


calculating all the remaining coefficients with the help of (13.23),
we obtain the following expression for the function u:
/ , *(* + *) «*-l I * ( * — * ) ( * + s) (ft + S — 1) __ \ __
« = ( — ') ( p -------- n— p -i 2ip
k
_ V I isk+ J . k - J ftl ( ft + S) l
— 2d ( ^ p yi(ft—y)i(ft+s—j ) \ ’ (13.24)
j=0

where s = 2 / - f l . The series (13.24) is an associated Laguerre


polynomial Q£(p) of order k and may also be represented in closed
form2*

« = QJ(P) = « V ^ ( e - pPUl). <13-25)

Thus, for the radial fimction Rnl(r) we finally obtain

Rm (P) = P' (p). (13.26)

where p= 2 y rA r.
Recalling that -y= = n, and substituting the
VA
value of B from Eq. (13.5), we find
22

na0 r (13.27)
fl'i
where a0= — m0elT is the radius of the first Bohr orbit. CalculatingD
the coefficient Cnl from the normalization condition (see below),
we obtain
Cnt - (\na„)
Z \'h VV n (n—I—41)1(n + /)! (13.28)

2We s h a l l sh o w h e r e t h a t t h e f u n c tio n u, w r i t t e n in th e c l o s e d form ( 1 3 .2 5 ), s a t i s f i e s


ICq. (13. 17). T h e f u n c tio n v = c ^pk +s s a t i s f i e s t h e e q u a t i o n p v + ( p — k ~ s)v ~ 0, a s c a n
be e a s i l y v e r i f i e d by t a k i n g t h e f i rs t d e r i v a t i v e o f v w ith r e s p e c t to p. D i f f e r e n t i a t i n g t h i s
e q u a t i o n k + 1 t i m e s a c c o r d i n g to L e i b n i t z ’s ru le , w e r e d u c e it to t h e form
p v {k + 2) + (p - s + 1)U(* + !) f (k + l ) v (k) 0 .
I n tr o d u c i n g t h e n ew f u n c t i o n oj l/ k \ ’^p s , w e o b t a i n th e d i f f e r e n t i a l e q u a t i o n
pw " t (s l 1 - p)w > i hw 0 ,
w hich is i d e n t i c a l with Eq. (1 3 .1 7 ) for t h e f u n c t i o n u [ s i n c e — /*]. S i n c e it
c a n he r e a d i l y sh o w n th at th e c o e f f i c i e n t of th e l e a d i n g t e r m ( p ^ ) o f t h e f u n c tio n

rPP~s d . (<■ ^ * +s)


tip
is identical with the corresponding coefficient of (13.24), we have demonstrated that the
relation (13.25) is correct-
THE THEORY OF THE H Y D R O G E N -L I KE ATOM 209

Therefore, the radial wave function is

1___________ (—-r- \'e nao Q n - l - I (— ) ■


l)!(n + /)! \na0/ ^ \no0 / (13.28a)

As we know, the normalization condition for the radial part of the wave function is

J dr = 1.

Substituting the expression for R ni from (13.26) and replacing r by ~ p, in accord­


ance with (13.27), we obtain ^

7a
Now let us represent one of the polynomials as a series (13.24), leaving the other
polynomial in the closed form (13.25). The normalization condition then becomes

c"' { s j Sp(_ i)ft { k {k+ 21+ («-pp'!+3i+i) = 1.

Applying the theorem for the transfer of a derivative (see Chapter 7, Section C) to this
equaUon, we find
CO
C °)3J
*, ( — <rP [(* + 1)! P2' +fc+2 - M k (2 1 + k + 1) d ( = I.

It is obvious that the remaining terms in the series representing the function Ql vanish,
since the order of the successive derivatives is higher than the corresponding powers
of p. Using the well-known integral
OO
\ e-Pp4dp = s!, (13.29)

we obtain the expression (13.28) for C„i . In a similar fashion, we can determine the
average value of (r~v) ^ = 1, 2, 3, 4), which will be useful in the later development:
CO

(r v) = £ ^ , mr - ^ „ i m d 3x [ * !//-’+* dr.
u
On the basis of the above equations we can rewrite this as

(r-’) = C%, { ^ y J p- +1 ( - I)* {P* - k (k 4- 21 + 1) P*-‘ +

k ( k ~ l) ( 2/ + f c + l) l fe(2/ + fe+ 1)!


+ ... + ( - I)*'2 2! (2/ + 3)! P2 + (-!)* -* (2/ + 2)1 P+
(21+ fe+ 1)!
+ (-!)* (21+ I)!
210 NON RE L A T I V I S T 1C Q U A N T U M MECHANICS

Setting •v = 1, 2, 3 and 4 in the last expression, and using once again the theorem for
the transfer of a derivative, we obtain after a few simple operations

1
(r-2) = (\ oo
- )/ ’ -n3(/ + V.) ’ (13.29a)
1
fr"*) _ Uo) «»/(/+*/.)(/+ 1) •
J - f i - V ______ 3n* — 1(1 + 1)
* 1 2n> \ a j '.) (/+ !)(/ + '/.)
I n calculating r~l, we retain only the leading term pft in the polynomial Q j . On the
contrary, in calculating r~s, we retain only the last term p». For r ' , we retain the last
two terms, and so on. The expressions for r~s and r~* are obtained under the assump­
tion that^ / + 0. For the s states (1 = 0), Interactions which are usually proportional to
either r~B or r-* are generally replaced by a contact interaction (see below. Chapters 19
and 20).

To supplement this general treatment, we shall calculate the


normalization coefficient of the wave function tynlm= Rn7 T for the
ground (or lowest) energy state, which is characterized by the
quantum numbers
n = 1, l — m = k = 0.

According to Eq. (12.9), the spherical harmonic K", is a constant


for l — m = 0, as is the polynomial Q*/+l for A:= 0. Consequently,
from (13.26) we have for <^100
Zr

'Woo= Ce °° > (13.30)

and, therefore, the normalization condition takes the form

J 'O .o .d 1* = J VL*mr!drdQ = 1. (13.31)

Substituting the explicit form of the wave function ^100 and bearing
in mind its independence of the angles 0 and <?, we obtain
<X>
p 2Z r

CH- } e "aortd r = 1. (13.31a)


0
*17r
Introducing the variable.v = and using (13.29), we find

The same value for C can also be obtained from the general equa­
tion (13.28) by setting n = 1 and ( = 0. Therefore, the wave function
for the ground state is
THE THEORY OF THE HYDROGEN—LIKE ATOM 211
Zr
(Jo (13.32)

The energy spectrum of the hydrogen-like atom is found from


Eqs. (13.20) and (13.5):
Z‘ej _ RhZ3 (13.33)
2a0n2 n3

where R is the Rydberg constant


n _ gp/no
2h3 •

This expression for the energy, which we note is in complete


agreement with the corresponding expression of the Bohr theory
[see Eq. (2.50)], depends only on the principal quantum number
n= (that is, only on the sum of the orbital and radial
quantum numbers I and k) and is independent of the magnetic
quantum number tn . At the same time, the wave function $nim= RniY'?
depends on all three quantum numbers rx, I and m individually.
Consequently, it follows from the Schrodinger wave theory that the
energy levels are degenerate. Since each value of I can vary from
0 to n — 1 and m can vary from —I to -j-l [see (13.20)], the degree
of degeneracy is

n —I I n—\

2 2
1=0 rn=—l
i = 1=0
2 (2/+ 1)==/z’-

It was shown in Chapter 12 that the degeneracy with respect to


m is characteristic of all central fields of force and is related to
the fact that there is no preferred direction passing through the
origin. The degeneracy with respect to the orbital angular mo­
mentum quantum number I appears, however, only in the case of
pure Coulomb interaction. In most other centrally symmetric
system s there is no I degeneracy; that is, the energy level for a
given value of n is split into n sublevels corresponding to different
I.3 If the system is placed in an external field (for example, a
magnetic field) which removes the central symmetry, the de­
generacy with respect to m also disappears. In this case, the nth
energy level is split into n2 distinct sublevels.

3In particular, as we shall see later, the degeneracy with respect to vanishes even in
I

the case of the hydrogen atom if we take into account the relativistic effects, the nuclear
volume, and the so-called vacuum corrections. Similarly, in the spectrum of alkali metals,
which have one valence electron in the outer shell, the influence of the electrons in the
inner shell removes the degeneracy with respect to I.
212 NON R E L A T I V I S T I C QUANTUM M E C H A N IC S

B. SEMICLASSICAL INTERPRETATION OF THE


PRINCIPAL RESULTS OF THE QUANTUM-MECHANICAL
THEORY OF THE HYDROGEN-LIKE ATOM

In classical theory the quantity


1 , _ . Zel p;
— p> = E + - ---------------------------------- (13.34)

[see (13.3a)] must be greater than zero. For elliptic orbits


(,E = — E |< 0 ) it is seen that this is possible only when the radius
r lies within certain lim its ('"min^S r <= ''max) which can be found by
setting the right-hand side of (13.34) equal to zero. Using the
quantum-mechanical expression for the energy (13.33), we find

n2ao f r \ (13.35)
'max
min ±v
The equation of an ellipse in polar coordinates is

r = ■1 +, pe ---- ,
COS tf ’ (13.36)

where the parameter P = -a is defined as the ratio of the square of


the semiminor axis b to the semimajor axis a, and the eccentricity
e= characterizes the elongation of the ellipse (for s = 0,
the ellipse becomes a circle). From (13.36) we readily obtain
equations for /-maxand /•min:

rmax — a (1 + e ) ,
(13.37)
rm i n = n (> - E)-

Comparing Eqs. (13.37) with (13,35), we find

nyo
= a, (13.38)

Pi (13.39)
iPh1

We see that the classical analog of the quantity — turns out to


be the semimajor axis of an ellipse whose eccentricity is given
by Eq. (13.39).
THE THEORY OF THE H Y D R O G E N -L I KE ATOM 213

Substituting into (13.39) the Bohr value of P%= 1Rn%= Hl (l- \)2
and then the quantum-mechanical value of P? = (I -f- 1), we obtain

(13.40)

e = ] / 1— IU + (13.41)
quan f n-

From this it is clear that the eccentricity becomes exactly zero


only in the Bohr theory (l — n — 1). In wave mechanics, for l = ti— 1
the eccentricity has a minimum nonzero value given by

cmin= l / i . (13.42)
quan V n

This shows that in quantum mechanics, we can speak of the cla s­


sical analog of states with circular orbits only when the quantum
number I has the value n — 1. Furthermore, it should be noted that
in the classical approximation, an s state (/ = 0) gives 1,
which corresponds to a parabolic orbit. This case, however, cannot
be associated with a parabolic orbit from the quantum-mechanical
point of view. As we can see from Eq. (13.33), in quantum mechanics
the energy is negative for all values of n (we recall that in the
classical case the total energy is zero for parabolic orbits); con­
sequently, the radius for / = 0 is limited only by its maximum
value
r max = 1 I*[i1I •

This lack of agreement between the quantum-mechanical and


classical solutions for the s state simply means that the case / = 0
has no classical analog.
In general, we are, of course, entitled to speak only of the
probability of particular events in the context of wave mechanics.
Therefore, all the results obtained from the Schrodinger theory
must be interpreted in terms of probability considerations. Let us
show, for instance, that the Bohr radius a - —°- [see Eq. (13.38)]
for the case of circular orbits (l = n — 1) corresponds in wave
mechanics to the most probable value of the position coordinate
of the electron, r — a.
According to the normalization condition
00

j r'lR%,dr = 1,
C
l
the distribution of radial probability density D{r) is
D(r) = rR'„i. (13.43)
214 NON R E L A T I V I S T I C QUANTUM M E CH A N IC S

In the case of circular orbits, when l = n — 1 and 6 = 0, Eq. (13.43)


gives
D(r) = rlR%,„-1 . (13.43a)

According to Eqs. (13.25) and (13.26),


_ _i_

f ln, n-i = co n ste

and, therefore, we find the following expression for D (r) (see Fig.
13.2):
_ 2Zr_
D (r) = const r ne n00 ■ (13.44)

Determining the value of r at which this function has a maximum:

we obtain
rn= a = " ^ c i<). (13.45)

It is interesting to note that if we set Z = 1, the most probable


radius r is the radius a0 of the first Bohr orbit.

F i g . 13 .2 . R a d i a l p r o b a b i l i t y d e n s i t y d i s t r i b u t i o n f u n c t i o n
in t h e c a s e o f c i r c u l a r o r b i t s .

If the radial quantum number k ^ 0, the orbits can be said to be


elliptical and the probability distribution D(r) assum es the form
_ ZJ
D(r) = consir!,+1e n''°(Ql'+'f. (13.46)

The equation for the extremals of this function is

= const P5,ne-p[ (21 + 2) Q%+1 —

— pQj!+] ~\~ 2P^Q2^'}Q"+' = 0 . (13.47)


THE THEORY OF THE H Y D R O G E N - L I KE A T O M 215

Since xQ^+‘ is a polynomial of the Ath degree, Eq. (13.47) has k


roots (not counting points r = 0 and r — oo) and k-\- 1 maxima. This
case is very similar to the probability distribution for the motion
of a free particle inside a potential well (sinusoidal variation of
the wave function, see Chapter 4) or for the motion of an oscillator
(see Chapter 10, Fig. 10.2).

C. SELECTION RULES, EMISSION SPECTRA OF


HYDROGEN-LIKE ATOMS

To determine the selection rules for hydrogen-like atoms


(Kepler’s problem), it is necessary to calculate the matrix ele­
ments

^Ynim' = J W'l'm' r^nlmd3x. (13.48)

Substituting here = Y™Rnl,we obtain

(13.48a)

As we know from Eqs. (12.24), (12.25) and (12.26), integration


over the angles a and <p gives the selection rules for the orbital
angular momentum quantum numbers (M = l — Z'= :±l) and the
magnetic quantum numbers (Am = m — m — 0, ±1). Using these
results we can write Eq. (13.48a) as
00

Wmm' = const{ 3 ^ +J ^ . ; ± i J ^ w r*Rnldr. (13.48b)


“ 0
Evaluating the integral4
OO O
O
I r*Rn.vRnldr ~ j r ^ l±'e~
* + ^ Q[ (-g l) X
o o
aw **)

4 T h i s i n t e g r a l c a n b e e v a l u a t e d by i n t r o d u c i n g t h e n e w v a r i a b l e p = — ( — + — \ Then,
I t± l . a° ^ U '
expressing Q a n d Q^/ in t h e form o f p o l y n o m i a l s , w e c a n p erfo rm t h e i n t e g r a t i o n term by
term. It i s f o u n d t h a t o n ly t h e i n t e g r a l
co

^ r2Rn ,l Rn l dr = 0 (*V n)
u
v a n is h e s b e c a u s e of the orthogonality condition.
216 N O N R ELA TIV ISTIC QUANTUM M E CH A N IC S

it is easily shown that it does not vanish for any value of n'\ that
is, for all allowed transitions the principal quantum number can
change arbitrarily.

E, ev

F ig . 13.3. S pectral series of the hydrogen


atom.

T h e w a v e l e n g t h s c o r r e s p o n d i n g to t h e i n d i ­
c a t e d t r a n s i t i o n s a r e e x p r e s s e d in A n g s tr o m
u nits.

Having obtained the selection rules for the hydrogen-like atom,


let us investigate its em ission spectrum. We shall first introduce
certain conventional symbols designating the energy levels of an
atom. The spectral terms (— Enl h) which depend, in the general
case, on both n and / are denoted by the symbol (til), that is,

(— -nL) = ^ (13'49)

where n = 1, 2, 3, ..., and I is replaced by one of the letter symbols


s, v, d, I, g, h, ... corresponding to 1= 0, 1, 2, 3, 4, 5, .... as
indicated in Chapter 12. Since the quantum number 1 n — 1, the
only possible terms are Is; 2s, 2/;; 3s, 3/;, 3d; 4s, Ap, Ad, Af; 5s, 5p,
5d, 5f, 5g; and so on.
There cannot, for instance, be a 1p term, since in this case we
would have n = 1 and / — l, nor can there be a 3/ term, since this
would give us n = l — 3. The radiation frequencies expressed in
term symbols (til) have the form
THE THEORY OF THE H Y D R O G E N - L I KE A T O M 217

u>n„‘ = 1~n- ^ = (n’l’) — (nl), (13.50)

where it is necessary to bear in mind the selection rules for the


orbital angular momentum quantum number I, namely, l' = l ± \ .
Using Eq. (13.33), the term (nl) can also be represented in the
form

(13.51)

where R is the Rydberg constant . We thus obtain the


following equation for the radiation frequency u w ;

<ann’= RZ~(^s — (13.52)

In the case of the hydrogen atom(Z = l), the Lyman series (see
Fig. 13.3), which corresponds to a transition to the lowest energy
level n ' = 1 (the Is level), is given by

V a n = ( 1s) - (np) = r [-^ ,(13.53)


where n = 2, 3, 4......... For the Balmer series (see Fig. 13.4),
which corresponds to a transition to the level n' = 2 from the levels
/ / > 2, there are three types of
allowed frequencies: Co If) tN
JBahner=(2s) — (np)-,
g
I § 1
“Bataer=(2P)-(nS);
W r = ( 2P)-(nd). (13.54)
"a Hy Hg Ho
Since the energy states of the hydro­
gen atom are degenerate with respect
to the orbital quantum number, these F i g . 13.4. B a l m e r s e r i e s .
three lines merge into one (see T h e w a v e l e n g t h s c o r r e s p o n d i n g to
Fig. 13.3), and, consequently, th e v i s i b l e l i n e s H a , Hfi, H y an d
H g a r e g i v e n in 5 A n g s tr o m s (A). H ffi
g iv e s the th e o r e tic a l p o s itio n of the
lim it of th e s e rie s .
•w = k ( tst- ^ ) - (13-55>

A similar result is obtained for the Paschen series, whose fre­


quencies (see Fig. 13.3) are given by

P aschen ^ f 32 /Is ) 1
(13.56)
where n — 4, 5, 6 ... .
218 N O N R ELA TIV ISTIC QUANTUM M E CH A N IC S

D. MOTION OF A PARTICLE IN A COULOMB FIELD


IN THE CASE OF A CONTINUOUS SPECTRUM

Although the wave function for a continuous spectrum can be


expressed in terms of a confluent hypergeometric function, the
usual procedure adopted in the investigation of the hyperbolic
solution is to consider the asymptotic behavior of the hypergeo­
metric function for large values of r. These asymptotic solutions
can be obtained directly with the help of the quasi-classical WKB
method. To investigate the hyperbolic orbits ( £ > 0), let us first
use this approximation to obtain certain general results applicable
to the case of central forces. According to Eqs. (11.21) and (11.51),
the equation for the radial part of the wave function in the case of
central forces is
u„+ ( ^ o £ _ ^ l/(r) _ ^ + _ l ) ) u = 0i (13.57)

where
u = Rr. (13.58)

In the WKB method, the potential energy V (r) is required to decrease


more slowly than r l as r-> 0. Furthermore, we are entitled to
choose the potential energy in such a way that

V (/■= oo) - 0.

Equation (13.57) has a singularity at the point/- 0 associated


with the term I (I + l)r~*. The effective potential energy here forms
a potential barrier of infinite height. Consequently, the asymptotic
forms of the wave function which we obtained earlier for con­
necting the WKB solutions across a slowly varying potential
barrier do not provide a good approximation in the present case.
Therefore, in order to use the WKB method in the case of central
forces, we must obtain either a different asymptotic expression
in place of the B essel functions of order %, or we must remove the
singularity from the point /• = () to the point x = — oo by introducing
the new variable x = ln r.
We shall use the second of these methods and introduce a new
wave function y(x):
u = ex/2y (x). (13.59)
Equation (13.57) then takes the form

2 ^ ( 0 - ( / + y ) V w) X= 0, (13.60)
to which the WKB method is applicable. For the argument z which
determines the asymptotic behavior o f [ s e e Eq. (5.55)], we obtain
THE THEORY OF THE H Y D R O G EN -L IK E A T OM 219

z = j § ex j /~| 2m0E — 2/n0V (ex) — ti1(l -f- —) e ‘ix | dx =


= { J \p\dx. (13.61)

It should be noted that here | p \ no longer represents the momentum.


Transforming back to the original independent variable r, we
obtain

z n 2m0£ — 2muV (r) — h dr. (13.62)

From this it follows, in particular, that in using the quasi-classical


expression for the one-dimensional radial equation, we must make
the following change in the orbital angular momentum:

/ ( / + l ) - > ( / + i - ) 2. (13.63)

Let us now apply Eq. (13.61) to the motion of a particle in a


Coulomb field

for the case of hyperbolic orbits (£^> 0). In the quasi-classical


treatment of the problem, it is necessary to use the connected
asymptotic solutions (5.66)—(5.69). This means that in our case
[see (13.62)] we must evaluate the integral
r
V c ]n 2c + fr r + 2 Y c /
S V7V) dr = V f
2 2c + b r — 2 Y c f "**
2ar + b + 2 Y a f
4 Y a 2ar + b — 2 Y a f ’ (13.64)
where
f = ar‘l -{-b r~ \-c , (13.65)

and the value of r„ is found from the condition f(ra) = 0.


First let us determine the asymptotic solution of the wave
function for r„> r -* 0. Substituting

c 2 m0Zel
a 2m0E
fta ' ft2
into Eq. (13.64) and introducing the notation

Y^nt^E moZel Ze\


k n > l ■~ wk ~ hv • (13.66)
220 NON R E L A T I V I S T I C QUANTUM M E C H A N IC S

we obtain
ro
z i I \ p 'd \n r^ +

i-i - 7 + n* +
ln — (* + ■§■) ^r + const.
'+ 2 -i-'I' + i

On the basis of Eq. (5.51), the asymptotic solution for this case
has the form

z = F f f l (C'<r' + i V ')’
where

|p| = *(' + y)

in accordance with (13.61). Since, according to (13.58) and (13.59),


the radial wave function R is related to x by the equation

R = -yr <13-67)
we find
R = Cr‘ + Dr-{,+1).

In order for the solution to remain finite at the origin, we must


set D = 0; that is, we choose the solution in the form

R = Crl, (13.68)

which is in complete agreement with the asymptotic solution


obtained by another method [see (13.13)].
For the other limiting case r-> oo, we have the following
asymptotic solution [see (5.67) and (5.66)]:

Zr>ro = — -sin(z + ! ) , (13.69)

where, according to (13.61), p Is equal to

P = rhk. (13.70)

Equation (13.69) is an analytic extension of the asymptotic solution


for /•-+(). To determine the quantity z from (13.62), we use the
integral (13.64) with the following substitutions:
THE THEORY OF THE H Y D R O G E N - L I KE A T O M 22

2maE . 2m0Z e l
«= b = —V JL
(13.71)
— ('+4 )’ .
Then, since V7 ***kr-\-y, we obtain at r->co

11 + T+ l[t + 2kr
zm k r-j-y- In ■ -rr + Tln (13.72)

We shall also make use of the relations

I" T+ i *+ T =
= In ] A 2 + (/ + -)* + arc tan (13.73)

In

arc tan (13.73a)


l+ 2
Here, in writing the imaginary part, we took account of the fact
that 7 > 0 and and therefore the angle determining the
imaginary part of the logarithm in Eq. (13.73) lies in the first
quadrant, and the angle of the logarithm in Eq. (13.73a) in the
fourth quadrant.
The expression for the argument z may now be transformed to

2 = Ar + Tln2ftr — y / - - + 8f, (13.74)


where the phase S? i s 5

B° = — (/ + Y )arc tan— -— —7 ln|/" (i + 7 ), +T* + 7 - (13.74a)


^+ *7T

F ro m a m ore a c c u r a t e c a l c u l a t i o n (a c a l c u l a t i o n o f t h e a s y m p t o t i c e x p r e s s i o n for th e
ar g u m e n t z from a n e x p a n s i o n o f t h e c o n f l u e n t h y p e r g e o m e t r i c f u n c t i o n ), t h e f o l lo w in g
v a l u e i s o b t a i n e d for t h e p h a s e 5^ ( s e e P . M. M orse a n d H. F e s h b a c h , Methods of T h e o ­
re tical P h y s ic s , P a r t II, N ew York: M cG raw -H ill, 1953):

®I = -argT (/ + 1 4 iy) .

If |/ + i y | ^> 1, t h e n u s i n g S t i r l i n g ’s formula

r o + / + iy) = i r o + / + i r ) | e ~ iS/° = .

we o b t a i n for t h e a p p r o x i m a t io n ( 1 3 . 7 4 a ) S®.
222 N O N RELA TIV IST1C QUANTUM M E C H A N IC S

In particular, when 1= 0 and f > 1, we may write as an approxima­


tion
5o= 7 (1 — In T) — j -
Hence, taking into account Eqs. (13.69), (13.67) and (13.70), we
find
C sin [ k r — +
Rl{k) = ----------------------------------------------- (13.75)

Here C is the normalization coefficient, and the total phase is

8i = 6?-{-f In 2kr. (13.76)

Setting if = 0 in (13.76), we obtain the asymptotic form of the radial


function for the case of free motion:

C sin
[kr~ l l) (13.77)
R?=0) (k) = ------

Thus the potential energy in a central field of force is taken into


account by means of the phase shift

= dr-f-
ro
r _____________________________________________

+ \ Y k' - 1w V { r) - { l + ^ r ' dr, (13.78)

where rl is the root of the first integrand.


In the special case of a Coulomb field, this phase is given by
Eq. (13.76). The presence of the logarithmic term depending on r
in the phase shift is the result of the long-range character of
electrostatic forces, which can influence the particle even at a
very large r.
The phase shifts 5, are functions not only of /, but also of ti1
ft
(that is, of the energy £ = -=— ). They are an essential character-
istic of the eigenfunctions of the continuous spectrum. The value
of the phase shift cannot be found in a general form. The quantity
S/ has to be determined separately for each specific problem,
usually by approximation methods. The phase shifts are of particu­
lar importance in scattering problems, where they are used to
express the effective cross sections (see Chapter 29).
m e wave functions of a continuous spectrum are normalized in
terms of a 5 function. The expression (13.75) for the wave function
is valid practically in all space; there is only a small region near
the center where this expression takes a somewhat different form.
THE THEORY OF THE H Y D R O G E N - L I KE A T OM 223

In evaluating the coefficient C we may, therefore, assume that Eq.


(13.75) gives a correct expression for the wave function; that is,
valid in all space. In this case, the wave function is normalized by
the relation

/ = ^ R, (k) R, (k’) r*dr = l { k - k').


o
We substitute the expression (13.75) for Ri(k) and neglect the
logarithmic term in the expression for the phase shift 8, because
it increases slowly compared with r. Consequently, we find
00 00

/ = -^- ^ cos r(k — k') dr — j cos [r (k k') — 28;] dr

and the value of the normalization coefficient in the asymptotic


solution for the central field of force is

C = j / ' |. (13.79)

An energy level diagram for the hydrogen atom, showing both


discrete levels and the continuous spectrum, is given in Fig. 13.3.
This diagram clearly shows the degeneracy with respect to /,
which results in a merging of all levels with the same n into a
single level.
Besides the ordinary transitions between discrete levels, two
other processes can take place—ionization and capture. Basically,
each of these processes is the reverse of the other. In ionization, an
electron jumps from a discrete level ( £ < 0), such as, for example,
the ground state, to the region of positive energies (E^>0) which
forms a continuous spectrum (hyperbolic orbits). This process
involves the absorption of energy. Conversely, in the process of
electron capture, a free electron jumps into one of the possible
discrete levels, at the same time liberating a corresponding amount
of energy.
A certain amount of energy is required to transfer an electron
from the ground state ( n = l ) to the region £ > 0 . This energy is
given by (see Fig. 13.3)
Eion = T _ Ei = Rn + T,
where T — is the kinetic energy of an electron which is no
longer bound to the nucleus. The energy Eion represents the ioniza­
tion energy of the atom. It is at its minimum when 7 = 0; this cor­
responds to the minimum energy (E = 0) transition of an electron
224 NON R E L A T I V I S T I C QUANTUM M E C H A N IC S

from the level n = l to the continuous spectrum. As a result of


this transition, the electron can leave the atom. For the hydrogen
atom

pi on
^-min= /? ^ = —
LQo
- = 13.59 ev.

E. CALCULATION OF THE EFFECTS OF THE


MOTION OF THE NUCLEUS

In developing the theory of hydrogen-like atom; we have until


now ignored the motion of the nucleus. Accordingly, our theory is
rigorous only for the case of infinitely large nuclear m ass.
In general, this is a relatively rough approximation, particularly
in the case of light elements such as hydrogen and helium. By
taking into account the motion of the nucleus, it is possible to
explain a number of important experimental facts.
We shall allow for the effects of the nuclear m ass Af in the
same way as we did in our discussion of the spectra of diatomic
molecules (Chapter 12), replacing everywhere the electron mass
ma by the reduced mass

1+ M
The Rydberg constant then becomes

^ = -2*r = * » (l ~ 5J). (13.81)

As a consequence of this change, the term values are slightly


shifted:

("0 = — (l - % ) . (13.82)

The radiation frequency is therefore given by the relationship

which differs from the previous one [see Eq. (13.52)] by the factor

Since the frequency of the radiation depends on the nuclear mass


Af, atomic weights can also be determined by spectroscopic methods,
as well as by conventional chemical methods. One successful
outcome of the application of the spectroscopic method was the
THE THEORY OF THE H Y D R O G E N - L I KE AT O M 225

proof of the existence of heavy hydrogen and ionized hellium. Pre­


viously, the average atomic weight of hydrogen (relative to oxygen)
was found by chemical means. The mass spectrograph made it
possible to measure the atomic weight of each atom. These
measurements gave somewhat different values for the atomic mass:

AW w- A l - ^ c 100% _ 0.0145%. (13.84)


”‘chem

This led Birge and Menzel to predict the existence of a hydrogen


isotope, called deuterium or heavy hydrogen, with anatomic weight
twice that of hydrogen ( D = ,H 2). The presence of deuterium in
natural hydrogen explains the greater atomic weight obtained in
chemical measurements. The mass spectrograph measures the
atomic weight of iH1 along, wince the spectral lines of iH2 atoms
fall at a different place on the scale.
Deuterium, like hydrogen, can Hydrogen (H1)
enter into reactions forming, for
example, heavy water D.^0, discov­
ered by Urey and Osborn in 1932.
Deuterium is usually obtained via
the electrolytic decomposition of
water. The rate of evolution of or­
dinary hydrogen, at the cathode,
greatly exceeds the rate of evolu­
tion of deuterium; thus, the concen­ F i g . 13.5. D iag ram o f t h e r e l a t i v e
tration of deuterium in the residual p o s i t i o n o f t h e s p e c t r a l l i n e s of
electrolyte increases. (It is almost hydrogen an d its iso to p es.
impossible to detect deuterium in
natural water because of its low concentration.) The presence of
deuterium was confirmed by spectroscopic studies which showed
that not only does the Balmer series (n' = 2) consist of the lines

0)^almer= floo (1 — —0) (jr — ^ ) , (13.85)

but that each of these lines is associated with a second line situ­
ated somewhat to the right. This second series of lines (see Fig.
13.5) is described by the equation6

B a lm e r n fi 1 \ /I 1\
cop = X « ’[1 (13.86)

A c c o r d i n g to t h e l a t e s t e x p e r i m e n t a l d a t a ,

Rm = 2 ttc • 109 73 7, R H = 2 t t c • 109 678, R D = 2 rrc • 109707,

w h e r e t h e n u m b e r s r e p r e s e n t t h e v a l u e s o f R s p ( s e e f o o t n o te , p a g e 20).
226 N O N R ELA TIV ISTIC QUANTUM M E C H A N IC S

which can be readily obtained from (13.83) by setting the m ass M


equal to twice the m ass of the hydrogen nucleus and substituting
Z = 1. It is worth noting that the large relative difference between
the m asses of the deuterium and hydrogen atoms causes far
greater differences in their physical and chemical properties than
is usual with isotopes of other elem ents. Thus, although heavy
water outwardly resem bles ordinary water, its melting and boiling
points are 3.81°C and 101.4°C, respectively, its viscosity is
greater and it is a poorer solvent for salts. With the development
of nuclear physics, heavy water has become particularly import­
ant because it is a good moderator for fast neutrons, and can also
be used as a source of deuterium.
At present, we know of a third hydrogen isotope, namely tritrium
(T = whose nucleus consists of two neutrons and one proton. It
forms a compound with oxygen sim ilar to water. The ratio of
tritium atoms to 1 H1 atoms in natural water is approximately ]0~18,
whereas the ratio of ,0* atoms to iH' atoms is 1/6800. In a mixture
with deuterium, tritium is a very important substance for the pro­
duction of thermonuclear reactions: the reaction between jDa and
iT3 nuclei leads to the formation of jHe* and one neutron. Each
such reaction releases more than 17 Mev of energy. Tritium is
also a beta emitter (with a half-life of 12 years) and consequently
is widely used as a radioactive indicator in chemical and biological
investigations. The positions of spectral lines of tritium are slightly
displaced relative to the hydrogen and deuterium lines (see Fig.
13.5) and are given by the equation

»?■"”" = (13-87)
Another very important consequence of accounting for the
motion of the nucleus was the discovery of ionized helium, first
detected in spectroscopic studies of the sun. The solar spectrum
was found to contain a series of lines, with positions described by
the equation

(13.88)

E n e rg y i s r e l e a s e d in th e f u s i o n o f d e u t e r i u m a n d tr itiu m in t o h e l i u m , j u s t a s in t h e
f issio n of U or P u ^ u n d e r n e u t r o n e x c i t a t i o n . F u s i o n , h o w e v e r , i s p o s s i b l e only
if Ihe p o t e n t i a l b a r r i e r o f th e C o u lo m b r e p u l s i o n b e t w e e n t h e D a n d T n u c l e i i s o v e r c o m e .
H ig h .'ftipera tu res 1 0 8 d e g r e e s ) a r e t h e r e f o r e r e q u i r e d i f t h i s r e a c t i o n i s to b e s e l f - s u s ­

t a i n i n g , w h e r e a s in t h e c a s e o f f i s s i o n e v e n l o w - e n e r g y n e u t r o n s c a n e a s i l y p e n e t r a t e t h e
n u c l e u s , an d t h a t e v e n a t low t e m p e r a t u r e s . T o o b t a i n a t h e r m o n u c l e a r r e a c t i o n , a m i x t u re
o f d e u t e r i u m a n d tritiu m m u s t f i r s t b e h e a t e d to a t e m p e r a t u r e o f t e n s o f m i l l i o n s o f d e g r e e s .
S u ch t e m p e r a t u r e s may b e c r e a t e d in a n a t o m i c e x p l o s i o n .
THE T HEORY OF THE HYDROGEN-LIKE ATOM 227

where n, takes the values

(13.89)

This series is, in effect, the Balmer hydrogen series (n, = 3, 4, 5,...)
with a number of intermediate lines, which form the so-called
Pickering series, characterized by the half-integral quantum
numbers n ,= 5/2, 7/2, 9/2........... At first, the Pickering series
was explained by assuming that hydrogen was in a special state
in the Sun, so that the quantum number n could assume half­
integral values. The spectral lines, however, were later found to
be located further to the right than is indicated by Eq. (13.89),
and consequently this assumption had to be abandoned. The second
hypothesis assumed that the observed spectrum arises from singly
ionized helium (,Hei)+ , whose nuclear mass is M = 7360m0 and
whose charge is Z = 2. According to (13.83), its radiation fre­
quencies then are

(13.90)

Setting n' = 4, we reduce (13.90) to the form

(13.91)

where n = 5, 6, 7, 8, . . . .
To answer the question of whether the Pickering series was
due to the radiation of hydrogen atoms (under the assumption that
the quantum numbers may assume half-integral values) or to the
radiation of ionized helium atoms (with the usual integral values of
the quantum numbers), it was necessary to find an experimental
value of the Rydberg constant. In the case of hydrogen.

(13.92)

whereas for helium

(13.93)

Careful spectroscopic studies confirmed that the Rydberg constant


has the value (13.93), and thus it was shown that the Pickering
series represents the spectrum of ionized helium.
228 NON R E L A T I V I S T I C QUANTUM M E C H A N IC S

Problem 13.1. Starting with Eqs. (13.26) and (13.28), show that the radial wave func­
tions R ni for the principal quantum numbers /: = 1, 2 and 3 are as follows:

Ri<t = 2Ne~f/2,
Ne~**{ 2 - p ) ,
'2/2
Rn ■ N e ~f/2 p,
'2/6
N e ~ ?/2 (6 — 6p -f- p3),
9 / 3
Ne~ p/2p ( 4 - p ) ,
9 /6
Ru = -—
9 /3 0

where

Zy/i . 2 Zr
N= — and p = ----- .
\j0/ nao

On the basis of these specific examples, show that the functions Rni are orthonormal,
that is.

r*R nlRn'l *r ®n/T*

Hint. In proving orthonormality, make use of the fact that at different n and n', the
corresponding values for p also differ from one another.

Problem 13.2. Show that the average electrostatic potential produced by a hydrogen­
like atom in the ground state is

2Zr
’ 00
r ^ \a 0 ^ r } \ m0e-J

Hint. Find the average electrostatic potential produced by the electron <I>j (r) =

— f o ^ ’I'* (r1) i" ( f ) d*x' and add it to the potential of the nucleus. Integrating
over the solid angle W, use the Identity

r'<r,
= 4it
§ r' > r.

Problem 13.3. Show that at Z = 1, the maximum of the probability density distri­
bution D = r !R J’ in the states Is, 2p and Ad occurs at distances of tia, 4fl,i and <)u„ from
the nucleus (circular orbits). Why do the 2s, As and Ap stales have several maxima
(see Fig. 13.6)7
THE THEORY OF THE H Y D R O G E N - L I KE A T O M 229

D!r) rs n;r} V(r) 3s

r (I;,
U(r) ?P D(r) 3p D(r) 3d

f=9aa r

Fig. 13.6. Graph of the radial probability density distribution D = fit­


te r various states.

Problem 13.4. Using the functions given in Problem 13.1, verify the following equa­
tions for n = 1, 2 and 3:

and

Find the spread (3r)- = r2 — r- of the radial deviations for these states. ___
On the basis of Fig. 13.6 and the uncertainty principle, explain why (&r)adoes not
vanish for circular orbits in the quantum theory.

Problem 13.5. By means of the quasi-classical WKB method, find the discrete energy
spectrum of the hydrogen-like atom.
Solution. According to (5.75) and (13.62), the eigenvalues of the discrete spectrum
(E <t 0) may be found from the equation

ri
where r i andr2 are roots of the integrand and
2 w0E „ nioZel
T '
Since the value of this integral is

I
we obtain exactly the same expression for the energy of the hydrogen-like atom as via
the Schrodinger theory.

8T h i s i n t e g r a l c a n b e e a s i l y e v a l u a t e d from ( 1 3 .6 4 ) b y s e t t i n g a = —A, b = 2B a n d
c = —(/ + 1 / 2 ) 2, a n d r e g a r d i n g Tj a n d r 2 > r j a s r o o t s o f t h e e q u a t i o n f (r) = 0. T h e l o g a r ith m
of t h e co m p le x q u a n t i t y [ a s s u m i n g t h a t a t e n d s to z e r o from t h e d i r e c t i o n o f p o s i t i v e num ­
b e r s ( a = + 0 ) | m ay b e t a k e n as :
, . ( i'O > 0 ,
In (fi + l a ) = In /3 + ] .
/ nr 0 < 0 ,

, | I i2 n fi > 0 ,
In — Ia) = + | p<0
230 N O N R ELA TIV ISTIC QUANTUM M E C H A N IC S

Problem 13.6. Determine that magnetic field intensity at the center of the hydrogen
atom which is due to the orbital motion of an electron. Find its numerical value for the
2p state. __ __
Answer. t l x = Hy = 0,

m0el\3 1______
H, = — m
ni0c

For the 2p state (m = 1)

Hz ~ 101 gauss.

Hint. Take the classical expression H = — r x /> for the intensity of the

magnetiv field produced by a moving charge. Transform to a quantum treatment of the


problem by replacing r x p by the_angular momentum operator L. Then, using Eq.
(13.29a), calculate the average value of H.
Chapter 14

Time-Independent Perturbation Theory

A. BASIC PRINCIPLES OF THE PERTURBATION


TREATMENT OF PROBLEMS

A relatively large number of problems in quantum mechanics


cannot be solved exactly with present-day mathematical methods.
Thus, various approximate calculations have to be used. One of
the most widely used approximations is the perturbation theory
method, which was first developed to handle problems in celestial
mechanics. It is well known that in Newtonian mechanics it is pos­
sible to solve exactly only the two-body problem (for example, the
Earth-Sun or the Moon-Earth problems). We cannot, however, ne­
glect interplanetary forces and consider only the attraction of the
planets by the Sun since many delicate phenomena are associated
with these additional interactions (it is worth recalling in this con­
nection that Leverrier predicted the existence of Neptune on the
basis of the orbital deviations of Jupiter, after which the planet was
discovered by astronomers). It thus became necessary to consider
the many-body problem, which has no exact solution in classical
mechanics. In celestial mechanics it was found that the perturba­
tion problems could be handled by means of an approximation based
on the fact that the forces between the planets are much sm aller
than the force of attraction to the Sun. In this method, one starts
by solving the two-body problem (the zero-order approximation),
then takes the “ perturbation” into account and finds the correction
to the solution (the first-order approximation). In other words, the
“ perturbation method” involves taking the principal forces acting
on a body, finding the rigorous solution for these forces, and then
taking the “ perturbing” forces into account.
Similarly, in the quantum-mechanical treatment of the motion
of several electrons in an atom, it is necessary to consider first
the principal forces, such as, for example, the force between the
nucleus and an electron. In this case, the perturbing forces may
be taken to be the Coulomb forces of mutual repulsion between the
electrons. In the problem of an atom subjected to an external
electric or magnetic field whose strength is small relative to the
electric field of the nucleus, the perturbation may be taken to be
the energy of the electron in the external field.
232 N O N R ELA TIV ISTIC QUANTUM M E C H A N IC S

B. FUNDAMENTAL EQUATIONS OF PERTURBATION


THEORY

We shall now develop the perturbation theory in a form suitable


for stationary problems, that is, problems in which the Hamiltonian
of the system does not depend on time. Suppose the Hamiltonian
has the form
H = T + K = T + F° + l/\ (14.1)

where the perturbation energy V' V°, and the main part of the
potential energy F° is chosen in such a manner that the Schrodinger
equation of the system
( £ — H) = 0 (14.2)

has an exact solution, characterized by £°and i]>°, when the pertur­


bation V’ is neglected. Setting T = H° (the zero-order approx­
imation) and using Eq. (14.1), we can write (14.2) as

(£ — H° — V') = 0. (14.2a)

The basic problem in perturbation theory is to calculate from this


equation the energy values En and the corresponding eigenfunctions
The solutions are sought in the form of the series

* = io + <|/ + * " + .... (14.3)

£ = £° + £'-{-£" + ..., (14.4)


where and £' are terms of the first order of sm allness relative
to 'jn; £°, 0" and £" are terms of the second order of sm allness;
and so on.
As a rule, the perturbation energy V can be represented in the
form of a potential energy of the same order as V'0, multiplied by
some small parameter X(X<^I). The solutions (14.3) and (14.4)
should then appear as expansions in terms of X. Thus, £° and 6°
will be independent of this parameter, £' and y will be propor­
tional to X, L" and y to X2, and so on. In the expressions for and
£, let us restrict ourselves to terms of the first order of sm all­
ness (that is, we shall retain only terms which are independent of
X or directly proportional to X). Substituting (14.3) and (14.4) into
(14.2a), we obtain the following equation for <]/ and £':

(£" -j- £’ — 1111— V') (’^° -[- ’I/) — 0. (14.2b)

Collecting terms of the same order, we have

t£° - 11") ■/’ 4- [(£' - V') y + (£» - 11")'/) + ( £ '- V') 'J/=0. (14.2c)
TIME-INDEPENDENT PERTURBATION T HEORY 233

Equation (14.2c) may be regarded as an exact equation, since we


have not neglected any term s in it, and since <]/ and £' may be taken
to represent the sums of all terms of different orders of sm allness
(that is, , £ '-* £ ' + £ " + ...). To obtain the first-
order approximation of perturbation theory, we neglect the second-
order term (£'—V') <j/ in (14.2c) and use the equation for the zero-
order approximation
(£° — H»)ft = 0. (14.5)

This equation yields the zero-order eigenvalues

*p-’!»
o po pn po
*‘“•tli •••j

and eigenfunctions
r?i° t»i° ,»in
Vl> ?■>> ?3i •••■
i!.°
?«> •••>

which are connected by the relationship


(£;)■ — H°) f t = 0. (14.6)

Keeping all this in mind, we shall now investigate the equation for
the first-order approximation of perturbation theory

(£u— H°)<j/ = — (£' — V') f t (14.7)

We assume that at the beginning, the system is in the quantum


state n' = n. Since £" = El and = ft in the zero-order approxima­
tion, and £' = £„ and'/ = r4> in the first-order approximation, Eq.
(14.7) becomes

{El - H") ft = - ( £ „ - V') ft. (14.7a)

We recall that an arbitrary function can be expanded in a series


of orthonormal functions forming a complete set and satisfying the
same boundary conditions as the original function; therefore, we
may assume the solution for ft to be of the form

ft = 2 f t f t . (14.8)
n‘
Our problem, therefore, reduces to determination of the unknown
coefficients C„■of a generalized Fourier series. Substituting (14.8)
into (14.7a), we have

2 ' Cn. {E%- H°) ft- = — (£; — V’) ft. (14.9)


234 N O N R ELA TIV IST1C QUANTUM M E C H A N IC S

or, taking into account (14.6), we find

2 cn. (£• - El’) W = -(£;- V') $•. (14.9a)

C. NONDEGENERATE CASE

Let us suppose that the system is nondegenerate. Therefore,


each energy eigenvalue Et, corresponds to one and only one eigen­
function t|)J. Then, multiplying Eq. (14.9a) on the left by <ji£* and
integrating over all space, we have

2 cn. (£ • - £ * .) Kn' = - £ ; + j r * V y ncPx. (14.10)


n'

In obtaining this equation, we have made use of the orthonormality


of the eigenfunctions '•}>£:

J W -' <?X=lnn..

Since the left-hand side of Eq. (14.10) is equal to zero (E°n— ££. = 0
for n’ = n, and 8n«' = 0 for n' n), the energy correction E'n is

E'n = V'n„, (14.11)

where

V'nn=^<ln*V'<fnd*X. (14.11a)

Thus, the energy correction E'n of the system quite naturally turns
out to be equal to the average value of the perturbation energy V'.
It is worth noting that the expression (14.11) for the energy
correction was obtained by setting the left-hand side of Eq.
(14.7a) equal to zero after it had been multiplied by <JC and in­
tegrated over all space. Since is a solution of the homogeneous
equation (14.6), it follows that the right-hand side of the inhomo­
geneous equation
Wj = f (14.12)

is orthogonal to the solution of the corresponding homogeneous


equation AV/ = 0 , that is,

{ <p*fdix = 0 . (14.13)
T IM E -IN D E P E N D E N T PER TU R BA T IO N THEORY 235

Let us now proceed to determine the wave functions (that is,


the coefficients C„ ) of the Schrodinger equation (14.7a) in first-
order perturbation theory. We write Eq. (14.9a) as

2 C„ ” (EJ - Ei» ) = ■ - (E ; - V)

Multiplying on the left by (n’ ^ n), using the orthonormality


condition, and integrating over all space, we obtain

^ = ’ (14.14)

where

^ = £ ^ '< 1 ^ . (14.15)

Thus, for ^ we have

= ^5 + 2 ’ (14. 16)
n’

where the prime on the summation symbol indicates that the sum
is taken over all n' except n' = n. Finally, the as yet undetermined
coefficient Cn of the zero-order wave function can be found from
the normalization condition

J < ^ nc f * = l (14.17)

for the total wave function

= tynT 'i'n= Cntyn ~f"2 (14.18)


rt'

where
c ;= l+ c „ . (14.19)

Substituting (14.18) into (14.17) and keeping only terms up to


the first order, we have

ICJI* J W<t'n<Px +
+ 2 { c r c n-
nr
J r*¥ n ' CPx + Cn'Cn J « dh) = 1. (14.20)
236 N O N R EL A TIV ISTIC QUANTUM M E CH A N IC S

Making use of the orthonormality condition and ignoring the phase


factor, which is of no interest to us, we obtain

C 5 = l. (14.21)

and therefore C„ = 0-
Consequently, for the wave function in the first-order
perturbation theory, we finally obtain

vv I/’*
% = 'r’n 4* 2 E° — Et. ■«' (14.22)

From Eqs. (14.22) and (14.11), it can be seen that both ^ and
E'n are proportional to the first power of the perturbation energy
(that iS, to the parameter X). If we were to compute the corrections
to the energy and the wave function in second-order perturbation
theory, both £" and ''/n would turn out to be proportional to the
second power of V' (that is, to X2).

D. DEGENERATE CASE

We shall now develop the perturbation theory in a form applica­


ble to degenerate system s, in which, in the absence of any per­
turbation, a given energy eigenvalue E% has associated with it j
eigenfunctions
6° y n 2t . . . i iti
Y/ii> Y 0/ y

It is obvious that any linear combination of these functions

j
(14.23)
/=*i

is itself a solution of the wave equation in the zero-order approxi­


mation

(£,; — n °)^ = o,

in which the eigenvalue of the energy is E'„.


If there is a perturbation V’, this arbitrariness is removed and
thf> coefficients C,! may become connected with one another by
certain definite relationships. Let us show this, proving first that,
just as in the case of nondegenerate states, any particular solution
of the homogeneous equation (14.6) is orthogonal to the right-
hand side of the inhomogeneous equation (14.7a) of first-order
TIM E-INDEPENDENT PERTURBATION THEORY 237

perturbation theory. For this purpose, we multiply (14.7a) on the


left by and integrate over all space. We then obtain

J {E"n— H") Y d'x = - J Yn* (En - V’) Y Px. (14.24)

Using the theorem for the transfer of a derivative [see Eq. (7.17)],
we have

j Y (En ~ H") tPx = - j Yn* (E'n - V’) %<Px. (14.25)

Since Y* is a solution of the Schrodinger equation (En— H0)'^* = 0,


we arrive at the equation

f Y* (E'„ - V') ^ C lY , d3jc = 0- (14.26)


i’= l

Without any restriction on the generality, we may assume that all


the eigenfunctions Y>r are orthonormal. 1 Then, since

J '^n'lYni,d3X='bn.nl,'
we obtain instead of (14.26)

Q (En- V'u) = v> ' Q-Vir, (14.27)

where

Vu= ^ Y n lVyni (Px, (14.28)

V'u’= J YSiV'Y«e (Px, (14.29)

and the prime on the summation sign indicates that the sum extends
over all i” s, except /' = /. Since the subscript i in (14.27) can take
any value from 1 to j, we have a system of j homogeneous equations
from which we can determine the energy E'„ and the coefficients C;:

c? (e; - 1/;,) - Clvti - . . . - qvij = o.


- Qv:n + c,{En- f.,2) - . . . - c;vv =o,
............................................................... (14.30)
- c y ii - q n - „ . + c ;(£ ;-i/rt) = o.

*If t h e f u n c t i o n s 0 ° . , a r e n o t o r t h o n o rm a l , it i s a l w a y s p o s s i b l e to c o n s t r u c t from them


by m e a n s of l i n e a r t r a n s f o r m a t i o n s , n e w f u n c t i o n s w h ic h p o s s e s s t h i s p r o p e rt y .
238 N O N R EL A TIV ISTIC QUANTUM M E C H A N IC S

Recalling that the wave function % must satisfy the normalization


condition

. (14.17a)

we see that the correction E'n to the energy of the unperturbed


state Ek of the system is uniquely determined, as are the coeffic­
ients C'l (and hence also ^ ). Since, in particular, the system of
equations (14.30) can have a nontrival solution only if its determi­
nant is equal to zero, we have the following equation for E'„:

(E'n- V n) , - V u...... - V [ ,
- V n, (En-V'n)....... -V',1
= 0. (14.31)

- V f r - V ' f i , .... (E'n-V'jj)

This equation is called the secular equation, a term taken from


celestial mechanics.
If the secular equation has several roots (not necessarily /
different roots), each one of them will correspond to a completely
determined set of coefficients C°, which can be found from (14.17a)
and (14.30) by substituting the given root Enii for E'n. Consequently,
the different corrections E'n to the energy lead to different zero-
order wave functions. Thus, if a system is /-fold degenerate in the
absence of any perturbation, a possible effect of a perturbation is
to reduce or completely remove the degeneracy [this will occur in
the case when Eq. (14.31) has j different roots].

E. THE STARK EFFECT

If an atom is placed in an electric field, its spectral lines are


generally split into components. This phenomenon was discovered
by Stark in 1913. Experiments have shown that the effect of an
electric field on a hydrogen atom is different than that on other
atoms. In a weak field, the splitting of the energy levels of hydro­
gen (for example, the Balmer series) is proportional to the first
power of the field intensity (the linear Stark effect), whereas for
all other atoms the splitting is proportional to the second power of
the field intensity (the quadratic Stark effect). In a stronger field
(of the order of 1 0 r* volts/cm ), there is an additional splitting ef­
fect, proportional to higher powers of the field intensity (second
power in the case of hydrogen). In very strong electric fields, the
spectral lines disappear completely.
The Stark effect could not be explained before the development
of quantum mechanics. In terms of classical concepts, the motion of
T IM E -IN D E P E N D E N T PER TU R B A T IO N THEORY 239

an electron in an atom can always be resolved into three mutually


orthogonal oscillations. Let us consider the oscillation along the
z axis, which can be taken to coincide with the direction of the
constant electric field E(Ex = Ev = 0, Ez = %). The equation describ­
ing the motion of the electron along the 2 axis 2 is (for e = — e0)

m0z m^\z = —e0 g, (14.32)

where m0 is the electronic mass and % is the angular frequency of


oscillations. It is readily seen that the solution of Eq. (14.32) is

*= - ^ + ' 4 co s(u,°/ + ?)- (14.33)

It is clear that in classical theory the only effect of the constant


force (—e0 g) is to change the position of the point of equilibrium
of the system. The frequency of oscillations remains completely
unaffected. Consequently, classical concepts imply that the fre­
quency of the radiation emitted by an atom remains the same
whether or not the atom is placed in an electric field, since the
frequency of the radiation is the same as the mechanical frequency
of oscillations of the atomic electrons. Accordingly, an electric
field cannot produce any shift of the spectral lines in classical
theory.
We shall now consider the Stark effect in terms of quantum
concepts. As we have just mentioned, there are two basic forms
of the Stark effect: namely, linear and nonlinear. The linear effect
is characteristic only of hydrogen-like atoms because they are
degenerate not only with respect to the magnetic quantum number
m, but also with respect to the orbital angular momentum quantum
number I (see Chapter 13), with which the linear Stark effect is
associated. In all other atoms, there is no degeneracy with respect
to I, and therefore the linear Stark effect is not observed.
Let us discuss in greater detail the theory of the linear Stark
effect for the hydrogen atom. We shall confine our treatment to
the second quantum level (n = 2 ).3 The external electric field
(which is of the order of 1 0 '*—1 0 s volts/cm in experiments) is much
weaker than the intraatomic field produced by the nucleus, which is

&n u c l. fly
• 1 0 s volts/cm ,

where is the radius of the first Bohr orbit. Therefore, we may


use the results of perturbation theory (in the form developed for

2T h e e l e c t r i c f i e l d i n t e n s i t y w i l l h a v e n o e f f e c t on t h e o s c i l l a t i o n s a l o n g t h e x a n d y
a x e s , w h i c h a r e p e r p e n d i c u l a r to E .
3T h e f i r s t q u an tu m l e v e l (n = 1 ) i s n o n d e g e n e r a t e an d , t h e r e f o r e , i s n o t s p l i t .
240 NON R E L A T I V I S T I C QUANTUM MECHANICS

degenerate problems). For the perturbation V', we must take the


potential energy of the electron in the external electric field

k' = e0 §z- (14.34)


In the unperturbed state, the energy of the electron is [see Eq.
(13.33)]
£o___Bb
— 4 *

which has four wave functions associated with it:

'h Ym,o Bi0(r)Y„ Rm(r)t (14.35)

^2 = ^,t,o = ( 0 ^ 5 = ] / ^ Bn (r) cosd, (14.36)

= = Bn (r) Y\ = Y \ Rn (r) p ± e!\ (14.37)

« = 4 -2 , = Bn (r) 7r‘ = - Y J B n (r) — er* (14.38)

When & and 9 are replaced by Cartesian coordinates, these wave


functions take the form:

4’i = / i (r). (14.35a)

Vi = fi(r)z, (14.36a)

(14.37a)
H

(14.38a)
Jl
1

where

U{r) yr«Rn{r)’ (14.39)


h (r)= Y l^ Y .
TIME-INDEPENDENT PERTURBATION T HEORY 241

The general wave function of an electron will have the form


4

(14.40)
i

Since the degree of degeneracy is four (/ = 4), we have the


following system of four equations for determining the unknown
coefficients E* and the corrections C° to the energy E\ of the un­
perturbed state:
Q (E% V'(,) C^Via — CIVi3 C\V 14 = 0,
- CtV'n + C5 (£i - V'n) - C®rn - C\V'U= 0,
- C \ V - ClV'n + Cl (El - V'„) - QV'm= 0,
- a— 4 j — CIV43 -f- C\ ( £ 2 — y 44) = 0,

where

V h = J rm }d*x =

= e ,& ^ W z ¥ ‘nPx. (14.42)

When we integrate over the volume, the matrix elements

^ n . V22> 1^331 VM , V13, V l4, V2 i , V24, V31, 1/32, V34, V4 l , V 42 and V 43

vanish, since for each of them the integrand must be an odd func­
tion with respect to at least one of the coordinates x, y and z. Only
the matrix elements

V19 and Vj| — V13,

which are even functions of all three coordinates, differ from zero

l/ n = ' / -2 i = Co& (14.43)

Let us substitute the values of Eq. (14.39) for /, (r) and (r) and
note that, according to Eq. (13.28a),

R»= — (2 -
2 1^2 a 3/* \ °o/

and
242 NONRELATIVISTIC QUANTUM MECHANICS

Then, integrating (14.43) over the angles 0 and 9 (remembering


that z = r cos b), we obtain

^ = = °>dr.(14.43a)

Next, taking into account the equality

e -y dP= r(s+l),
0

we obtain
= — 3^60,. (14.44)

Using the obtained values of the matrix elements Vn, we find,


in accordance with (14.31), that the secular equation for the correc­
tions E'i is

E:| 3a en8 0 0

3a e0 8 E\ 0 0
(14.45)
0 0 0

0 0 0

which can be rewritten in the form

£ '= (£ /- 9 a M 8 4) = 0 . (14.45a)

This equation has four roots:

E-t 1 —
£ ; i2'= 3a0 e0 8, (14.46)

each one of which, according to Eq. (14.41), is associated with a


quite specific set of coefficients:
C?' = c?- c,r = C1“ = 0
cr= -cr- c? = Cl,,= 0
(14.47)
C 7=C 7 = 0 ; c.r , C? 0.
C,*' = C;’= 0; c r . Ci" ^ 0 .

Here the superscript / of the coefficients OJ' — Cj indicates the so­


lution (or root) of Eq. (14.45a) with which it is associated. Thus, it
follows from (14.40) and (14.47) that the energy level

£," = £ 5 + £;"■ = — 3e(la„ 8 (14.48)


T IM E -IN D E P E N D E N T PER TU R B A T IO N THEORY 243

is associated in the zero-order approximation with the wave func­


tion

f (» = Ci,' ( K o , o + ^., .o).

Because of the normalization condition

y (p x = i,

this wave function becomes

+0 ( ,) = 7 ^ #.#+ ' ^ ‘.,)- (14-49>

In a sim ilar fashion, it can be shown that the state with energy

£i!'= E°2 + £ ; is’ = - ^ - f 3a0 e0§ (14.50)

is described by the zero-order wave function

'ij0(2) = 7 f (^ #. o - K ,.o)- (14.51)

To describe the states with energy


(3)
-- £>EV4>
% --- CO

£ •2

which remain unperturbed by the electric field in the first-order


approximation, we may equally well take the function

f (3, = <Ki,. (m = 1 ),
or

(m = — 1 )

or, alternatively, we may take a linear combination of these func­


tions, since the system remains degenerate for m= + I even in the
presence of an electric field. Thus, when the z component of the
angular momentum is not equal to zero (m = ± 1 in units of fl), that
is, the electron is moving largely in the xy plane (see Chapter 12,
Section A), there will be no splitting of the energy levels (and,
therefore, of spectral lines) in an electric field. If, on the other
hand, the orientation of the angular momentum is such that its z
component is equal to zero /n = 0 and, consequently, the electron
244 NON R E L A T I V I S T I C QUANTUM MECHANICS

is moving in a plane which includes the z axis (again, see Chapter


12, Section A), then an electric field does lead to splitting of the
spectral lines (see Fig. 14.1).

----------------------- T +3e*°oS
-Ri Rh
T j
------------------------------% - 3 e 0 a0 6
a b

F i g . 14.1. S p l i t t i n g o f t h e s e c o n d s p e c t r a l ter m o f h y d r o g e n in
a n e l e c t r i c f i e l d ( t h e l i n e a r S ta r k e f f e c t ) :
a) e n e r g y l e v e l in t h e a b s e n c e o f a f i e l d (S = 0 );
b) e n e r g y l e v e l in t h e p r e s e n c e o f a f i e l d ( & ^ 0 ).

Qaulitatively, the Stark effect for n — 2 may be interpreted as


follows. Since the wave function describing the motion of the
electron for n = 2 (see Fig. 12.1) is not centrally symmetric, the
atom has a certain electric moment p . Consequently, when the
atom is placed in an electric field

(Ex = Ey = 0, £ z = &),

it acquires an additional energy

V' = — (pE) = — p& cos 7 , (14.52)

where 7 is the angle between the direction of the electric dipole


moment of the atom and the z axis. Comparing this expression
with (14.46), we see that the electric dipole moment of the atom
is p = 3aue0. The solution mcorresponds to the case 7 = 0 and
the solution '/M2) to the case 7 = -. For the third and fourth solu­
tions, it is necessary to set 7 = ± :-. We note that, in the last case,
the electric dipole moment is oriented perpendicularly to the elec­
tric field and, consequently, no additional energy appears. To con­
clude this discussion, the linear Stark effect arises because of the
intrinsic electric dipole moment of the hydrogen atom at n = 2 .
Predictions obtained on the basis of quantum mechanics are in
good agreement with experimental data only in the case of weak
fields (101 volts/cm ). At higher field intensities (~1(F volts/cm ),
there is an additional splitting (the quadratic Stark effect) due to
the removal of the degeneracy with respect to the magnetic quan­
tum number in. Finally, at field intensities greater than 10" v o lts/
cm, the Stark effect completely disappears. This is the result of
autoionization of the atoms, that is, removal of electrons from the
excited levels.
TIM E-IN D EPE N D E N T PERTURBA TIO N THEORY 245

F. PRINCIPLES OF THE CLASSICAL THEORY OF


DISPERSION

Perturbation theory has many important applications in studies


of the interaction between light and matter. The predictions obtained
in this way differ from classical results and receive excellent
confirmation from experiment. This section is concerned with the
classical theory of dispersion (that is, the scattering of light) in
a dielectric medium. According to classical notions, a dielectric
is characterized by the index of refraction

n= V e”,
where s is the dielectric permittivity (the magnetic permeability
(i is taken to be equal to unity). If the index of refraction n becomes
larger as the frequency of light increases (that is, — ^>0 ),th e
dispersion is said to be normal. A typical example of normal
dispersion is the spectral resolution of white light by a glass or
quartz prism (the deflection of violet rays from their initial direc-
dti
tion is larger than that of red rays). If, however, ^ < 0 i n a certain
range of frequencies, the dispersion in this region is said to be
anomalous. As a rule, anomalous dispersion occurs at frequencies
at which light is absorbed by the medium.
To determine the index of refraction (one of the most important
problems in the theory of dispersion), we use the equation relating
the electric field intensity E, the displacement vector D, and the
polarization vector P 4

D = e E = E ^ 4 -P . (14.53)
Since t = we have

P =n
^ E . (14.53a)

Thus, to determine n we are required to find the relationship


between P and E on the basis of the microscopic picture of the
structure of matter. We shall be better able to appreciate the
contribution of quantum theory in this connection after we have
completed our review of the basic principles of the classical theory
of dispersion.
According to the Lorentz classical theory, atoms may be
regarded as harmonic oscillators in which, in the sim plest case,
all the electrons oscillate with the same angular frequency u>. If

T h e p o l a r i z a t i o n P i s d e f i n e d a s t h e t o t a l d i p o l e m o m en t of t h e a t o m s p e r u n i t v o lu m e.
246 NON R E L A T I V I S T I C QUANTUM M E CH A N IC S

we take the z axis to be parallel to the direction of propagation of


the electromagnetic wave, then, since the wave is transverse, we
may direct vector E along the x axis (Ex = &, Ey = Ez = 0), and
vector H along the y axis. If we neglect the force exerted on the
atomic electrons by the magnetic field (since the magnitude of this
force is only a fraction ^ 0 of the force exerted by the electric
field), the oscillation of the electrons can be described by the
equation
m0x -j- m0u>lx= — e0 g. (14.54)

We shall assume that the frequency of the incident light wave is o>.
Then

6 = So cos {mt — . (14.55)

If the energy transported by the wave is much sm aller than the


bonding energy of the electrons in the atom, it follows that the
ratio ^ may be neglected because it is sm all compared to the
atomic dimensions ( x ~ a ^ 1 0 ~s cm, while the wavelength l i s of the
order of 10~“ cm). Thus, under the assumptions we have made, the
electric field of the wave may be considered to be quasi-stationary
inside the atom. As a result of this simplification, Eq. (14.54)
becomes
iJ+ (o2*= — n
—iff^g = — ■

Wo'gocosiirf. (14.54a)

Multiplying (14.54a) by (— e„N), where N is the number of atoms


per unit volume, and substituting
— e„x = px, and Px = Npx = ®‘, Py = P2= 0,

we reduce this equation to the form

& -f- (oJeT5 = g0 cos mt. (14.54b)

from which we obtain


0 »= Ne<
Wq cos ait. (14.56)

Comparing Eqs. (14.56) and (14.53a), we obtain an equation for the


index of refraction which should be familiar from optics:

“S—«■ (14.57)
TIME-INDEPENDENT PERTURBATION T HEORY 247

We note that if the atom is assumed to contain electrons with


different eigenfrequencies

“ o. “ I. “>9. • • • . “ a >•••>

a more general equation is obtained instead of (14.57):

na— 1 jL y n* (14.57a)
An rrto Zj “ 1 —<->3 *

where Nk is the number of electrons per unit volume oscillating


with the frequency <«*.
It follows from (14.57) that at radio frequencies (u> <u0) the
index of refraction may be assumed to be constant, without signif­
icantly affecting the accuracy. Its value is given by the relation

n1 — 1 _ _ Ne\j
4t. ~ m0 w§ (14.58)

On the other hand, for frequencies u)J>u>0, the value of the index of
refraction is given by the relation

—1 Ne\
4t. ~ m0“* ' (14.59)

The index of refraction is, therefore, a constant greater than unity


for <D<^a)o , whereas for it is less than unity, approaching it
as (u—■co.
At frequencies close to u>0, the magnitude of the index of refrac­
tion increases without limit, and at o>= co0 it has a discontinuity
(see Fig. 14.2). The reason for this be­
havior of the function is that Eq. (14.54)
does not include the radiation damping of
_ o
the electron /•’damp — 3 c3 ’ which arises
from the interaction between the moving
electron and its own field. When Edamp is
included, the dispersion curve has the form
indicated by the dotted line in Fig. 14.2.
Consequently, the dispersion is anomalous
near the resonance frequency io0. Since the
region of anomalous dispersion coincides p e r s i o n c u r v e \ m0 “0 /
with the region of the eigenfrequencies of
oscillation of electrons in the atom, it follows that anomalous
dispersion is accompanied by strong absorption.
248 N O N R ELA TIV IST1C QUANTUM M E C H A N IC S

F. QUANTUM THEORY OF DISPERSION

Let us now develop a quantum theory of dispersion. By analogy


with the classical case, we shall assume that all the electrons in
the atoms are in the same quantum state k.s We shall use the
perturbation method to solve our problem, since the energy of the
interaction with the external field is generally sm all compared
with the bonding energy of the electrons. In the nonrelativistic case
(when we may neglect the “ magnetic” force), the external force
which acts on an electron can be obtained from Eq. (14.55)6:

Fx = — e0gocos w t > Fy = F z = 0.

The perturbation energy is given by

V' = e0 xg0 cos coL (14.60)

Consequently, the Schrodinger equation for the electron is

( - ^ - H° - r ) < M 0 = °- (i4.6i)

Let us suppose that Eq. (14.61) has an exact solution for V"= 0:

(0 = ^ e - m)Ek‘ = , (14.62)

where <|4 and Ek satisfy the equation

( £ * - H ') ^ = 0. (14.63)

In accordance with perturbation theory, we shall look for a solution


in the form

= (O + MO- (14.64)

Since (14.62) and (14.63) yield

( - 7 £ - 1 1 ° ) *2 (0 = 0, (14.63a)

T his assum ption is analogous to t h e assu m p tio n made in t h e c l a s s i c a l tre a tm e n t,


a c c o r d i n g to w h i c h all the e l e c t r o n s h a v e the s a m e e i g e n f r e q u e n c i e s of o s c i l l a t i o n .
J u s t a s in t h e c l a s s i c a l c a s e , w e h a v e a s s u m e d h e r e t h a t t h e e l e c t r i c f i e l d i s q u a s i -
s t a t i o n a r y o v e r d i s t a n c e s of th e o rd e r of t h e d i m e n s i o n s of th e atom .
T IM E -IN D E P E N D E N T PER TU R BA T IO N THEORY 249

the equation for < } > * (0 and the first-order correction to the energy
£* is
(14.65)

Substituting V' from (14.60), we have

(— y — — H"j^(/) = - c 0 jcg0 '{'Me-wl“*“ “) + e -',f“*+“>}. (14.65a)

To eliminate the time t from this equation, let us look for a solu­
tion ty't, (() in the form
(t) — ue~ u (“*-") ve~11(“+<u/(). (14.66)

We then have the following equations for the functions u and v :

{*(“* — w) — H0} u = y e0 xgo<jj*, (14.67)

{h (coA-f- U)) — H0} V = y <?0 *g0 <]i*. (14.68)

We note that these two equations have exactly the same form.
Consequently, it is only necessary to solve Eq. (14.67) for u, since
the solution for v of Eq. (14,68) can be obtained from u by substi­
tuting — io for (o. Since the time does not appear explicitly in Eq.
(14.67), we can find u by the perturbation method in the form ap­
plicable to stationary problems. Thus we shall look for a solution
in the form of an expansion in eigenfunctions of the unperturbed
problem [see (14.8)]:

(14.69)
k'

where the tyl" satisfy the equation

(£ » » -H ')fr = 0 . (14.70)

Accordingly, we may reduce (14.57) to the form

ti J Ck- (»**.■ - <■») W, (14.67a)

where the frequency of radiation is

(14.71)
250 NON R E L A T I V I S T I C QUANTUM M E CH A N IC S

Let us multiply (14.67a) on the left by and integrate over all


space, taking into account the orthonormality of the eigenfunctions
(^ (Px = w e then obtain the following equations for the
coefficients C*- and the function u :

_ ■ gpSo xk'k (14.72)


2 ft «*,*+•'

2 ft ) ’ (14.73)
h'

where the matrix element x^k is equal to

Xk’k = ^ ifPxif%<Px. (14.74)

Substituting —o> for co in (14.73), we obtain an expression for the


function v:

v= Iq§o Xh'k (14.75)


2 ft “V* — “
W'-

From (14.64), (14.66), (14.73) and (14.75), it follows that the


total wave function I* (/) is

(0= c- - -?#- 2 -^r— ^ K* cosWt - tosinorf]j . (14.76)


' A ' '

From the wave function <})*(/) of the electron in the external


field, we can readily obtain the polarization vector of the medium
<SP. In the classical theory we had

& = N p = — Nenx.

To generalize this expression to the quantum case, we must replace


p by its average value. Then

& = N p = - N e , J yh{t)xt?k{t)d'x. (14.77)

Substituting from (14.76) and retaining only first-order terms


in we have
TIME-INDEPENDENT PERTURBATION THEORY 251

In deriving this expression, we used the relation

J j d'x—0,
which follows from the fact that the integrand is an odd function of
x. Comparing Eqs. (14.78) and (14.53a), we obtain the dispersion
formula

n2— 1 __ 2Ne\ V ak’k Ixk'k I2


4 71 tl ^ U>l.k U8
(14.79)
V
By introducing the new variable

2 Wlo
fk’k n ^k'k | Xk'k[ , (14,80)

which is called the oscillator force, we transform Eq. (14.79) to

n2 — 1 N e\ V I f k'k
47u m0 £ u2k,k—u: (14.81)
k'

Here let us make an observation sim ilar to the one made in regard
to the classical treatment: namely, if we had included the radiation
damping in the quantum-mechanical treatment, we should have
obtained a finite value of a* for frequencies u> in the neighborhood
of u>k'k (see Fig. 14.3a, the dotted line).

F i g . 14.3. D i s p e r s i o n c u r v e s .
a) p o s i t i v e d i s p e r s i o n (o)^=
b) n e g a t i v e d i s p e r s i o n

Equation (14.81) has a structure sim ilar to the classical equa­


tion (14.57). In actual fact, however, the quantum results are
fundamentally different from the classical results. From quantum
theory it follows that anomalous dispersion occurs in the neighbor­
hood of frequencies corresponding to allowed transitions, and not,
252 N O N R ELA TIV ISTIC QUANTUM M E CH A N IC S

as in classical theory, in the neighborhood of the eigenfrequencies


of oscillation of the electrons. This particular conclusion can be
seen to be directly related to the role of the oscillator force
in (14.81), which is specified by the matrix element xk'k [see (14.80)]
which characterizes the selection rules (and thus the allowed
transitions). This prediction of quantum theory was experimentally
verified by D. S. Rozhdestvens.kiy.
A second, very important difference from the classical results is
that quantum theory leads to negative dispersion (see Fig. 14.3b)—a
phenomenon which has no classical analog. This can be under­
stood by simply noting that when light is scattered by excited
atoms, it is necessary to take into account the states with
for which

fk<k lOk'k = * ft—- <C0 .

For these states the dispersion formula (14.81) becomes

n2- l = Ne% V \fk’k\


4n ma —<i) 2 ’ (14.81a)
k'
and the dispersion curve is represented by the dotted line in Fig.
14.3b. The experimental discovery of negative dispersion was made
by Ladenburg; thus, this prediction of quantum theory was also
confirmed.
Let us now find the value of the oscillator force /*>* and the
dispersion formula for a harmonic oscillator. The only nonvanish­
ing matrix elements in this case are [see (10.55)]

hk
xk1 1 . k V h<k+ ) and xk \.k
1

2/7 lo°7|)

“ By chance,” it turns out that the quantum-mechanical frequencies


of radiation are identical with the eigenfrequencies of oscilla­
tion

u4 , ii77= ,,)u and uja_ ! — «)„.


We thus obtain

f k \. k-- fk-l.k--------------- — It- (14.82)


Consequently (since^ = 1J, the dispersion formula (14,81) can
be written as
TIME-INDEPENDENT PERTURBATION THEORY 253

n- — 1 ___ M>ii_ It -|- I _ Nc- k ______ M ’S


■Iji ill,, 0 )^ — o)" rn Q u)’ — a)- ma toj- — u>
(14.83)

We can see that in this particular problem the quantum and clas­
sical theories yield the same value for the index of refraction n.
The phenomenon of negative dispersion is not observed. The
reason for this is that the regions of positive and negative disper­
sion coincide since I | 4 = I“*
■i, * I1, so that the stronger effect of
positive dispersion masks the negative dispersion.

G. RAMAN EFFECT

Let us consider the phenomenon of dispersion from the stand­


point of energy diagrams. Suppose that a photon with energy

e= Hm (14.84)

impinges on an atom with only three energy levels Ek"< ]£ * < £V


(see Fig. 14.4). In general, the scattering of this photon (that is,
dispersion) will be a second-order effect. The first form which
this process can take is absorp­
tion of the photon. This is ac- /
companied by excitation of an
electron from level k to some huj hoi'
intermediate state (which may ■Ex‘
even be a forbidden state7: see
Fig. 14.4,1) and, subsequently, ZP
by emission of a photon. If, as Ex-
a result, the electron returns hoi1 flU J
to its initial state, it follows
from the law of conservation of 11
energy that the frequency m of
the scattered photon is the same F i g . 14.4. E n e r g y d i a g r a m for p h o t o n
as the frequency u> of the inci­ s c a t t e r i n g:
dent photon. hei) i s t h e e n e r g y o f t h e i n c i d e n t p h o to n ;
hco , t h e e n e r g y o f t h e s c a t t e r e d p h o to n ;
Alternatively, the order of
I a n d II r e p r e s e n t e l a s t i c s c a t t e r i n g
the process may be reversed: (ho)^hcok i k a n d hco£ hajk k n); III a n d IV
the atom first emits a photon r e p r e s e n t i n d u c e d t r a n s i t i o n s (h e o ^ h c o ^ k
(see Fig. 14.4,11) and then ab­ or hco'^->hcok k u).
sorbs the incident photon. As
in the preceding case, the frequency u>' of the scattered photon will
be equal to the frequency <u of the incident photon if the atom re­
turns to its initial state.

7
More p r e c i s e l y , t h e law of c o n s e r v a t i o n of e n e r g y may b e v i o l a t e d in i n t e r m e d i a t e
s t a t e s . It i s r e q u i r e d to h o ld on ly in t h e f i n a l r e s u l t .
254 N O N R ELA TIV ISTIC QUANTUM M E C H A N IC S

Finally, resonance occurs when ui^uik>k. in this case, both


p rocesses—scattering and absorption of the photons—take place
(see Fig. 14.4, III); as a result of the last process the electrons in
the atom undergo induced transitions. The probability of these
transitions is given by the Einstein
coefficient fl**- [see (9.21)]. An
fib) tiuj' external field increases the number
of downward transitions (see Fig.
14.4, IV), which results in some
additional radiation proportional to
■ % the coefficient B**".
So far, we have been concerned
F i g . 1 4 .5 . T h e R a m a n e f f e c t . with cases in which atoms return to
Hco i s th e en erg y of th e in c id e n t their initial state after scattering.
p h o t o n ; H co', t h e e n e r g y o f t h e s c a t ­ It may happen, however, that after
te r e d p h o to n c o r r e s p o n d i n g to “ S to k e s *
l i n e s ; a n d H c jn , t h e e n e r g y o f t h e
the atom has absorbed the incident
sca tte re d pho to n c o r r e s p o n d i n g to photon, the electron does not return
“a n ti-S to k e s " lines. from the intermediate state to the
level k, but instead makes a transi­
tion to the level k' or k" (see Fig. 14.5). In this case, the frequency
of the scattered light (uj' or u>") is not equal to the frequency of the
incident light. This type of scattering is called the R am an effect,
after the Indian physicist who first discovered this phenomenon in
liquids. In solids the Raman effect was discovered by the Soviet
physicists L. I. Mandel’shtam and G. S. Landsberg (1928).
From Fig. 14.5, it can be seen that the frequency of the scattered
photon may be either lower or higher than the frequency of the
incident photon. In the former case, the lines

u/ = u) — to*'* to

correspond to excitation of the atom, since the atom ends up in a


higher energy state. These lines are known as “ Stokes” lines (the
levels are shifted towards the red part of the spectrum). The
second case corresponds to “ anti-Stokes” lines (shifting towards
the violet part of the spectrum):

“>"= “>+ mkk" ]> u>;

these lines appear only when the light is scattered by excited atoms.
It is obvious that at low temperatures only Stokes lines can be
observed. As the temperature increases and the atoms of the
substance begin to undergo transitions to excited states, anti-
Stokes lines appear.
The Raman effect provides much important information in
studies of molecular structure. In Chapter 12, Section C, we saw
TIME-INDEPENDENT PERTURBATION THEORY 255

that the rotational and vibrational levels (and also the vibrational-
rotational levels), which provide data on molecular structure, are
all located in the far infrared region of
the spectrum and are very difficult to
observe. In studies of the Raman effect,
it is possible to use visible light in gI I ------------ 1
CO
determining molecular spectra, since a/ of
these spectra are superposed on the bI I I I
lines in the spectrum of the incident
light. The experimental values of <o' and F ig . 14.6. S u p e r p o s i t i o n o f th e
a>" (see Fig. 14.6) yield the molecular m o l e c u l a r f r e q u e n c i e s on t h e
frequencies f r e q u e n c y of i n c i d e n t l ig h t:
a ) s p e c t r a l l i n e a> in t h e a b ­
s e n c e of m o lecu lar o s c illa tio n s ;
aik'k = u>—«/ and Whh"= id"— u),
a n d b) s h i f t o f t h e s p e c t r a l l i n e
d u e to m o l e c u l a r o s c i l l a t i o n s
from which the selection rules can be ( < u '= co— b>k ' k a n d a i " = “ A * ")-
derived.

Problem 14.1. Find the energy correction for a system in second-order perturbation
theory.
Solution. Including the terms up to and including the second order in the expansions of
the wave function 4 (14.3) and the energy E (14.4), and substituting these expansions into
the Schrodinger equation (14.2a), we obtain the equation

(£ “ - H °) V n = - ( £ ’„ - V )V n - K C

Since the solution 4^* of the homogeneous equation (£° — H°) 4„ = 0 is orthogonal to the
right-hand side and since we can substitute the expression (14.22) for 4 n ,we have

n'

The value of V'n,n is given by (14.15), and we have used the relationship

V
v /m , = V'h
n n*,
which holds for Hermitian operators.
We note that the second-order correction (14.85) to the energy of the ground state is
always negative, since all the other levels Eh' are higher than the level Eh, that is.
Eh' > Eh.
Problem 14.2. Using the results of the perturbation theory, find the energy of the
anharmonic oscillator including the terms up to the order of take the Hamiltonian of
the system to be equal to
maa-x-
H= t2m
;—0 + ■y\
where V’ = ax3 -(- fU4 (the constants a and ? are classical quantities).
Solution. The energy of a harmonic oscillator (V' = 0) is

Ert ^ flta (d 4/a).


Taking V' as the perturbation energy, the first-order approximation gives

£ ; = i / ; n = a (A- \ n + p(A-1u
256 NO N R ELA TIV IST1C QUANTUM M E CH A N IC S

It can easily be shown that


+ oo
(x% n = J
- oo
l+ „ |'x » ^ = 0,

since the integrand is an odd function.


To calculate the matrix elements P (x*)nn, we use the multiplication rule for matrix
elements (see Problem 10.4), obtaining

(X % n = ^ (* % k ( * \ n = ((* % . n V + ( ( * % . „)* + ( ( - a . it + 2>*.


k
Substituting the value of (je3)„fc from Problem 10.5, we obtain the following expression
for the first approximation of the perturbation energy En:

En= ( " 2 + n + ~t ) ■ <14-86>


Our problem, however, is not yet fully solved, since in the second-order approximation
there is a contribution proportional to — ~ fta which arises from the first term of
the perturbation energy ax3, and we must take this contribution into account. The
second-order contribution from the term $x* is proportional to —— ~ Tl\ and accordingly
it may be neglected in our approximation. The second-order correction arising from
the first term of the perturbation energy can be calculated from Eq. (14.85):

E’n= JlL V
tla ^ (n —il')
The only nonvanishing matrix elements are [see Eq. (10.55) and Problem 10.5]

(x 3 )n, n - i = (x 2 ) n Mn ( x ) n t n - 1 H " ( x 2 )n, n - s ( x ) n ~ 9j r t - l =

/ .- a \ __ / a *2 \ /*a v 3 ~ \ f ~ n (n 0 ( 11 2)
v ' /n. n -a — ('*• /n . n- 2 \ x )n -* > / i - a — * '■ 0 1/ g »

(-*■ ) n , n + i = = (''■3) n + i , /it

)rij /i+a — (•^3)n+8, m

h
where x 0=

Hence
/ III uwo

15 n- a-
«'iV «’ + « +
En (14.87)
4
P a r t II

R ela tiv istic Q uantum M echanics


Chapter 15

The Kl ei n-Gordon Scal ar Rel ati vi sti c


Wave Equation

A. RELATIVISTIC MECHANICS AND THE KLEIN-GORDON


EQUATION. RELATIVISTIC INVARIANCE

The Schrodinger wave equation is nonrelativistic: it is suitable


only for particles whose velocity v is much sm aller than the
velocity of light c. It is not invariant with respect to the Lorentz
transformations of the special theory of relativity since there is
an asymmetry between the time and space coordinates (the
equation contains a first derivative with respect to tim e, and
second derivatives with respect to the space coordinates). Ac­
cording to the special theory of relativity, it is necessary for the
time and space coordinates to be treated on the same basis.
It is interesting to note that the de Broglie relations

p = hk, E — %<0 (15.1)

are relatively invariant. In the Lorentz transformation, they


behave like a four-vector p^ with components

(15.2)

This indicates that it is possible to generalize quantum mechanics


to the case of particles traveling with a velocity of the order of
the velocity of light.
A method of extending the nonrelativistic wave equation in a
way consistent with the special theory of relativity was proposed
by Klein and Gordon in 1926. (This method was also put forward
by Schrodinger and by Fok.) The sim plest way of obtaining the
Klein-Gordon equation consists in taking the relativistic relation­
ship between the energy E, momentum p, and rest mass m0 of a
free particle

— cV — mjc* -0 , (15.3)
260 R E L A T IV IS T IC QUANTUM M E C H A N IC S

substituting the energy and momentum operators

E = *7Z~, p = - i h V , (15.4)

which act on the wave function | (r, (). Replacing tn0 by hk^fc and
dividing by tl2c-, we obtain 1

V2 ---- ______ £2 ) a —
c2 d t2 *n l v — & : A - ko ) ^ = 0- (15.5)

Here
xt - - x, x.3= y, x3= 2 , Xi = id (15.6)

(a double occurrence of the subscript ju in a term indicates that it


should be summed from 1 to 4). Since our initial equation was the
relativistic relationship (15.3), Eq. (15.6) is relativistically in­
variant and, therefore, it is symmetric with respect to the space
and time coordinates. We shall not attempt to prove the invariance
of the Klein-Gordon equation more rigorously, and shall now pro­
ceed to examine its properties.

B. THE CHARGE AND CURRENT DENSITY

As in the nonrelativistic theory, equations for the charge and


current density can be obtained on the basis of the equation of
continuity

s/- f + 4 r = ° - (15*7)

We multiply (15.5) on the left by •[)*, and the complex conjugate


of this equation [that is, Eq. (15.5) with <1** substituted for | ] by
•\. Then we subtract the second of the resulting equations from
the first

d2 d2 (15.8)
(r * - <!< "r r ) = 0 .

After some simple transformations, Eq. (15.8) becomes

;. ^ v 7 *} + L (;* -lt * - * £ r ] = 0 . (15-9)

Here a n d in t h e s u b s e q u e n t c h a p t e r s , w e s h a l l n o t w r i t e ^ a s a function of t, as was


d o n e i n t h e S< h o d i n p . e r t h e o r y . In t h e w a v e e q u a t i o n f o r m o n o c h r o m a t i c (/£ - c o n s t ) w a v e s ,
for w h i c h o n l y the lim e- in d e p e n d e n t part of the /' f u n c t i o n m u s t he considered, we shall
u s e tile e n e r g y e i g e n v a l u e i n s t e a d o f t h e o p e r a t o r .
T H E K L E I N —GORDON SCALAR RELATIVISTIC WAVE E QUATI ON 26

Defining the charge density and the current density as

P
. 'fi.L * d't (15.10)
2m0c2 L‘ M

and

ell
(15.11)
J 2im0

we note that these expressions satisfy the equation of continuity


(15.7). Moreover, they define a four-dimensional vector

(15.12)

where
A'i = id, j 4 = icp. (15.13)

The current density (15.11) is identical with the nonrelativistic


expression (5.21), and the charge density (15.10) reduces to the
nonrelativistic expression (5.20) when Substituting ih E
[see (15.4)] into (15.10), we obtain

d 5 .i4 )

which becomes the usual expression p= e<]>*fy in the nonrelativistic


approximation E ~ m„c2. Thus we have selected a normalization in
which the relativistic values of p and j reduce to the corresponding
nonrelativistic expressions when
It is worth noting that the definition of the particle density
(as distinguished from the charge density)

k a !-=5 ? 1 4 4 t] <15-15>
gives rise to some difficulties in the relativistic theory. The
Klein-Gordon wave equation is a second-order differential equation
and, therefore, both ^ and ^ can be arbitrarily defined at some
dt
given time t. Consequently, the density p0 (15.15) is not positively
defined, unlike the nonrelativistic probability density

Po= 'W’- (15.16)


262 R E L A T IV IS T IC QUANTUM M E C H A N IC S

Accordingly, the expression (15.15) cannot be interpreted as the


particle density (that is, the “ number of particles per unit
volume” ). The underlying reason for this is that the same rela­
tivistic equation describes particles with either positive or nega­
tive charge (and, indeed, it mesons, to which the Klein-Gordon
equation is applicable, may be either positive or negative in charge).
The quantity p0 , therefore, can have both signs.

C. RELATIVISTIC THEORY OF THE HYDROGEN ATOM


(NEGLECTING THE ELECTRON SPIN)

To treat the interaction of a particle with an electromagnetic


field (defined as usual by a vector potential A and a scalar po­
tential <I>), we introduce the same operators as in the nonrela-
tivistic case

F = ih-^ — e®, P = — ihV---- e—A. (15.17)

From (15.3), we can obtain the Klein-Gordon equation 2

{(«'* -g f ~ e4> ) 2 — c'( 'sv + - r ^ ) 4 — ^ c 4|<Ji = °. (15.19)


We shall use this equation to study the spectrum of the hydrogen-
like atom. Setting A = 0. and V = e& = — ~~ in (15.19), we have

^ + — { ( E - F ) ’ - m ^ } t = 0. (15.20)

Since the potential energy does not depend on time in this equation,
we can transform to the time-independent case by means of the
substitution

tp(r, t) = ty(r)e n (E+moca,, .

In this equation, we have not Included the rest mass energy m0 c4 of


the particle in the energy E. As a result, Eq. (15.20) becomes

^ ^ [(£ + "W* + -S -) - m0V ] * = 0. (15.21)*I

2T h i s e q u a t i o n c a n a l s o b e o b t a i n e d from t h e r e l a t i v i s t i c H a m i l t o n i a n for a p a r t i c l e in
an e l e c t r o m a g n e t i c f i e l d

II 1/ c 2( p - - A ' j I m b ' » <•<!> .(1 5 .1 8 )

It is onl y n e c e s s a r y to t r a n s f e r to t h e l e f t - h a n d s i d e , s q u a r e b o th s i d e s , a n d r e p l a c e
p an d A by t h e i r q u a n t u m - m e c h a n i c a l o p e r a t o r s .
T H E KL E I N —GORDON S C A L A R R E L A T I V I S T I C W A V E E Q U A T I O N 263

Just as in the Schrodinger theory, we shall look for a solution to


this equation in the form
<|. = /?(r)/,"(», 9 ). (15.22)

The equation for the radial part is


w (15.23)
rr- A r

Here = is a dimensionless quantity, called the fine


structure constant, and

A
(15.24)

When c’ - o o , the expressions (15.24) reduce exactly to their non-


relativistic counterparts (see Chapter 13).
The somewhat improved values that we have obtained for A
and B by taking into account the relativistic effect do not change
in any way the general character of the solution that was obtained
in nonrelativistic theory. Formally, the additional term ~j- in
Eq. (15.23) can be treated as a relativistic, attractive potential
energy, which obeys an inverse square law and which affects the
solution under certain conditions. A detailed analysis of the role
of this term will be given below.
First of all, let us consider the asymptotic solution as
r —«0 . In this case, Eq. (15.23) reduces to

± t M i > _ n i + n - z v R t= 0 (1 5 25)

We shall look for a solution of this equation in the form

Ro = Crs.
We then obtain an equation for s

s(s-j- 1 ) — (((-(- 1 ) + ZW — 0 , (15.26)


the solution of which is

sliS= — L ± y p + D ’- z v . (15.27)

Consequently,
R. -}■- C./5*. (15.28)
2 64 R E L A T I V IS T IC QUANTUM M E CH A N IC S

If

both roots s, and s.2 will be real for all values of I (I = 0 , 1 , 2 , . . .).
We retain only the solution for rR0 that does not diverge at r = 0;
that is, we set Ca= 0 . Similarly, only the exponentially decreasing
solution for the wave function as r —*oo should be kept when
£ < / 0 (/I > 0 ) . The asymptotically decreasing solutions for the two
limiting values of r yield the same equation for the energy spectrum
as in the Schrodinger theory, as can be seen from Eq. (13.20) by
substituting s, for I. Thus, for the eigenvalues we have the equation

7 = - = * + i+ l/ ’ ('+ 4 -)'-z v - <15-29>


Substituting the relativistic values (15.24) of the constants A and
B, we have (for n = k + I + 1)
2

Z 2al 2
Fnl = m ^ 2 —m^c1.
(15.30)

Expanding this expression into a series of powers of Z2 a2 and


retaining only the first two nonvanishing term s, we obtain an
energy spectrum which includes relativistic effects:

RhZ2 a2Z 2
En l i+ (15.31)

The first term is identical to the nonrelativistic expression. The


second term, which is proportional to the square of the fine
structure constant 1/137, gives the relativistic correction.
The relativistic correction for the hydrogen atom ( Z = l ) is
interesting because it removes the degeneracy with regard to I.
The level for a given n is split into n closely spaced sublevels
(the close spacing is a consequence of the sm allness of a2) because
the orbital quantum number I can assume n different values
(/ = 0, 1, 2, . . . , /I — 1). In order to compare these results with
experiment, let us compute the distance between the doublet states
of the Balmer series (71 = 2). We find

Aw 'h i - J L ™ _ i £ 7! (15.32)
7l 3 1C ■

Experimental data show that the actual distance between doublet


states in the Balmer series is only one third of the distance
THE K L E I N —G O R D O N SCALAR RELATIVISTIC WAVE EQUATION 265

given by Eq. (15.32). This discrepancy arises because the fine


structure of the hydrogen levels cannot be explained entirely in
terms of the relativistic relationship between mass and velocity.
As we shall see in Chapter 19, it is also necessary to consider the
electron spin (that is, the intrinsic angular momentum of the
electron, which gives rise to a magnetic moment). At first it
was thought that the Klein-Gordon equation could be used to describe
a relativistic electron. As a result of the discrepancies between
its predictions and experiment, however, it was established that
it describes particles with spin of 0 , whereas the electron has a
spin of 1/2. Consequently, the Klein-Gordon equation can be used
for t mesons, which have a spin of 0.
Finally, let us consider the case in which

Za>l (15.33)

in Eq. (15.27). In this case, the solution does not consist of a


correction added to the nonrelativistic solution, but is fundamentally
new. Indeed, for / = 0 bothroots and s.2 are complex. Therefore,
the asymptotic solution (15.28) is

/? 0 = -y 7 (C,e'i' + Cle - <T'). (15.34)

where 7 = j / Z V — . We cannot impose the condition C.2 = 0 (or


C, = 0) on our problem because both solutions have the same
singularity as r-> 0. Since the solution remains unrestricted by a
potential barrier at sm all r, even when E<^0 ,the energy spectrum
for 1= 0 is continuous. Consequently, the particle will “ fall” to
the center.
The question of the stability of the motion of the particle is very
important in studies of the central forces. The above results can be
used to analyze the solution of the Schrodinger equation in the
general case of an attractive potential
n2 g2
2Mo r t ' (15.35)

On obtaining the asymptotic solution for r-+0 [15.25], we see that


the solution will vanish at the origin only for a maximum value of
q equal to 2 and that the particle will not fall to the center if

It is interesting to note in this connection that the r- 3 dependence


of potential energy occurs fairly frequently in the theory of
266 RELATIVISTIC QUANTUM MECHANICS

elementary particles since the potential energy of this form charac­


terizes the interaction of two elementary magnetic dipoles. Actually,
two cases have to be distinguished. In the first case, V'~r~3 only at
relatively large distances, while at small distances V varies as
r-*. This behavior of V is observed for the spin-orbit interaction
in the Dirac theory and it does not give rise to any difficulties.
Moreover, it is found that stable motion corresponds to a value of
Z greater than any in the present periodic system of elements
(Z < 137) because the spin effects reduce the influence of the
relativistic effects (whereas in the Klein-Gordon theory Z is con­
fined to relatively sm all values Z < y • 137).
In the second case, V continues to vary as r s even at small
distances (/--►0), and particles cannot be combined into atomlike
system s. This case can be observed in the meson theory of
nuclear forces, where quasi-magnetic interactions are of con­
siderable importance. The formation of an atomlike system be­
comes possible only if the potential is cut off at sm all distances
from the origin.

Problem 15.1. Find an expression for p and j if the scalar relativistic equation con­
tains a term arising from the presence of an electromagnetic field.
g
Solution. Let us substitute p —■p — — A, E — E — e<b in the Klein-Gordon equation,

wnich now describes the motion of a particle in an electromagnetic field.

Repeating the calculations that lead to Eq. (15.15), we obtain the generalization

where the four-dimensional potential has the components


A^= {A, <$}.
Problem 15.2. Show that in the case of time-independent potentials A and 3>, the
space and time coordinates in the Klein-Gordon equation can be separated and the wave
function can be written as
. E
4' (r, 0 = 4>(r)e ' h
Problem 15.3. Find the wave function of a free particle described by the Klein-
Gordon equation using for normalization the expression for the density p. Show that in
the relativistic case, p is the charge density rather than the particle density.
Solution. Suppose the momentum of the particle is directed along the z axis and
tha the particle travels In a segment of length L (the one-dimensional case). The
solution of the Klein-Gordon equation can be written in the form
THE K L E I N —GORDON SCALAR R E L A T IV IS T IC WAVE E Q U A TIO N 267

where E = ± chK Is the energy of the particle. Since the charge density Is given by the
expression

v 2m0c‘ I ' dt dt +}•


the total charge Is [see (15.14)]

It follows that p is the charge density rather than the particle density, since this re­
lation can be Interpreted only if it is assumed that particles described by an amplitude
B (negative energies) have a charge of opposite sign from particles described by an
amplitude A (E >■ 0).
Chapler 16

Motion of an Electron in a Magnetic Field.


Electron Spin

In 1896, Zeeman found that when atoms are placed in a strong


magnetic field, their spectral lines are split into several compo­
nents. This phenomenon is known as the Zeeman effect. The
Zeeman effect played an important role in the investigation of the
structure and magnetic properties of the atom. It led in particular
to the discovery of the spin (intrinsic mechanical moment) and
magnetic moment of the electron. Accordingly, it is worth elaborat­
ing the theory of this effect in some detail.

A. THE CLASSICAL THEORY OF THE ZEEMAN EFFECT


The simplest model of the radiating atom in Lorentz’ selection
theory is based on the assumption that the electron moves under
the influence of an elastic force

F= —kr. (16.1)

The elastic constant k is related to the electron m ass and the


angular frequency of oscillation u>0 by the expression

k = m0t»l. (16.2)

The equation for the oscillations of an electron in a homogeneous,


constant magnetic field H, therefore, becomes

m0r -|- iiwolr = — r x H, (16.3)

where e = — e„ is the electron charge. Taking the components of


(16.3) along the coordinate axes (the z axis is chosen in the direc­
tion of the field H, so that Hx = Hy = 0, llz = jyf), we find

(16.4)
h (,,i”z = 0 .
MOTI ON OF AN ELECTRON IN A M A G N E T I C FIELD 269

Multiplying the second equation by*(i=l^-I) and adding it to the


first, we obtain

E+ O.JS— 2|V.'=0 (16.5)

where o- 2m ac is the Larmor frequency of precession and l = x-\-iy.


For o< w 0 the solution of (16.5) is of the form 1

\z=e'°‘ {Ae™*1-\-Be-'™*1} (16.6)

and it follows from (16.4) that the z component is


z = Ce-iw«t. (16.7)

From the above expressions it can be seen that the frequency of


oscillation of the electron (a three-dimensional oscillator) changes
under the influence of a magnetic field. An atom placed in a
magnetic field should emit radiation at three frequencies:

u>0— 0, “o. “o+ °- (16.8)


According to the classical theory, an oscillator does not emit
energy in the direction of oscillation. Therefore, when we observe
the light emitted by an atom in the z direction (the direction of the
magnetic lines of force), we are able to detect only two lines (there
will be no component u>o due to oscillations along the z axis). In
other directions, we are able to observe all three components (the
normal Zeeman effect). Equations (16.6) and (16.7) indicate that
the oscillations are resolved in a longitudinal component in the
direction of the magnetic field (the z axis) and two transverse
components corresponding to two directions of rotation (a right-
handed rotation and a left-handed rotation). Thus, the magnetic
field has no effect on the longitudinal oscillations and acts only
on the circular rotations in the plane perpendicular to the magnetic
field.
In quantum theory a change in the frequency of oscillation is
always associated with a change of energy. At first glance, it may
seem strange that the magnetic field changes the energy of the
electron, since the Lorentz force F = v x H is perpendicular to
the velocity, and therefore the work done by this force, just as the
work done by any centripetal force, must be equal to zero. On the

^ v e n in t h e c a s e o f very s t r o n g f i e l d s ~ 106 g a u s s ) t h e q u a n t i t y 0 i s o f t h e
o r d e r o f l O 1^ s e c w h e r e a s t h e f r e q u e n c y o f o s c i l l a t i o n of a n e l e c t r o n in an atom (the
o p tical spectrum ) is ^ 10*^ s e c T h e r e f o r e , t h e i n e q u a l i t y 0 « <±>q i s p r a c t i c a l l y
alw ay s satisfied .
270 RELATIVISTIC QUANTUM MECHANICS

other hand, it is well known that an electron rotating in a circle


(that is, a current loop) forms a dipole. The energy of this dipole
in a magnetic field is equal to
l/mag= (16.9)
These two conflicting conclusions may be explained in the following
manner. As the magnetic field changes from zero to a certain
constant value Hz= b%~, the electron experiences a force directed
along one of the components of the electric field g. This force
imparts an additional energy to the electron. The magnitude of
this component of the electric field can be found from Faraday’s
law of induction (second Maxwell equation):

1 1 j <3TdS.
c dt

Assuming thats/T and S depend only on time and that the switching
on of the magnetic field does not alter the radius of the stationary
orbit, we find
r
8 = — 2c dt ‘
The additional velocity imparted to the electron ( e = —e0) by this
electric field can be found from the equation

ft mag____ fo &__ rgp djff*


dt m0 2m0c dt ’

which gives
ymag _reo_ ^
2m„c

As we can see, v ma& is independent of the rate of change of the


magnetic field when it is switched on. Since the magnetic field is
directed along the z axis and the induced electric field (and there­
fore also umag) is perpendicular to it and to the radius of the orbit,
we may write in vector form
®mag= Hxr
2m0c
From this it is clear that the magnetic field produces an additional
rotation of the electron (Larmor precession) with an angular
v elo city o =

vve may now determine the unknown additional energy acquired


by a rotating electron when the magnetic field is switched on. Since
the energy of an electron placed in a magnetic field is determined
entirely by its kinetic energy, we have
M O T IO N OF AN ELECTRON IN A M A G N E T I C F IE L D 27

y mag — ^m ag^i___ ^

where v is the velocity of the electron before the magnetic field is


switched on. Retaining only terms that are linear with respect
to ©mag, we obtain

Vmag= e_°_v . ( f / X r ) = £e f f . ( r x v ) . (16.10)

Comparing (16.10) with (16.9), we see that the magnetic moment


of an electron moving in a circle is given by the expression

V.= - fec r x v . (16.11)

Recalling that its angular momentum is equal to

L = ml) r x v ,

we find a simple relation between these two quantities

(16.12)

It is worth noting that the magnitude of the magnetic moment can


also be found from other considerations. As we know, the magnetic
moment of a current loop is

where n° is a unit vector normal to the plane of the current loop.


In the above relationship the current strength is equal to
, __ _ £o»
T ~~ 2 n r’
2nr
where T = - - is the period of rotation, while the area enclosed
by the current is
S —- r \

Combining these last relations we again obtain the expression (16.11)


for the quantity ji.

B. THE ZEEMAN EFFECT IN THE NONRELATIVISTIC


SCHRODINGER THEORY

In order to obtain the Schrodinger equation for an electron


moving in a magnetic field, we shall use the general rule for trans­
formation of the classical Hamiltonian to the quantum case (see, for
272 R E L A T IV IS T IC QUANTUM M E C H A N IC S

example, Chapters), To do this, we substitute the momentum oper­


ator p into the classical expression for the energy of an electron in
the presence of electrostatic and magnetic fields. The Schrodinger
equation for the central forces in the presence of a magnetic field
characterized by the vector potential A then takes the form

(E — Hs ) | = 0 (16,13)

where the Hamiltonian of the Schrodinger equation is

H S = e©(r) + — (16.14)

and the operator P = p — —A is called the generalized momentum


operator. For the case of a constant, homogeneous magnetic field
directed along the z axis (Hx = Hy = 0, Hz = s 5 T ) , we may write

Ax = - ^ y ^ T , Ay = \ x J r .

Using the fact that

(P•A) t = — ih (V •A) <]) + (A •p) <ji,


where
V A = 0,

and neglecting the terms proportional to the square of the magnetic


field strength we find

| E —e'l> (/•) , e + = 0. (16.15)


2m0 ' m0c

Remembering that

m0c 2m0c (16.16)

where
d
L, — ih d't -

we reduce the Schrodinger equation to the form

{V‘‘ + 2mo e:r r e<l> Co]} + = 0 . (16.17)


M OTION OF AN ELECTRON IN A M AG NETIC FIELD 273

It is easy to show that this equation is satisfied by the usual


wave function for a centrally symmetric field:
<[»= /?(/•) 1 7 (0 , <p). (1 6 .1 8 )

Substituting this solution into (16.17) and recalling that

Lt Y? = hmY?.
we obtain the equation

which also includes the effect of the magnetic field on the atom.
This equation may be written as

(£ — Hs ){| = 0,

Hs + <16-19>

where we take the charge of the electron as

e = — e„.
The last term in the Hamiltonian may be attributed to the presence
of the orbital magnetic moment of the atom, which gives rise to an
additional energy
Vm&s = — \i-H = — p ^ = -£j^in-££ (16.20)

Therefore, the orbital magnetic moment obtained on the basis of


the Schrodinger theory is

Recalling the expression for the z component of the angular momen­


tum
t z = hm,

we obtain the same relation between the magnetic moment and the
angular momentum as in the classical theory [see (16.12)]:
Fz __ go (16.22)
Lz 2 m ac

It follows, therefore, that the components of the orbital magnetic


moment are multiples of a certain unit magnetic moment
2 74 R E L A T IV IS T IC QUANTUM M E C H A N IC S

liz = — Hm>
^ = J ^ = 9-273- 10’4' erg.gauss-', (16.23)

which is called the Bohr magneton.


The orbital magnetic moment of an electron is one of the most
important magnetic properties of an atom. It can be seen from
Eq. (16.20) that the additional energy of an orbital electron placed
in a magnetic field is given by the expression

£mag= p m a g _ £ o ^ r m = oAm (16 . 24 )


Ih\qC
since l/mas is a constant, where o is the Larmor frequency. Because
of the selection rules for the magnetic quantum number (Am= 0, ± 1),
the additional radiation frequencies due to the Zeeman splitting
are the same as in the classical theory [see (16.8)], namely,

Aa) = A£^ aS : - o Am= 0, ± o . (16.25)

The normal Zeeman splitting2 of the spectral lines (triplets


and doublets) is encountered only in the case of a strong field (the
Paschen-Back effect) or in the case when the total spin of the
electrons in the atom is equal to zero (for example, in parahelium,
whose outer shell consists of two electrons with oppositely directed
spins). In cases in which the spectral lines are split into more than
three components, the Zeeman effect is said to be anomalous.
The anomalous Zeeman effect is connected with the spin properties
of electrons, and an explanation of this effect can be constructed
only on the basis of Dirac’s theory, which takes into account the
spin effects (see Chapter 20).

C. THE EXPERIMENTAL DISCOVERY OF ELECTRON SPIN

It was shown in the last section that the Schrodinger theory is able
to explain only the orbital angular momentum and magnetic moment
of an electron. The basic equations characterizing these properties
are Eq. (16.22) for the ratio of the orbital magnetic moment and

L e t u s n o t e t h a t t h e r e a s o n s f or t h e u s e o f t h e t e r m s “ n o r m a l Z e e m a n e f f e c t " a n d
"an nSTlous Z e e m a n e f f e c t " a r e p u r e l y h i s t o r i c a l . U e f o r e t h e d i s c o v e r y o f e l e c t r o n s p i n ,
o n l y t h e c l a s s i c a l t h e o r y o f t r i p l e t s p l i t t i n g ( n o r m a l Z e e m a n e f f e c t ) w a s k n o w n . Whe n a
m o r e c o m p l i c a t e d s p l i t t i n g w a s d i s c o v e r e d it w a s c a l l e d t h e a n o m a l o u s Z e e m a n e f f e c t
b e c a u s e n o t h e o r e t i c a l e x p l a n a t i o n c o u l d b e g i v e n f o r it u n t i l t h e d e v e l o p m e n t o f t h e t h e o r y
of e l e c t r o n s p i n
MOTION OF AN ELECTRON IN A MAGNETIC FIELD 275

the orbital angular momentum and Eq. (16.24), which indicates that
the number of possible orientations of the magnetic moment relative
to the z axis is necessarily odd, since the
number of states with different quantum
numbers m is equal to 2/ + 1. The Schrodinger
theory, however, does not adequately account
for all the experimental data, the analysis of
which led to the discovery of the spin prop­
erties of electron. Let us briefly discuss
these experimental results.
1. First of all let us consider the Einstein-
de Haas experiment (1915), which was carried
out in order to verify Eq. (16.22):

^L z = —gs —
2 m ac '

where g , the Lande factor, should be equal to


unity for orbital moments. In this experi­ F i g . 16.1. D ia g r a m o f t h e
ment, a ferromagnetic rod is suspended on E in stein -d e H aas ex p e ri­
a quartz fiber and magnetized by passing a m ent for th e d e term in atio n

current through a coil (see Fig. 16.1). As of th e L a n d e g factor.

a result, the rod acquires a magnetic moment 1) q u a r t z f i b e r ; 2 ) c u r ­


re n t c a rry in g c o il; 3) fer-
and an angular momentum whose magnitude r o - m a g n e tic rod.
can be determined from the angular rotation
of the quartz fiber. If an alternating current is passed through the
coil, an alternating torque will arise, causing torsional vibrations
in the ferromagnetic sample. In addition, resonance can be used
to enhance the rotational effect. Experimental measurements of
the gyromagnetic ratio (16.22) show that the sign of this ratio is
negative, so that it can be definitely concluded that the magnetiza­
tion of the ferromagnetic sample is due to the motion of electrons.
The value of the Lande g factor, however, turned out to be equal
to two (g = 2), rather than to the unity that was required by the
classical or Schrodinger theories. This g value was not explained
until the development of the theory of electron spin (see below).

F ig . 1 6 . 2 . D i a g r a m o f t h e S t e r n —G e r l a c h e x p e r i m e n t f o r t h e
d eterm in atio n of th e m a g n e tic m om ent o f m o n o v a le n t atom s.
276 R E L A T IV IST IC QUANTUM M E C H A N IC S

2. Stern and Gerlach (1921) studied the behavior of a beam of


atoms in an inhomogeneous magnetic field in order to check the
theoretical result (16.23)

(*■*= — IVrc,

which describes the spatial quantization. In their classical experi­


ments a beam of monovalent atoms (hydrogen, lithium, silver),
traveling along the x axis, crossed a magnetic field directed along
the z axis (Hx = Hy = 0, Hz = e%T). This magnetic field was very
inhomogeneous, so that it had large gradients. Then a magnetic
dipole of moment
P = emag ( 1 6 .2 6 )

where emag is the “ magnetic charge” and I is the length of the


dipole, will experience a force directed along the z axis3

c mag
i V qS L drJT
— e mag ^ co s a qz — P* dz (16.27)

Let us calculate in a simplified fashion the displacement ex­


perienced by a particle under the action of the force Fz during the
time t. If the particle moves with a velocity v perpendicular to
the magnetic field (that is, to the z axis) and travels a distance
L — vt, the displacement along the direction of the z axis will
equal

(16.28)
2 2 v1 M dz

In this case the acceleration is w = - ~ t where the force Fz is given


by (16.27), and M is the mass of the atom. Consequently, a beam of
particles possessing a magnetic moment p will be split into
components as it passes through an inhomogeneous magnetic field.
The number of components is determined by the possible number
of projections of the magnetic moment p on the direction of the
field.
In their experiments. Stern and Gerlach studied the splitting
of a beam of atoms in the s state. In this state, the angular
momentum and consequently the magnetic moment of an atom are
equal to zero (l = m = 0), and therefore there shouldbe no splitting.
If the electron is in the p state ( / = ] ) , then triple splitting should

We n o t e t h a t in o r d e r to d e t e r m i n e th e m o tio n of th e c e n te r of m a s s o f th e m a g n e tic
d i p o l e , it i s q u i t e i m m a t e r i a l w h e t h e r w e r e g a r d i t a s a r i g i d d i p o l e o r a c u r r e n t l o o p .
MOTION OF AN ELECTRON IN A MAGNETIC FIELD 277

be observed because of the three possible values of the magnetic


quantum number

8z = 0 (m — 0), = + (m = zh i ) .

Experiments on hydrogen, lithium, silver, and other atoms show,


however, that the beam is split into only two components. This
proved the existence of a magnetic moment for atoms in the
s state. The projection of this magnetic moment on the z axis can
assume only two values. The measurements of the quantity jj.
showed that it is equal to one Bohr magneton

e„h (16.29)
f1"- 2m„c

In order to reconcile the results of these two classical experi­


ments, Uhlenbeck and Goudsmit introduced the hypothesis that
an electron posses an intrinsic angular momentum in addition
to its orbital angular momentum. At first it was believed that
this intrinsic angular momentum could be treated by analogy
with a top spinning about an axis, and therefore it was called the
electron spin. It must, however, be emphasized that no rigorous
classical theory of spin exists. According to the hypothesis of
Uhlenbeck and Goudsmit, the intrinsic angular momentum of an
electron is equal to

5, = ± i * ; (16.30)

that is, the quantum number characterizing its projection on the


z axis takes on half-integral values = ^ . The important
distinction between the integral (orbital magnetic quantum number
m) and the half-integral (spin quantum number ms ) values of
quantum numbers lies in the number of possible states. Integral
quantum numbers always give us an odd number of states (for
/ = 0 we have one state m — 0\ for 1= 1 there are three states
m = 0, -j-1, —1; and so on). On the other hand, half-integral
quantum numbers give us an even number of states (for example,
1 1 1 3
for s— ~2 there are two states ms = ~ ; for there
are four states; and so on).
The assumption of the half-integral quantum numbers was
introduced even before Uhlenbeck and Goudsmit in order to explain
the double splitting of terms for the monovalent atoms. The Stern-
Gerlach experiment showed that there are two possible electron
states in a monovalent atom; that is, the electron spin must be
278 R E L A T IV IS T IC QUANTUM M E C H A N IC S

described by the half-integral quantum numbers corresponding to


two opposite orientations. Recalling that the Einstein-de Haas
experiment showed that the Lande g factor in Eq. (16.29) is equal
to two (g = 2) and the intrinsic angular momentum is given by
Eq. (16.30), we find the following expression for the z component
of an intrinsic magnetic moment:
gp
H'iz — IH qC
(16.31)

which is simply one Bohr magneton. The introduction of the


electron spin also made it possible to explain the multiple splitting
of the spectral lines of atoms, as well as their magnetic properties.

D. PAULI EQUATION

A nonrelativistic wave equation that includes the intrinsic


magnetic moment of the electron was first proposed by Pauli. For
this purpose he took the ordinary Hamiltonian of the Schrodinger
equation and added to it a term representing the interaction between
the magnetic moment of the electron p and the external magnetic
field H :

Pp = — \ i - H. (16.32)

Then the time-independent Schrodinger equation takes the form

{£ — HS -fjx • H] <ji = 0, (16.33)

where the Hamiltonian Hs is

+ <16-34)
Next, it was necessary to find suitable quantities to describe
the intrinsic magnetic moment of the electron. It is well known
that introduction of the spin is related to introduction of a
fourth quantum number, characterizing the internal properties of
an electron. On the other hand, the wave function ^ of a particle
depends only on three quantum numbers, corresponding to quantiza­
tion of the three spatial coordinates. In order to describe spin,
Pauli introduced two wave functions T, and 'F.2in place of the single
w a.c function ^. One of the wave functions describes a state with
one spin orientation and the other wave function describes a state
with the opposite spin orientation. The actual wave equation
represents a system of two equations. It is possible to represent
a system of two or more equations, such as
MOTION OF AN ELECTRON IN A MAGNETIC FIELD 279

(16.35)

by a single equation in matrix notation

(16.35a)

where the multiplication is carried out according to the rule for


the multiplication of matrices (c) = (a) (b): namely, an element of a
matrix product is obtained by multiplying each element in the
appropriate row of the first matrix by the corresponding elements
of the appropriate column of the second matrix and taking the sum
of these products, that is.

cik = ^ciinbnk. (16.36)


n

Pauli suggested selecting the wave function lF in the form of a one-


column matrix *F= and setting the intrinsic magnetic moment
of an electron equal to
H= — (16. 37)

where p.0 is the Bohr magneton and a' stands for the three 2 x 2
Pauli matrices
0 1' 0 —i ./I 0
10 1 0 ■* \0 - 1 , (16.38)

These matrices are denoted by the letter a with a prime (the same
letter without a prime will be used to denote the 4 x 4 Dirac
m atrices). These matrices characterize the components of the
spin vector along the coordinate axes.
Using the rule (16.36) for matrix multiplication, it can be
readily shown that the square of each Pauli matrix is equal to
unity
ajs = Q-i = °3S— I > (16.39)

where T denotes a 2 x 2 unit m atrix^ ^ . It can also be shown


that different matrices anticommute with one another:
280 R E L A T IV IST IC QUANTUM M E CH A N IC S

In term s of the above matrix expressions, the nonrelativistic


Pauli equation has the form

{ < £ - h s )(; ? )-!* .[(? “ »H +

+ (o - l ) H-]}(t'l) = 0' (16.41)

This matrix equation is equivalent to a system of two ordinary


equations, each of which corresponds to one of the rows of the
matrix
(£ _ H S _ ^Ht) V, - m, (Hx - iHy) Vs = 0,
( £ _ H S + HHz) W , ~ [L0(Hx + iHy)'Vi = O. * ‘ ’

Let us consider the case of an electron moving in a magnetic field


directed along the 2 axis (Hx = Hy = 0, HZ= H^'). Using the Hamil­
tonian (16.19), which includes the effect of a magnetic field, we
obtain two equations of motion for the electron

{ £ + e0 <l»— 1^ m— ^ 1 - -0 ,
| £ -j- eo® — -f- (i.03/^ — | 1 .2= 0,

where >n is the energy of interaction between the magnetic


field e^ ’and the orbital magnetic moment, and is the energy
of interaction between the magnetic field and the spin magnetic
moment. In the s state the magnetic quantum number m is equal
to zero, so that the Pauli equation takes the form

(16.44)
(£ + M> + wX" — 2"-) Vi = 0 ;

that is, the wave function T, describes a state in which the intrinsic
magnetic moment of the electron is parallel to the z axis, and the
wave function T,, a state in which the magnetic moment is anti-
parallel to the z axis. These are the orientations of the intrinsic
magnetic moment which were observed in the Stern-Gerlach
experiment.
As the function 'I'1’ Pauli suggested taking the Hermitian adjoint
of '[', that is, the matrix >I;v = (M;*'I'*), whose elements are ob­
tained by taking the complex conjugates of the elements of lIr
and transposing them (interchanging rows and columns). Thus, if
MOTION OF AN ELECTRON IN A MAGNETIC FIELD 20

is a column matrix, *F1' will be a row matrix. The probability


density will be given by

V1>F= (V
F*’F 1j = ' F * ' F , _|_ ' F j 'F,, (16.45)

which includes the possibility of two spin orientations.


The other matrix elements are formed in a sim ilar manner.
For example.

_ ipnp _ipnp ■
(16.46)

that is, 'F f ' F , and lF * * F s represent the probability densities of states
in which the electron has a spin orientation parallel and antiparallel
to the z axis, respectively. Using the expression for the intrinsic
magnetic moment in the Pauli theory

and the Einstein-de Haas relation between the intrinsic magnetic


moment and the angular momentum

lir = — -P-°- S,
m 0c

we find that
S -- -.y Tift'. (16.47)

Thus, in agreement with the other experimental facts, the z com­


ponent of the spin angular momentum is equal to rh 1 .
Since the spin operator is expressed in terms of the matrices
o', the spin components do not commute. In this they resemble the
components of the orbital angular momentum, which are operators
depending on derivatives [see (11.75) and (11.76)]. The commutation
relations satisfied by the spin operators can be easily established
from (16.40) and (16.47):

S ^ , - S vS , = i7 iS ,,
svs, - s,sv= ihsx, (16.48)
SZSX— SVS_. = itiSy.
282 R E L A T IV IS T IC QUANTUM M EC.HAN1CS

Concluding our discussion, we note that the Klein-Gordon theory,


which includes the relativistic effects but neglects the spin effects,
and the Pauli theory, which, on the contrary, neglects the relativistic
effects but includes the spin effects, were predecessors of the more
rigorous theory of the electron developed by Dirac, which predicts
all the elementary properties of the electron. It should be noted
in this connection that the absolute value of the intrinsic magnetic
moment was introduced in the Pauli theory from purely empirical
considerations.
Problem 16.1. Show that in nonrelativistic quantum mechanics, just as in the classical
theory, the Zeeman effect is due to the precessional motion of the orbit in a magnetic
field, the motion having the Larmor frequency.
Solution. From the Hamiltonian of the Schrodinger equation for the case of an electron
moving in a magnetic field directed along the z axis

we can find the time derivatives of the angular momentum

= ( i ^L-v La-H) = oLy, f ( Ly = oLxt ~ t L, = 0,

where o is the Larmor frequency. It follows that the component of the angular momentum
in the field direction (z axis) is a constant of the motion. The components along the x
and y axes, however, precess around the z axis with the frequency o .

Problem 16.2. Show that the spin operator S is vectorial; that is, if we construct the
linear combination

SA.— S,—a2Sx + -j- 7 0 S.,


S ; = aaSx + 'PaSj, + 7aS; ,

where a, 3, 7 are the directional cosines, then

S'vSj, — S',S't = HiS', and so on.

Problem 16.3, Show that in a homogeneous magnetic field which is a function of time
only, the wave function of the Pauli equation can be resolved into a product of coordinates
and spin functions. What form does this solution take if the field is time Independent?
Solution. Let us look for a solution of the Pauli equation in the form

It is readily shown that the coordinate part of the wave function 41(r, t) satisfies the
ordinary Schrodinger equation without the spin

= t).

while the spin part of the wave function may be obtained from the equation

•sS: !!;)— <*«>(§ S)-


MOTION OF AN ELECTRON IN A MAGNETIC FIELD 283

The spin part of the wave function Is normalized as follows

(CfC,*i(§)=CfC, + « C , = l.

In the case of a stationary magnetic field It Is easy to determine the time-dependent


part in the above equations. We simply set

Then the time-independent parts of the wave function are determined from

(E — Es) ^ = H S

Problem 16.4. Find the eigenvalues of the operator of a spin component along the
direction specified by the spherical angles 8 and <p. Investigate the particular cases in
which this direction is the x , y or z axis.
Solution. Consider first the case in which the spin is directed along the z axis. Then
the initial equation takes the form

where
n /1
'l 2 \0

This matrix equation is equivalent to two homogeneous algebraic equations

{ c , - i c , = o,

1 c , + > .c , = 0.

The normalized solutions of these equations have the form


1 and I = — —
2’ ■*/:
The first evidently corresponds to the case in which the spin is directed along the z
axis; the second to the case In which the spin Is directed along the - z axis.
The operator for the component of spin along the direction defined by the spherical
angles 9 and <p with respect to the coordinate axes is equal to

S = sin 9 cos <pSx + sin 9 sin <p + cos 9 Sz,

where

0 n 0 s
S.* 10 9 o,r
284 R E L A T IV IST IC Q U ANT UM M E CH A N IC S

Hence from the equation

we find two solutions:


(a) the solution corresponding to the case in which the spin is parallel to this direction

(b) the solution corresponding to the case in which the spin is antiparallel to this
direction

Setting 8 = 0, y = Owe obtain the same solution as above. The cases in which the spin is
directed along the x or y axes may be obtained by setting, respectively.
n
”*

Problem 16.5. The electron spin is parallel to the z axis. Find the probability that
the component of the spin (a) in a direction parallel to the a- axis, and (b) in a direction

making an angle 0 with the z axis, will have the values-g- ft and---- ry ft.

Hint. Take the wave function describing the state in which the spin is parallel to the
z axis and then expand it in terms of the functions corresponding to the cases in which
the spin is parallel and antiparallel to the direction forming an angle 8 with the z axis.
Both these functions are given in Problem 16.4. Without loss of generality we may set
the angle <p= 0.
Then the squared modulus of the expansion coefficients gives the probabilities

of the components of the spin along the corresponding directions; these are equal to

In order to find the x component of the spin, we must set 8 = — in the last equations,
Clinplrr 17

The Dirac Wave Equation 1

A. LINEARIZATION OF THE ENERGY OPERATOR.


DIRAC MATRICES AND THEIR RELATION TO
PAULI MATRICES

As indicated in Section 15, relativistic quantum mechanics is


based on the well-known relativistic relation between the energy E,
momentum p, and rest mass m0:
E = c V p l ^rm\cl. (17.1)
To obtain the wave equation describing a free particle, we substitute
the appropriate operators into this equation

E= p= — ih v, (17.2)

and act with these operators on the wave function. It is impossible,


however, to make a direct transition to operators in (17.1) because
we cannot determine the action of the differential operator under
the radical sign. It is therefore necessary to get rid of the square
root in (17.1). One way of doing this is to take the square of Eq.
(17.1) . This gives the relativistic Klein-Gordon wave equation with
a one-component wave function.2 As already noted, this equation
describes the motion of spinless particles and is not applicable to
electrons, whose spin is equal to 1/2 (in units of h).
A different method of obtaining a linear relativistic wave
equation was adopted by Dirac (1928). This method gave a first-
order wave equation and consisted in linearizing the relation
(17.1) . It led to the discovery of the relativistic wave equation for
the electron. This equation plays a fundamental role in relativistic
quantum mechanics and quantum field theory since it provides a
See P. A. M. D i r a c , P r i n c i p l e s o f Q u a n tu m M e c h a n i c s , N e w Y ork: O xford U n iv e rsity
P ress, 1958.
2 M o r e e x a c t l y , w e i n f a c t h a v e a f u n c t i o n w i t h t w o c o m p o n e n t s ypi = \b a n d 4>2 —
cQt
s i n c e a s e c o n d d e r i v a t i v e w ith r e s p e c t to tim e a p p e a r s in t h e f u n d a m e n t a l e q u a t i o n . O n e
d e g r e e o f f r e e d o m c o r r e s p o n d s to p a r t i c l e s w ith p o s i t i v e e n e r g y , t h e o t h e r to p a r t i c l e s w ith
n e g a t i v e e n e r g y . It w a s s h o w n b y P a u l i a n d W e i s s k o p f t h a t t h e n e g a t i v e e n e r g y s t a t e s c a n
be e lim in a te d by c a rry in g ou t a s e c o n d q u a n tiz a tio n of th e s c a l a r e q u a tio n a n d in tro d u c in g
s p in le s s p a r t ic l e s w ith c h a rg e s of o p p o s ite s ig n s .
286 R E L A T IV IS T IC QUANTUM M E C H A N IC S

suitable description of the motion of particles of spin 1/2. The


discovery of this equation was the most important advance in the
theory of the electron since the Maxwell-Lorentz equations of
classical electrodynamics. Bohr’s sem iclassical theory and non-
relativistic quantum mechanics served only as transitional theories.
The relativistic relation between energy and momentum is
linearized by “ extracting” the square root of the four-term poly­
nomial with the aid of m atrices. For this purpose we represent
(17.1) in the form
3

E= c + mlc1= c J W (17.3)
p-.=o

where
p0= m0c, Pi = px, Pi = Py, Pi = Pz■ (17.4)
We note that
3

£ 2 = c 2 2 j w ==c8 (p, + m«c')- (i7 -5)

To determine what conditions the quantities must satisfy, we


square both sides of (17.3). Then, if the operators p^ and p com­
mute, we have3

El = c4 2 2 W *'°W = T 2 2 A P|1' (“h-V + V<v)- (17.6)


M-' ^ H-'
Equation (17.6) coincides with (17.5) only if

— 20^^-, (17.7)
that is, all four quantities <*., anticommute with one another

°yv + v an= 0 htV (17.8)


and the square of each of them is equal to unity

<=1. (17.9)
We recall that the 2 x 2 Pauli matrices also possess analogous
properties

0 1 o —i '
a
10 c,
o
a (17.10)

' T h o s e o p e r a t o r s co m m u te w ith e a c h o t h e r if t h e r e i s no e l e c t r o m a g n e t i c field . T h e r e ­


fore, D irac p r o p o s e d th a t o n e s h o u l d f i rs t e x t r a c t t h e s q u a r e roo t o f t h e o p e r a t o r for a fre e
p a r t i c l e , a n d t h e n g e n e r a l i z e t h e r e s u l t i n g e q u a t i o n to the c a s e w h e n f i e l d s are p r e s e n t .
THE DIRAC WAVE EQUATION 287

since they anticommute [see (16.40] and the square of each is equal
to unity [see (16.39)]. To extract the square root of the four-term
polynomial, however, it is necessary to have four relations (17.7)
(p = 0, 1, 2, 3), instead of three [Eqs. (16.39) and (16.40)] that are
satisfied by the Pauli matrices.
Accordingly, Dirac proposed that we take a system of 4 x 4
matrices cn and P„ that are related to the 2 x 2 matrices by the
expressions
0n Cn 0' ( « = 1 . 2, 3),
o' < (17.11)

(17.12)

where °'n are the Pauli matrices

0 0 and l'= 1
00 ?)■
Hence we find, for example,
/0 1 0 O'
/ 1 0 0 0 and so on.
3i I0 0 0 1
\0 0 1 0 .

The properties of these matrices are sim ilar to those of the


Pauli m atrices, as may be easily checked by direct multiplication.
In particular, it turns out that their squares are equal to unity

o“= P; = I (17.13)
or, more exactly, are equal to the 4 x 4 unit matrix

/I 0 0 O'
0 1 0 0
(17.14)
I0 0 1 0

\ 0 0 0 1 /

As in the case of Pauli m atrices, we have

°ia -2 = — = PiP>= — p.2pi = ip3 and so on.


(11. lo)
c,iP-i' = P«'3« («, n = 1, 2, 3).

From this it follows that the different matrices a anticommute with


one another (a sim ilar conclusion is also true for the system of P
matrices):
°n°n’+ =«'3n= PnPn’ + Pn'Pn= 23„„-. (17.16)
The matrices a„ and P«; however, commute with each other.
208 R E L A T IV IS T IC QUANTUM M E C H A N IC S

Dirac proposed that the matrices [see Eq. (17.3)] be


chosen as follows:

= 1, 2, 3), (17.17)
T 0'\
“o—Pa= O' —I')- (17.18)

In conventional notation these 4 x 4 m atrices have the form

i /°0 0 0 i\ /° 0 0 ■
a ,=
0 1 o\ a - 0 i o\
0 1 0 or [ 00 —i 0 O’
Vi 0 0 0J \i 0 0 0/
(17.19)
/I 0 0 0
//°0 0
0 1
0 - °\
-M, (0 1 0 0
ao= Pa='100--1
—\l 0 0 °' 0
Vo --i 0 0/ \o 0 0 -- 1.

Multiplying the above matrices by one another, it is easy to show


that they satisfy the relations (17.7).

B. THE DIRAC EQUATION. CHARGE DENSITY


AND CURRENT DENSITY

Let us substitute the corresponding operators into the linearized


relativistic relation (17.3) between the energy and momentum. We
obtain the Dirac equation for a free particle4
(E — H)t = 0, (17.20)

where the operators E and p are, as usual, equal to

E= P ~ ~ iflV’

^ B e c a u s e of Ihe fo u r c o m p o n e n t s o f t h e w a v e f u n c t i o n 0 , e a c h s t a t e c a n h a v e e i t h e r
p o s i t i v e or n e g a t i v e e n e r g i e s ( s e e b e l o w ) a n d tw o d i r e c t i o n s o f t h e s p i n ( s e e C h a p t e r 18).
In th e c l a s s i c a l c a s e t h e r e l a t i o n (1 7 .1 ) b e t w e e n t h e e n e r g y a n d m om entum c a n b e
r e p r e s e n t e d in a form s i m i l a r to (1 7 .2 0 )

K-u • p- \/l- /?2 m0 <' 2 " 0 •


'Phis e q u a t i o n is e a s i l y v e r i f i e d if it i s r e m e m b e r e d th a t for a free p a r t i c l e

m0r2 m0u
._ and p -— —— .
y'l- £ 2 \/l-/32
C o n s e q u e n t l y th e ma trix at • v c m u st p l a y t h e ro le o f t h e v e l o c i t y , w h i l e p 3 * \J 1 —/32 i s
a s c a la r that c h a r a c te r iz e s the L orcntz contraction.
THE DIRAC WAVE EQUATION 209

and the Hamiltonian H is given by


H = <"(a» p)-j- pj»ac‘. (17.21)
Since a and p., are 4 x 4 m atrices, the wave function must have
four components, which we combine to form a single-column matrix

'<Pr
(17.22)
^3
•'W.
The complex conjugate of this function is understood to be the
Hermitian adjoint, that is, the row matrix
^ = (17.23)

The Dirac wave equation (17.20) is therefore equivalent to a


system of four equations
(E — m0c-) tyi — c (p* — ipy) — cp^a= 0,
(E — /acc2) <]>4— c(p* + ipy) - f cp^4= 0, (17.24)
(E-I- m0c2) f c - c f p , - ipy) <jj.2— cp^, = 0,
(E -+ mhc2) ^ 4 — c (p* -f- ipy) <!>,-1- cp^j = 0.
In the case of motion of an electron in the electromagnetic field
specified by the given vector and scalar potentials A and <!>, we
can still use Eqs. (17.20) and (17.24), but the energy and momentum
operators have to be generalized in accordance with the general
laws of quantum mechanics:
F = t7 )A —e®, P = — iHV — j A . (17.25)
The complex conjugate of the wave equation may also be repre­
sented in the form of a single matrix equation
<1>+(F — c(a • P) — p3m0c2) = 0, (17.26)

where the action of the operators ih and —ihV on the wave


function which is on their left should be taken to be the same as
in Eq. (17.20) but with opposite sign, that Is,

- ^ i H V = ihvy-, = — (17.27)

For a free particle, Eqs. (17.20) and (17.26) now become

iH W + ich (a ‘ V) '1*~ Psm0c24»= 0, (17.28)

ih -gf (J)+- f ich (V** • a) -f- mocVp, = 0. (17.29)


290 R E L A T IV IS T IC QUANTUM M E C H A N IC S

Multiplying (17.28) on the left by <{>+ and (17.29) on the right by <(>
and adding them, we obtain

--J-'T'I’ + V- <!>+a* = 0, (17.30)

which may be interpreted as the equation of continuity for the


probability density p and the current density j :

4 -p + v -y = o , (17.31)

where5
p= e<]>+<jj, j=ec<Sf+a.']/. (17.32)

If we write the last equation in term s of components of the wave


functions, rather than in term s of m atrices, we obtain

'h\
p0= - = ^ = ( M W ) = W t + W * + W . + K 'hi (17.33)

that is, p0 is a matrix consisting of a single element (it is just a


number). In exactly the same way it is readily shown that

0 0 0 1
= ^ = o 1 0 0 = r ■=
\1 0 0 0 i
= « < ! • « + w. - (17.33a)
We note that, contrary to the Klein-Gordon theory, the density
p„ is a positively defined quantity. This does not mean, however,
that in the Dirac theory p„ can be considered the particle
density. Just as in the Klein-Gordon theory, there will be particles
with a sign opposite to that of the electrons (positrons). From
Eq. (17.32) it can again be concluded that ca should be regarded
as the velocity operator.

^ i ^ i l a r r e l a t i o n s w i l l a l s o h o l d in t h e c a s e w h e n a f i e l d i s p r e s e n t .

f’ l n second q u an tizatio n , the d efin itio n of P q as a p o sitiv e q u an tity m e a n s o n ly th a t


I*'ermi s t a t i s t i c s s h o u l d b e a p p l i e d to th e p a r t i c l e s (fo r e x a m p l e , in t h e c a s e o f t h e D ir a c
e q u a tio n ), if p 0 m a y t a k e e i t h e r p o s i t i v e or n e g a tiv e v a lu e s (for e x a m p le , in t h e c a s e o f
th e K le i n - G o r d o n e q u a t i o n ) , th e n F iose s t a t i s t i c s s h o u l d b e a p p l i e d to th e p a r t i c l e s .
THE DIRAC WAVE EQUATION 29 1

C. TRANSFORMATION PROPERTIES OF THE WAVE


FUNCTION UNDER LORENTZ TRANSFORMATIONS
AND SPATIAL ROTATIONS

According to the general principles of the special theory of


relativity, physical laws must be independent of the choice of the
Lorentz frame of reference. Therefore, the Maxwell equations,
the Klein-Gordon equation, and the Dirac equation must all be
invariant under the Lorentz transformations. Let us investigate
the transformation properties of the Dirac wave function. The
Lorentz transformations can be written as

c/' = c/coshy— xsinhy, x! — x cosh 7 — ct sinh y, y' = y, z’= z, (17.34)


where
cosh v - !-- sin h y P V

T VT=?’ c

This transformation must be satisfied by all four-dimensional


vectors, including, in particular, the charge and current densities
cp' = cpcosh y jx s i nh y, j'x = jx cosh y — cpsinhy, j'y . *= j y ,z . (17.35)

The definition of these quantities, according to the Dirac theory,


gives

6’+<j/ = <j>+ ( c o s h y — a, s inh'r) ’]) = e iau|i,


</+ai</ = F (a, c o s h y — s i n h y ) (i) = (17.36)
= F a-2.3,TJ-

Here we have used the fact that e-1"1 = cosh y*, - sinh ya, = cosh
f — a, sinh y, since a?" = 1, *?"+'= where n is an integer. In
order to satisfy the above relations, we must set

^c o s h — a, s i n h l) y = e 2
(17.37)
O'1"= F ^ cosh 1— a, s i n h =i}»+e 2

Then, since

(17.38)

it is easy to show that the relations (17.36) are correct. From


(17.37) it can be seen that the wave functions do not transform as
292 R E L A T IV IS T IC QUANTUM M E CH A N IC S

vectors (whole angle ) or as tensors (double angles ) , but as


7 7

spinors, the transformation of which is characterized by the angles


7 / 2 . Spinors are also called tensors of rank 1/2.

In a sim ilar manner, it may be shown that the transformation


law of a spinor under an ordinary spatial rotation (for example,
around the 2 axis by the angle cp) is as follows:

(17.39)

The above relations follow from the transformations for the cur­
rent vector:
U = ix cos <P + /, Sin <P,
j 'y ~ = jy COS j x s i n cp, (17.40)

these transformations are represented in the Dirac theory as

(/ "a,!]/ = t|j+(a, cos <p a.2 sin <p) (17.41)


== <Jj+a3ty and so on.
By substituting the values for <]/ from (17.39), and using the fact
that

we can obtain the relations (17.40).


(llmplcr 18

The Dirac Theory of the Motion of an Electron


in a Central Field of Force

A. ORBITAL, SPIN AND TOTAL ANGULAR MOMENTA.


CONSERVATION LAWS

We shall determine the angular momentum of an electron from


the conservation laws characterizing the motion of an electron in
a central field of force:
V — e© (r) (18.1)

(for example, an electron moving in the Coulomb field of a nucleus


V = — — j . It was shown in the nonrelativistic Schrodinger theory
that the orbital angular momentum is conserved in a central field
L = rxp. (18.2)

In the Dirac theory, however, which takes into account the electron
spin, the component of the orbital angular momentum does not
commute with the Hamiltonian
H = c(a.p) + p3 m(1c‘!- f V{r) (18.3)

and therefore it is not a constant of the motion. Indeed

H L ,- L,H = ^ (*lPj, - asPx) * 0. (18.4)

In order to generalize the law of conservation of angular mo­


mentum to particles having spin, we shall use the relation

Ha, — o,H = - (a4 px — <xlPj,). (18.5)

It follows from this that the operator

J = L + --/b = L + S (18.6)
2 94 R E L A T IV IS T IC QUANTUM M E C H A N IC S

commutes with the Hamiltonian operator H and thus is a constant


of the motion. 1
This result may be interpreted in the following manner. The
electron has an intrinsic angular momentum (spin). We have just
found that only the total angular momentum is conserved (the
orbital angular momentum plus the spin). The orbital angular
momentum in the s state is equal to zero, and therefore we have
here the law of conservation of spin angular momentum. For the
square of the spin we obtain

S2= 4 - U‘ (oj -f-°l -(- 0 5 ) = s(s -j- 1 ) ft2 = — HL\ (18.7)

that is, the electron spin takes half-integral values s = 1 / 2 (in


units of h).

B. PROPERTIES OF THE TOTAL ANGULAR MOMENTUM


OPERATORS. QUANTIZATION OF THE TOTAL ANGULAR
MOMENTUM. VECTOR MODELS

We shall now show that the operators for the components of the
total angular momentum in the Dirac theory satisfy the same com­
mutation relations as the operators for the components of the
orbital angular momentum in the nonrelativistic quantum theory
(see Chapter 11). It can be seen that the operators L and S com­
mute with each other, because they act on different variables.
Therefore,

J / , - J )1J . = ( L ^ S . t)(L, + S)l) - ( L jl-hSJ,)(Ll4 S l ) =


= L vLv L,L, + S,S>; - S,S, = ifi (Lz + Sz)
and

-- Iftj ^,
JyJz - J zJ, = i7LJ,, (18.9)
J J A— J A.J . = ihiy.

The last two relations are obtained from the first by cyclic per­
mutation of the coordinates

x->- y, (/->- z, z -> x.

Since only the total Quantity (18.6) is c o n s e r v e d , the se p a ra ti o n of the a n g ul a r mo­


mentum into s p in a nd o rb i t al p a r t s i s no t r i g o r o u s in t he g e n e r a l c a s e . T h i s s e p a r a t i o n is
f o u n d t o b o p o s s i b l e o n l y in c e r t a i n s p e c i a l c a s e s ( s e e C h a p t e r 20).
THE DIRAC THEORY OF THE MOTION OF AN ELECTRON 295

The operator of the square of the total angular momentum is seen


to be
J* = (L + S) 2 = L2 + S2 + 2 (L,S, + L y Sy + L,S,), (18.10)
which commutes not only with the Hamiltonian but also with any of
its components, for example, with the z component:

J*JZ— J,J2 = 0. (18.11)

By analogy to the orbital and spin angular moments, we conclude


that the square of the total angular momentum and one of its com­
ponents (for example, J;) may have simultaneous eigenvalues.
The quantization rules of the total angular momentum may be
found from the quantization rules of its orbital component (for a
spinless particle):
I* = «*/(/ + 1 ), (1 = 0,1,...), L2— hm, = — .... + /) (18.12)

and its spin component (for example, for / = 0 )

S 2 = / r s ( s + 1), ( s = 7 4), S, = hms, (ms = :±7,). (18.13)


This problem can be solved exactly in general form in the Dirac
theory. For the sake of simplicity we shall, however, solve it in
the Pauli approximation, that is, taking into account the spin of
the particle and assuming that the particle itself is nonrelativistic.
If the particle moves in a central field of force, the components
of the wave function

>F = 'Ti\ (18.14)


>I\.

which obeys the Pauli equation (see Chapter 16), can be related by
means of the law of conservation of angular momentum

(18.15)

where L = r x p is the orbital angular momentum operator and <z'


are the two-component Pauli m atrices. We shall look for a solution
of the system of equations (18.15) in the form 2

IJT,=c,yr1. T., = c,y7 , (18.16)


2In t h i s p a r t i c u l a r c h o i c e o f a s o l u t i o n , o n ly th e s q u a r e of t h e o r b i ta l a n g u l a r mom en
turn i s c o n s e r v e d , n o t i t s p r o j e c t i o n on t h e z a x i s .
296 R E L A T IV IS T IC QUANTUM M E C H A N IC S

where K,m are the spherical harmonics (see Chapter 11). Then,
since

L4 © = w + » K ) . (18.17)

Eqs. (18.15), (18.12) and (18.13) give

_l_
n
or

l[ ( L je- i L J,)qri + L,T1] = ^ 1,


(18.18)
1 [(L, + /Lj,)T 1 - L zT8 ]=< 7 '1 -2>

where

(18.18a)

Using the relations (11.87) and (11.88), we have

L,Y? - — ih YT = tnhYf, (18.19)


(L, ±~ iLy) YT = - f l / ( / + l ±m)(/zpm) Y7~. (18.20)

Then, taking into account (18.16), we may write Eq. (18.18) as

(q — m - 1)C,+ ] / ( / + 1 — m ) ( l - \ - m ) C i = 0,

V ( l + 1~ m ) (l + m) C, + (q + m) C, = 0.

From the requirement that the determinant of the system be equal


to zero, we find two values of q corresponding to the two possible
types of solution

7 = '. / = i + 7„ (1.8.22)

7= i = i ~ % c, = Y (18*23)

The coefficients C, and Clt which determine the relationship 3 be­


tween the spherical harmonics in the sum of the orbital and spin
angular momenta, are called the Clebsch-Gordan coefficients.

Wo n o t e Ih <i t t h i s i c l o t ion s h i p b e t w e e n th e s p h e r i c a l h a r m o n i c s i s o b t a i n e d o n l y in t h e
c a s e of s p i n - o r b i t i n t e r a c t i o n , w h i c h we h a v e t a k e n in t o a c c o u n t w i t h t h e a i d o f t h e r e ­
l a t i o n s (18 .1 5 ). If t h e r e i s no i n t e r a c t i o n , t h e two s o l u t i o n s w ill b e c o m p l e t e l y i n d e p e n d e n t .
T H E D I R A C T T H E O R Y OF T H E M O T I O N OF AN E L E C T R O N 297

Using also the normalization condition Cj-)-C-= 1, we may write


the first type of solution when/ = / + -7, / = 0, 1................in the fol­
lowing form:

>F( ; = i + 1/s) =
Y/ + 1—m
l + '«
'll f I
y m —I
/

y m
v u ^ t + ' h)
*I m (18.24)
- V 2/+ I
For /' = / —‘/s. / = 1, 2, . . . (the second type of solution), the wave
function has the form
f /- m l y m - \
qn;=/-'/2) V 2/ + 1
y m
\ - YIu m= ‘ i / 2)
(18.25)
l/" —
V '21 +

where Y^m are the so-called spherical spinors. The orthonormality


condition for the spherical spinors can be written as

dQYlP+. V'<(/,b = 8a , 8 „ . 8 B(B. (18.26)

where /' = / + 7* corresponds to the case in which the spin and orbital
angular momenta are parallel, and / = / —7* to the case when they
are antiparallel. This condition follows immediately from the fact
that the spherical spinor Y\i]m is a single-row matrix and from the
orthonormality condition for spherical harmonics. The spherical
spinors (18.24) and (18.25) are spinor generalizations of the ordinary
spherical harmonics (see Chapter 11) and represent the angular part
of the solution for all problems involving the motion of a particle of
half-integral spin in a central field of force.
Substituting these solutions for the function T into (18.15), we
find that the component Jz of the total angular momentum takes the
value Jz — hrrij, where the quantum number is equal to rtij= m — -7.
For the first type of solution (/ = / + --). it can be seen from (18.24)
that it ranges from — / (m; = — I ——= — /) to I -f- 1( my= / -(- -7 = j ),
since the coefficient at the function Y?~\ which does exist tor the
last value of m, vanishes. In exactly the same way the number m
in the second type of solution (/ = / — -7) ranges from — / - f 1
(rrij = — j) to I (rrij = j).
Thus our results can be summarized as follows. The square
of the total angular momentum has the eigenvalues
298 R E L A T IV IS T IC QUANTUM M E C H A N IC S

that is, it is quantized sim ilarly to the orbital angular momentum,


except that the quantum number j, which is called the total
angular momentum quantum number, takes half-integral values. 4
The eigenvalues of the component of the angular momentum along
any axis are also characterized by half-integral quantum numbers

Jz = flnij, m.j = —/'....... +/■ (18.27)

From the relations (18.6) and (18.7) and the quantization rules
(18.26) and (18.27), it is easy to obtain quantization rules for the
scalar products L • S and J • S', which are important in spectros­
copy

L- S = l ( P - L 2- S 2) = - { / ( / + l ) - / ( / + l ) - s ( s + l ) } , (18.28)

and by analogy

J • S = i - ( J 2- L '2+ S2) = | { / ( / + l ) - / ( / + l) + s(s+ l)}. (18.29)


We shall consider here the vector model of the addition of angular moments. In
spite of the lack of mathematical rigor of this model. It enables us to resolve a number
of complicated questions and often gives accurate results.
As we know, the orbital angular momentum does not have
a specific direction in space In quantum mechanics. The
absolute value (square) of the angular momentum and one
of Its components, for example, along the z axis, have,
however, simultaneous definite values. These facts can be
represented geometrically by an angular momentum vector
that describes a cone about the z axis. The projection of
the angular momentum on the z axis will then have a well-
defined value, whereas the projections on the x and y axes
remain indeterminate. These arguments apply with equal
validity to the spin angular momentum since it has the
same commutation properties as the orbital angular mo­
mentum. The spin and orbital angular momentum vectors
are oriented in space in such a way that their sum forms a
vector J that is constant in magnitude. Thus, the vectors
L and 5 do not have arbitrary directions; they precess about
J like two coupled gyroscopes.
The dimensionless quantities j*, /* , and s* are drawn
in Fig. 18.1. Each of the vectors I* and s* is defined on
the surface of a cone. They "precess” around J* like a
coupled system. We note that, according to Eq. (18.26),
the addition of the vector I* ( /= 0, 1, 2, . . .) and the
l i p . III. I. A d d i t i o n o f t h e v e c t o r s * ( s = ; ! _ i / a ) leads to the total angular-momentum
spin and orbital angular vector J* with half-integral values of the total angular mo-
m oments. mentum quantum number j = I i 1/2.

T h e num ber j is also called the internal quantum number. This number was introduced
by speclroscopisl s before the discovery of spin on a purely em pirical basis. Il e x p r e s s e d
certain internal properties of p a rticle s that were s ti ll u n c l e a r at th a l stage.

^T h e lack o f m a t h e m a t i c al rigor lie's, for example, in the fact thal the square of the
vector j is e q u a l t o j ( j i 11 r a t h e r th an to / 2.

fT * ' r o m the1 s t a n d p o i n t of the clas s ic al theory, this coupling can be i n t e r p rctecl as a


coupling o f t he- o r b i t a l m o t i o n by the m a g n e tic field.
T H E DI RAC T H E O R Y OF T H E M O T I O N O F AN E L E C T R O N 299

From the vector model we can quickly find a number of quantities. For example, we
can find the quantization rule of the angle between the vectors I* and s*. From the
oblique triangle we obtain

cos (l* s ’ ) 2/*S:l - {j*‘ — I*' —s*2} (18.30)

that Is,

cos (Z*s*) = •/ ( / ! ' n ‘ 1 (/ ■+1>—s (18.31)


'2 Vl(l+ l)s(s+l )

C. MOTION IN A CENTRAL FIELD OF FORCE INCLUDING


SPIN EFFECTS. THEORY OF THE ROTATOR

If we wish to investigate the motion of a particle in a central


field in the nonrelativistic approximation with the inclusion of
spin effects, we must use spherical spinors Y\Jm ' characterizing
the states in which the total angular momentum (orbital plus spin)
is conserved, instead of the spherical harmonics Y'" describing
the states where only the orbital angular momentum is conserved.
Since the spherical spinors (in the nonrelativistic approximation) are
composed of spherical harmonics having the same quantum number /,
we obtain the same radial equation as for a nonrelativistic spinless
particle, that is,
f?R + R = o. (18.32)

For the wave functions of an electron in a central field we obtain

»Ft(i = RYY!„, (18.33)

where the spherical spinor is defined by the expressions


(18.24) and (18.25).
In particular, for the rotator we may set r = a = const and the
radial part of the wave function R = l. It is seen that the spin
effects in the nonrelativistic approximation do not give any ad­
ditional terms for the rotator energy, which will be given by the
same expression (12.7) as for a spinless particle, that is,

F _u-i(/ + I)
1 2ma(i-

The wave function will be given by the spherical spinor Y]1^;


therefore we must find the selection rules for the quantum num­
bers I, nij and / . These selection rules hold not only for the
rotator, but also for any problem of motion of a particle in a
central field of force (for example, the electron in the hydrogen
atom).
300 R E L A T IV IS T IC QUANTUM M E C H A N IC S

In place of Eq. (12.19), from which the selection rules for


spinless particles were established, we now have

{q)limly = ) ' ( ’/ 'Pm-)+ qY\nmdQ, (18.35)

where q may have three values

q — z = cos&, q — x ± i y — %\n%e'~it (18.36)

(for simplicity let us take the radius of the rotator equal to unity:
a = l ) . If in place of the spherical spinors we substitute their
values from (18.24) and (18.25), the matrix element (18.35) be­
comes

(?){£/= DW> J ) ( y f - y qY?-' dil - f Oi’i) (| (Yf)* qYT dQ. (18.37)

From this it is seen that the two integrals in (18.37) agree exactly
with the integrals in Eqs. (12.19)-(12.22). The selection rules for
the quantum numbers / and m will therefore be the same as for
a spinless rotator, that is,

M = l — /' = z b l , Am= 0 (q = z), Am = ± l (q = x ± i y ).


We shall now find the selection rules for the quantum numbers
nij and /. Since is related to m by the relation m, = m—
for both types of solutions, the selection rules of will be the
same as for m, that is,
hnij = 0, dt 1.

To determine the selection rules for j let us consider first the


case in which the transitions occur between states characterized
by the same type of solutions (j' = /'-)- ‘/s ->/ = ’/4 or f — V— '/.2->
/ = / — ' ,), It follows from (18.24) and (18.25) that the coefficients
D'i'i) and CJi’i) are always positive and therefore such transitions
are always allowed. In this case the possible change of / must
be the same as the change of the orbital quantum number /, that is,
A/ = A/ = ~± 1.
If the transitions occur between states characterized by different
+ypes of solution (/ = /' -)-1/«-> j = / — '/2 or j' = i — '., -*■ j = l -f ’/2),
then by taking into account A / = d i l , we obtain three possible
values for A/ = 0, +2, -2. Here, however, we must consider the
fact that the coefficients Di'i) and Qi'i> have different signs. For
A/ = Jr 2 the two terms cancel each other, so that this transition
T H E DI RAC T H E O R Y O F T H E M O T I O N O F AN E L E C T R O N 30

is forbidden. For A/ = 0 the difference between the two terms is


not zero, but owing to the fact that the two terms occur with
different signs, the intensity of the radiation will be weaker
than in the case of transitions between states characterized by
the same type of solutions, when A/ = zb 1. This can be shown with
the help of a specific example. Let us suppose that the initial
state is j = l — */.> and the final state is / ' = / ' -|-’/2. We shall
consider the case A//i = mr— m = 0. Then, using (12.22), we re­
duce the appropriate matrix element (18.37) to the form
{zj;$J' = lr, l+,\DV-»A{l, m)] +
+ 8,-./-, m— \) + Oi^»B{l, m)\, 1 '

where /' = /'-f-’/o and j — l — ‘/». Substituting the expressions

D</ = ''+ */2 . j=i-'h) VI' + m7 Vl + 1 — m


Vi 2/'+l)(2/+l)

and
______ I_____
C ( / = ' ' + 1/o . / = ! - ' / : ) = _ V r + l — m'Vl + m
/(2 /'+ I)(2/+ 1)

from (18.24) and (18.25), and the expressions for A and B from
(12.22a), we find that the coefficient of V , + 1 vanishes; that is,
the transition Aj = — 2 is forbidden. At the same time, the co­
efficient of 8/'. ;_i does not vanish, that is, the transition Ay = 0
is allowed, but the intensity of the lines is weak in comparison
with A/ = ± 1. In a sim ilar manner it is easy to show that the
transition A; = 2 is forbidden not only for q = z, but also for
q = xAziy (Am = zbl).
In accordance with the above discussion, the selection rules
for the quantum numbers which characterize the state of a particle
in a central field of force, when the spin is taken into account,
have the form

A/ = ± l , Am; = 0, zb I, (18.39)

. . / — I (normal intensity),
^ 10 (weaker intensity).

D. PARITY OF A STATE

In connection with the formulation of the law of conservation of


angular momentum for Dirac particles, we shall now define more
clearly the meaning of the quantum numbers I and j in the Dirac
theory of a particle in a central field. In nonrelativistic quantum
302 RELATIVISTIC QUANTUM MECHANICS

theory, the orbital quantum number / is associated with the square


of the angular momentum L- = ffl (I + 1), which is a constant of
the motion; therefore in both Schrodinger’s and Pauli’s theory
the quantum number I represents a quantity that is constant in
time.
In the Dirac theory the orbital angular momentum does not
commute with the Hamiltonian, and therefore it is not a constant
of the motion. Consequently the quantum number I has only an
approximate meaning when used in connection with the law of
conservation of angular momentum.
It turns out, however, that / characterizes an additional property
of a particle in quantum theory, namely, the parity of a state. By
the parity of a state we mean the behavior of the wave function
with respect to space inversion:

x = — x', y = — y\ z = — z'. (18.40)

The parity operator is defined as follow s7

= /•); (18.41)

that is, it reverses the signs of the space coordinates. The eigen­
values X of this operator may be found by applying this operator
twice:

I^ = X^. (18.42)

This double application of the parity operator leaves the co­


ordinates unaltered. From (18.41) and (18.42) it follows that

X= zb 1; (18.43)

that is, either the wave functions remain unchanged with respect
to space inversion (even functions, X= 1), or they reverse their
sign (odd functions, X= —1).
We shall now find the quantities which determine the parity of
a wave function in a central field. In the spherical coordinate
system r, ft, ?, space inversion affects only the angular part

r' = r, ft' = - —ft, (18.44)

as can easily be seen from the fact that the sign of the coordinates

a-= r s i n ft c o s 'f, y = / - s i n ft s i n 9 , z = r c o s ft (18.45)

T h i s o p e r a t o r c o n v e r t s a r i p h i - h a n d e d s y s t e m of c o o r d i n a t e s into a l e f t - h a n d e d system ,
a n d v i c e v e r s a . In t h e D i r a c t h e o r y I </ Cr) ^ ^
T H E DI R AC T H E O R Y OF T H E M O T I O N OF AN E L E C T R O N 303

changes. The radial part of the wave function remains unchanged


with respect to space inversion, whereas the angular part changes
in accordance with the relationship

lr;" (0, <p)=J'7(« — », ic + <p) = const P"‘ (— cos 0)e"» <*+ *>=
—(—l)'Y'in, (18.46)
because
p/n( - x ) = ( - i

Thus the parity of a state in a spherically symmetric field is deter­


mined by the parity of the number /.
Furthermore, it can be seen that the Hamiltonian in a central
field remains unchanged under space inversion; therefore, the in­
version operator 1 and the Hamiltonian operator H commute with
each other. It follows that the parity of a state is a constant of the
motion, since

- 1 = i - ( H I - I H ) = 0. (18.47)

The law of conservation of parity has no classical analog, unlike


the other conservation laws (energy, momentum, angular mo­
mentum). Consequently, in nonrelativistic wave mechanics the
number I characterizes two constants of the motion: the square
of the angular momentum and the parity of a state. In the Dirac
theory, the number I does not have the significance of the square
of the angular momentum, but the relationship between this num­
ber and the parity of a state is preserved.
We shall see later that parity plays a particularly important
role in the physics of elementary particles. All wave functions—
whether for one or more particles—can be classified as functions
of odd or even parity. Dipole transitions can occur between states
with different parities. In the case of two or more particles, the
parity depends on the total spin of the system and also on the type
of statistics obeyed by the particles. These concepts will be
analyzed in some more detail in the treatment of specific examples.

E. SOLUTION OF THE DIRAC EQUATION


FOR A FREE PARTICLE

Let us consider the motion of a free particle of spin 1/2 and con­
stant momentum such as, for example, an electron. Without loss of
generality we may take the z axis to lie along the direction of
momentum; that is, in Eq. (17.20) we set

Px = Py = 0, Pz 0. (18.48)
304 R E L A T IV IS T IC QUANTUM M E C H A N IC S

The Dirac equation then takes the form

(ifl -h ca3ih — p3m0c j ^= 0. (18.49)


We shall look for a solution of this equation in the form

<|»= pr; be- ic*Kt + ikz, (18.50)


where L3 is the normalization volume, and the wave number k has
the same value as in the nonrelativistic Schrodinger theory,
namely,

k = ^ { n = 0, 1, 2, ...),

K = VW+%, =
and the 4 x 4 matrix

satisfies the normalization condition


bvb = b\b, + btbt + b%b, + bjbk - 1. (18.51)

To determine the quantity e and the coefficients b^ we use the


system of equations (17.24), setting

P* = P.y= 0. Ety=cfaKty, pz’i>= hty.


We then obtain

{zK - *.) b, - kb, = 0, (e/< - klt) b, + kb, = 0, (18.52)


( s / < + kt ) b, — kb , = 0 , ( s / C + k v) bx+ kb, = 0.

Hence we find two values for the quantity e : e = 1 (the energy of


the electron is positive) and e = — 1 (the energy of the electron
is negative); while we obtain four values for the matrix b, the
components of which satisfy the normalization condition (18.51).
Two of the values of the matrix b refer to states with e = l:

&'■>= (18.53)
\'2
T H E DI RAC T H E O R Y O F T H E M O T I O N O F AN E L E C T R O N 305

and two of them refer to states with e = — 1:

1
M3) = —L bM (18.54)
V2 \/2

that is, the states (18.53) differ from the states (18.54) in the
sign of the energy.
To determine the physical significance of the states bU) with
different i = 1, 2, 3, 4, let us find the projection of the spin on
the direction of motion, that is, on the z axis. First of all, we
note that, since L, = xpv— ^ * = 0 when a particle is moving
along the 2 axis, the projection of spin on the z axis must be con­
served. This follows directly from the fact that the matrix s3
commutes with the Hamiltonian in (18.49).
We can find the eigenvalues of this operator o3 by applying
it to the spin functions b[,). We have then

/1 0 0
0 — 10 0
/V* V1
\ 0 = _L6(d. (18.55)
0 0 1 0 — 2° ’
lo 0 0 - 1/ \yiY'0
that is, for this solution the eigenvalue of the operator l/i a, equals
V«. In exactly the same way it is easy to show that

1 o36(2)= — !&<*>, i 0,6(3) = y 6 (3) and i 0 = - I //*>.

Thus, the four possible states correspond to the four possible com­
binations of the sign of the energy and the spin direction. The solu­
tion bn) corresponds to positive energy ( s = 1) and the projection of
this spin along the positive z direction (5 = 1). In a similar way we
have e = 1, s = — 1 for the solution 6(4'and we have e = — 1, s = 1
and s — —1, s = — 1 for the solutions6(3,and b(i), respectively, where
s is double the projection of the spin in the direction of the mo­
mentum.
In the nonrelativistic limit (v < c), the wave functions <t3 andijj4
will be of the order of tim es the wave functions ^ and<|)2

( |3~ y ^ i) for positive energy states (e = 1). For states with nega­

tive energies [see (18.54)], on the contrary,


306 RELATIVISTIC QUANTUM MECHANICS
We have considered the special case of motion of a particle along the z axis. This
does not restrict, however, the generality of the investigation of the general motion of a
particle. Whenever the direction of momentum is characterized by spherical angles 8
and <p, it is always possible to choose a primed coordinate system in such a manner
that the z' axis is directed along the momentum. Then, by carrying out two rotations,
one rotation through an angle 8 around the y' axis directed perpendicularly to the zz'
plane, the other, second rotation through an angle f around the z axis, we may transform
from the solution in the primed coordinate system (momentum along the z' axis) to the
general case (direction of momentum characterized by the spherical angles 8, <p).
Using the fact that under a rotation of the coordinate system the wave function
changes in accordance with (17.39), we may write the solution for this general case

/> ^ ( ,+ - if

- 2 H®Ja) —UcKt + Ikr


iL= ----- e (18.56)
V
•V'

\ - v r = ~ ’ ('-4 T
which is a generalization of Eqs. (18.50), (18.53) and (18.54).
8
Problem 18.1. Show that the energy and momentum operators E = ift ^ andp = — iftV,
0
respectively, transform like a four-vector under the Lorentz transformations:
E' | „ < > E'
E 7 + PP* _ _ P* + T P _ _f

c y i __1 y j ps ’ Pv
where

P = — , E' = ih , and so on.

Hint. Use the Lorentz coordinate transformation [Eq. (17.34)] and change to new
variables in the process of differentiation.
Problem 18.2. Prove the relativistic invariance of the scalar equation for a free
particle.
Hint. First, let us prove the invariance of the operator relation

E'a — c2p'2 = E 2— c2p2


by using the results of the preceding problem.

Problem 18.3. Show that in the case of spatial rotation of the coordinate system
around the y axis by the angle 8 , the wave function transforms according to the relation

C) ^ V »
, t.__j.,
( cos + (o3 sin —J ip= e 2

Hint. Use the method which leads to the relation (17.39).

Problem 18.4. With the aid of the vector model of addition of angular moments,
find the angles between j * and s* and between j* and /* taking into account the geo­
metric vector addition in quantum mechanics; that is, find
cos (j*s*)\ cos

T h e s e p r o b l e m s a l s o ref er to the m a te r ia l in C h a p t e r 17.


T H E DI RAC T H E O R Y O F T H E M O T I O N OF AN E L E C T R O N 307

Hint. Use a method similar to the one that led to Eqs. (18.30) and (18.31).

Problem 18.5. Show that the wave function where C = ia3pa, and t];*ls
the complex conjugate (but not the llermitian adjoint, that is, noti^+) of the Dirac wave
function for an electron with negative energy satisfies the Dirac equation with positive
energy and opposite (positive) sign of the charge, that is, describes the motion of a
positron (a charge conjugate transformation).
Solution. The Dirac equation is

ft d_ i in 0 e
i dx + “2 ^Tdy~T Ay +

for the complex conjugate of this equation, we may write (taking into account the fact that
of - «i, a* = — “ 2 , of = o8, pf -- p8):

{ t + c [“■( t S + 7 ^ ) - ' (7 ^ + 7 ^) +
+ °a ~ ’!'* = 0.

We note that the complex conjugate ip* differs from <|i+ (the Hermitian conjugate): namely,

I* =

whereas

Let us substitute <jJ= io.p,^* into the complex conjugate of the Dirac equation. We

then find that IJi satisfies the Dirac equation if the charge e is replaced by - e.
-f|£ |,
4> *
Since the state <Ji(r, t) = e i}' (r ) Is treated as a state with positive energy (E = | E | ),

and the state <|/* (r,t) = e 71


~lLr t
<|/* (r) is treated as a state with negative energy
(E = — \E | ), we must Interpret the sign of the energy in ^ differently than in
Chapter 19

The Dirac Equation in Approximate Form

In many problems which are solved by the Dirac theory, we


retain only the relativistic corrections of the order of the
final results. Therefore we may immediately write the Dirac equa­
tion in in approximate form, retaining quantities of the order o f^ y j.
It will be shown below that the role of both the relativistic and spin
terms is clearly displayed in this approximation.
Let us consider the motion of an electron with positive energy
E^> 0 in an electromagnetic field which does not depend on time. In
this case we may replace the energy operator by its eigenvalue,
separating out the rest m ass energy m0c2:
E— (19.1)
The wave equation (17.20) then becomes
(E - e(I>) g ; ) = c (o'-P) g j , (2muc- + E - e*) ({;) = c (ff'-P) ({;), (19.2)
where a' stands for 2 x 2 Pauli m atrices [see (16.18)], and P =
p— 1 A. Equations (19.2) are simply a different form of the exact
Dirac equation. For the components of <]», we obtain from (19.2)
(E - <*»]») %- c (P.t - iPy) ^ - cP^a= 0
and so on. This equation can also be obtained from the first of Eqs.
(17.24), if we substitute into it both (19.1) and (17.25).
As was mentioned in the preceding section, in the nonrelativistic
lim it the components 0, and ^ are “ sm all” for positive energy
states, since they are of the order of y tim es the “ large functions
•j, and ^.2. The transition to the approximate Dirac equation consists
in eliminating the “ sm all” components and -!>, from Eqs. (19.2)
and retaining terms of the order of (^—'j in the remaining equations
for the “ large” components and <^2. Thus from (19.2) we obtain
(19.3)
T H E D I R A C E Q U A T I O N IN A P P R O X I M A T E F O R M 309

First of all we change from the four-component wave functions

to the two-component functions If by setting

N (19.4)
W— »F .j.

where N is the normalization coefficient. This coefficient may be


determined from the “ renormalization” relationship

/'t'A
» w ) = < w j ) ( J;). (i9.5)

Since the “ sm all” wave functions and <Jj4 occur as squares in


(19.5), we may set

'h' a'*p
2 m„c
N (19.6)

that is, in (19.3) we neglect second-order terms and replace P


by p. This change of operators is permissible in calculating the
normalization coefficient since they differ from one another by a
first-order term (inversely proportional to the velocity of light).
When applied to second-order terms (squares of the “ sm all”
wave functions), this term of P gives only third-order term s,
which we discard. Then, substituting Eqs. (19.4) and (19.6) into the
left-hand side of (19.15) and using the equation1

{o’- a) (o’ - b) = (a • b)+ ip • (ox 6)J, (19.7)

which holds both for the Pauli and Dirac m atrices, we find

( W i) J;) (19.8)

From this we obtain


p-
N = 1— 8m;c2
o r d e r to p r o v e t h i s e q u a l i t y , w e m ay , w i t h o u t l o s s o f g e n e r a l i t y , a s s u m e t h a t v e c t o r
a i s d i r e c t e d a l o n g t h e z a x i s (Qx = ay = 0 , a z = a), a n d t h a t v e c t o r b i s l o c a t e d in th e zx

p l a n e ( by = 0). T h e n we o b t a i n

(t7 • a ) (£ 7 ■ 6 ) = (cr^a){cr^bz + C \ b x ) = abz + icr^aby =

= a • b t i [a • (h x </)l = a ■ b + i l a " ■ (a x 6)]


310 R E L A T IV IST IC QUANTUM . M E C H A N IC S

This approximation gives us, in accordance with (19.3) and


(19.4),

='l 8m?cs - ) ©
\ •
PS (19.9)
8my

Substituting (19.9) into the first of Eqs. (19.2) and neglecting terms
of the order of ( v / c )3, we find

{£ — e$ 8maca <E-C4.)p>}© =
P1 (19.10)
= (h ; t« -p ) 16m jica Hi

Using the relation (19.7), we have

(a'- P) (o'- P) = P* - f i [a'.(P x p )]=


= PJ— (^[(p x/l)-i-(/l x p) ] ) = P 1— — [o'.(v Xi4)]= (19.11)

= PJ

where / / i s the magnetic field since the operator v acts only on the
vector potential A, and not on b.
In exactly the same way, with the aid of the relations
(o', p) (£ — eO) (o', p) = (£ — e<D) p* — ihe (o'- £) (o', p) =
= (£ — e<t>) p’ — ihe (£*p) -f- eh [o’.(£ x p) ]
and
= p ^ £ _ e<t) = ( £ _ ea , ) p ^ 2-le £.p (19.13)

where £ = — v(1> is the electric field intensity, we may reduce the


Dirac equation (19.10) to the following approximate form:

, Ps _ P*_ eh
e ‘I> -------- 5 -
2m0 8 mlc- 2m0c
(o './/)-
eh
4 m lc-
[o'.(£xp)] + (19.14)

The left-hand side of Eq. (19.14) describes the motion of a particle


with a nonrelativistic velocity in a stationary electromagnetic
field. The right-hand side of (19.14) contains an additional inter­
action energy that describes the relativistic and spin corrections.
THE DIRAC EQUATION IN A P P R O X I M A T E F O R M 3 11

The first term on the right-hand side of (19.14)

1/ r e l _________ ____
(19.15)
— 8m2ca

takes into account the correction due to the relativistic velocity of


the particle. A sim ilar additional energy must also appear in the
relativistic Klein-Gordon equation. The classical analog of this
term will be obtained if the relativistic expression for the Hamil­
tonian is expanded in a series, retaining terms of the order of
(v/c)2

H = V + p V = m Qc 2 +

The second term on the right-hand side of (19.14) may be written


as

l/mag— — jx*//. (19.16)

From this it is clear that the quantity p = --en a’ may be treated


ZIHqC
as the Dirac magnetic moment of an electron, which appears ex­
plicitly in the nonrelativistic approximation only through this
transition. This interaction energy turns out to be of the order of
v / c. From the intrinsic angular momentum [Eq. (18.6)] of the
electron

S= -ff\ (19.17)

we find the relationship between S and p that is required by ex­


periment and follows automatically from the Dirac theory

0= — S. (19.18)

The next term of the expansion characterizes the so-called spin-


orbit interaction

which describes the interaction of a moving magnetic dipole v/ith


an electric field.

This Interaction may also be Interpreted from the classical point of view In the follow­
ing way: a magnetic dipole moving with a velocity v (the spatial component of a tensor
312 R E L A T IV IS T IC QUANTUM M E CH A N IC S

quantity) acquires an additional electric moment (space-time component of the same


tensor quantity)

l*el = - - * >'!*= (19.20)

which Interacts with the electric field of the nucleus. This additional energy of inter­
action Is

eti
Vcl = — £.jiel
2/»;c: [o’. (Exp) ] (19.21)

This classical expression for the interaction energy is twice as large as the correspond­
ing quantum expression [see (19.19)]. Even before the advent of the Dirac theory, an
attempt was made to explain the fine structure by the sem iclasslcal introduction of spin-
orbit interaction. To obtain an agreement with experiment. Thomas and Frenkel sug­
gested that we substitute the coefficient 1/2 into the classical expression for the inter­
action energy (19.21). This interaction, which follows automatically from the Dirac
theory, is called the Thomas-Frenkel correction.

In particular, for the Coulomb field of a nucleus

= E = -°r, e = - e 0. (19.22)

The interaction between the moving magnetic dipole and the


nucleus according to (19.19) becomes

Vs-°-= S-L, (19.23)


2mic2n
where S = n <r'/2 is the spin, and L=-=rxp is the orbital angular
momentum.
We note that there is no spin-orbit interaction for an atom in
the s state since the orbital angular momentum in this state
vanishes. Finally, the last term of the interaction, which in the
case of the Coulomb field is equal to 3

(19.24)

is called the contact interaction. The additional energy corre­


sponding to it

A£cont= \ U' l/cont'F^:lx (19.25)

2.|. I. F r e n k e l , W'nur Mr r h a n i c s , N ew York: Oxfo rd U n i v e r s i t y P r e s s , 1938, Vol. 2;


s e c i n s ’6 ( 1 6 .1 1 ), w h e r e Ihe c o e f f i c i e n t 1 / 2 a p p e a r s in t h e m a g n e t i c m o m en t p r o d u c e d by a
moving charge.

J In th e d e r i v a t i o n of (19 24), w e h a v e u s e d t h e f a c t th at, a c c o r d i n g to (4 .7 8 ),

- 47r8(r) - 4tt8(x) 8(v) 8 (2) . (19.24a)


THE DIRAC EQU A TI O N IN A P P R O X I M A T E F O R M 313

is proportional to i *lr (0) i2, and it will differ from zero only for the s
state since, according to (13.28a), only in this case | ( 0 ) r/ 0.
For all other states (I 0) this square of the wave function vanishes
when r = Q. In this sense the contact term may be regarded as
the spin-orbit interaction for the s. state. We can see now that
the last two terms in the interaction energy (19.14) characterize
the spin properties of an electron.

Problem 19.1. Show that the matrix ca Is the velocity operator, and that In the case
of a free particle a is not a constant of the motion, unlike the momentum operator
(p = — . Explain this difference. Determine In what case the average velocity

® ^ ^ c a ^ d '.v , and so on,

will be related to the average momentum by the classical relation

» = c2§r. (19.26)

Hint. From the Hamiltonian (17.21) it is possible to obtain the velocity operator
. /
r = (Hr — rH) = ca,

It can also be shown that the velocity Is not a constant of the motion, since a ^6 0 . Con­
sequently, if for a given k we take a linear combination of positive and negative energy
states [see Eqs. (18.53), (18.54)] there will exist interference terms that will fluctuate
with time ( ~ e— 2,cKt). As a result, Ehrenfest’s theorems will hold only on the average
in the Dirac theory. The interference terms will disappear in the calculation of the
average value of the momentum operator.
Equation (19.26) holds only if the states with positive energies are retained (e = 1).
Chapter 2 0

The Fine Structure of the Spectra of


Hydrogen-like Atoms

A. STATEMENT OF THE PROBLEM.

The problem of the motion of an electron in a hydrogen-like atom


(Kepler’s problem) is rightfully considered as the touchstone of all
forms of quantum theory. There are two main reasons for this.
First, it has great physical significance, since the problem of
motion in a Coulomb field can be solved exactly. Second, the
results may be compared with experiment to a high degree of
accuracy; for example, the em ission spectra of atoms can be
observed by optical and microwave spectroscopy.
The solution of the problem of motion of an electron in a
Coulomb field of a nucleus (hydrogen atom) on the basis of the
Schrodinger equation, obtained in Chapter 13, gives an expression
for the energy

( 20 . 1)

which is in good agreement with experimental data. This ex­


pression for the energy may be taken as the zero-order approxi­
mation. A more detailed study of atomic spectra shows, however,
that the spectral lines have a fine structure which, of course, must
be associated with the detailed structure of the energy levels. The
Schrodinger theory does not give an adequate description of the
regularities frequently occurring in spectra, since it neglects at
least two important facts: the relativistic dependence of mass
upon velocity and the spin properties of the electron. Both these
facts, as we already know, are accounted for by the Dirac theory,
and therefore application of the Dirac equation to the Kepler
problem gives results that accurately describe the multiplet
structure of energy levels.
Ajs was pointed out, the Kepler problem can be solved exactly
in the Dirac theory. The solution, however, requires many
tedious calculations (much more complicated than in the Schrodinger
theory, because in this case we have not one but four equations).
Moreover, in the course of these calculations one does not always
THE FIN E STRUCTURE OF SP E C T R A 315

perceive the physical meaning of the results, the analysis of which


is of primary importance to us. We shall therefore use a more
elementary method, based on the approximate equations of the
preceding section. This method not only enables us to obtain
formulas characterizing the fine structure up to terms of the
order of I/ v- \2
1 » but also to interpret the individual terms as
manifestations of relativistic or spin properties of the
electron.

B. RELATIVISTIC AND SPIN EFFECTS

As follows from Chapter 18 [see (18.24) and (18.25)], the wave


function of a particle obeying the Dirac equation, taking into
account the spin properties, is

xW = RniY(». (20.2)

Here Rn[ is the radial part of the wave function and is a spher­
ical spinor: for / = /-)- ‘/.2the spin is parallel to the orbital angular
momentum and for j = l — ‘/a it is antiparallel.
Although terms of the order of {vjc)'1 are not formally accounted
for in Eq. (20.2), the relationship between the spherical harmonies
in the spherical spinor that determines the zero-order approxima­
tion of the wave function is established by the spin-orbit interaction,
which is of the order of (vjcfl
The spherical spinor can therefore be used only when the atom
is not subject to external perturbing forces of magnitude greater
than those involved in the spin-orbit interaction. If that is not the
case, the spin-orbit coupling will be disrupted and a new set of
prem ises must be set forth in order to establish a relationship
between the spherical functions.
Spherical spinors, just as spherical harmonics, satisfy the
equation
Vo, <pYim= —; (* + 1) (20.3)
therefore, taking into account (11.17), the radial function in (20.2)
satisfies the same equation that was derived in the nonrelativistic
Schrodinger theory:

„2 d | ( 2moEn | 2 m0Zel I (/ + 1)
VA ( T ft2 T ft3 rfi Rnl= 0.

^ T h e r e a r e s e v e r a l c a s e s in q u a n t u m m e c h a n i c s in w h i c h a s m a l l i n t e r a c t i o n e n e r g y
e n a b l e s u s to f in d a r e l a t i o n b e t w e e n t h e c o e f f i c i e n t s o f th e f u n c t i o n s in t h e z e r o - o r d e r
a p p r o x i m a t io n . We h a v e a l r e a d y e n c o u n t e r e d a s i m i l a r s i t u a t i o n in t h e t r e a t m e n t of t h e
S tark e f f e c t ( s e e C h a p t e r 14).
316 R E L A T IV IS T IC QUANTUM M E CH A N IC S

The wave function (20.2) completely determines the selection


rules for all quantum numbers. The selection rules for the quantum
numbers I, j, and nij are given by formula (18.39), while the sele c ­
tion rules for the principal quantum number n will evidently be
the same as in Schrodinger’s theory [see (13.48c)], since the radial
function remains unchanged. Considering all this, we obtain the
following selection rules for a theory of the hydrogen-like atom
which takes into account spin effects:

A/ = zh 1, A/ = 0, d il, Am; = 0 , ± 1 ( An is an integer). (20.4)

As for the expression for the energy, we cannot restrict ourselves


in this problem to its nonrelativistic value (2 0 . 1 ), since the
latter does not determine the fine structure of the energy levels.
Knowing the zero-order approximation of the wave function
(2 0 .2) , , and also the additional perturbation energy describing the
relativistic [see (19.15)] and spin [see (19.23) and (19.24)] effects,
we may find the energy levels characterizing the multiplet struc­
ture of the spectrum.
According to formula (19.15), the relativistic correction to the
energy levels is

A£rel = — j (»FW)+ WU)d3x. (20.5)

Since in the present case

= (£ •+ £ ? )* (/),

(V(/,)+£ 0= ( ’F<;))+ (20.6)

we see that this additional energy will be independent of the solid


angles i), 9 ; that is, integrating over the solid angle we get

§dQ (¥%)+¥% = 1. (20.7)

Then the additional energy characterizing the relativistic effects


is

A£rel = - 2 Jo-, ( ( £ '/,) 2 + 2E"nZc'l {r~T) + Z*e\ ( O l =


_ RhZ'S j n 3\
~ n' W +7i 4 j’ (20.8)

where a = tic
^ Vm is the fine structure constant.
THE FIN E S T R U C TU R E OF S P E C T R A 317

In the derivation of (20.8) we have used (13.29a)

— _ Z J_ 2RhZ
a0 n 2 e'iir ’

' ~ \< J h3 (i'-\-'ja) h i 3 (I + ‘/a) ’

Equation (20.8) agrees exactly with the formula for the rela­
tivistic energy, which was calculated in identical approximation by
means of the relativistic Klein-Gordon equation [see (15.31)].
In a similar manner, with the aid of (19.23), we find the addi­
tional energy due to the spin-orbit interaction

A£s' ° * = W ( S ' (20.9)

Using expression (13.29a) for (/* :i)


/ / 3 1

= «*/ (/ + >/>) (/ -H) ’

and expression (18.28) for(S • L)

n2 .
~2 9 for 1^0,
S•L
o for / = o,

we obtain the following value2 for the energy (20.9):

AEs-°.— Rh, 2n3


a- 9(1 — 5/o)
1(1 + (/+ 1)'
(20.10)

In these equations

/ for / = ^+ '/2 ,
?= /(/+l)-i(' + l)-s(s+l) = - ( f + l ) f o r / = *-«/*,
( 20 . 11 )

z At f i r s t g l a n c e it may s e e m t h a t t h e s p i n - o r b i t i n t e r a c t i o n , w h i c h i s i n v e r s e l y p r o p o r ­
t i o n a l to t h e th ird p o w e r o f t h e d i s t a n c e , c a n n o t g i v e a s t a b l e s t a t e . T h i s , h o w e v e r , i s n o t
so. At s m a ll d i s t a n c e s t h e s p i n - o r b i t i n t e r a c t i o n b e h a v e s j u s t l i k e th e r e l a t i v i s t i c i n t e r ­
a c t i o n ; t h a t i s , it i s i n v e r s e l y p r o p o r t i o n a l to t h e s q u a r e of th e d i s t a n c e . I n d i r e c t p ro o f of
t h i s i s th e f a c t t h a t / \ E S,°' d i f f e r s from A E rel o n ly by a n u m e r i c a l f a c t o r o f th e o r d e r of
u n ity .
318 R E L A T IV IST IC QUANTUM M E C H A N IC S

and the quantity

f 0 for I 9 ^ 0 ,
!" = | . for ( _ 0 . (20' 12)
Finally, the energy corresponding to the contact interaction, accord­
ing to (19.24), is given by

A£c°nt = Kft*Zel |W(0) |»,


2m\c2

where

I^ (0) I2 = (0) Yfy. (20.13)

Furthermore, considering the expression for

[see (13.28a)], and using the fact that 1^ 1*= -^ when/ = 0 and
j = 1l2, we find

l lJr(°)l2 = i ( f ) 3’ (20.1.4)

that is,

A£cont== ftfi 8/0.3 (20.15)

From this we obtain the following expression for the additional


energy which accounts for the relativistic effects, the spin-orbit
and contact interactions:

A £ = A £ re l f- A £ s-°--|~ A £cont =
— - ______ <7"(l ~ 5/o)__
' n' [ l + lh 4 21 (l + lh)(l+ 1 )

Incidentally, Eq. (20.10) for the contact interaction may be obtained when the expres­
sion for the spin-orbit interaction (Eq. 20.10) is allowed to go to the limit as / 0, if we
■*

discard the factor 5/q in 20.10. Therefore, many authors use this procedure and neglect
the contact interaction in deriving the fine-structure formula. However, the agreement be-
lw<err the two formulas is accidental since for the slates the numerator of Eq. (20.10) is
s

always zero, while the denominator vanishes only in the nonrelativistic approximation. In
a number of other problems, for example, an atom containing several electrons, the energy
associated with contact interaction is no longer a limit of the expression for the spin-orbit
interaction.
THE F IN E STR U CT U RE OF S P E C T R A 319

Substituting here the value of q from (20.11), we have4

AEnj = - R h ^ r (20.16)
J+2
Therefore, summing both results [(20.1) and (20.16)], we obtain
the fine structure formula for the spectrum of a hydrogen-like atom5
72a 2
RhZa n (20.17)
Enj — Eh + AEnj lla
I +I —
n2
j +2
From this it is seen that the splitting of the levels is proportional
to the square of the fine structure constant.

C. THE FINE STRUCTURE IN THE DIRAC THEORY

When we take the fine structure into account the position of the
energy levels in the hydrogen atom is found to depend also on the
total angular momentum quantum number /. Therefore the terms
will be denoted in the following manner:
EnIJ RZ2 1 + £ l ( ___
n (20.18)
(ntj) n -b „» [J+ >,,
From this formula it is seen that the fine structure, according
to the Dirac theory, depends only on the principal quantum number
n and the total angular momentum quantum number /. In contrast to
the Klein-Gordon theory, it is independent of the orbital angular
momentum quantum number / (up to terms of the order of a2).

“'T h i s p ro b lem c a n b e s o l v e d e x a c t l y in t h e D i r a c t h e o r y . We t h e n o b t a i n a c l o s e d for-


m u la for t h e e n e r g y l e v e l s ; i n t h i s fo rm u la, t h e f i r s t e x p a n s i o n term ( w h i c h i s i n d e p e n d ­
e n t o f a 2) g i v e s t h e n o n r e l a t i v i s t i c form ula (2 0 .1 ). T h e s e c o n d term , w h i c h i s p r o p o r t i o n a l
to a 2, g i v e s t h e a d d i t i o n a l e n e r g y ( 2 0 .1 6 ). T h e th ird e x p a n s i o n term , w h i c h i s p r o p o r t i o n a l
to a 4 in t h i s a p p r o x i m a t i o n , c a n b e n e g l e c t e d , s i n c e i t i s s m a l l e r t h a n t h e s o - c a l l e d
v a c u u m c o r r e c t i o n s , w h i c h a r e p r o p o r t i o n a l to ( s e e C h a p t e r 22).

SAn e x a c t s o l u t i o n o f t h e D i r a c e q u a t i o n g i v e s t h e f o l lo w in g g e n e r a l i z a t i o n o f Eq.
( 1 5 .3 0 ), w h i c h t a k e s in to a c c o u n t t h e r e l a t i v i s t i c e f f e c t s in t h e c a s e w h e r e s p i n i s a l s o
p resen t:

E nj = m 0c 1+ i 0c ( 2 0 .1 7 a )
( n - j - ■/, + V o' + 54)2 - z V ) 2_

E q u a t i o n (2 0 .1 7 ) may b e o b t a i n e d from ( 2 0 . 1 7 a ) , if t h e l a t t e r i s e x p a n d e d in a s e r i e s a n d
w e r e s t r i c t o u r s e l v e s to t h e f i r s t two term s.
S i n c e t h e minim um v a l u e of j i s e q u a l to 1 / 2 , w e fin d t h a t s t a b l e m o tio n in t h e C o u lo m b
f i e l d o f a p o i n t n u c l e u s , a c c o r d i n g to t h e D i r a c th eo ry , w ill e x t e n d to Z cr = 137, w h e r e a s
in t h e K le i n - G o rd o n th e o r y i t w a s l i m i t e d b y Z cr = * 137 [ s e e ( 1 5 .3 3 )]. Such a n i n c r e a s e of
Z cr i s , a s w e h a v e a l r e a d y m e n t i o n e d , d u e to t h e s l i g h t c o m p e n s a t i o n of t h e r e l a t i v i s t i c
e f f e c t s by t h e s p i n e f f e c t s .
320 R E L A T IV IS T IC QUANTUM M E C H A N IC S

The diagram given in Fig. 20.1 shows that all terms are doubly
split, since to each value of I there correspond two values of /;
for example, instead of a single term 2p (/= 1 ) we now have two
terms 2pn, and 2p.,,.,. The exceptions
■E /h =■0 are the s terms (1 = 0), for which j
can have only one value (j = '/.2). Thus
the relativistic and spin effects som e­
--------------------3ds/2 what reduce but do not split the s terms
3 /23j>3/2,3d (see Fig. 20.1).
3st/2, 3p,/2
The degree of degeneracy also
changes owing to splitting of the energy
----------------------------------3-P 3/2 _ levels. We know that the principal
3s t/2, 3Pt/2 quantum number may take the following
values: n = l , 2, 3, 4 ,___ The orbital
*St/2 angular momentum quantum number I
varies from 1= 0 (s state) to I = n—1.
F i g. 20. 1. Energy level dia­ The total angular momentum quantum
gram of t h e h y d r o g e n atom. number / takes the values j — lzh '/.2(I ^ 0)
and j — l/i (l = 0) and, finally, the mag­
netic quantum number m;-= — / ....... that is, for a given / there
are 2 /+ 1 half-integral values of tiij. The degree of degeneracy,
which is characteristic for any central field of force and is related
to the equivalence of the various directions in space, is therefore
equal to 2y'-f-1 for particles with a spin of 1/2 (we remember that
for spinless particles it was equal to 2/-f- 1). In contrast to the rela­
tivistic spinless theory, the degeneracy with respect to I is still
present when we take into account terms of the order of a2 and
even the following expansion terms proportional to a4. When the
finite size of the nucleus is taken into account, the degeneracy with
respect to / is removed. We note incidentally that even greater
splitting with respect to I is due to vacuum fluctuations (see
Chapter 22). The magnitude of the splitting of spectral lines can
be determined from the selection rules (20.4). For the Lyman
series we then have two lines (instead of a single one):

0)<1) = ( 1S i/„ ) — (npy/„),

(weak line, since A/=0) (20.19)


(,/iJ = ( I S , / . ) — (« /> :l/2 ).

The Balmer series lines are split as follows:


o/'» = (2si/a) :h
(U,4>= (2.s’i , ) — ( HP
u,f:" = (2 P ^ , ) --- ( , ( S |/L
(o'11 = (2/>«2) --- ( , l S ‘. ■
w,:|1 - ( 2 / > , ..) — (/id. ( 20 . 20 )
J.
THE FI NE STR U CTU RE OF S P E C T R A 321

and, finally, the transition 2/)w., -*■m / i s forbidden, because in this


case A/'~ 2. If the degeneracy with respect to / is not removed,
the lines o>(l) andiou) coincide, since the initial and final levels have
the same values of the principal quantum number n and the total
angular momentum quantum number /. In a sim ilar manner we
may determine the splitting of all other lines. The lowest split
energy level corresponds to n = 2. Let us consider in greater
detail the splitting of this level in the case of the hydrogen atom
(Z = 1), which is the one most carefully investigated experi­
mentally. In general, the n = 2 level would be split into three sub-
levels, and, according to our theory, two of these sublevels would
combine:

(2sVa) = (2pVa) = | [ l + j ( 2 -

(2pa/a) = ^ [ l + a- ( l
( 20 . 21 )

The transition frequency between these levels is, according to the


Dirac theory,

Ao)d = (2/j./a) — (2/>,/,) = /? jg , (20.22)

which is about 1.095 • 104 Me.6 If only the relativistic effects are
taken into account (Klein-Gordon equation) the corresponding
splitting is [see (15.32)]

AcoK-G = (2 s ) — (2 p ) = = 4 l6 - ; (20.23)

that is, the frequency is almost three times greater than the one
found from the Dirac theory. Consequently, the spin properties
of particles somewhat reduce the influence of the relativistic
effects.
The conclusions of Dirac’s theory have been accurately con­
firmed by experiment.
It is interesting to note that the fine structure of the spectrum
of the hydrogen atom was first theoretically calculated by Sommer-
feld who applied a relativistic Hamiltonian to the steady states
of the Bohr classical theory. Sommerfeld obtained [see (2.61)]

6 1 Me = 1 0 6 s e c t h a t i s , t h e a n g u l a r f r e q u e n c y a), e x p r e s s e d in s e c is connected
w ith t h e f r e q u e n c y v, e x p r e s s e d in Me, by t h e r e l a t i o n

0) = 277 • 10 V
322 R E L A T IV IS T IC QUANTUM M E C H A N IC S

the following expression for the relativistic theory (20.22) without


taking into account the spin effects:

Ao)Somm,= (2s) — (2p) = ^ r . (20.24)

Agreement of the Sommerfeld result with the conclusion of Dirac’s


theory was, however, only accidental. Sommerfeld’s theory did
not take into account the spin effects, and therefore it was unable
to predict the splitting of the n = 2 level into three sublevels, the
presence of which was later confirmed experimentally.

D. EXPERIMENTAL VERIFICATION OF THE FINE


STRUCTURE THEORY

The major accomplishment of Dirac’s theory was its treatment


of the fine structure. The theory was in good agreement with the
experimental facts and was able to explain this structure as a
manifestation of the relativistic and spin effects caused by the
motion of the electrons within the atom. However, further and
more detailed studies showed divergencies between the theory and
fact. Thus, special attention was given to the 2si/2and 2pi/a levels
which, according to the Dirac theory [see (20.21)], should coincide
in a hydrogen atom. Among spectroscopists, doubts about the
validity of this conclusion were expressed as early as 1934. How­
ever, the techniques of the time did not allow greater experimental
accuracy, and the discrepancy between the theory and optical
observations (that is, the splitting of the levels) b ein g sm a ll.n o
great attention was paid to it. Better experimental data on this
splitting were obtained considerably later, when microwave spectro­
scopic techniques were used.
The microwave spectroscopic method was invented and rapidly
developed in the postwar years as a result of technical progress in
microwave engineering.7 Microwave spectroscopy, which has now
developed into a special branch of physics, gives valuable results
when used in the investigation of nuclei, atoms and m olecules.
Microwave spectroscopic methods are also applied to the physics
of solids and liquids. In 1947 Lamb and Rutherford employed this
method to studies of the 2si/u and 2pi/a levels, making use of
a special property characteristic of the 2si/a state. This state is

By m i c r o w a v e u l t r a h i g h - f r e q u e n c y r a d i o e m i s s i o n we m e a n t h e r e g i o n o f t h e e l e c t r o ­
m a g n e t i c s p e c t r u m l o c a t e d in t h e w a v e l e n g t h r a n g e from m i l l i m e t e r s to t e n s o f c e n t i m e t e r s
( 1 0 6 — 1 0 3 Me). S u c c e s s f u l a p p l i c a t i o n o f m i c r o w a v e s p e c t r o s c o p y to t h e i n v e s t i g a t i o n of
a t o m i c s p e c t r a i s d u e to t h e f a c t t h a t t h e d i s t a n c e s b e t w e e n t h e c o m p o n e n t s o f t h e l e v e l s
s p l i t by t h e r e l a t i v i s t i c , s p i n a n d v a c u u m e f f e c t s are o f t h e s a m e o r d e r o f m a g n i t u d e a s th e
w a v e l e n g t h s in t h e m i c r o w a v e re g io n .
THE FINE STRUCTURE OF SP E C T R A 323

metastable, since a dipole transition from the state to the


lower lsi/2 state is forbidden by the selection rules A/ = 0 [see
Eq. (20.4)].8
Transition from the metastable state may be associated either
with the em ission of two photons (the probability of such a transi­
tion is 108 lower than that for the allowed transition), or with
a preliminary transition to the 2p level. Lamb and Rutherford
investigated the latter type.
Let us describe the general features of their experiment
(see Fig. 20.2). A beam of hydrogen atoms in the unexcited lsi/„
state is obtained as a result of dissociation of molecular hydrogen
at high temperatures (tungsten furnace). A bombarding beam
of electrons then excites some fraction of the atoms in the hot
beam (approximately one out of 10s) to the metastable state 2si/a.
The metastable atoms, unlike the unexcited atoms, readily give
up their energy of excitation upon striking a metallic target. In
so doing they remove electrons from the metal. The resulting
current is measured by a sensitive galvanometer.

F i g . 2 0 .2 . D iag ram o f t h e L a m b - R u th e r f o r d e x ­
p e r i m e n t s on t h e d e t e c t i o n o f t h e s p l i t t i n g o f
t h e 2si/, a n d 2pi/, l e v e l s : 1) t u n g s t e n f u r n a c e
e m i t t i n g a b e a m o f h y d r o g e n a t o m s ; 2 ) b e a m of
e l e c t r o n s e x c i t i n g t h e h y d r o g e n a to m s; 3 ) r a d i o
f r e q u e n c y f ie ld ; 4 ) t a r g e t ; 5) g a lv a n o m e t e r .

If the beam of metastable atoms is subjected in transit to a


perturbation capable of causing a 2s-*2p transition, then the atoms
will almost instantaneously pass to the lsi/2 state (prior to reaching
the target). As a result, the current reading on the galvanometer
is lower.
In the Lamb-Rutherford experiment such transitions were
induced by microwave radiation (the probability of the corre­
sponding spontaneous transition, proportional to w4, is vanishingly
small as a consequence of the smallness of «>); a strong damping

T h i s i s c o r r e c t for a d i p o l e t r a n s i t i o n , b u t c a l c u l a t i o n s h o w s t h a t t h e q u a d r u p o l e
t r a n s i t i o n b e t w e e n t h e s e s t a t e s i s a l s o f o rb id d en .
324 R E L A T IV IS T IC QUANTUM M E C H A N IC S

action, resulting in a decrease of target current, was observed at


some frequency to. This oj was assumed to be the resonance
frequency which causes transitions 2si/a-* 2pi/3 or 2si/o -»■2p.\h with
a subsequent practically instantaneous transition to the lsi/s level;
the energy difference between these corresponds to fiw. Thus,
one can very precisely measure the relative positions of the
levels

2si/„2/;i/2 and 2p3/l9.

These measurements showed that the level 2si,., is shifted


upwards relative to the level 2/n,., by approximately one tenth of
the distance between the doublet levels 2pya —2pi/a, which is equal
to R. The arrangement of the levels of a hydrogen-like atom
(n = 2) derived from the Lamb-Rutherford experiment is given
in Fig. 20.3. The disposition of these levels according to the
Dirac theory is given for comparison. According to the latest
data, the shift of the 2si/a level is approximately 1057.77 Me
or, in wavelengths, ~ 28 cm.

' 2 P3/2 2p3/2

1
•2s,>/2
1058
2 P t/? 2S//J, 2p,/2
b

1'ig. 20..'i. S p l i t t i n g o f e n e r g y l e v e l s in t h e h y d r o g e n
atom, a) e x p e r i m e n t a l d a t a ; b) a c c o r d i n g to t h e D i r a c
theory ( n e g le c tin g vacuum effe cts). T h e fre q u e n c ie s
of th e c o r r e s p o n d i n g t r a n s i t i o n s a n d t h e d i s t a n c e s
a r e g i v e n in Me.

This apparently negligible discrepancy between theory and ex­


periment led to remarkable progress in theoretical physics and,
in particular, in quantum electrodynamics. This subject will be
considered in greater detail in Chapter 22.

In t h e e x p e r i m e n t s of L am b a n d R u t h e r f o r d , t h e f re q u e n c y of t h e m i c r o w a v e r a d i a t i o n
w a s f i x e d a n d t h e r e s o n a n c e c o n d i t i o n , c o r r e s p o n d i n g to t h e d i f f e r e n c e in t h e Z e e m a n co m ­
p o n e n t s b e t w e e n th e s t a t e s 2>si/, a n d 2/d/, or 2p y , w a s o b t a i n e d by a d j u s t i n g t h e m a g n e t i c
f i e l d ■Z /C . T h e n , e x t r a p o l a t i n g t h e r e s u l t s to th e c a s e 'O iT -- 0, t h e a u t h o r s f o u n d t h e l e v e l
sh ift.
THE F I NE STR U CTU RE OF SP E C T R A 325

E. ANOMALOUS ZEEMAN EFFECT

The complete theory of the Zeeman effect (both normal and


anomalous) must be based on the Dirac theory, because the latter
takes into account both the relativistic and the spin corrections.
Since the anomalous Zeeman effect is due to the spin effects in
the atom, neither classical theory nor Schrodinger's wave me­
chanics was able to give a satisfactory explanation of the Zee-
man effect, and for obvious reasons.
As a starting point of the theory let us take the approximate
Dirac equation (19.14), in which these effects are taken into account
up to the terms ( y ) . Let the magnetic field be directed along the
z axis, that is, Hx —-Hy = 0, Hz = <S/f'. Then, using the fact that,
according to (16.16),

Jo
2m0 2w0 lll0C 2hi. d-i (20.25)

we reduce Eq. (19.14), describing the motion of an electron in the


Coulomb field of a nucleus, to the form

[e + ■ ^ 7)(j-i)= (vrr*i + v' s-o* + i'cont+ i'mag) ( y . (2°-26)

where Frel, F s>0* and l/cont are given by Eqs. (19.15), (19.23) and
(19.24), respectively. Upon averaging of these terms

AEn/ = ^ (T f T f )( F r e l-l-F s .o ._ |_ l/c o n t )f J j .‘jrf'JC (20.27)

we obtain the fine structure formula (20.16), that is,10

H , - _ » i £ ( y £ * .- f ) . (20.28)

When a magnetic field is present, we obtain on the right-hand


side of Eq. (20.26) the interaction

pmag — ^- + 0 3 ], (20.29)

^ G e n e r a l l y s p e a k i n g , t h e s p i n - o r b i t i n t e r a c t i o n V s ’0, i s o f f u n d a m e n t a l i m p o r t a n c e in
th is c a s e . Since, how ever, the r e la tiv is tic term s are of th e sa m e order a s the spin-orbit
interaction, we may se t
s . 0 . •v AE
AE nj ■
326 R E L A T IV IST IC QUANTUM M E C H A N IC S

which gives the following value for the additional energy of the atom:

A£ mag = J (T*T*) ( - i + oi) ( d'x. (20.30)

The appearance of either anomalous (case of a weak magnetic field)


or normal (case of a strong magnetic field) Zeeman effects depends
on the relative proportion between the additional energies on
the right-hand side of (20.26).
Let us assume that we have a comparatively weak magnetic
field, whose interaction with the atomic electrons is sm aller than
the relativistic or spin-orbit interaction.
Then the zero-order approximation will be expressed by the
wave functions (20.2) that are obtained when the spin-orbit coupling
is retained.
Substituting these functions into (20.30), the additional energy
becomes

A£mag= 1,oa5r J | Rnl |V dr j dQ (¥'£)+(- i £ + « ;) Y\». (20.31)


6
In (20.31) we should note that the integral over r is equal to
unity

J !ff» /|V d /-= l. (20.32)


6
Substituting in place of the spherical spinors their values from
(18.24) and (18.25), and using the orthogonality condition for the
spherical harmonics

(Ff)*(y™)d2 = 1.

we find the following expression for the additional energy when


/ — ^~r 'id-

AZ^ag = [(/ m) m _|_ (/ _j_ i _ tH) (m - 1)] =

In exactly the same way when j = l — we obtain

A£mag = [(/ _ m + 1) m + (/ + m ) (m - 1)] =

= ij-o y f (m v 2 ) 2t + \•
THE FI NE STR UCTURE OF S P E C T R A 327

Recalling that m/ = in— '/* * the last two expressions may be written
as a single formula
A£mag_ ^yjTgrtij = ofignij, (20.33)

e0SJf is the Larmor frequency, and the Lande g factor is


where o 2 m 0c

t= 'M - <2°-34>

Thus, in the case of the anomalous Zeeman effect, the expression


for the additional energy contains the Lande g factor, which in the
case of the normal Zeeman effect [see (16.23)] is equal to unity..
The additional energy (20.33) does not lead to the usual triplet
splitting (normal Zeeman effect), but to a more complex splitting
pattern (anomalous Zeeman effect).
In view of the fact that m, can assume 2j-\-\ different values,
each level in the case of the anomalous Zeeman effect is split
into 2/-(-l separate sublevels; that is, the external magnetic field
completely removes the degeneracy, which is present even in the
relativistic theory of the hydrogen atom.

F i g . 2 0 .4 . Z e e m a n e f f e c t : a) p o s i t i o n o f t h e e n e r g y l e v e l s
in t h e a b s e n c e o f a f ield ; b) a n o m a l o u s Z e e m a n ef fe c t;
c ) n o rm al Z e e m a n e f f e c t .

To obtain the splitting pattern, it is necessary to take into


account the value of the Lande g factor (g = 2 for the s:/„ states,
g = 7a for the pi/2 states, g = i/3 for the p3/., states, and so on) and
also the selection rules for the magnetic quantum number trij. In
particular, when Amy = 0, the emitted components are polarized
parallel to the z axis (that is, parallel to the magnetic field), and
when Aitij = z h 1 the components are polarized perpendicular to the
magnetic field.
328 R E L A T IV IS T IC QUANTUM M E CH A N IC S

Equation (20.33) gives us the following value for the frequency


of the radiation:
(o= «o + o (g^n'l — gmj), (20.35)

where cd0 is the frequency of the radiation in the absence of a mag­


netic field (a?r = 0); g' and g are the Lande g factors of the initial
and final states; the magnetic quantum number my of the final state
may take three values: m;-= m], in'} z t 1.
Figure 20.4b shows the splitting of the spectral levels Fsi/2 and
2-pi,, in a weak magnetic field, the Larmor frequency being taken
as the unit of the splitting. From Fig. 20.4b it is seen that in this
case there are four, and not three (as in the normal Zeeman effect)
shifted lines. The magnitude of the displacement is given by
(20.35).
In ithe case of a weak field [according to (20.34)] we find

g° = 7s. g = 2.
Hence
. 2 . 4
All)! = CD, — C00= - j0 , A co.2= — 0,

Acd3= -^-
Oo, A<d4= — u o. (20.36)

Equation (20.34) for the Lande g factor is applicable to the hydrogen


atom and to atoms having a single valence electron. In the general
case, the Lande g factor becomes

g = 1 +-IV + ') —L(L+ 1) + S(S + l) (20.37)


“ ' 27(7+ 1) - v ;

where L, 5 , and J are the orbital, spin and total angular momenta
of the atoms and

J = \L±S\.

In particular, for elements of the first group (•/ = /, L = Z, s —


Eqs. (20.37) and (20.34) are identically the same. The Lande g
factor attains its maximum value for s states (1 = 0, /' = s = '/.2):

gs = 2. (20.38)

For atoms with two electrons in the outer shell (for example,
helium atoms), single lines (S = 0, J = L) are possible along with
the triplet state S = l . For the single lines we have g = 1, and,
therefore, in this case spin effects should be of no importance;
only the normal Zeeman effect (that is, triplet splitting) should be
observed in either a weak or strong field.
THE FINE STRUCTURE OF SPECTRA 329

F. STRONG MAGNETIC FIELDS. PASCHEN-BACK


EFFECT

It has been indicated that the anomalous Zeeman effect appears


in the case of weak fields, when the external magnetic field cannot
disrupt the spin-orbit coupling.
Mathematically this means that A£mae [see (20.33)] is much
smaller than the natural splitting of the lines A£s*°-given by Eq.
(20.28)
A£s-°-> A £ m a g . (20.39)

In the latter case we first solved the problem by taking into account
the spin-orbit interaction; this establishes a relation between
the spherical harmonics that form the spherical spinor (18.24) or
(18.25); then we found an additional energy that leads to the
anomalous Zeeman effect, since the Lande g factor does not equal
unity.
In the case of strong fields, when the splitting due to the external
magnetic field is greater than that due to the spin-orbit interaction

A£maS > A£s-0*, (20.39a)

the magnetic field “ breaks” the spin-orbit coupling and the zero-
order approximation solutions, expressed in terms of spherical
spinors [see (18.24 and 18.25)], are no longer true.
In this case we may neglect the interactions Frel, Fs'°* and
p c°m (20.26), which, when (20.29) is taken into account, becomes

(E + ¥ - - S r ) ® = !> .> * •(-< £ + » ;)® ■ <2 0 -4 0 >


Using the fact that the functions T, and 4\2 must be proportional to
the spherical harmonics Yf, with — i ^ Y f = mYf, we find two
independent equations for these wave functions:

Ze\
r ^ - - Na5 r (m+ l ) ) T 1= 0,
Zel (20.41)
£ -f

from which it is evident that the additional energy equals

A£^ag = N25T(m + 2mi), (20.42)


and
A(A£mi)mag = Na5T (Am+ 2Am,);
330 RELATIVISTIC QUANTUM MECHANICS

that i s , the wave function Vi corresponds to the case in which the


electron spin is directed along the magnetic field (mi = 1/j)» and the
wave function T.2 corresponds to the case in which the spin direc­
tion is opposite to that of the magnetic field.
If we choose tn such that the same energy value is obtained
for both functions, then we must set mx= m— 1 for the function T j,
and = 1- 1 for the function 'F.j.
In this case the wave functions

>Fi = RnlY ? ~ \ ¥ , = RnlY r + ' (20.43)

will be mutually orthogonal, so that


CO

J «7'Fo d3x = j IRnl I2r-dr j) d2 (Y” ~ ')* Y ” + 1= 0. (20.44)


o

Since the interaction between the atom and the external magnetic
field (20.29) contains only the matrix oj, which does not couple the
wave functions lF , and ' F j , transitions from the state with m s = ‘/ a to
the state with ms = — */2 > Educed by this interaction, will be for­
bidden in this case and hence Ams= 0.
Taking into account this circumstance, and also the selection
rules for the quantum number m (Am= 0, rkl), we find from (20.42)
an expression for the Zeeman splitting of the spectral lines

Aa) = oAm= 0, dio, (20.45)

which agrees with the result of Schrodinger’s theory, which explains


the normal Zeeman effect (triplet splitting of the spectral lines).
Thus, in strong fields (A£mas>>A£s*0>), the anomalous effect
becomes the normal effect, which is in agreement with experi­
mental data (Paschen-Back effect). It is interesting to note that
the passage from the anomalous Zeeman effect to the normal effect
can be illustrated by Fig. 20.4, if the Lande g factor is set equal to
unity (see case c). Then the splitting will be

Au)1= Aooa= 0, Au)3= o, and A(«4= — o;

that i s , we obtain three components of the split line instead of four.


In special cases, when a£ s‘°'<A £ma8 for one energy level and
conversely A£s-°-^> A£ma® for the other level, or when the energies
of both levels are of the same order of magnitude, the Zeeman
splitting becomes complex. Since these are all special cases, we
shall not elaborate them here.
THE FINE STRUCTURE OF SPECTRA 331

Problem 20.1. Investigate the diamagnetism and paramagnetism of atoms by placing


them In a constant homogeneous magnetic field (//v = H y =»0,//, = J2 D ; contrary to the
Investigation of the Zeeman effect, keep all terms containing gj^as well as terms pro­
portional to * In the Hamiltonian [see (20.25)].
Indicate the atoms In which diamagnetism may be observed.
Solution. When terms pordonal to a J f 2are taken into account, we have, instead of
(20.25)
pa
2«0 2nio d<f 1 8m0c2
-(A-*+y) cpf'.
Consequently, on the right-hand side of (20.26) we have another term
ydiam_ (Aa+ y ) e5r2,
8m0ca

which In conjunction with (20.33) gives the following expression for the additional energy
of an electron In a magnetic field:

A£mag = ^ ^ T g n i j + f 2■ (2(T' 46)

Here when calculating the perturbation energy proportional to x 2 + y 2, we have used the
spherical symmetry, which must occur in the zero-order approximation(g/f = 0),an d
have set

^ ¥+(*2+.y2)¥rfs.*:= |- ^ ¥+r2¥ d*x = — r 2•


d
Hence the magnetic moment of an atom in a magnetic field is
dA £mag elJT r2 nparam , diam
- - j y — Ngm/' 6m0:2 " - p (X (20.47)

The latter relationship is a generalization of a familiar equation relating A£maS


in the case where A£mag a nonlinear function of cj%".
The diamagnetism of atoms is characterized by the second term of Eq. (20.47), which
is proportional to oTT■
The magnetic susceptibility per gram-atom is
3 diam
ydiam _ ^ ___ -a
(20.48)
6m„c2
where N is Avogadro’s number. _
The quantity /diam i s never zero and must always be negative (r 2 y> 0). Therefore,
the diamagnetic effect must occur in all atoms.
As for the first term on the right-hand side of Eq. (20.47), which is proportional to
1 3
nt], it may take either positive or negative values, since nij = ± , + - , ..., ± j.
In a state of thermodynamic equilibrium, however, the negative values for m;, which
give a smaller value for the energy, will be preferred. On the average we therefore
obtain a positive value for the paramagnetic susceptibility 11
y P aram _ [ijjg2 j ( j + 1)
~ kT 3
Expression (20.47) is obtained for a weak magnetic field, when the anomalous Zeeman
effect occurs. It can, however, be easily extended to the case of a strong magnetic field.
To do this we must set g — I, nij = m In (20.47) [see (20.42)].

11S ee R. B e c k e r , Electron Theory.


332 R E L A T IV IS T IC QUANTUM M EC H A N ICS

Since the paramagnetic susceptibility Is considerably larger than the diamagnetic


susceptibility (/P aram '/ dlam)f the atoms exhibit paramagnetic properties when j * 0.
For hydrogen-like atoms j differs from zero (the minimum value of / equals */a) and,
therefore, they are always paramagnetic. Only for atoms with an even number of
electrons can the quantum number / vanish (for example, parahellum In the ground state,
see Chapter 24). Such an atom will be diamagnetic.

Problem 20.2. Adding geometrically the orbital and spin angular momenta, show that
the anomalous Zeeman effect Is associated with the fact that the total magnetic moment
p Is not parallel to the total angular
momentum J. With the aid of the geo­
metric model, also explain the Paschen-
Back effect.
Solution. First let us find geometri­
cally the angular and the magnetic mo­
ment vectors

J = L -(- S; (i = ■
2 ^ L+ 2S>-
In the geometric representation (see
Fig. 20.5) ‘we may choose the scale so
that u. = — „ g° (it is immaterial at
2m0c
the moment whether the vectors are
parallel or antiparallel).
Then
F i g . 2 0 .5 . G e o m e t r i c i n t e r p r e t a t i o n o f t h e
a n o m a l o u s Z e e m an e f f e c t . f-i = L, and pJ = 2S;

that is, the total magnetic moment ji will undergo two rotations in a magnetic field: one
with an angular velocity u around the direction of the total angular momentum (this
angular velocity corresponds to the frequency associated with the transition between
components of the spin-orbit splitting of the spectral lines <u / ? a ‘J), and the other corre­
sponding to the Larmor frequency of precession around the direction of the magnetic
field H (H X = H V= 0, II, = p5T).
When m ;j> o, the additional energy should be calculated from the relation

mag __(|t • J) (J • H)

Since the magnetic moment Is directed on the average along J, we have

A £ mag _ gohmj,

where a = ' , and the Lande g factor Is equal to


Zitifi c

g = -J cos (L -J) + 2 - J cos (S-J).

Substituting the values of the cosines of the angles

J2— I.2+ S2 S2+ La


cos (5 - J) cos (L • J) = ^
2JS 2JL
and remembering that .]2— h - J ( J (- I), and so on, we obtain the expression (20.37) for
the Lande g factor. Let us pay attention to the fact that if the vectors |i and J were
parallel, the Lande’ g factor would be unity.
THE FIN E STRU C TU R E OF S P E C T R A 333

In strong fields o « we must consider Independently the rotation of the orbital and
the spin moments |i around the 2 axis. Then the additional energy becomes

AC mag = _ (L, + 2SZ) = 0ft (m + 1),

which leads directly to the normal Zeeman splitting (Paschen-Back effect).

Problem 20.3. By means of the relativistic scalar wave equation and the Dirac equa­
tion, find the frequency of the allowed transitions between the n = 2 and n = 3 states.
Show that, according to the Dirac theory, there are seven lines, five of which are distinct,
and that, according to the scalar theory, there are only three distinct lines.
Chap ter 21

The Effect of Nuclear Structure


on Atomic Spectra
A. INTRODUCTORY REMARKS

As has been mentioned in Chapter 13, the position of the spectral lines is shifted when
the finiteness of the nuclear m ass is taken into account. The Rydberg constant R in the
expression for the energy of a hydrogen-like atom

R nz«
( 21. 1)
n~ n?
is somewhat reduced and becomes equal to

( 21. 2 )

where

D
*°° 2h3
is the Rydberg constant corresponding to infinite nuclear m ass. Consequently, the
Rydberg constant will have somewhat different values for hydrogen, deuterium, and
tritium. With the great accuracy attainable in modern spectroscopic techniques, this
effect can be used to detect the presence of different isotopes (see Chapter 13).
In a sim ilar fashion, the finite size and the magnetic moment of the atomic nucleus
have certain effects on atomic spectra.

B. EFFECT OF THE FINITE SIZE OF THE NUCLEUS


When the motion of an electron in the field of a nucleus is treated as a problem in
classical theory, it is quite immaterial whether the nucleus is regarded as a point or as
a particle with finite dimensions. All that matters is that the electron should at all
times be outside the nucleus and that the nuclear charge should be spherically sym­
metric, since the potential outside a spherically symmetric charge distribution is the
same as the potential of a point charge.
In quantum mechanics the situation is somewhat different. The wave function must
differ from zero inside the nucleus; therefore, there is a certain probability (however
small) that the electron will be located inside the nucleus. Consequently, the charge
distribution inside the nucleus must in some manner influence the energy levels of the
electrons in the atom.
To estimate the effect of the finite nuclear size on the energy spectrum of a hydrogen­
like atom, we shall assume that the nucleus can be represented by a sphere of radius
R n with charge distributed uniformly throughout the volume. The potential energy will
be given by (see Fig. 2 LI)

Zel ,'3_ _ J_ r 2
for r < R A'. (2U)
X'A* 2 **A.
T H E E F F E C T O F N U C L E A R S T R U C T U R E ON A T O M I C SPECTRA 335

V= — ^ for r > Rk. (21. 4)

The shift of energy levels due to the finite size of the nucleus can be calculated with
the help of perturbation theory. We shall assume that the perturbation energy consists
of the difference between the potential energy of a point nucleus and the potential energy
of a nucleus with charge uniformly distributed over the nuclear volume

(2 1.5 )
hr
*5 *(r).

with
1 for r < R V)
e (21.6)
0 for r Rn.
The perturbing force therefore differs from zero only inside the nucleus,

th e v a ria tio n o f th e p o te n tia l energy


w h ich w o u ld b e o b ta in e d i n s id e th e
n u c le u s , if the p o te n tia l w ere d e ­
s c r ib e d by th e C oulom b law both in s id e
and o u ts id e the nu c le u s.

In first-order perturbation theory the shift of the levels is given by


AEvol = ^ <j>*vol (21.7)

Since | ^ |2 does not change appreciably in the region r ^ R t this Integral can be readily
evaluated by substituting for | <{/12 Its value at the origin.

Rn
i£V<i ', [7 -jfe(T- 7 &)] dr--
2n ( 21. 8 )
j Z W ' K O ) I*.
336 R E L A T IV IS T IC QUANTUM M EC H A N ICS

Substituting the expression (20.14) for 11 (0) |a , we obtain

AE M l - (21.9)
a\n* ’

that is, in nonrelativistic theory, the shift of the energy levels is different from zero only
for s states (/ = 0;.
( R A*
—- I
n* °n
(where an — a ^ -p ) , and for d states a factor of the order of j , and so on. *
1 Con­
sequently, the shift in the energy levels for p and d states can be neglected in the first
approximation.
For hydrogen, the first-order shift In the energy levels Is about 1 Me; this Is much
too small to account for the Lamb shift, which Is equal to approximately 1,057 Me (see
Chapter 20),
The volume of the nucleus Is important In connection with the Isotope shift, that Is,
the shift In the energy levels of atoms with the same atomic number Z and different
m ass numbers A . The chief factors that give rise to the Isotope shift are the different
m asses (the mass effect) and different volumes (the volume effect) of the Isotopes. The
m ass effect Is manifested In a shift of the spectrum lines towards the ultraviolet as the
m ass number A Increases. For example, for Z = 1 the highest frequencies are found
in tritium with A = 3, then deuterium with A = 2', and, finally, ordinary hydrogen with
A — I [see (21.2) and also Chapter 13]. On the other hand, the volume eftect is mani­
fested In a shift of the spectrum lines towards the Infrared as A Increases. For Instance,
it can be seen from Eqs. (21.1), (21.9) and (21.13) [see below], that the energy levels of a
hydrogen-like atom will be given by the following expression when the shift due to the
volume effect is taken into account:
RZAh 1 2lR
5 a\n

Experiment shows that an Isotope shift towards ultraviolet Is observed In elements whose
atomic number Z Is le ss than 40-50. For elements with a larger value of Z an isotope
shift in the opposite direction Is observed, that Is, towards the Infrared. This Indicates
that, for relatively light elements, the Isotope shift is caused mainly by the mass effect,
whereas for heavier elements It Is caused by the volume effect. This, however. Is a
rather simplified picture of the isotope effect, and other features associated with the
structures of the atom (for example, nuclear spin and polarization of nuclei by electrons)
also have to be taken Into account.

C. MESIC ATOMS

The finite size of the nucleus has a particularly Important effect on the position of
energy levels in a m esic atom—an atomic system consisting of a fi meson revolving about
a nucleus. The }i meson is a particle that has the same spin as the electron (that is, spin

T o c a l c u l a t e t h e s h i f t o f t h e p l e v e l s w e m u s t s u b s t i t u t e t h e s e c o n d le r m in t h e e x -
p a r s i o n o f k l( r ) i n t o ( 2 1 . 7 ) , n a m e l y ,

10(012 r2I <h{drj I


I <)r I' - 0 ’

s i n c e t h e m a i n te r m | i / / ( 0 ) |2 v a n i s h e s fo r p s t a t e s .
T H E E F F E C T O F N U C L E A R S T R U C T U R E ON A T O M I C S P E C T R A 337

1/2 in units of /l) and a mass 207 times greater ( m |1 = 2 0 7 m „ ) , so that the p meson Is
basically a "heavy” electron. Meslc atoms can be produced by passing negative p mesons
through matter. After losing Its energy and slowing down, a p meson may be captured
In an orbit about a nucleus, forming In this way a ji-rnesic atom. Meslc atoms have
been obtained for almost all elements of the periodic system, from hydrogen up to
the heavy elements (uranium, neptunium, and so on).2
The motion of a p meson about the nucleus Is determined mainly by the Coulomb
attraction. Just like the motion of an electron. A p meson, however, also has nonelectro-
magnetic Interactions with the electron—neutrino and nuclear fields; these Interactions
may result In spontaneous decay of the p meson into an electron, neutrino, and antineutrino
(the lifetime of a p meson at rest is t = 2.2 x 10~6 sec). The p meson has, in addition,
a definite probability of being captured by a nucleus. Thus the lifetime of a p-m eslc
atom Is determined by two competing processes: natural decay of a p meson Into an
electron, neutrino, and antineutrino and nuclear capture of the p meson. In light meslc
atoms ( Z < 10 ) the probability of the first process Is greater than the probability of
the second; that is , the lifetime of a meslc atom Is determined by the lifetime of a
p meson at rest ( t ^ l O -0 sec). For Z >■ 10, nuclear capture begins to predominate
and the lifetime slowly decreases to ~ 7 -10“9 sec (for Z = 82).3
In the theory of p -m eslc atoms, ordinary electrostatic Interaction plays a funda­
mental role. In the first approximation, we can regard the nucleus as a point charge
and calculate the energy of the meslc atom and the radius of the orbit using the equations
derived for an ordinary hydrogen-like atom, replacing the electronic mass by the mass
of a p meson. Then the energy and radius of the orbit will be given by [see (13.33),
(13.45)]

_ ft2 na
( 21. 10)
' r"—m~el Z ’

where w|1= 207w0. It can be seen that the energy of the p meson In a m eslc atom Is
207 times greater than the corresponding energy of an electron In the atom, and that the
radius of the orbit on the contrary is reduced by the same factor. If electrons remain
In the atom along with the p mesons, they will move about the nucleus In considerably
larger orbits than the p meson and therefore cannot exert a significant Influence on the
p meson rotating around the nucleus. A meslc atom, therefore, may be regarded as a
hydrogen-like atom that can have both large and small values of Z.
Since the radius of the "Bohr” orbit of the meson Is 207 times smaller than that of
the electron orbit, the probability that the meson will be located In the nucleus is con­
siderably greater than for an electron in a hydrogen-like atom. The main correction to
the energy levels of a meslc atom will therefore come from the volume effect. The
equation for the energy of s states can be obtained from (21.9) and (21.10) by replacing
the Bohr radius aa by the corresponding radius In a m eslc atom
7l2
a
f- (21. 11)

obtaining, therefore.

F _ m ^7Jel /. 4 Z*R*N
( 21. 12)
— 2 h 2n 2 \ 5n o 2

2 T h e s e s u b j e c t s a r e t r e a t e d i n m o r e d e t a i l in a p a p e r b y D. D. I v a n e n k o a n d G. E. P u s -
t o v a l o v a , U s p e k h i f i z i c h e s k i k h n a u k , 6 1 , 27 (1 9 5 7 ) .

^ F o r 7 t m e s o n s , w h i c h s t r o n g l y i n t e r a c t w i t h n u c l e i a n d a r e r e s p o n s i b l e fo r n u c l e a r
f o r c e s , t h e l i f e t i m e w i t h r e s p e c t to d e c a y i n t o a y m e s o n a n d n e u t r i n o i s e q u a l to 2 . 6 x
i c r 8 s e c , w h e r e a s t h e l i f e t i m e w i t h r e s p e c t to c a p t u r e b y a n u c l e u s i s m a n y t i m e s s m a l l e r .
In p a r t i c u l a r , f o r 7 7 - m e s i c h y d r o g e n t h e c a p t u r e t i m e o f a n e g a t i v e p i o n fr om t h e o r b i t i s o f
th e o rd e r o f 1 0 sec.
338 R E L A T IV IS T IC QUANTUM M EC H A N IC S

Using the fact that the nuclear radius Is related to the m ass number A ~ 2Z by the
expression

R N = R„Ai,>«=7?„ (2Z ) ' \ (21.13)

where Rt>= const, we obtain In the first approximation

It follows that, for the s levels, the energy correction due to the nuclear volume will
be proportional to 2 */a • and therefore attains very large values for heavy elements.
In heavy elements, the orbit of a p meson may even be inside the nucleus, at large
Z and small n. The Bohr radius of the m esic atom becomes equal to the nuclear radius
for Z = Z cr = 45
For orbits inside the nucleus (Z > Z c r ) , the main part of the potential will no longer
be determined by the Coulomb law (21.4) but instead by formula (21.3) which corresponds
to the potential of a three-dimensional harmonic oscillator (on the assumption of a
simplified, model of the nucleus in which the charge is uniformly distributed over the
volume). Thus the energy has to be determined from the following equation Instead of
(13.4):
d2R . 2 dR . 2m^
dr 2 ^ r dr + ft2' X
1 Zelr 2 /( / 4- I)ft3 (21.15)
x ( £ + 2 TiV 2 R\N 2m^r- I = 0.

In Eq. (21.15) let us change the variable by setting R = Y rR', r = V P- Since

vS/? = 4p ^ f ^ ' + l W 3_ \
P I dp2 ^ p dp ^ 16p2 H )

it follows that the equation for the energy of a three-dimensional harmonic oscillator is
formally identical with the equation for the hydrogen-like atom

d 2R ‘ , 2 dR 1 (21.16)
dp2 p dp + H + T - f ) s '= ° ’
but has different values of the constants, namely,

WRV D= i k ( E ' 3Zel

c = j ( '! + i - { ) = n r + i), (21.17)

-i('-T)-
To determine the eigenvalues of the energy we may use Eq. (13.20), according to which

= e + k4
V a

Substituting for /?, A and I' their values from (21.17) and using the fact that k = n I 1,
we can find the energy of the meson in an orbit inside the nucleus:
o+ ft“ (2/j f / + 3i), (21.18)
T H E E F F E C T O F N U C L E A R S T R U C T U R E ON A T O M I C S P E C T R A 339

where die frequency of mechanical vibrations of the three-dimensional oscillator Is

The quantity represents the zero-point energy of the three-dimensional oscillator;


It Is three times greater than the corresponding zero-point energy of a one-dlmenslonal
3 Ze 3
oscillator. The quantity K0= 7t - Is the greatest depth of the oscillator potential well.
1 “n
Equation (21.18) Is correct on the assumption that the potential energy varies In ac­
cordance with (21.3) from zero to Infinity. If the finite size of the nucleus Is taken Into
account, an additional energy is obtained that represents the difference between the
potential energy of the particle In the oscillator well and Its potential energy In the
Coulomb field (this difference being averaged over the space outside the nucleus). The
equation for the additional energy Is
uu
I3 r-
AE vol = Ze\ (j) dQ ^ *1* r-dr.
Kn
RN ( 2 '27?l (21.19)

Since the wave function of a spherical oscillator Is sim ilar to the wave function for a
Coulomb field (It Is determined from the same wave equation with r replaced by p = r-),
It decreases exponentially as r Increases. The
correction (21.19) is therefore significant only . — 2S t/ 2
when the radius r„ of the meslc atoms Is close
to R s
As an example, let us consider the 2/? —* Is 2P3/2
transition in lead {Z = 82). If the m eslc atom 2 P,/2
of lead Is assumed to have a point nucleus, the
energy released In this transition can be found tS ,/s
from (21. 10)
F i g . 2 1 .2 . E n e r g y l e v e l s in a m e s i c
3 m Z-p 1
E, — El = = 14 Mev- (21.20) atom.

The relativistic and spin effects in the 2p , /a — Is1/o transition increase this energy by
the amount

A (A E )^ 2 Mev. (21.21)

Incidentally, such a significant role of the relativistic and spin effects Is due to the fact
that the energy Is expanded in terms of(Za)s. This quantity Is comparatively large for
lead (Z = 82, a = 1/137). Comparison with experiment shows, however, that In this
transition an energy of 6 Mev Is liberated Instead of the predicted 16 Mev. This dis­
crepancy between theory and experiment arises because the Is state for lead lies Inside
the nucleus. In the Ip state the orbit lies outside the nucleus and the volume effect of
the nucleus Is small. If we take the energy of the Is level from (21.18), and the energy
of the 2p level from (21. 10) for a point nucleus, the energy of the 2p — Is transition
will be 3.6 Mev. If we add to this the correction (21.19) for the energy of the Is level,
the energy of the transition will come to about 5 Mev, which Is relatively close to the
experimental value.
A study of the multlplet structure of the 2p level In meslc atoms enables us to
determine the spin of a p meson. For a particle with Integral spin, the level splits
Into an odd number of components (for spin 0 no splitting occurs, for spin 1 three lines
are observed, etc.). Since the 2p level splits Into two components (2pi/a and 2/73 ., ), It
was established that the spin of a p meson is 1/2. On the assumption of a point nucleus,
the theoretical splitting calculated from Eq. (20.17) should amount to about 0.55 Mev,
with the 2pi, and 2si/a levels coinciding. When the finite size of the nucleus Is taken
Into account aln a meslc lead atom, the splitting of the 2/73/2— 2pi/a levels Is reduced
to 0.2 Mev and the 2sJ/2 level Is raised above the 2p ,/o level. This is Illustrated In
340 R E L A T IV IS T IC QUANTUM M EC H A N IC S

Fig. 21.2. From the above data It follows that heavy m eslc atoms will emit gamma
quanta having energies of several Mev. Lighter meslc atoms emit x-rays.
Because of the significant influence of the size of the nucleus on the spectra of heavy
m eslc atoms, the charge distribution Inside the nucleus can be determined from an
analysis of the spectra. It has been found that the value that should be substituted for
Ro in the formula for the electromagnetic radius of a nucleus with mass number /I:
R n = R (,A'/‘ (21. 22)
is L 2x 10“ 13 cm rather than 1.4 x 10-13 cm (the value assumed for nuclear Inter­
actions).
Similarly, the multiplet structure of the spectral lines of m eslc atoms can be used to
determine the magnetic moment of the p meson; Its value is close to the muon magneton
ge
(21.23)
2m^c
where the Lande g factor Is £ = 2.
The theory of ji -m eslc atoms Is based mainly On electromagnetic Interactions. By
contrast. In the theory of v -m eslc atoms, a great part Is played by the nuclear Inter­
actions, the theory of which Is far from complete. Further experimental study of
7r-meslc* atoms and an explanation of the semlempirlcal laws that describe their be­
havior will have important bearings on future work in the theory of nuclear forces.
These topics, however, lie beyond the scope of this textbook.

D. APPLICATION OF THE DIRAC EQUATION


TO THE NEUTRON AND THE PROTON

The Dirac equation describes the motion of particles with spin 1/2. It applies to
electrons as well as to protons and neutrons. In the presence of an electromagnetic field
it is necessary to take into account the charge of the proton, as well as the so-called
anomalous magnetic moments of the proton and the neutron. We recall that the energy
of interaction between a charged Dirac particle and an electromagnetic field is
Ve = e<b— e a . A . (21.24)
In the nonrelatlvistlc approximation this expression contains a magnetic moment due to
the intrinsic (spin) angular momentum ( h / 2 a)
eh
V-e = o. (21.25)
2mac
This quantity Is known as the kinematic or Dirac magnetic moment. In passing to the
relativistic equation, we must replace the mass w„in Eq. (21.25) by its relativistic value
mo
~ y —-y--, and, therefore, the Dirac magnetic moment vanishes as the velocity approaches
V *—
the velocity of light (v ~ c).
In addition to the Dirac magnetic moment, which appears only In the nonrelatlvistlc
approximation and which depends on the charge, a particle may have an anomalous
magnetic moment that does not vanish even In the relativistic case and is Independent
of the particle’s charge.
We shall now find the energy of interaction due to the anomalous magnetic moment.
The energy of Interaction (21.24) of an electron with an electromagnetic field Is a
scalar quantity, since In four-dimensional space i4>= At, Ax = A t, A y = A«, A: = AS.
In the same way the unit matrix I Is the fourth component of the velocity matrix a |i
(that Is, a, = (T).4 The Interaction energy (21.24) may, therefore, be represented as a
scalar quantity In four-dlmenslonai notation

Ve = —e ^ V 'V (2L26)
ii=i
^ M o re p r e c i s e l y , t h e q u a n t i t y j ^ < > v.' b + a. ^b | Se e ( 1 7 . 3 2 ) 1 , w h e r e a a l , 2 ,3 , a 4 w ill
transform a s a four-vector.
THE EFFECT OF NUCLEAR STRUCTURE ON A T O M IC SPECTRA 341

'Ihe electromagnetic field forms an antisymmetric tensor of second rank


// d/L- __ 0-/ V
— (21.27)
^ Ox,± Oxv ’
where
x t = lct.
It follows that
HV 11bit
(21.28)
i h x = I I 4it ifcy = II\2i ib.z = I I \ J.

The Interaction energy of the anomalous magnetic moment with the electromagnetic field
Is, therefore, given by
•i
= n y V 'V - (21.29)

where «fl Is a second rank tensor composed of the Dirac matrices .5


Using the rules for transformation of a wave function under the Lorentz [see (17.38)]
and spatial rotations [see (17.39)], we can show that the quantities
a 28 = P 3 °li 031= P8a 2, a 12 = p3^3> °41— <p:l'li

a 12 = — i p 2a 2, a ]3 = — [ P 2 33
(£•1*«3U)
are the matrix elements forming a second rank tensor. The energy of Interaction between
the anomalous magnetic moment and the electromagnetic field takes the form
l/m = F[P3='-// + P2i- £ l. (21.31)
An electron has a charge, a spin, and also a Dirac magnetic moment. Its anomalous
magnetic moment Is relatively small (see below). A neutron has no charge, but it does
have an anomalous magnetic moment; this magnetic moment determines the interaction
between the neutron and the electromagnetic field. As for the proton, it has both a
charge and a spin, and hence a Dirac magnetic moment; in addition, it has an anomalous
magnetic moment. It should be noted that nuclear interactions are of great importance
in the theory of nucleons.

E. EXPERIMENTAL DETERMINATION OF THE MAGNETIC


MOMENT OF THE NEUTRON AND THE PROTON

The procedure for determining the magnetic moment of the neutron, proton, and
complex nuclei is basically the same as for the magnetic moment of the electron (the
Stern-Gerlach experiment). The basic principle consists in applying a magnetic field
perpendicular to the direction of motion of the particle. The particle will react dif­
ferently depending on whether Its magnetic moment is oriented parallel or antiparallel
to the field.
Let us first consider the possibility of determining the Dirac magnetic moment
and the anomalous magnetic moment of a free particle. Suppose a free particle moves
perpendicularly to the z axis. The Hamiltonian describing its motion has the form

H= c p i^ P jc + c p ,a vPy + psm0c2. (2 1.32)

The component of the intrinsic angular momentum perpendicular to the direction of this
motion

(21,33)
S‘ = 2 n3

M ore p r e c i s e l y , t h e q u a n t i t y i s a s e c o n d rank ten so r.


342 R E L A T IV IS T IC QUANTUM M EC H A N ICS

does not commute with this Hamiltonian. The component of the total angular momentum
along the z axis does commute with the Hamiltonian

J z = y V x — x V 3/ + \ f a z , (21.34)
and can therefore be determined exactly together with the energy.
Let us evaluate the error in the determination of the orbital angular momentum by
means of the uncertainty relation. If the origin of the coordinate system Is taken to be
at the center of the wave packet, the error in the orbital angular momentum will be
AZ.* A yipx — AxApy.

In accordance with the uncertainty relation, we have Apx ~ — , Apy ~ — . Since the

errors may be either positive or negative, we find

+ (■ $ )• (21-35)
The error A[,z will be minimum when | Ax | = | Ay |. Thus the error AL: due to the
translational motion of the particle is of the order of the spin, and therefore the per­
pendicular components of the intrinsic angular momentum and the Dirac magnetic
moment cannot be determined simultaneously.

F i g . 2 1 .3 . E x p e r i m e n t s for d e t e r m i n i n g the m agnetic


m o m en t of t h e n e u t r o n .

We recall that the Stern-Gerlach experiment allowed for the determination of a mag­
netic moment of a bound electron; since, however, the orbital angular momentum in the
s state is zero, it was the spin (or Dirac) magnetic moment that was actually measured.
In accordance with (21.31), the interaction energy associated with the perpendicular
component of the anomalous magnetic moment is

Fm= fip,<.,Q5 r . (2L36)

This component commutes with the Hamiltonian (21.32) and can therefore be measured
exactly. Consequently, it is possible to measure the magnetic moment of a free neutron
when the magnetic field is perpendicular to its motion, as was done by Bloch and Alvarez
(1940).6 In their experiments when a beam of neutrons was passed through a piece of
magnetized iron ,7 the most pronounced scattering was observed for those neutrons whose
magnetic moment was parallel to the magnetic induction vector inside the iron. The
emerging beam consisted, therefore, mostly of neutrons whose magnetic moment was
antiparallel to the magnetic induction vector. If the neutron beam passes now through
two magnetized iron plates in succession, the experiment is completely analogous to
the transmission of light through two Nlcol prisms; that is, the first iron plate acts as

S i m i l a r l y , it m a y b o s h o w n t h a t o n l y t h e l o n g i t u d i n a l c o m p o n e n t o f s p i n c o m m u t e s
w i t h t h e H a m i l t o n i a n . In p r i n c i p l e , t h e r e f o r e , it a l s o c a n b e m e a s u r e d e x p e r i m e n t a l l y .
7N e u t r o n s h a v e n o e l e c t r i c c h a r g e a n d t h e y p a s s q u i t e f r e e l y t h r o u g h m a t t e r . A c t u a l l y ,
t h e i r o n l y i n t e r a c t i o n s o c c u r on c o l l i s i o n s w i t h t h e n u c l e i .
T H E E F F E C T O F N U C L E A R S T R U C T U R E ON A T O M I C S P E C T R A 34 3

a polarizer and the second as an analyzer. This phenomenon was used to determine the
magnetic moment of the neutron.
A schematic diagram of the Bloch-Alvarez experiments Is given In Fig. 21.3.
Unpolarized neutrons moving along the direction of the x axis pass through the
polarizer (first Iron plate, with a magnetic induction vector directed upwards). The
emerging beam consists mostly of neutrons whose magnetic moment Is directed down­
wards. These polarized neutrons will pass freely through the analyzer if its magnetic
Induction vector Is directed upwards, like that of the polarizer. On the contrary, they will
be transmitted much more weakly if the iron plates are oppositely magnetized.
Between the polarizer and analyzer there Is a device that reorients the neutron
spin. This device Is sim ilar In principle to the one used by Rabl in his nuclear magnetic
resonance experiments to determine the magnetic moment of the proton and of heavier
nuclei. The basic principle of the Instrument Is as follows. In the space between the
polarizer and the analyzer, a relatively strong, constant magnetic field Is applied
parallel to their magnetization vectors (see Fig. 21.3). A neutron whose magnetic
moment is antiparallel to this magnetic field acquires the additional energy

A£ami= ^ a5f . (21.37)

If, however, the magnetic moment Is parallel to the field, the neutron loses an energy

AE Par = - | (21. 38)


In addition, there is a relatively weak oscillatory magnetic field II applied perpendicu­
larly to the field
H y = A cos (o0f.
This oscillatory field will reorient the spin of the neutron particularly strongly when the
frequency w0 is close to the resonance frequency

A /;an ti_ AEPar _


(21.39)

At this resonance frequency, the number of neutrons undergoing a reorientation of the


magnetic moment reaches its maximum value. To find when this happens. It is necessary
to determine when the number of neutrons passing through the analyzer Is minimum In
the case of parallel magnetic Induction vectors, or maximum in the case of antiparallel
magnetic vectors.
Once the frequency <■>,> has been determined (It Is, In effect, twice as great as the
Larmor frequency of precession), the magnetic moment of the neutron can be found.
According to recent data It is equal to 8
Fn == 1.9131 H-nucl
where the unit for measuring magnetic moments is the nuclear magneton
_
^ nucl- 2nipC £ i*= Tskr>“ ” 0-505 • l0~“ erE •
where mp is the mass of a proton and p0 Is the Bohr magneton.
From the resonance frequency of the oscillatory field, we can determine the magnitude
of the magnetic moment but not its sign. If, however, we replace the oscillatory magnetic
field by a rotating magnetic field, we can also determine the sign of the moment, since
for resonance it is necessary that the vector equation Wo = — 2o should hold, where
M
-
o = l A-L—. is the Larmor frequency of precession of the neutron spin. The minus sign

shows that the magnetic moment of the neutron, Just as for the electron, Is directed
opposite to the spin.

Subsequent Improvements on the Bloch and Alvarez experiments are described in


E. Segri, Experimental Nuclear Phyeica, Vol. I, 1953.
THE EFFECT OF NUCLEAR STRUCTURE ON A T O M IC SPECTRA 34 5

we obtain

= 6(r), (21.42)

Consequently, in the first approximation, the interaction of magnetic moments—Just like


the contact interaction—influences only the s state. The expression (o'-op) in (21.42)
can be found from the following simple considerations.
The spin matrices of the proton op and the electron o' must satisfy the relation

| f t > ' + o p)2 = 7l2S ( S + l ) , (21.43)

where S is the absolute value of the total spin, which is equal to either zero (antiparal-
lell spins) or to unity (parallel spins). Then

| b ' 2 + ^ 2 + 2(o'-o;)] = S ( S + I ) .

Using the fact that a12+ ap2= 6, we obtain

(s '. b;) = 2 S ( S + I ) - 3 . (21.44)


Since integration when the 6 function is present gives

J ■*6 (r)= * | (0) | 2,

we obtain the following expression for the shift of s levels (hyperfine structure) of
hydrogen:

A lls = N pp J - [2 S (S + 1) - 3] , ( 2 1. 4 5 )

%2
where a0 = -----j- is the radius of the first Bohr orbit, and the value of | ^ (0) | 2 is taken
m0ei
from Eq. (20.14),
Two cases should be distinguished:
(1) Spins of the proton and the electron are antiparallel (S = 0); then

i £ 5 = 0 = - 8 H-oFp— • (21.46)

(2) Spins of the proton and the electron are parallel (S = 1); then
8 1 (21.47)
iES=l— 3 n»a» •
The difference between these levels represents splitting of the s term due to the
Interaction between the electron and the magnetic moment of the nucleus
AES= i A/; 5=0 _ 32 poFp _1_
Aoj= 3 % n 3a3'
(21.48)

If we use (21.48) to calculate the s-term splitting for the case n = 1, then substituting
the value of pp obtained in Rabi’s experiments and setting p0 equal to the Bohr mag­
neton, we find
Autheor= 1417 Me.
On the other hand, a careful experimental verification of the splitting of this level by
microwave spectroscopic methods has given the value

Ao)exp = 1420 Me.


346 R E L A T IV IS T IC QUANTUM M EC H A N ICS

The relativistic corrections and the corrections for the finlteness of the nuclear mass

do not raise the frequency iu ^ 601" to the required value AwexP . The proton’s
magnetic moment has also been measured very accurately. To explain this anomaly,
therefore. It remained to assume that the magnetic moment of the electron is not exactly
equal to the Bohr magneton, but is instead somewhat larger. Kusch and Foley showed that
to obtain agreement with experiment the magnetic moment of the electron must be taken
to be

I1el ——Fo (1 + ®)i (21.49)


where, according to recent data.

8 = 0.00116.
These considerations show that an electron will have a very small anomalous mag­
netic moment p ^ om = p05 in addition to the Dirac or kinematic magnetic moment

(— po). We shall discuss the nature of the anomalous magnetic moments in the next
chapter.
Concluding this section, it is necessary to point out that the hyperflne structure can­
not explain the Lamb shift of the 2si/2 level (1.058 Me relative to the 2/» i /2 level). First
of all, itfollows from (21.48) that the splitting of the 2si /2 term Is of the order of 200
Me, and, in addition, the center of mass of the s terms is not shifted. Suppose, the
level with S = 1 (parallel spins of the electron and the proton) is raised by a certain
amount [see (21.47)]; then the level with S = 0 (antiparallel spins of the electron and
the proton) is lowered by an amount three times as large [see (21.46)]. Since a state
with S = 1 is three times more probable than a state with S = 0 (when S = 1 the
spin may be directed along the z axis, opposite to the z axis and perpendicular to the
z axis), the center of mass of the s states remains unaltered, so that It occurs in the
same position as when the magnetic moment of the nucleus is neglected. The hyper­
flne structure cannot therefore account for the Lamb shift. The theory of this effect
will be considered In the next chapter.

Problem 21.1. Find the shift of the Is levels in the light p-m esic atoms as a result
of the influence of the nuclear structure, taking into account the nuclear motion and
also the variation of the wave function within a distance comparable to the size of the
nucleus.
Hint. The shift of energy levels should be calculated by the perturbation method. We
use Eqs. (21.5) and (21,7) where the wave functions are those of hydrogen-like atoms with
the meson mass substituted for the electron mass. Because of the finiteness of the
nuclear m ass, the motion of the nucleus has much greater Influence on the position of
the energy levels of a meslc atom than on the levels of an ordinary atom, it is there­
fore necessary to use the reduced meson mass in more accurate calculations. The
wave functions of a meson change appreciably inside the nucleus; therefore, the quantity
| ’'j iL
' can be replaced by its value at the origin only in a comparatively crude qualita­
tive estim ate, as in the derivation of (21.14).
Answer. In the general case

For sfnall b and ,U — oo, this expression may be obtained from Eq. (21.12).
(Hiapter 22

The Electron-Positron Vacuum and the


Electromagnetic Vacuum

A. DIRAC THEORY OF “ HOLES.” DISCOVERY OF


THE POSITRON

The Dirac theory, which includes spin effects and relativistic effects, was able to
account for the fine structure of the spectral lines of hydrogen-like atoms and the
anomalous and normal Zeeman effects. The Dirac theory, however, also gave rise to
a number of major difficulties in connection with the interpretation of negative energy
states. These difficulties were not overcome for some time, but they eventually led to
fundamental new discoveries in relativistic quantum mechanics.
In our treatment of the motion of a free particle (Chapter 17), we mentioned that the
Dirac equation allowed solutions corresponding to both positive and negative values
of energy. It is worth noting in this connection that solutions with negative energy are
not characteristic of the Dirac theory alone, but appear in any relativistic theory, in
relativistic mechanics, the energy of a free particle is connected with the momentum
and rest mass by the well-known expression

E- = c-p- + m-uc\
which has two roots

E = i t Yc-p- + /njc1.
The regions of positive and negative energies are separated by an interval equal to
2moc2 (see Fig. 22.1). At first glance, states with negative energy do not appear
to have a real physical meaning, since the region of negative energies extends to Infinity
( E — — oo) and, therefore, there is no lowest
state. This would imply that no ordinary state
is stable, since a spontaneous transition to a
lower energy state would always be possible.
Furthermore, a particle with negative mass
(negative energy) would have a number of
strange properties: for example, it would repel
a particle with positive mass. -E=0
In classical physics, states with negative
energy do not cause any difficulties, because
the energy of a moving particle can only change
continuously; therefore, transitions from the
states with positive energy to the states with
negative energy cannot take place, since the
energy would have to change discontinuously by F ig . 22.1. A llo w e d e n e rg y le v e ls of a
the amount &E 5 : 2 m0c2. Defining the energy free D irac p a rtic le .
to be positive from the start, we may, therefore,
neglect the negative energy states.
The situation is quite different in quantum theory, where transitions can take place
between the states in a discrete spectrum, as well as in a continuous spectrum. States
with negative energy cannot be excluded simply by defining the energy to be positive
348 R E L A T IV IS T IC QUANTUM M EC H A N IC S

at the initial time, because the probability of the transition between the states with
energy + m 0c2 and — m0c- is not equal to zero.
In order to avoid transitions of electrons to negative states, Dirac suggested (1931)
that we regard all negative energy levels as occupied by electrons (see Fig. 22.2),
so that an electron with a positive energy cannot
• Jump into a negative energy level under ordinary
---------------------------------------conditions. 1 The state In which all negative
• ~~' — energy levels and no positive energy levels are
occupied is called the "electron vacuum.”
Let us now assume that a 7-ray photon with
energy t i> 2 m0c2 excites an electron from the
electron vacuum into a positive energy state. In
this case, the absorbed 7-ray photon will be
replaced by an electron with positive energy, and
a "hole” will appear In one of the negative energy
states (see Fig. 22.3).
The decisive factor that led to the success of
F ig. 22.2. Z e ro -p o in t e n erg y d ia ­ Dirac’s hypothesis was that he interpreted the
gram of the e lectro n -p o sitro n "hole” as a particle (the "positron” ) with posi­
vacuum .
tive mass equal to the mass of the electron, but
with the opposite charge. 2 Let us suppose that
there are no particles at the initial time. Then
the "zero-point energy” EVac (the energy of the electron vacuum) is equal to the sum of
the energies of the electrons In the negative energy states /i_

E —V E ( 22. 1)
c vac — Z . n

The "zero-point charge” is equal to

eCvac v e0.

Thus, from the standpoint of the hole theory, the absence of any real particles means
that all positive energy states are empty, and all negative energy states are occupied.
This case corresponds to the electron vacuum (see Fig. 22.2).
When an electron jumps from a negative energy state n_ to a positive energy state n+ ,
the total energy change of the system is

AU= E’\ + 1n’' £»1- 2 £»L= E"+- E»_ = En++ IEnJ . (22.2)
n'

This change represents the sum of the positive energies of two nascent particles .3 Similar
arguments with regard to charge show that the charge of the nascent particle correspond­
ing to the "hole” is opposite to the electron charge

e = — en+ — fo + ^ **0 = — + en_ = — e0 + e0. (22.3)

*In a c c o r d a n c e w i t h t h e P a u l i e x c l u s i o n p r i n c i p l e ( s e e C h a p t e r 6), o n l y o n e e l e c t r o n
can occupy each stale.
A s i m i l a r c o n c l u s i o n w a s r e a c h e d in Ih o t r e a t m e n t o f t h e h o l e t h c o r y o f s e m i c o n d u c t o r s
( s e e t h e d i s c u s s i o n o f th e b a n d t h e o r y o f c o n d u c t i o n in C h a p t e r 6).
* T h e p r i m e a t t a c h e d to t h e s u m m a t i o n sip.n (?. ) m e a n s t h a t t h e s u m m a t i o n e x t e n d s o v e r
all s t a t e s e x c e rp t t h e s t a t e n ‘_ .
ELECTRON POSITRON AND ELECTROMAGNETIC VACUUMS 349

Ihus, the transition of an electron from a negative energy state to a positive energy
state (as a result of the absorption of a 7-ray quantum with energy greater than 2 muc'J)
leads to the creation of a pair of particles. The unoccupied negative energy state ("hole")
may be regarded as a state occupied by a particle with positive charge + e 0 and positive
energy.4 This particle, which was predicted by
Dirac, was called a "positron” and was dis­
covered by Anderson in cosmic radiation (1932).
Once It is Interpreted In this way, the Dirac
theory describes In a natural manner both the mnc ‘
electron and the positron. The positron Is an
antiparticle; Its wave function satisfies the Dirac
equation with positive energy and positive charge
(see Problem 18.5). -mr
In the Dirac theory, pair annihilation — a
process which Is the reverse of pair creation—is
also allowed. This process takes place when an
electron with positive energy Jumps Into a hole.
In this case the electron and positron are con­ F ig . 22.3. F o rm a tio n o f a n e le c tr o n -
verted Into 7-rays. p o s itr o n pair.
In these transformations, the laws of conser­
vation of energy and momentum are rigorously
obeyed. As already mentioned, pair creation due to the absorption of a 7-ray photon can
occur only In the presence of a third particle (for example, a nucleus), that takes up the
excess momentum of the photon

7 Ze„ —*■Zbq -f- b + b (22.4)


Similarly, the conversion of an electron-positron pair Into 7-rays takes place In accord­
ance with the laws of conservation of energy and momentum; as a result of pair annihila­
tion, at least two 7-ray photons are created
b+ ~ \- b_ = 2y. (22.5)

In order to show this, we may choose, without the loss of generality, a coordinate system
In which the electron and positron move with opposite momenta, so that = — k+ = k
(the center-of-mass system). Then, according to the law of conservation of momentum,
the total momentum of the two photons which are formed as a result of annihilation must
also equal zero
n ( k l + k») = 0. (22.5a)

Using the law of conservation of energy

cTi (ki + It.) = 2 V'mlc* + c-n2k-, (22.5b)

we find that k i = — k 2l and the energy of each of the photons Is equal to

e = ctiki - ctiki = y in5c4+ c2H2k-.

The lowest value of the photon energy is obtained when k = 0 (that Is, when the electron
and positron are at rest). Then ( mjn = moc'-. These two 7-ray photons move apart with
the same energy and oppositely directed momenta. It is easy to see that the electron-
positron pair cannot be converted Into a single 7-ray photon ( k 2 = 0), since the laws of
conservation of energy and momentum cannot be satisfied simultaneously with only one
7 -ray photon.

4
With t h e h e l p o f q u a n t u m f i e l d t h e o r y , w e c a n c o n s t r u c t a t h e o r y o f t h e e l e c t r o n -
p o s itro n vacu u m th a t is s y m m e tric w ith r e s p e c t to c h arg e. H o w ev e r, ev en w ith th e theo ry
d e s c r i b e d a b o v e , w h i c h i s a s y m m e t r i c w i t h r e s p e c t to e l e c t r o n s a n d p o s i t r o n s ( a n e l e c t r o n
i s a p a r t i c l e , w h e r e a s a p o s i t r o n i s a “h o l e ” ) w e c a n g i v e a c l e a r e x p l a n a t i o n o f m a n y
pheno m en a in v o lv in g th e tra n sfo rm a tio n of p a r tic le s .
350 R E L A T IV IS T IC QUANTUM M EC H A N ICS

The law of conservation of total angular momentum (orbital plus spin) is also very
Important in the annihilation processes. If an electron moves with a nonrelativlstlc
velocity, this law (as shown in Chapter 18) can be resolved into a law of conservation of
orbital angular momentum and a law of conservation of spin.
The law of conservation of spin can be observed particularly clearly in the annihila­
tion of positronium (a hydrogen atom in which the proton is replaced by a positron, or
more exactly, a system in which an electron and a positron rotate about their common
center of mass). In this atom, the nucleus (that is, the positron) and the electron have the
same m ass, and therefore the reduced mass Is The energy of levels In the positron­
ium atom will, therefore, be one half as large as in the hydrogen atom

= _M _
2n1 '
while the radius of the orbit will be twice as large. The velocities of the electron and
positron may be regarded as nonrelativistic. Just as in the hydrogen atom. If the spins
of the electron and positron are antiparallel (paraposltronlum), positronium can decay
into two 7-ray photons (the corresponding mean life Is 1.25 • 10~10 sec).
The total spin of parapositronium Is equal to zero, and therefore the two photons move
apart with opposite directions of spin (that is, their total spin Is zero).
If, however, the spins of the electron and positron are parallel (orthoposltronlum) the
system must decay Into three 7-ray photons (the corresponding mean life is 1.4 • 10- 7sec).
The spin of orthopositronium is equal to unity. Orthopositronium cannot decay into one
7-ray photon with spin 1, because in this case the law of conservation of momentum
would be violated. It cannot decay into two 7 -ra y photons because then the law of con­
servation of spin would be violated (the total spin of two 7-ray photons is either two or
zero). Only if orthopositronium decays into three 7 -ray photons will the law of con­
servation of momentum and the law of conservation of spin be satisfied.
The discovery of the positron opened a new stage In the study of elementary particles.
This discovery showed that particle shad a new fundamental property—lnterconvertlblllty—
and confirmed the existence of antiparticles. We can regard the creation of a positron
as the conversion of a 7-ray photon into an electron-positron pair, and the annihilation
of an electron-positron pair as the conversion of an electron-positron pair into 7-ray
photons.

B. THE LAMB SHIFT OF ENERGY LEVELS OF


ATOMIC ELECTRONS
When an electron moves in an atom, it interacts with the electromagnetic vacuum,
as well as with the atomic nucleus and the electron-positron vacuum. The interaction
between the electron and the electromagnetic vacuum exerts a particularly strong
influence on the motion of the electron in the atom, and it explains the shift of the 2s1/s
level upwards relative to the 2/ti/o level (in the hydrogen atom).
A complete theory of this phenomenon can be constructed only by means of quantum
electrodynamics, which Is based on the theory of second quantization. But even without
referring to this theory we can still obtain the appropriate equations with accuracy up to
coefficients of the order of unity, while using comparatively simple physical arguments.
One of the basic ideas of the quantum field theory Is that each wave or field can be
associated with a particle. ’Ihus, for example, the Dirac waves correspond to electrons
and positrons, and light waves correspond to photons. It Is very well known, however,
that Maxwell’s equations describe not only light waves, but also electrostatic and magnetic
fields, which depend on charges and their velocities (an accelerating charge produces
electromagnetic or light waves). The electrostatic and magnetic fields can be associated
with "pseudo-photons," which have observable effects only in the presence of charges.
An electrostatic field can be expanded in a Fourier series, that is, schematically repre­
sented as a set of oscillators with different frequencies. An analogous expansion holds
for the vacuum field of "pseudo-photons”

( 22 . 6 )
ELECTRON-POSITRON AND ELECTROMAGNETIC VACUUMS 35

Since the rest mass of a "pseudo-photon” Is zero, we may write the relation between Its
frequency <*> and the wave number k as follows:

“ = -«- = I T = e 1/ * T F * j T * * . (22.7)

The components kx , ky, k, are related to the Integers n,, n2, n, and the period I. (see
Chapter 6) by the expressions

kx = - — - , and so forth,

where «i = ± 1, ± 2 , 3............ Hence


\b _ 27t3;j, _ 2 n
^ k X-- Sky Sk

so that
Skx Sky Sh2 = k 2 dkdQ,

m2dus 8n3
—3
c3— dQ= ~rr
L■

If the system Is spherically symmetric, this relation may be written as

o- da* I (22. 8)
L1 “ca—'2nr
With the aid of Eq. (22.6), we find that the energy of the electrostatic field Inside a region
of volume V is

ep. P. = i J (£)2 (i'x = w Z (£ (u))2- (22*9)


O)

In deriving this expression, we have used the relation

(cos U c o s u '0 a v =

Just as In the theory of the harmonic oscillator, the energy of the field In the lowest
energy state is not equal to zero, but instead is equal to the sum of the zero-point
energies of the harmonic oscillator (see Chapter 10)

S« = 2y { s"- (22.10)
(1)

The coefficient 2 In front of the summation sign takes into account the fact that each
harmonic corresponds to two different states of polarization of the "pseudo-photons.”
In the state where there are no real photons the total energy of an electromagnetic
field (the vacuum), must be equal to this zero-point energy (e p^p# = e0). Hence, taking
(22. 8) into account, we find

( £ (» ) )* = 71“ = - ^ - ~ P - . (2 2 .10a)

In the absence of real particles and external fields, the vacuum (including the electro­
magnetic one containing photons and pseudo-photons) does not, as a rule, have any
observable effects because it is isotropic. On the other hand, when real particles
352 R E L A T IV IS T IC QUANTUM M EC H A N IC S

and external fields are present, the Isotropy Is disrupted and virtual particles (“pseudo­
photons” or electron-positron pairs) are created and subsequently annihilated (vacuum
fluctuations).
A simple physical picture of a few basic notions Involved In the theory of the vacuum
was obtained somewhat unrlgorously by Welton by means of a sem iclassical, nonrelatlvlstlc
treatment of the motion of an electron, taking Into account Its Interaction with the vacuum
fluctuations. As an Illustration of Welton’s calculations, let us attempt to give a more
concrete meaning to the zero-point fluctuations (22.10X In a rough approximation we
can use the ordinary classical equation
/n0r = ££p>p> (22.11)
to find how an electron will be affected by the vacuum field of "pseudo-photons." Ex­
panding £_ _ in a Fourier series
P* P*

£ p .p> = ^ £ H cos wt (22.12)


<l>
and integrating Eq. (22.11), we find the displacement of the electron positron due to the
vacuum field:

(22-13)
<D
The mean-square deviation of the position is given by

(17F = ^ - ( r ) ’ = ^ 2 i ^ ^ , (22.14)
U)
since

COS u f • = 0, COS u>t COS <1>'t = ~ 60>(u.

and therefore r — 0. Substituting (22.10a), we obtain a divergent Integral

(Ar? = -2 f ( -~ )2 ,la , (22.15)


' it he \ nioc / j ’

from which a finite (observable) part can be separated out If the range of variation of the
frequency is cut off from above at a frequency corresponding to the rest energy of the
electron

(l)max —
m0c2 (22.16)
ft ’
and from below at a frequency corresponding to the minimum energy of the electron In
an atom 5

_ I/-U_ "'n«_4 (22.17)


m in — ft —

Substitution of (22.17) and (22.16) Into (22.15) gives

2 _h_
(22.18)
(ir)2 —a Mn
»v.

’ T h e l i m i t s o f t h e r a n p e o f v a r i a t i o n o f fv a r e s p e c i f i e d m o r e a c c u r a t e l y in r e n o r m a l i ­
z a tio n theory.
ELECTRON PO SITR O N AND E L EC TR O M A G N E TIC VACUUMS 353

Thus it can be seen that the vacuum field of "pseudo-photons" will cause the electron
to perform a motion somewhat resembling Brownian motion with a definite value of the
mean square displacement. It is well known that the Brownian motion of a particle Is due
to Its collisions with randomly moving molecules of the surrounding medium. In similar
fashion, an electron undergoes "collisions” (of a special kind) with the assembly of
virtual particles forming the vacuum.
As was shown by N. Bogolyubov and S. Tyablikov, the vacuum fluctuations cause a
certain "smearing out” of a point electron. As a result, the electronic radius turns out
to be the geometric mean of the classical radius and the Compton wavelength

fT a ~ (22.19)
r «ioc »i0c muc
The existence of this effective radius should have several consequences. In particular,
the interaction between the electron and the nuclear charge should be changed, and this,
in turn, should lead to an additional coupling energy and thus to a shift of the energy
levels. The usual expression for the potential energy of an electron In the field of a
nucleus
V = — e0‘I>(r) (22.20)
Is replaced by

K '= -<?„<!>(r + Ar) = - * 0 [1 + (ArV) + - (Ary)2 + . . - l * ( r ) . (22. 21)

Changing to the average values and using the relations

A>= 0, (A77?' = j(A 77y2, (22.22)


we obtain
l / ^ - e o J l + I t T T j V + ... } *( r) . (22.23)

The additional energy of interaction between the electron and the nucleus Is given by

a^p. p - ^ -^ = -^ w ^ = 4 e»a( i ) 2ln? 8(r)- <22-24)


since the Coulomb potential of the hydrogen nucleus satisfies the Poisson equation

V2*K= - 4c<?0!>(r). (22.25)

To obtain an expression for the shift of energy levels In the hydrogen atom. It Is necessary
to average the additional Interaction energy over the corresponding state

A£p>p= £ (V' — V) | * (r) I2d*x = - ela | * (0) |» In (22.26)

This shift occurs only for s states, since the quantity | ^ (0) |2In the approximation under
consideration vanishes for other states ( / = 1, 2, 3 , . . . ) , whereas for the s state

14*(0) I2= (22.27)


r.n3al ’
where a0= ti-/mael is the radius of the first Bohr orbit of the hydrogen atom, and n Is the
principal quantum number. If we now substitute this value Into (22.26), we obtain a
formula for the s-level shift
i a3—r-
R U In
in 2na
—rr (22.28)
AEP. P.— 3c n3 a-

This formula was first derived by Bethe,


354 R E L A T IV IS T IC QUANTUM M EC H A N ICS

A substitution of numerical values for the 2s state (n = 2) gives

A£p>p> = 17.8/?= 1040Me . (22.29)

This Is in fairly good agreement with recent experimental data (A E = 1057.77 Me) for
the Lamb shift (see Chapter 20),
A complete study of the shift of energy levels of atomic electrons with the help of
relativistic quantum field theory gives considerably better agreement between theoretical
and experimental results than the sem iclassical formula (22.28). The discrepancy is
reduced to le ss than 1 Me.
For the sake of brevity, we shall not discuss the modern theory of the vacuum In
greater detail, and we shall confine ourselves to a mere enumeration of the main results
that this theory has yielded.

C. ELECTRON-POSITRON VACUUM

Equation (22.28) for the Lamb shift was obtained from a calculation of the Interaction
between electrons and the electromagnetic vacuum. In addition to the electromagnetic
vacuum,. there exists an electron-positron vacuum and also vacuum states of other
particles. The method of second quantization (which is In some measure applicable to
all fields) can be used to calculate the influence of the electron-positron vacuum.
In modern quantum field theory, the study of the properties of the vacuum states of
different particles plays a particularly important role. The vacuum gives rise to inter­
actions between particles. In particular, the electromagnetic Interaction of two electrons
(Coulomb’s law) may be regarded as an interaction which takes place through the electro­
magnetic vacuum, with one electron emitting a virtual photon and the other electron
absorbing it.
On the other hand, the vacuum represents a sort of "reservoir,” from which real
particles are "drawn” when they are created, and to which they "return" when they are
annihilated. We have already come across the electron-positron vacuum as a "collection”
of electrons in negative energy states. Unfortunately, it has no classical analog and
therefore we cannot use a sem iclassical analysis, as was possible in the case of the
electromagnetic vacuum. The Coulomb field of a nucleus can polarize the electron-
positron vacuum (so that an electron will behave as if it were in a dielectric), giving rise
to an additional interaction

<2 2 - 3 0 >

Comparing (22.30) with (22.24), we see that the level shift due to Interaction with the
electron-positron vacuum is about 1/40 times as large as the level shift due to the fluc­
tuations of the electromagnetic field, and Is opposite in sign.
The electron-positron vacuum exerts a particularly strong influence on the magnetic
properties of the electron. It was shown by Schwinger that the magnetic moment of the
electron becomes somewhat larger than the Bohr magnetron

Fe.-p.= (22.31)

The change in the magnetic moment of the electron, with the second-order term taken
Into account.

Au „ = - ( £ ■ — 0.328 - V o = -0.0011596 n„ (22.32)


7. j

is in good agreement with experimental data obtained with the aid of microwave spectro­
scopic methods (see Chapter 21). Qualitatively, the appearance of the additional magnetic
moment of the electron and the sign of the moment can be explained as follows. The initial
electron A (see Fig. 22.4), whose spin is directed upwards (we specify in this manner a
preferred direction), creates a virtual pair—electron /!1 and positron fl1—which generally
E L E C T R O N -P O S IT R O N AND EL E C T R O M A G N E T IC VACUUMS 355

have opposite spin directions. Since we have a preferred direction (determined by the
spin of electron A) the spins of electron A' and positron B‘ can be directed in two ways:
either the spin of A will be antiparallel to the spin of /V and parallel to the spin of B'
(case 1 in Fig. 22.4), or the spin of A will be parallel to the spin of /V and antiparallel
to the spin of B' (case 2 in Fig. 22.4).
In determining the additional magnetic mo­
ment, it Is necessary to consider the following
possibilities of pair creation and annihilation: bY a)
(a) Palr/VB 1 is created and then annihilated.
This is possible in case 1 and case 2. No cor­
rection to the magnetic moment should arise,
because the probability of the creation process
is the same in both cases, and the magnetic
moments have opposite signs in the two cases.
(b) Since ,1 and B' have opposite spins in
case 2, positron B' may be annihilated together
with A ', as well as with the initial electron A.
In case 1, however, this process is less prob­ >1 if-
able, since the spins of pair A and B' are
parallel to one another. This process gives a A n t i p a r a l l e l Parallel
preferred state in which the additional magnetic s p in s of spins of
moment is parallel to the Initial magnetic mo­ electro n s electrons
ment of electron A, since, as can be easily A and A1 A and A1
seen, the magnetic moment of all three particles F i g . 2 2 . 1. V i r t u a l c r e a t i o n o f a n e l e c ­
(A, A' and B') is directed downwards. Conse­ tr o n - p o s itro n p a i r by a n e le c tr o n . T h e
quently, there arises an additional magnetic s p in s of th e real a n d v irtu al p a r tic le s
moment of the electron, equal to
a r e d e p i c t e d by s o l i d a n d d a s h e d a r ­
^ e .-p . =—2fM. row s, r e s p e c tiv e ly .

where f Is a numerical factor which determines the probability that the initial electron
A will be annihilated together with positron B'.
These simple considerations give the correct sign of this anomalous magnetic mo­
ment. More rigorous calculations led to Eq. (22.32).
In addition to the Dirac magnetic moment, u = — y 0o , an electron will have an
a ^
anomalous magnetic moment |ian = — ,-j- ^oP«®, which arises owing to the interaction
between the electron and the electron-positron vacuum.
The anomalous magnetic moment of nucleons (neutrons and protons) can be explained
in meson theory. The accuracy of the results, however, is much le ss striking than in
calculations of the anomalous magnetic moment of the electron. This Is due to the fact
that meson theory is still in a much less satisfactory state than quantum electrodynamics.
According to the meson theory, protons and neutrons interact with the 7r-meson field.
Since the proton can dissociate into a neutron and a positively charged ir meson, and the
neutron Into a proton and a negatively charged ir meson, the anomalous magnetic moments
of protons and neutrons due to the ir-meson field must be approximately equal in magni­
tude and opposite in sign.
The appearance of the additional magnetic moment of a neutron is explained as follows.
The neutron has a definite probability of dissociating into a proton and a ir meson. Since
the spin of a tt meson equalsO.it does not possess an intrinsic magnetic moment. There­
fore, there will be a contribution to the magnetic moment of the neutron from a plon
which is, for example. In a p state (we recall that the orbital angular momentum in the
s state is also equal to 0). The magnetic moment of a ir meson is —£ «= 7 times greater
than the nuclear magneton, and therefore the tt meson produces the main contribution to
the anomalous magnetic moment of the neutron. For the virtual process to fulfill the
law of conservation of angular momentum, it is necessary that the direction of the
orbital angular momentum (equal to 1) of the virtual ir meson should coincide with the
direction of spin of the neutron, while the spin of the virtual proton (equal to *,», just
like the spin of the neutron) should be directed opposite to the spin of the neutron so that
s„ = s„ — sp, s„ = Sp = 4j-, s „ = l . Since the ir meson has a negative charge and its
angular momentum is parallel to the spin of the neutron, it gives rise to the negative
magnetic moment of the neutron.
356 R E L A T IV IS T IC QUANTUM M EC H A N IC S

In order to explain the magnitude of the anomalous magnetic moment of the neutron,
one must assume that the neutron spends ‘A of Its time in Its dissociated state. This
estimate is perfectly reasonable and gives the correct sign and order of magnitude of the
magnetic moment of the neutron(n®n ~ — 2t± nuc). Similarly, it can be shown that the dis­
sociation of the proton into a neutron and a ji+ meson gives rise to a positive anomalous
magnetic moment of the proton (|J-pn~ + 2m-nuc)- Moreover, the proton has a Dirac mag­
netic moment (|J-jp= p- nuc)- More accurate data on the magnitude of the magnetic moment
of the proton and neutron are given in the preceding chapter.
Thus we have a basis for regarding the anomalous magnetic moment of Dirac particles
as a secondary effect which can be explained on the basis of the field theory. This
moment does not appear in the initial equations, but arises as a result of the interaction
either between the electric charge and the electron-positron field (in the case of electrons),
or between the nuclear charge and the 7r-meson field (in the case of protons and neutrons).

D. RENORMALIZATION

One of the most important sections of modern quantum field theory is concerned with
the question of renormalization. This subject is not yet in a mathematically satisfactory
state, but a number of important results have been obtained.
The basic idea involved in renormalization is the separation of finite, observable
terms from the divergent terms describing the interaction between an electron and the
electromagnetic or electron-positron vacuum.
In effect, the question of renormalization first arose in classical electrodynamics—
for example, in the theory of the electromagnetic mass of the electron. If we assume
that the electron charge is distributed inside a sphere of radius r0, its classical electro­
magnetic m ass will be equal to

field — y e‘ (22.33)
~ Tr0c2’
where 7 is a factor of the order of unity which depends on the charge distribution inside
the sphere. The attempts to construct a classical electrodynamics in which a finite
radius r 0~ 10“13 cm would give a reasonable value for mfield (the Lorentz theory, the
nonlinear Bom-Infeld theory, the Boppe-Podolsky theory with higher derivatives) did not
give any satisfactory results. Moreover, all these theories gave rise to fundamental
difficulties in connection with quantization.
On the other hand, the theory of a point electron leads to an infinite value of the mass
as r0 — 0 both in the classical and quantum forms of the theory. It was therefore a major
achievement of renormalization in modern quantum field theory that it was able to
separate, from the Infinite Interaction energy, finite terms associated with the Lamb
shift of the atomic levels and the additional magnetic moment of the electron.
Modern renormalization theory extends to the problem of the self-m ass and charge
of the electron. It has been found that when the electron-positron vacuum and the electro­
magnetic vacuum are taken Into account, the electromagnetic m ass (22.33) drops out
completely. Therefore, the main mass of the electron (the so-called “bare” mass) will
not be associated with the electromagnetic field. The Interaction with the vacuum (in this
case the main contribution is obtained when the electromagnetic vacuum and the electron-
positron vacuum are taken into account simultaneously) yields an additional m ass which
diverges only logarithmically

. field .
Am — am In ‘max (22.34)
bare m 0c-

Renormalization is faced with the important question of finding the value c max at
whLh the logarithmically divergent expression should be cut off. This problem Is a
theoretically important one, although in practice the logarithmically divergent terms may
be regarded to be of the order of unity, and therefore the quantity Am will remain of the
order m bare /137 over a comparatively wide range of values e max.
ELECTRON PO SITR O N AND E L E C T R O M A G N E T IC VACUUMS 357

In exactly the same way, the Interaction of the electron with the electron-positron
vacuum (rather than with the electromagnetic vacuum) leads to a decrease of Its "bare
charge” by the amount

■It -P._ ac In e max (22.35)


bare
The field corrections to the mass and charge of the electron have not yet been experi­
mentally separated, and this important subject requires further investigation.6
Problem 22.1. When can the emission of photons take place for the free motion of an
electron In a medium with Index of refraction n > 1(the Cherenkov effect)?
Why does this emission become impossible in a vacuum (n = 1)7
Solution. In order for the emission to be possible. It Is necessary that the laws of
conservation of energy

Y + c'~Ps = Tiw-)- Y Mot1 + c*Pn


and momentum

p = p ’ + tik
be satisfied, where p and p' are the initial and final moments of the electron and tik is
the photon momentum.
Squaring these relations and then subtracting one from the other, we obtain the follow­
ing expression for the cosine of the angle of photon emission:

C0S°
= ^ + f ['“ A }
Using the relation for the index of refraction n of the medium

e = Hoi = c'hk = —

(c' = -^-is the phase velocity of light in the medium^, we obtain

a 1 , nk(, 1\
COSe=F + ^ ( 1 - ^)-
Radiation can take place when p« > 1 (that is, when the electron velocity v remains less
than the phase velocity of light in vacuum, but becomes greater than the velocity of light
in the medium c > v' > c'). In the classical case (ti = 0), the angle at which the photon is
emitted satisfies the relation

COSt) = S
s-/t .

The emission of radiation in vacuum is impossible because for n = 1 we have


c
cos 0 = “ » the velocity v cannot become greater than the velocity of light c.

6 S ee G. W en tzel, Quantum Theory o f F ie l d s , N e w York: I n t e r s c i e n c e , 1949; A. I.


A k h i y e z e r a n d V. B. B e r e s t e t s k i y , E le m e n ts of Quantum E le ctrodynam ic s (tra ns.)* L o n d o n :
O l d b o u m e P r e s s , 1962; N. N. B o g o l y u b o v a n d D. V. Shirk ov, Introduction to the Theory of
Q uantized F ie ld s ( t r a n s . ) , N ew York: I n t e r s c i e n c e , 1959.
Chapter 23

Theory of the Helium Atom Neglecting


Spin States

A. BASIC PRINCIPLES OF THE THEORY OF


MULTIELECTRON ATOMS

The helium atom is the sim plest multielectron atom; it con­


sists of two electrons moving about a nucleus of charge Z= 2. In
spite of its simplicity, this system exhibits several important
features characteristic of the many-body problem in quantum
mechanics.
In classical theory two electrons can always be identified by
subscripts 1 and 2, and the motion of each electron can be followed
separately. According to quantum theory two electrons can,
in practice, be distinguished from each other only when the dis­
tance between them is large. If electrons 1 and 2 are so close
to each other that there are points in space where their wave
functions are both simultaneously different from zero, then,
since electrons are identical particles, we shall be unable to
distinguish whether an electron occurring at the point is electron 1
or 2.
This indistinguishability or identity of particles is a special
feature of quantum theory. It gives rise to the so-called exchange
forces, which have no classical analog.
In multielectron atoms, spin properties are very important;
these properties are neglected in the classical and Bohr theories
and are taken into account only in quantum mechanics. In fact,
only in the case of a one-electron atom can the spin corrections be
neglected in the first approximation. This explains why the pre­
dictions of the Bohr theory are applicable only to hydrogen-like
atoms. The Bohr theory could not be extended to atoms with two
or more electrons, since it could not account for either the ex­
change forces or the spin states.
In order to investigate the basic features of the quantum theory
of the many-body problem, we shall consider in some detail the
problem of helium-like atoms (for example, neutral helium, singly
ionized Li+, doubly ionized Be++, and so on).
THEORY OF THE H EL IU M ATOM N E G L E C T IN G SP IN STATES 359

B. SOLUTION OF THE PROBLEM OF THE HELIUM ATOM


VIA METHODS OF PERTURBATION THEORY

To start with, let us determine the physical nature of the ex­


change forces which are related to the indistinguishability of
electrons. We shall neglect the spin properties of the p articles.1
We shall assume that the position of
the first and second electrons is given
by the position vectors /*i and r2, re­
spectively (their origin coincides with
the stationary nucleus; see Fig. 23.1). For
the sake of brevity, states with quantum
numbers (/t2, 11 , m2) and (n.2, /.2, m.t) will
be denoted by, respectively, «, and n., (in
this notation, therefore, n stands for the
whole set of quantum numbers). If there
were no interaction between the electrons,
we could determine the motion of each
electron separately from the Schrodinger F i g . 2 3 . 1 . T h e h e l i u m a to m .
equation

(£». — Hj) = 0, (23.1)


where

H; = T, + VV T, = = (23.2)

and the subscript j takes two values: / = 1 in the case of the first
electron, and / = 2 in the case of the second electron. From these
equations we can obtain the energy values Enj (see Chapter 13)

E = — m r' (23.3)
ni n-j

The eigenfunctions <i„. will be the same as the wave functions of a


hydrogen-like atom; they will satisfy the orthonormality condition

^*«j{r)</nj,(r)d»x = *n/l/- (23.4)

If we now take into account the interaction of the two electrons

V' = \r2—r-. | (23.5)


Fl!

*We a r e a l l o w e d t o d o t h i s b e c a u s e in t h i s a p p r o x i m a t i o n t h e p r o b l e m h a s a s o l u t i o n
in t h e fo rm o f a p r o d u c t o f tw o f u n c t i o n s , d e p e n d i n g o n t h e s p a t i a l a n d s p i n c o o r d i n a t e s ,
r e s p e c t i v e l y ( s e e C h a p t e r 2 4 ).
360 R E L A T IV IS T IC QUANTUM M EC H A N IC S

their motion can no longer be regarded as independent. The


Hamiltonian of the complete system will be

H = H1+ H, + y '= H " + y', (23.6)

and to describe this system we must take the Schrodinger equation


in the form
( £ - H' — F) *(/■!. rt) = 0, (23.7)
where E is the total energy, and <])(/*i, ra) is the total wave function,
which depends on the coordinates of both electrons. The quantity
i*(/*i, r2)^(/*i, ra) characterizes the probability of finding the first
electron at position o and the second electron at position r2.
Therefore, the normalization condition for !(/*!, /*a) takes the form

r 2)^ (rlt r.2) tfxt d3X i= 1, (23.8)

where the integration extends over the coordinates of both particles.


Since the exact solution of Eq. (23.7) entails insurmountable
difficulties, we shall use the perturbation theory2 developed in
Chapter 14, assuming that the mutual interaction of the electrons
causes only a sm all change in their individual motions in the
Coulomb field of the nucleus (the justification for this approxima­
tion will be examined in greater detail later).
We shall first consider the zeroth approximation, in which the
perturbation energy V can be neglected. The Schrodinger equation
(23.7) becomes
(£° — Hn)^°(r1, ra) = 0. (23.9)

Since the zero-order Hamiltonian H° can be resolved into a


sum of two Hamiltonians H, -f- Ha, each of which depends only on
a single variable (either r t or r2), it is obvious that in the zeroth
approximation the wave function may be written in the form (<]>“= «)

« = M n ) <!>»,(/•*)■ (23.i°)
Substituting (23.10) into (23.9) and using (23.1), we get

(£" - I I") a = {£" - (Hi + Ha)| (r,) ^ (ra) =


= Ehi — (ra) H,%, (/-,) + (r (/-a)} =
= E°u — {%., (ra) E„$,,, (/*i) -j- %i (fi) Enj!>ns (ra)} =
— {£" — (£„,+ Ena)} u = 0.

The problem of the h e liu m - li k e atom is a th ree-b o d y p roblem an d c a n n o t be s o lv e d


e x a c t l y , e v e n in t h e c l a s s i c a l a p p r o x i m a t i o n .
THEORY OF THE HELIUM AT O M NEGLECTING SPIN STATES 36

Hence, the energy in the zeroth approximation

(23.11)

where Eni and En., are the energies of the two electrons on the as­
sumption of no mutual interaction. This result can be explained as
follows. In the absence of the perturbation V', the motion of the
electrons is determined by their interaction with the nucleus Ze„;
that is, their motion is completely described by the Schrodinger
equation (23.1) which has the eigenvalues En. [see (23.3)] and
eigenfunctions •}‘nj. Since one of the electrons is in a state ih and
the second in a state n4, the total energy of the system is E„, -f- En,
when V' = 0.
Since the two electrons move independently of one another, the
total wave function will be the product of the corresponding two
independent one-electron wave functions obtained in the one-
electron problem. By direct substitution into Eq. (23.9) it is,
however, easy to show that, in addition to the first solution (23.10),
there will be a second solution (<]>0 = y) corresponding to the same
energy eigenvalue (23.11):
V= tym(f“i) 'i'ni (^l)‘ (23.12)
This solution differs from u by a permutation of the electrons. The
first electron is now in state n4 and the second electron is in
state «i.
This state of the system has, therefore, an additional degeneracy
which is due entirely to the indistinguishability of the electrons;
this is known as exchange degeneracy.
If both electrons are in identical states (n, = «.,), the wave
functions u and v are identical and there will be no exchange de­
generacy since
w^ u= (]<ni (/*i) <]<n. (/"c)). (23.12a)

In the case ni ^ n4, however, functions u and v are different, and


therefore the following linear combination should be taken as the
general solution <j>° of the Schrodinger equation (23.9):

^«= CIu4-Csi», (23.13)

where and C4 are arbitrary constant coefficients, which are


related only by the normalization condition

^(]i0 *(ji0 rf3^,d3x4= 1 .

In order to find the values of the coefficients Cj and C.2 and the
energy E of the perturbed system (the system with V' taken into
362 RELATIVISTIC QUANTUM MECHANICS

account), according to the perturbation theory, we shall look for


the solutions of £ and 'J in the form:

£ = £° + £';
^= t/>-{-f . (23.14)

We use a first approximation of the Schrodinger equation (23.7),


and write it in the form

(£° — H°) f = — (£' — V') (C,m+ Cjw). (23.15)

From Chapter 14, we recall that the solution of the homogeneous


equation for the unperturbed problem must be orthogonal to the
right-hand side of the inhomogeneous equation for the first-order
approximation of the wave function [see (14.13)]. Since the functions
u and v are solutions of the unperturbed problem, we have

J u* (£' — V') (Ciu + C2v) d% d3x2= 0, (23.16)

I (£' — V') (C,m+ C2v) d3xt d3x2== 0.


V* (23.17)

In (23.17), let us substitute r2 for rx and /”i for r2. The function
v [see (23.12)] then becomes u [see (23.10)], and the function v
becomes u , while the perturbation energy V' remains unchanged
since \r2— r2\ = \r2— r x\. Thus Eq. (23.17) takes the form

ij M* (£' _ V) (C2u + Cyv) d3x, d3Xi = 0. (23.17a)

It is, therefore, sufficient to consider only Eq. (23.16) since the


results can be extended to Eq. (23.17a) by substituting C, C2 and
C2— *■Ci.
Let us substitute into Eq. (23.16) the explicit expressions
(23.10) and (23.12) for u and v, and introduce the notation

(23.18)

^n, (/•.,) (r2) - - P22(r,), (23.19)

^ , ( / ' l ) ^ 2(/',) = Pl-2 (n), (23.20)

'i'nj ( P i ) 'r'ni t) = Pai (f s). (23.21)

Here Pn(rj) and P«(ra) characterize the probability density distri­


bution of electrons in the states rq and nt , respectively, whereas
T HEORY OF THE HELIUM ATOM NEGLECTING SPIN STATES 3G3

pn (/-,) and p.i (/".,) characterize the so-called density of mLxed (or
exchange) states , 3 when each electron is partially in state n, and
partially in state nt.
The orthonormality condition gives

^ u*u d3xt d*xt = ^ Pn (r,) d3xt ^ pj 8 (r2) d% = 1 ,

and

^ u*v d3xi d'x, = ^ pu (/-,) d3xl ^ p2, (r,) d3x.t = 0 ,

and therefore we can reduce (23.16) to the form

£'C, - {c.ej $ Pll|(rr; ^ ; a([a) ■cPXi d3x, +

+ C-M 5 Pl|(r? - r . (['a) d*x' d3je*} = °- (23> 22)

The first integral in Eq. (23.22) is simply the Coulomb interaction


of two “ smeared-out” electrons

K = el $ fl\iy_!^!]~ d 3xl d%. (23.23)

The second integral gives the so-called exchange energy

A = e\ ^ d3x, d3x.,, (23.24)

corresponding to the interaction of two electrons when each is in


a mixed state nx and n2. Unlike the Coulomb energy K, the ex­
change energy A has no classical analog; it is essentially a
quantum-mechanical concept.
Using Eqs. (23.23) and (23.24), we obtain instead of Eq. (23.22)
the following expression:

C, (£' — K) — C.,A = 0. (23.25)

From Eq. (23.25), we obtain the equation corresponding to (23.17a)


by substituting C8 -> C, and C, -> C,2 :

C* (£' — /() — C,,4 = 0 . (23.26)

T h e s e d e n s i t i e s h a v e no c l a s s i c a l a na l og.
364 RELATIVISTIC QUANTUM MECHANICS

Equations (23.25) and (23.26) give


1 ) E’= K + A, C, = C.z> (23.27)
and
2) E' = K — A, C, = — C2. (23.28)

Accordingly, we have two solutions for the wave function [see


(23.13)] and for the total energy
1 ) symmetric
«>s = C1(W+ y)( (23.29)
Es= £° + a; + A, (23.30)
and
2 ) ..antisymmetric4
^ = Ct (u — v), (23.31)
E3= £» + /< _ A, (23.32)

\Ve can determine the coefficient C, from the normalization


condition of the wave functions <}>s and i],a:

^ (l>*stysd:'xid:ix2= ^ (})*a(J)ac(3.K:1d3 X2 = 1 .

Hence, 2C[=1 or C, = —^ . Thus for <jjs and we finally have


V^

^ = 7 f (W+ U)’ (23.29a)

(23.31a)

When both electrons are in the same quantum state (nl = n.l ),
the functions u and v are identical. In this case Eqs. (23.16) and
(23.17) reduce to the same equation

^ u* (E'— V ^ u d ^ c P x ^ 0. (23.33)

From this it is readily seen that

E' = K, (23.34)

4 Wc r e c a l l t h a t u n d e r a p e r mu t a t i o n o f c o o r d i n a t e s ( t hat i s, wh e n a n d T2 a r e i n t e r ­
c h a n g e d ) t he f un c t i o n u a n d u t r a n s f o r m i nt o o n e anot her . T h e w a v e f un c t i o n d o e s not
c h a n g e s i g n a s a r e s u l t of t h i s o p e r a t i o n ( s y m m e t r i c f unc t i on) , w h e r e a s t he f unc t i on 0
r e v e r s e s i t s s i g n ( a n t i s y m m e t r i c f unct i on) .
THEORY OF THE HELIUM ATOM NEGLECTING SPIN STATES 365

and thus the exchange energy disappears in th iscase. For the wave
function we obtain a unique symmetric solution

$* = ,, = 1^ (23.35)

with the corresponding energy of the system

Es = E°-\-K. (23.36)

Summarizing the results of this section, we can make the fol­


lowing statement: Application of the perturbation method to the prob­
lem of the helium atom 5 leads to two types of solutions—a sym­
metric or antisymmetric solution. This is in complete agreement
with the general theory of system s of identical particles (see below).

C. COULOMB INTERACTION BETWEEN ELECTRONS

Let us find an expression for the Coulomb energy of two


electrons in the lowest energy state («i = n1 = l ) . In this case the
energy and the wave function of each electron are given by

Z*el , , v 1 (Z \ 3 / 3 e- fa»
1 (23.37)
Ex *l(r>= 7 rU r)
ft2
where °o— is the radius of the first Bohr orbit.
The Coulomb energy of interaction between the two electrons is

K= (n) (/-*) [7 7 = 1 7 7 1 (PxidW (23.38)

Here |rj — ri \= Vr\-\-r\ — 2 r2 cos t), and & is the angle between the
vectors and r To integrate (23.38), let us choose the direction
of the z axis along the direction of the vector rim Substituting the
expression (23.37) for the wave functions into Eq. (23.38) and in­
tegrating over the angles, we find 6

T h i s i s d u e to t h e f a c t t h a t the p e r t u r b a t i o n r e m o v e s t h e d e g e n e r a c y , a n d t h e r e f o r e
th e c o e f f i c i e n t s th a t w e r e i n d e t e r m i n a t e in t h e z e r o t h a p p r o x i m a t io n c a n now t a k e s p e c i f i c
v a l u e s . ( S e e a l s o C h a p t e r 14, t h e S tark e f f e c t . )

6 In i n t e g r a t i n g o v e r th e a n g l e t? (x c o s i9), w e u s e d th e r e l a t i o n

2
for r j < r 2 ,
d x _______ >~2
I
v ri > i - 2 ri'Y: —2 , r , >^ r2 .
for
(continued)
366 RELATIVISTIC QUANTUM MECHANICS

on yeas 2Zr i 5° __ IZr^


K= — ^ r\drie °» ^ r2e °» dr2. (23.38a)

Next, integrating over rx and rit we finally obtain


i s __5_ Ze\ (23.39)
8 a0

Since the zero-order energy is

£o = 2£, = — (23.40)
a0 ’
the total energy of two electrons in the ground state is equal to

£ = P + i( = - ^ i - f | z J (23.41)
it
Let us now find the ionization energy of the helium atom, that
is, the energy that must be expended in order to remove one
electron from the first orbit. For a singly ionized helium atom
(a hydrogen-like atom) the bonding energy between the electron and
the nucleus is simply £, [see (23.37)]. The ionization energy of a
helium-like atom is therefore
£ ‘ioiw ,£1_ £ = ^ -J z 2 — - Z ) , (23.42)

so that for helium (Z = 2) we have

£ lor = 0.75 —. (23.43)


ao
According to experimental data, the ionization energy of helium
is

£exp = 0 . 9 —■= 2 4 . 4 8 ev. (2 3 .4 3 a )

The discrepancy between the theoretical value and the experimental


data is a result of the fact that the perturbation energy K = 'j ~ o

is not very sm all as compared with the zero-order energy | £°| = ~


(their ratio is —1/3). Perturbation theory, therefore, gives us
C o n s id e rin g that the integrand ) ^ ? ( r 2) i s s y m m e t r i c w i t h r e s p e c t to t h e v a r i a b l e s
r | a n d £3 , w e r e p l a c e (for rj »
r 2) t h e r a d i u s £j by r 2 a n d v i c e v e r s a , o b t a i n i n g

for r» < r 2 .
r2

0 for r| > r 2 •

T h i s e x p r e s s i o n w a s u s e d in e v a l u a t i n g t h e i n t e g r a l ( 2 3 . 3 8 ) .
T HEORY OF T HE HELIUM AT O M NEGLECTING SPIN STATES 367

only qualitative aspects of the problem. The accuracy of the method


Is not very great because K and£"have the same order of magnitude.

D. THE VARIATIONAL METHOD


The variational method developed by Ritz, Hylleraas and others
was first successfully used to find the ground-state energy of
atomic system s, and in recent years ithasbeen applied in collision
theory.
As we know, the average energy of an atomic system is given by

E= (23.44)

If the wave function is represented as

* = .£Qpn, (23.44a)

where the coefficients Cn give the probability of an electron being


in state n, the average value of the energy will be given by the
relation [see (7.21)]

£ = 2 l C"l2£«- (23.45)
n
Replacing each eigenvalue En in the summation by the lowest
eigenvalue £ min and using the fact that for normalized functions

2 |C J 2 = 1 ,
n

we find that
£ min sg ^ (23.46)

that is, the lowest value of the integral ^ ty*K''?d3x can be used to
determine the upper limit of the ground-state energy of the system.
It was found that the variational method gives very good results
when the perturbation energy £' is of the same order as the energy
£ 0 of the zero-order approximation. The variational method can
therefore be used in cases where perturbation theory gives poor
results (for example, in calculation of the ground-state energy of
the helium atom). When a problem is solved by the variational
method, both the additional interaction V’ and the main interaction
are treated equally in the Hamiltonian H of Eq. (23.7). A test
function <Ji then depends on several parameters and is selected
in such a way that the integral can be calculated exactly. The
energy £ will then be a function of these parameters and it is
obvious that the minimum value of this function will be close to
the true value if the test function resem bles the true function.
368 RELATIVISTIC QUANTUM MECHANICS

The most difficult part of the method lies in the choice of the
best test function. All the available information on the properties
of the system must be used in making this choice. It is impossible
in the general case to indicate the form of the test function, and
therefore it is frequently necessary to rely on physical and
mathematical intuition. Very often, a test function is sim ilar in
form to the solution of the unperturbed equation.
We shall now use the variational method to calculate the ground-
state energy of the helium atom. Our procedure will be based on
that of Hylleraas (1927). At the end of the discussion we shall
compare both perturbation and variational methods.
For the test function, Hylleraas chose the ground-state function
(23.27) of the hydrogen atom, replacing the charge Z by a certain
effective charge Z'. The quantity Z is the unknown parameter which
has to be determined from the variational principle. The test
function

<23-4?>

will obviously be normalized to unity, just like (23.37), since its


normalization is independent of the value of Z'.
The Hamiltonian H in (23.46) must include both the zero-order
approximation Hamiltonian H# and the perturbation potential energy
V' . We thus obtain

H = T1 + l/ 1 + T2 + F 2 + F', (23.48)

where Tt and V j ( j = 1, 2) are given by (23.2) and the perturbation


potential energy F'is given by (23.5).
Since the wave functions are normalized and since Tj + Fi does
not depend on /* 2 and T2 -)-Va is independent of the coordinate r u
the average value of the Hamiltonian is

H = 2T1-\-2Vi -\-V', (23.49)

where

r , ) ( **>) ' Mr , (23.50)

- v7= -$< W (ri) (23.50a)

V' = J r, (/*.)«(/•*) [— 7^ d^ d 'x>- (23.50b)


THEORY OF T HE HELIUM ATOM NEGLECTING SPIN STATES 369

The integral (23.50b) agrees exactly with integral (23.38); therefore,


setting Z = Z’, we obtain in accordance with (23.39)

(23.51)
8 a0 ’

The quantity T, in Eq. (23.49) represents the average value of


the kinetic energy of a hydrogen-like atom with atomic number Z'
when the electron is in the lowest state. This value is related to
the total energy of the hydrogen-like atom by the well-known ex­
pression

T\ = — El = ^ ^ - . (23.52)

In exactly the same way, replacing Z'in (23.50a) by Z, we can ob­


tain the average value of the potential energy of a hydrogen-like
atom, since it is well known that the potential energy is twice the
total energy ( V = 2Et ). Consequently,

7, = -£ 2 £ 1 = — a0 (23. 53)

It follows from (23.49) that the average value of the energy is


given by the expression

E (Z') = O
—q
(Z,a — 2ZZ' —
o
Z'), (23.54)

which is a function of the parameter Z'.


Let us now find the value of the parameter Z' corresponding to
the minimum energy of the system . Differentiating E (Z') with re­
spect to Z' and setting the derivative equal to zero, we find

The minimum energy of electrons in the helium atom is therefore

£ min= _ ( Z _ 5 ) * l . (23.55)

For the ionization energy we have

£■« = £ , - £" ‘“= 5 , (Z‘ - 1 Z + * ) .

In particular, for helium (Z = 2) we obtain

el (23.56)
00
370 RELATIVISTIC QUANTUM MECHANICS

This result is considerably closer to the experimental value [see


(23.43a)] than (23.43) obtained from the perturbation theory.
Hylleraas later improved the agreement with the experiment by
using several variational parameters. The result (23.55) for £ min
has a simple physical interpretation, namely: the interaction between
the electrons results in a screening of the positive nuclear charge.
The variational method can also be used to find the upper energy
limit of one or several excited states. In this case the test function
has to be chosen in such a way that it is orthogonal to the wave
functions of all the lower states. When the energy levels are
arranged in order of increasing magnitudes (£„, £,, £ , . . . ) , it can
easily be shown that since <]» is orthogonal to functions tyo. 'J'i , <|>o, . .,
the corresponding expansion coefficientsC0, C,, C2f. . . areallequal
to zero. Therefore, in accordance with (23.45), the energy is
00

£ = 2 En\Cn\\ (23.57)
n=n0

where n0 is the quantum number of the given excited state. Using


the relation ^"l4==*’ we find that the minimum value of this
2
n
energy corresponds to the unknown energy E„0 of the excited state.
The application of this method to the calculation of the energies of
the highest excited states is rather difficult, because of the neces­
sity of introducing a large number of additional conditions to ensure
that the wave function of a given state will be orthogonal to the
wave functions of the lower states.

E. DERIVATION OF THE SCHRODINGER EQUATION


BY THE VARIATIONAL METHOD

We shall consider one of the most general forms of the variational problem, when
the choice of the test wave function 4' that is used to find the average value of the
Hamiltonian

E= (23.58)

describing the motion of a single particle is restricted only by the normalization con­
dition

^ d3x = 1. (23.59)

Upon varying E with respect to ^ and using the hermitlcity of the operator H, we ob­
tain

5E = \ (S^*H4/ + 51-H*>(/*)d,x = 0. (23.60)


T HEORY OF THE HELIUM ATOM NEGLECTING SPIN STATES 371

Here the variations H and cannot be regarded as Independent since they are con­
nected by the normalization condition (23.59). These variations can be made Independent
by varying the condition (23.59)

^ ^ d‘x = 0.

Multiplying this equation by a constant Lagrangian multiplier X, which is chosen In such


a way that the variations are now Independent, we add the resulting equation to (23.60).
Since the variations E'j' and are now arbitrary, the variational principle gives auto­
matically the Schrodinger equation for ^ and ■}*:

(H — £ H = 0, (H* — £)^* = 0, (23.61)


and the physical meaning of the parameter X becomes clear: It Is the negative of the
energy E (X= — E).
Consequently, the variational principle and the normalization condition together lead
to the Schrodinger equation. It Is apparent from the above results that the eigenvalues
of the Schrodinger equation (23.61) give the extremals of the variational Integral. A
more detailed analysis shows that these extremals are minima and that the energy of
the ground state corresponds to the absolute minimum—the smallest possible energy
value. In order to calculate excited states, it Is necessary to impose orthogonality
conditions on the wave functions of the lower energy state (as mentioned above), whereas
orthogonality follows automatically In the Schrodinger theory.

F. HARTREE-FOCK METHOD OF SELF-CONSISTENT


FIELDS
We have considered two extreme cases of the variational method.
In one case (the Ritz-Hylleraas method) the variation of the wave
function amounted to a determination of the “ best values” of
parameters in a specially chosen wave function. In the other case,
the choice of the wave functions was restricted only by the normal­
ization condition. This case led to the Schrodinger equation. There
is also an intermediate case in which the wave function is not
specified, but is assumed to be a product of one-electron functions
depending on the coordinates only. The specific form of these
functions is found by using the method of successive approxi­
mations to solve an equation derived on the basis of the variational
principle.
One such method was proposed by Hartree (1928). This method,
whose interpretation from the point of view of the variational
principle was given later by Fock, may be described as follows.
Let us start with the variational principle for two particles in
general form 7

E= (ru r.2) H<J>{ru r.z) d3x.2. (23.62)

As an additional condition, we require that the total wave function


should be a product of one-electron functions
'MOi ^’■
2 ) ==3t'i (o) ^ 2 (23.63)

7T h i s i n t e g r a l c a n b e g e n e r a l i z e d in a s i mi l a r f a s h i o n to t he c a s e of t h r e e or more
p ar ti cl es.
372 RELATIVISTIC QUANTUM MECHANICS

It is also necessary to allow for the normalization condition

$ (23.64)

which can be written for each particle separately

Substituting the test function (23.63) into (23.62) and varying


and 9 a» we obtain

^ [(^ 9 * 9 2 4 " 9*® 92) ( ^ 1 + H .j - f - — j 9i'Pa 4 ~


(23.65)
4 " 9 *9 * -(- h 2 -|- --°-j (89,<|>2 -(- 9iS9a) d3xld3xi = o,

where Y\j = y)-^--\-Vj(rj) is the Hamiltonian describing the


g2
motion of a single electron ( / = 1 , 2 ), and — represents the in­
teraction energy between the electrons.
From the normalization condition (23.64) we obtain a relation
between the variations

^ (09*9*9i92 4" 9*^9*9l9a “1“ 9*9*®9i9i 4“ 9*9*9i^92) d^Xyd^X^ = 0.

Multiplying this equation by the Lagrange multiplier X= — E and


adding it to (23.65), we can select X so that all the variations
S’p*. <4*, and so on, will be independent. Therefore, we obtain the
Hartree equations

(i J I, + 9 .H 4 A + J 9? — 9 - e) 9i= 0,
(23.66)
( h 4 4~ ^ 'PiHrPi^i “I- ^ 9? 9irf,1*i — 9-2= 0-

Analogous equations can also be obtained for the complex conjugate


functions. We multiply the first equation by 9 * and integrate it
over the whole coordinate space of the second particle; in a
sim ilar fashion, we multiply the second Hartree equation by 92 and
integrate it over the coordinate space of the first particle. Adding
the two resulting equations and dividing the total sum by 2 we
obtain the expression for the energy

C = V j- 9 ;iI/P/i%. + i y V 5 9W ^ - 9 ,9 (23.66a)
i
THEORY OF THE HELIUM ATOM NEGLECTING SPIN STATES 373

where in the case of two particles /, / ' = ! , 2. This equation can


also be used successfully for a larger number of particles.
If we neglect the interaction energy (that is, if we set the terms
containing —— equal to zero) and use the relations
rn

and
E j = J

Hartree’s equation can be resolved into a system of two inde­


pendent equations

(H; Ej) = 0

describing the motion of each particle individually.


In problems treated by the Hartree method, the field in which
each electron moves is composed of an external field (for example,
the nuclear field) and the field produced by all the other electrons.
This method is, therefore, known as the self-consistent field
method.
Fock (1930) generalized the Hartree method by taking into
account the exchange effects. For this purpose it is necessary
that the test function in the initial equation (23.62) be consistent
with the Pauli exclusion principle. The choice of test functions
is, therefore, further restricted by the requirement of anti­
symmetry (antisymmetric functions satisfying the exclusion prin­
ciple will be considered in greater detail in the next chapter).
Hartree’s system of equations (for example, for an electron shell
in an atom) is solved by the method of successive approximations.
First of all, the wave functions in the zeroth approximation (that
is, neglecting mutual interaction between the electrons) are deter­
mined. These wave functions together with the interaction potential
between the electrons are then used to obtain the first approxi­
mation equations. The first approximation wave function is sub­
stituted back into the Hartree-Fock equations to obtain the next
approximation and so on. This process is continued until the
solutions obtained in successive approximations are identical
(within the desired accuracy), that is, until the solutions are self-
consistent. Hartree’s system of equations can be solved only by
means of numerical methods of integration. With the help of
modern computers it is now possible to use Hartree’s method to
determine the energies and the wave functions for both light and
heavy atoms.
Another approximation which can be used in treating heavy
atoms is the Thomas-Fermi statistical method. Although this
method is not as accurate as the self-consistent field method,
3 74 RELATIVISTIC QUANTUM MECHANICS

it is comparatively simple and predicts many properties of multi-


electron atoms. This method will be used in our subsequent in­
vestigations, and we shall consider it in Chapters 25 and 26 in con­
nection with the theory of the periodic system of elements.

G. INVESTIGATION OF THE EXCHANGE ENERGY

Let us consider in somewhat greater detail the physical mean­


ing of the exchange energy (23.24), which represents the average
value of the Coulomb interaction between two electrons that are
both partly in state n, and partly in state n2. In accordance with
Eqs. (23.30) and (23.32), the total energy of the system is re­
lated to the Coulomb energy K and the exchange energy A by the
expression
E = E? + K ± A , (23.67)

where the plus sign corresponds to <]js , and the minus sign to i|»a.
In order to analyze the exchange energy in greater detail, we shall
examine the behavior of the system in time, taking into account the
exchange energy. The wave functions of the symmetric and anti­
symmetric states may be written as

t{is (t) = tyse * £ and ^{t) = ^e ^ £ (23.68)

Introducing the notation


(23.69)

Eq. (23.68) may be represented as

(23.70)

Let us consider a state of the system described by a super­


position of the solutions and <jA(f)
T(r„ r2> 0 = ^ (0 = ^ SU) + CV(0- (23.71)
It is easy to show that the function T (t) represents the general
solution of the Schrodinger equation (23.7) in the first-order
perturbation theory.
We shall now assume that at the initial instant of time (t = 0)
one. of the electrons is in state nlt and the other is in state «2.
Then the function

T (0) = -L r {(Cs+ Ca) u + (Cs - Ca) r} (23.72)


THEORY OF THE H ELIU M ATOM NEGLECTING SPIN STA TES 375

is simply equal to the function it. It follows that


~=r(Cs+ C ') = l.and Cs- C a= 0 ,

that is.

Cs= Ca= — (23. 73)

According to Eqs. (23.70), (23.71) and (23.73), at a certain later


time t, the function yY (t) becomes

if (l) = e~‘wC{u cos ot — iii sin o^} = e" 1*' {Cuu -|- Cuv], (23.74)

where

Cu= cosht, Cv = — isinB/. (23.75)

It is obvious that the amplitudes Cu and C„, which satisfy the


normalization condition

|C„ r + |C„ |*=1, (23.76)

characterize the probability of a system being in states described


by u and u, respectively.
Since at ( = 0 we have Cv = 0 and C„ = 1, the system is initially
in a state represented by the function u. However, after a time

~ (23.77)

(so that ht = -^ ), it can be seen from (23.75) that the coefficients


Cu and Cv, will have the values

Cu= 0 and Cv = —l.

At the time / = x,the state of the system will therefore no longer


be described by the function u, but instead by the function v. This
indicates that whereas at time f = 0 one of the electrons of the
system is in state nx and the other in state n.2, after a time A/ = ',
the first electron will be in state n2 and the second electron in
state /ij. The time - (23.77) during which the “ exchange” of
electronic states occurs is called the “ exchange tim e.” It is con­
nected with the exchange energy A by the simple relationship
376 RELATIVISTIC Q U A NT UM MECHANICS

It is readily seen that if there is no exchange energy (/1 = 0) then


X ------ C O .

To conclude this section, let us evaluate the exchange energy


when the first electron is in the state «i = ls, and the second
electron is in the state n.2 = 2s. The wave function ([>„, = <]>, is given
by (23.37) and the wave function can be found from (13.26)
and (13.28)8

1 Z_ 3/2 .
^ (r) I e (23.79)
2 f/2n (to

For the exchange energy, instead of (23.38), we obtain

A= J -MM |fi (r4) «[>>(/■>) d % d W (23.80)

Carrying out the integration in the same way as was done for the
evaluation of energy, we obtain, instead of (23.38a),
00

Zr\ - (3/2) Zro \ e - (3/2)


2a o
e. °» drl
2a0 )
a“ du.

Evaluating these integrals (which is a relatively simple process)


we find that

A. 2 *Zel (23.81)
3°a0

In accordance with (23.78), the exchange time of electrons in the


Is and 2s states in the helium atom (Z = 2) has the value

3
0 .8 - 1 0 ' 1 5 s e c .
w
The exchange time in the case where one of the electrons is in
the Is state and the other in the 1 0 s state is of the order of several
years, and there is practically no exchange. Consequently, it can
be seen that the exchange energy plays a significant part only
when the probability densities | ^ n ; | 2 of different states mutually
overlap to a considerable extent. When the overlapping of the
wave functions is insignificant, the exchange energy A is very
small. This situation somewhat resem bles the transfer of energy
from one coupled pendulum to another. It is well known that an
oscillating pendulum transfers its energy to another pendulum,

See also Problem 13.1.


THEORY OF T HE HELIUM ATOM NEGLECTING SPIN STATES 377

initially at rest, and its amplitude of oscillations after a certain


time interval becomes equal to zero. The exchange time of the
oscillatory energy depends to a great extent on the relationship
between the natural frequencies of oscillation of the two pen­
dulums; the time of exchange attains a maximum when the two
frequencies coincide (the case of resonance). It must be em­
phasized that this analogy is purely formal and is possible only
because of the wave properties manifested in both phenomena.
C hapter 2 4

Elementary Theory of Multielectron Atoms


Including Spin States

A. SYMMETRIC AND ANTISYMMETRIC STATES

As we pointed out at the beginning of Chapter 23, the quantum-


mechanical theory of assem blies of identical particles contains
several features that have no classical analog. The most im­
portant of these features arise from the fact that the state of a
system is unaffected by interchange of identical particles. This is
known as the principle of indistinguishability.
We shall consider here the general properties of a wave func­
tion that describes a system composed of two identical particles.
The state of the system is characterized by the position vector r,
three spatial quantum numbers n , I and m , and the spin quantum
number s. For the sake of brevity, we let n denote all three of the
quantum numbers /?, I and m. The wave function of the system
has the form

T (nts,r,; /2.,s.,r.>), (24.1)

where subscripts 1 and 2 refer to the first and second particles,


respectively.
Let us introduce an operator P which permutes the position
coordinates r x and rt or the quantum numbers s, and n.lt s.2

PT (/j,s,r,; «.2s.,r4) = 'F(/i,s,rJ; n^r,); (24.2)

It is easy to find the eigenvalues of this operator

PT (njSi/*,; /z.2siri) = XT (n1 s,r1; rhs^). (24.3)

It follows from (24.2) that two successive applications of P lead


to the initial wave function

PnF (/!is,r,; n1 s1/-2) = PT (^spv, n1 V 1) = 'Ir (/hspY, tusSi). (24.4)


T HEOR Y OF MULTI E LE CT RON A T O M S I NCLUDING S P I N S T A T E S 379

On the other hand, it follows from (24.3) that

PIlF (/Jjs,/-,; titStrt) — X1*^ (n,s,r,; n2s2r,). (24.5)

The eigenvalues of the permutation operator are therefore

X= =bl. (24.6)

This result means that interchanging the particles leaves the wave
function unchanged (symmetric wave functions, with k = 1 )

Ts (/ijii/'i; / w s) = Ufs (n^r*; n ^ri), (24.7)

or causes the wave function to reverse its sign (antisymmetric


wave functions with X= — D

Wa (n1 sir,; rc2 s2 r2) = — Ta (/qsj/y, n2 s2 r,). (24.8)

According to a principle of quantum mechanics, a set of


identical particles can only exist in states with a definite type of
symmetry. A state is either symmetric (described by a symmetric
wave function) or antisymmetric (described by an antisymmetric
wave function). Quantum transitions between symmetric and anti­
symmetric states are impossible; states with different types of
symmetry do not mix. This suggests that there are two kinds of
particles: one kind described by symmetric wave functions,
and the other by antisymmetric wave functions.

B. FERMI-DIRAC AND EINSTEIN-BOSE STATISTICS.


THE PAULI EXCLUSION PRINCIPLE
Experimental and theoretical investigations of the properties
of system s of identical particles have shown that there are two
kinds of particles with fundamentally different statistical prop­
erties, and that this difference is essentially related to the spin.
/ 1 3
Particles with half-integral spin = ... in units of the
Planck constant /i) obey Fermi-Dirac statistics. These particles
are known as fermions and include electrons, protons, neutrons,
U mesons and hyperons (all with spin */a). On the other hand, par­
ticles with integral spin (s = 0 , 1, ...) obey E instein-B osestatistics.
These particles are known as bosons and include tv mesons, K
mesons (both with spin 0 ), and photons (spin 1 ).
We may note here, without going into a detailed analysis of the
statistical properties of aggregates of particles, that in Einstein-
Bose statistics an unliminted number of particles can occupy each
state. On the contrary, in Fermi-Dirac statistics only a single
particle can occupy each state defined by four quantum numbers.
300 RELATIVISTIC QUANTUM MECHANICS

This characteristic feature of fermions was established empirically


by Pauli (1923) before the discovery of quantum mechanics and
quantum statistics; it is known as the Pauli exclusion principle.
In order to establish the relationship between the type of sym ­
metry of a state and the kind of statistics that is appropriate for
this state, let us consider a system of two particles which have a
negligible mutual interaction. The wave function for the system
can be written in the form of the product
T (n^ru nis.iri)_= <pBlSl (rt) %iSi (r9), (24.9)

where and are the wave functions of the individual parti­


cles. The solution (24.9) corresponds to a state in which the
particles with positions rx and r2 are in states characterized by the
quantum numbers n^i and n9 s9, respectively. Because of the
identity of the particles, the state of the system is not changed
when the particles are interchanged. The following solution is
therefore just as valid as (24.9):

W{n^ry, n,s,r9) = < ^ 3 (r,) (/*9) (24.10)

(that is , particles with coordinates /"i and r* are in the states n2 s.2
and 'h^i , respectively).
Both wave functions (24.9) and (24.10) describe the same
physical state of the system . We note that any linear combination
of these functions will also describe the same physical state. One
of the possible linear combinations is the sum of the solutions
(24.9) and (24.10), namely,

li's = y=- { W i (O) K.s, (r„) + W i (ri) (o) } (24.11)

(we have assumed that the functions and are mutually


orthogonal and normalized to unity, and therefore we have intro­
duced the normalization factor "jTf)* This solution is symmetric
with respect to interchange of the spatial and spin coordinates

xIfs (tz1s,/*1; nis.,ri) = Ws (riiSiri', n^r^). (24.12)

Another possible linear combination is the difference between


(24.9) and (24.10)

a I 'r'ni-Sl (ri) (f*2 ) 'Prti$i (r 2 ) 'PrtoSo (r l) }, (24.13)

which gives an antisymmetric solution since

vl ‘a ( a^ s ,/-,; n1s.2r1) = — ( h 2s .2/ ' 1; /i,s,r2). (24.14)


THEORY OF MU LT I E LE C TR O N A T O M S I NCLUDING S P I N S T A T E S 3 BI

From the form of lirs and T;‘ it follows directly that particles
obeying Fermi-Dirac statistics can be described only by anti­
symmetric solutions. The Pauli exclusion principle (Fermi sta­
tistics) is satisfied only by a solution of the form (24.13) since
when both particles are in the same state (that is, ni — tii — n and
s1 = s.i = s), we have

'l'a (/(sty, nsr.i) = Q, but lI;s (nsrx\ nsr*) 0. (24.15)

On the other hand,'fa does not satisfy the requirement of Einstein-


Bose statistics, in which an arbitrary number of particles can
occupy the same state.
It is easy to generalize the antisymmetric and symmetric wave
functions to the case of a system composed of N noninteracting
particles. The antisymmetric solution may be represented by the
determinant

'Nisi (r )1 'Nm (r-i) ... 'iWt ( r t:)


l|ra — ' W a fo ) W a fo ) ••• % ss3 (Ov) (24.16)
yw .

^nNsN ^nNsN * • ^nNsN

The antisymmetric nature of this solution is obvious since inter­


changing any two columns changes the sign of the determinant.
Moreover, it is obvious that the function (24.16) satisfies the Pauli
principle because if any of the sets of quantum numbers are equal
(for example, ni = nt, si = si) the determinant vanishes.
The symmetric wave function can be written in the form of a
sum of products

= y = {W , ('"i) IW. (ft) ■•• %N*tr (rn) +


+ <I'n! s3 ( r i ) ('•*)•■• % N > tf ( r * r ) + ...}■ (24.17)

C. ADDITION OF ANGULAR MOMENTA. RUSSELL-SAUNDERS


COUPLING. CLEBSCH-GORDAN COEFFICIENTS

Let us consider more fully the antisymmetric wave function


for a system composed of two particles

^"a = ~yy { (fi) 'Kusa (^2 ) — 'i'niii (r 2 ) (ri) }. (24.18)

In the many-body problem, we are faced with the question of how


to add the angular momenta. In the two-particle case, each
382 RELATIVISTIC QUANTUM MECHANICS

particle is characterized by orbital angular momentum quantum


numbers /, and I,, respectively, for which

L* = ***i (*i + 1 ) (24.19)

and spins and s.2> for which

S| = /S2Sl( S i + !)• (24.20)

There are two different ways in which these four quantities can be
added. One of these involves adding the orbital and spin angular
momenta separately and then finding their total sum

L = L1 + L, 1 S = S, + S2) J = L + S. (24.21)

This type of coupling of the particles in an atom is called LS


coupling or Russell-Saunders coupling. It corresponds to the case
in which the total orbital angular momentum and the total spin
angular fnomentum are conserved separately. This situation is
encountered most often in light elements.
The second way of combining the angular momenta involves the
addition of the orbital and spin angular momenta of each electron
individually and then finding the total angular momentum of all the
particles

J, = L, H- S,. J2 = L 2 + S2; J = J , + J2. (24.22)

This type of coupling is called jj coupling and is encountered mainly


in heavy elem ents. It corresponds to the case in which only the
total angular momentum is conserved, and therefore it is im­
portant when there is a strong spin-orbit interaction.
We shall now compare the order of magnitude of the Coulomb
and spin-orbit interactions in the helium atom. The Coulomb
energy [see (23.23)] has the form

K = e\ J P» (r»>p» y d3x->. ^ (24.23)

(T lies approximately between 0.1 and 1). The existence of LS


coupling depends on the Coulomb energy since the orbital and spin
angular momenta are conserved separately in this interaction. The
spin-orbit interaction for the helium atom

£s.o. _ L. S (24.24)
2 m ic* rJ

(a = '/in) is considerably sm aller than the Coulomb interaction, and


therefore the coupling is of the LS or Russell-Saunders type. It can
THE MULTI ELECTRON ATOMS INCLUDING SPIN STATES 383

be seen from (24.24) that the order of magnitude of the spin-orbit


interaction depends strongly on the nuclear charge z ( ~ Z ‘), so
that for large Z (heavy elements) the value of Es ° may be quite
large. In this case the coupling will be of the jj type.
Let us consider the problem of the addition of the orbital angular momenta of two
particles and find the function y la1 (1,2) which Is an eigenfunction of the operator for the
z component of the total angular momentum L = L2 L.

L , r iA1(1.2) = (Llz + L2z) YL M (\.2) = h M Y LM (1.2), (24.25)

and of the operator for the square of the total angular momentum

Lsy l m (1>2) = {Lf + L? + (Li* — 'Liy) (L,x + lLly) + (Lljc iL^) (L2jf— iL2>1) +
+ 2 L „u ,} YLMo.2) = n n y LM(),2y, \ = L ( L + d (24.26)

As we shall show, the eigenfunction ylm 0 ,2) can be written as a linear combination of
products

W 1.2)= 2 bmlmi YTl ' 0 ) Y?i (2). (24.27)


77Zj772g

where the wave functions K™1 (1) and Y™2 (2) refer to the first and second particles,
respectively, and

L„ C ( 1) = *",‘r'?(D: L2z ^ ( 2 ) = ^ ^ (2);


L f ) ^ I (l) = 7t, / i ( / , + l) y“ *(l); L\Y?*(2) = % *h(l,+ \) y£=(2). (24.28)
The numerical coefficients in the linear superposition (24.27) are called the vector
summation coefficients of Clebsch-Gordan coefficients.
For simplicity, we shall restrict ourselves to the case h = l2 = 1. Substituting (24.27)
into Eq. (24.25)

L* Y lm (1,2) = 2 + = (24.29)
77ij772<$
we find that M = fll| + '"sand = bm„.
Without loss of generality we may set 711= 0; that is, we make the z axis perpendic­
ular to the total angular momentum. Then it follows from (24.27) that
K£0(l,2) = b ^ Y \ (1) Lr‘ (2) + M V (1) Y\ (2) + (1) K° (2). (24.30)

Taking into account the equation for spherical harmonics [see (11.88)]

(Li* i fLij,) (1) = — h V(/, + l ± /«,)(/, + m,) ^ * ± ‘ (1), (24.31)

and using Eq. (24.26), we obtain

V Y lo(1,2) =- ft* {*_, [2KJ Vf1 + 2K» KJ] + ft, [2Kf‘ Y\ + 2 +


+ f-o [4KJ yj + 2Y\ + 2>V K{]} = X(ft_, yj yr* + ft, yr«yj + y»). {24.32)

This equation is equivalent to a system of three homogeneous equations for the unknown
Clebsch-Gordan coefficients

2f>0 + (2 — X) b, =0
2b0 + (2 — X) b_i = 0 (24.33)
(4 — X) bo + 2 bt +26_,=0.
384 RELATIVISTIC QUANTUM MECHANICS

This system has a nonvanishing solution if its determinant is equal to zero


2 2—X 0
2 0 2— X = 0. (24.34)
4— X 2 2

Calculating this determinant and subjecting the wave function YLM to the normalization
condition

J rfas = i,
that is.
bi + b\ + bU = \,

we obtain three possible solutions:

1) X= 0, = = -$„= — •
V*
From the conditions X= L (L + 1) we find that L = 0; that Is, this solution corresponds to
the case in which the vectors A and fa are antiparallel

A f J A.

2) X= 2, bt = 0, - b l = b_l = y = , L = \:

This case corresponds to the addition of the angular momentum vectors at an angle
of 60°

This solution corresponds to parallel A and A

A f t l».

We conclude, therefore, that two vectors A and A , whose absolute values are Integral
quantum numbers, can be oriented at angles such that the vector representing their sum
L is also characterized by integers. For A 55 l3 these values are

L = A + A, A + A — 1........A — A + 1, A — A (24.35)

(a total of 2A + 1 values).
This method of adding the angular momenta is based on a vector model of the atom.
We can obtain our previous results by setting A =A = 1 andM = 0. It is interesting to
note that the same values of the Clebsch-Gordan coefficients would also be obtained in
adding the orbital angular momentum to a spin angular momentum equal to unity (for
example, for photons). In this case, however, one would have to use the normalized spin
THEORY OF M ULTI ELEC TR O N A TO M S INCLUDING S P IN STATES 305

function Instead of the spherical harmonics I7™3. It Is easy lo show that the Clebsch-
Gordan coefficients will satisfy the following conditions:

V h L’M _ , V b L M b I. M _ ,
tn$ L

In general the Clebsch-Gordan coefficients bm^ will depend not only on the quantum
number m, = M — mlt but also on L and M. If the orbital angular momentum of the
second particle Is h = 1, then adding both orbital angular moments, we obtain
l
yLU= 2 i (2 ).
mo ——1

for which the Clebsch-Gordan coefficients are given in Table 24.1.

D. WAVE FUNCTION OF THE HELIUM ATOM


INCLUDING SPIN

We shall now consider more fully the wave function of the


helium atom, for which, as we know, the interaction between the
spins and the orbital angular momenta of the electrons is of the
Russell-Saunders type. Since, therefore, the orbital and spin
angular momenta are added independently of one another, the wave
function can be written as a product of two parts, one of which
depends on the spins of the particles and the other on their co­
ordinates. The wave function must be an antisymmetric function
with four quantum numbers

Wa = C (s„ s2)<|>ni„3(r,, r2) = — C(s2, s ,)^ 2ni(r„ rs) =


= — C(s„ s4) (/*, /*,). (24.36)

There are two possible cases: either the function is symmetric


with respect to the spins and antisymmetric with respect to the
coordinates, or the reverse case is true. We have therefore the
following two types of solution:

Ta = C - (s,, Sj) <(£,„„ (r,, /*), (24.37)


T a = C a (s1, s2) <$,„,(/*„ ra). (24.38)

We recall that the spatial part of the wave function has already
been obtained (see Chapter 23). For n i ^ n it

tim A fi. r 9) = y = ( u + v), (24.39)

tfmAri, n) = — (u — v).
(24.40)
366

Table of Clebsch-Gordan coefficients (for i2 = i)


RELATIVISTIC
QUANTUM
MECHANICS
THE MULTI ELECTRON ATOMS INCLUDING SPIN STATES 387

where
« =% .(''! ) ^ 3 (/"a).
v = tynl (ri)%1(r |)- (24.41)

Let us now investigate the spin part of the wave function. It has
already been mentioned that in Russell-Saunders coupling the spin
angular momenta are added independently of the orbital angular
momenta. The law of conservation of spin, therefore, provides a
basis for the construction of the spin wave function for the two-
electron system. We shall take the spin function of each electron
to be an eigenfunction of the operator for the z component of the
spin
q Tl
2 °3- (24.42)

and also of the operator for the square of the spin

Si = -j-(oi + a 2 + °3)‘ (24.43)

Here we write 2 x 2 Pauli matrices a (see Chapter 16) without


primes

Thus the spin function C= ( j of a single particle satisifies two


equations

(24.44)
s , c = i .,c = |.

s*c = + °j)(c‘) = ^ (cj). (24.45)

Since c | = l , and so on, we obtain from Eq. (24.45) X.2 3/4. The
matrix equation (24.44) for X, is equivalent to a system of two
homogeneous algebraic equations

ci(‘A - M = o ,
c-2 ('A + xi) = 0, (24.46)

from which it follows that there are two solutions corresponding to


the two possible orientations of the spin with respect to the z axis

1 ) X, — i/i, C\ — 1 , c2 — 0 . (24.47)
380 RELATIVISTIC QUANTUM MECHANICS

Here the spin is parallel to the z axis. The wave function corre­
sponding to the eigenvalues l/s is

(24.48)

2) (24.49)

In this case the spin is antiparallel to the z axis. The correspond­


ing wave function is

(24.50)

In Eqs. (24.48) and (24.50) the value of the z component of the


spin is indicated in the parentheses following the amplitude C.
It is easily shown that both spin parts of the wave function satisfy
the orthonormality condition. If, in accordance with the definition,
we take the Hermitian conjugate spin function to be a single-row
matrix
C+= (c^),

then from (24.48) and (24.50) we see that

C+ = 1,

and

= 0.

The effect of Pauli m atrices on the spin function, according to Eqs.


(24.50), (24.48) and (24.44), is

“C ± 7 = c - 7 ; =*=7 *
=:,C ± - = ± C . (24.51)

We shall look for the spin function C of the two electrons of the
helium atom in the form of a superposition

C=a,

-r-a.iC, (24.52)
THE MULTIELECTRON ATOMS INCLUDING S P IN STATES 389

where C, [i!r and are the spin functions of the first and
second electrons, respectively, while ax, a,, a, and are the
Clebsch-Gordan coefficients that have to be determined. Let us now
attempt to select the coefficients a^ of the function (24.52) in such
a way that the latter will be an eigenfunction of the operator for the
z component of the total spin

S ,— — "2 "(°a °a)> (24,53)

and also of the operator for the square of the total spin

S1= (S' - f S y = - {o'* + a" 1 + 2 a'- a"} =

= + i *'■*")• (24.54)

Here the primes and the double primes on the Pauli matrices
indicate that these matrices act on the spin functions of the
first and second electrons, respectively. We thus have two
matrix equations

4 (° ;+ ° i ) c = n i tc. (24.55)

n-''-y^a'.a")c=n% c. (24.56)

Substituting the total spin function (24.52) and taking into account
the effect of the Pauli matrices on the spin functions of the in­
dividual electrons [see (24.51)], we find from (24.55)

Equating the coefficients at the same spin functions on both sides


of the equation, we obtain the equations for the parameter X, and
the coefficients

M l — *i) = 0, M , = 0, a3Xj = 0, a4(I + X ,) = 0 . (24.57)


390 RELATIVISTIC quantum m echanics

Sim ilarly, using the relation


(»' a") C = a.C, ( I ) C.} ( 1 ) - a.2 [ c , ( i } c , ( - 1 ) -

— 2C, (— t ) C*( t ) ] — [C,( ~ t ) Ci( t ) “ 2Cl (t ) C’3(— t )] +


+ fl4C1( - i ) c 2 ( - l ) > (24.58)

which can be obtained with the aid of (24.51), we obtain from (24.56)
2a, = Xsa,, 2 a4 = X2 a4,
aj = (^s— l ) ai. a 2 = (X2 — l)aj. (24.59)
As usual we set X2 = i ’(6’+ 1 ) , where S is the eigenvalue of the
total spin. We note that the system s of equations (24.57) and
(24.59) must be satisfied simultaneously. From these equations
we find that the possible nonvanishing solutions have eigenvalues
X2 = 2,with X, = + 1, 0, — 1 (triplet),
X2 = 0, with Xj = 0 (singlet).
Consider now four cases. Case 1. Total spin is equal to unity and
is directed along the z axis. The spins of both particles are par­
allel. The corresponding solution, given by (24.52), is symmetric:

c‘ = c 1(t ) c *( t )> <24-60)


x2 ‘ 2 , = 1 ,
Xi = -}-1 , o3= a2= a4 = 0 .

Case 2. Total spin is equal to unity and is antiparallel to the z


axis. The spins of both particles are parallel. The corresponding
symmetric solution is

c* = c 1( - I ) c 2 ( - i ) , (m ^ ) , (2 4 -6 1 )
X.2 = 2, a4 — 1,
X, = — 1, al = a .1= ai = 0.

Case 3. Total spin is directed perpendicular to the z axis and is


equal to unity. The spins of both particles are parallel. The cor­
responding symmetric solution is

X, = 0. ai = a i = Q.
THE MULTIELECTRON ATOMS INCLUDING S P IN STATES 39

Case 4. Total spin is equal to zero. The spins of the particles are
antiparallel. The solution is antisymmetric:

(24.63)
c' =7f-(c' ( R H ) - c' R ) R ) f
k2= 0, a4 = —a;t = -— ,

X1= 0, a1= a i = 0.

We note that the choice of the nonvanishing Clebsch-Gordan co­


efficients was such that all four solutions are normalized to unity.
Returning once again to the spatial part of the wave function for
the helium atom and using the fact that the total solution must be
antisymmetric we have, in accordance with (24.37) and (24.39),

= C S (s„ s.2)^a (r„ n) (24.64)

(three states) and

W = C a (s„ s.2)<>s (/*,, r.2) (24.65)

(one state). The spin functions are given by Eqs. (24.60)-(24.63),


and the spatial part of the wave function for ^ n2 is

r= y= (u ±v). (24.66)

In the case where both electrons are in the same state («i — n2)»
there is only one solution with a symmetrical spatial part:

’= O (slt st) 6 s ; <i-s = u = R (n) ^Bl (/•„). (24.67)

E. PARAHELIUM AND ORTHOHELIUM

We have obtained the wave functions that describe two states.


One state (parahelium) is characterized by antiparallel spins of the
electrons [the wave function (24.65), which is symmetric with
respect to interchange of the coordinates]. The other state (ortho­
helium) corresponds to parallel spins [the wave function (24.64),
which is antisymmetric with respect to interchange of the co­
ordinates; see Fig. 24.1]. A very interesting property of these
states is that both types of helium are stable; in other words,
parahelium and orthohelium are not converted into one another.
We can convince ourselves that both system s are closed by direct
392 RELATIVISTIC QUANTUM MECHANICS

calculation. The matrix element of the dipole moment correspond­


ing to a dipole transition from orthohelium to parahelium

''s-a= j + ^2 ) (ritr.2) d3Xid3Xi =


= ^ 't'3 (n + / j) {r2,r,) d3x.!d3xi = (24.68)
= — ^ * s (/T/-,)(/-,+ /-,)'/ (/'i,r.2) d'Xid'x,,

turns out to be equal to zero, since

f s-a — r s-a — 0 . (24.69)

[In (24.68) we have changed the


variables of integration and used
the symmetry properties of the
wave functions.] Dipole transitions
from one state to another are
therefore forbidden.
F i g . 24.1. E l e c t r o n s p i n o r i e n t a t i o n Mutual conversion of these
in t h e h e l i u m atom . states may, however, take place
under the action of other parti­
c les. For example, the bombardment of orthohelium by electrons
may lead to the replacement of an ejected electron by another elec­
tron with opposite spin, and thus orthohelium can be transformed
into parahelium, and vice versa.

F. ENERGY SPECTRUM OF THE HELIUM ATOM

It has already been noted that the total orbital angular momentum
L, which is a result of the addition of the orbital angular mo­
menta li and of two electrons, assum es integral values (R ussell-
Saunders coupling). In the particular case where f, = f4 = 1 (both
electrons are in the p state), the total orbital angular momentum
takes the values L = 2, 1, 0. In terms of the vector model, these
values correspond to the following situations:
1. L = 2. The angular momenta are parallel;
fit t fa L = l i - \ - l - i = 2.

2. L = \ . The angular momenta are at an angle of 60°;


THE MULTIELECTRON ATOMS INCLUDING SPIN STATES 393

3. L= 0. The angular momenta are antiparallel;

/ifj/i L = ll —li = 0.

In the general case where /, 5 s /2, the number L takes all possible
integral values

I = /, + /* 1, + — 2.........l y - h - (24.70)
In denoting the energy levels for multielectron atoms, we follow
the same rules as for hydrogen-like atoms, except that the states
with a definite value of the orbital angular momentum L are denoted
by capital letters

L= 0 S state
L= 1 P state
L= 2 D state
L= 3 F state, and so on,

Let us enumerate the energy levels of the helium atom. The lowest
level corresponds to the total orbital angular momentum L = 0 with
both electrons in the s state. This singlet term of parahelium is
denoted by

(lsls)'So
and corresponds to an antiparallel spin orientation. The super­
script at the upper left-hand side of the letter characterizing a
particular term indicates the number of states (the multiplicity
of states).
The next term corresponds to the case when one of the electrons
is in the Is state and the other is in the 2s state. In this case, both
parahelium

(ls2s) *S0
and orthohelium

(ls2s) 3 S,

are possible. Orthohelium, unlike parahelium, has a spin angular


momentum and exhibits anomalous Zeeman splitting in a magnetic
field.
We note that the (ls2s)3 S 1 level of orthohelium is metastable,
because a transition to the lower energy state belonging to para­
helium is forbidden by the selection rules. The levels of para­
helium are singlets (spin 0 ), and those of orthohelium are triplets
(spin 1 ).
394 RELATIVISTIC QUANTUM MECHANICS

The splitting can be easily explained in terms of the vector


model of the orbital and spin angular momenta. According to the
vector model, the sum of the two vectors L and S (that is, the
total angular momentum vector)
f,e v can take the values
Parahelium •g Orthohellum
24 | | ^329 ,(/s2p)
22 Jfs 2fi) V ( I s 2p) 3P, J = L + S, L + S — l,
20,55. zilsMJh. ( t s Zp) % ... , \ t — 5 |. (24.71)
WT 20
M etastable '((s2s) 3S,
18 state For L^> S, the total number of
16 these values is
/4
12 2S+1, (24.72)
10
whereas for L < S it is
8
6 2L +1. (24.73)
4
2 It follows that for the S term
0 ■(/sis) % (L = 0), the total number of pos­
Ground sible states for both ortho­
level
helium and parahelium is equal
F i g . 2 4 .2 . E n e r g y l e v e l d i a g r a m for t h e
to one. Inexactly the same way,
h e l i u m ato m ( t h e s p l i t t i n g o f t h e 3P the s term of the hydrogen
l e v e l i s n o t in s c a l e ) . atom (one valence electron) is a
single, and the number of states
is equal to two (that is, the levels are doublets) only for p, d terms
(for which l ^> s).
The total spin S of helium assumes two values: 5 = 0,1. For
S = 0 the number of states (2 S + 1 ) is equal to unity, and for S = 1
it is equal to three (2 S -|-1 = 3 ); that is, the state is a triplet. A
general diagram of the energy levels of the helium atom is given
in Fig. 24.2.
For elements of the third group ( s = y and S = -i-)> the states
are quartets or doublets. Thus the number of valence electrons
(see Chapter 25) completely determines the splitting of the spectral
lines.
The theory of multielectron atoms is treated in several books. 1

Problem 24.1.2 A particle Is In the Yukawa potential well

^cc, f o r e x a m p l e , D. I. B l o k h i n l s e v , I ^ r i n c i p l c s o f Q u a n t u m M e c h a n i c s ( I r a n s .) , A llyn
and Bacon, 1964.

2 T h c s c p r o b l e m s r e f e r to C h a p t e r s 2 3 a n d 24.
THE MULTIELECTRON ATOMS INCLUDING SPIN STATES 395

Choose a test function In the form r and determine the upper lim it of the
values of z at which at least one discrete level will exist. Using the obtained solution,
consider the special case of a Coulomb potential (g‘ = ef„ z = 0). Show that discrete
levels always exist for the Coulomb potential. Determine the lowest energy level and the
corresponding wave function.
Solution. The normalized wave function Is of the form [see (23.47)]:

7i — —Yr.
!— . 0 a / 2 e. _p‘
1

The average values of the kinetic and potential energies are [see (23.52), (23.53)]

T= lX,il
2Wo ’ (2Pi + *)*
The appearance of the first discrete level Is possible under the conditions

dE i
E, = T + V = 0 , =0 .
from which we find

max jfs •

D iscrete levels exist for E <C 0. This happens when z < zmax. For a Coulomb field
z = 0 , and therefore this condition is always satisfied. The average energy in the Coulomb
case is

E, 2 m0 Pl*0-
Using the condition that must be minimum, we find

where o0 = — - r ls the radius of the firs t Bohr orbit. The expressions for the wave func-
m0<?5
tlon and the average energy will be exactly the same as those yielded by the Schrodlnger
equation for the Is level of the hydrogen atom (see Chapter 13).

Problem 24.2. Calculate the diamagnetic susceptibility of the ground state of one
gram -atom of parahelium , using the expression (23.47) obtained for the wave function by
the variational method.
Solution. In the ground state of parahelium , the orbital and spin angular momenta are
equal to zero (j = 0), and therefore parahelium must be diamagnetic (see Problem 20.1).
In accordance with (20.48), the diamagnetic susceptibility per gram -atom is given by
the expression
fi]N
6h/oCs Ol +rp.
(24.74)

where N Is Avogadro’s number and

r\ + r\ = ^ (rj + r|) | | (ru r s) |2 d \\\ d3x». (24.75)

From (23.47) we obtain an approximate expression for i (ru r«)

7-'~_ e— Z’{rl + r»)/a


~a[i
396 R E L A T IV IS T IC QUANTUM M E C H A N IC S

where for helium

Z'

Evaluating the integral (24.75) with the above expression for 4,» we obtain

—. — 2al
“1“^*2 £12 ■
The diamagnetic susceptibility Is therefore
ytheor — ] .67 • 10-“,

which is in fairly good agreem ent with the experimental value

XexP = — 1.90 • 10-“.


Chapter 25

Optical Spectra of A l k a l i Metals

A. GENERAL DISCUSSION OF THE STRUCTURE


OF COMPLEX ATOMS

It has already been pointed out that the position of an atom in


the periodic table of elements is determined by the atomic number
Z. This number characterizes the nuclear charge in units of e0;
that is, it is an integer that is equal to the number of protons in
the nucleus and also to the number of electrons in the neutral
atom. Thus, for example, sodium has 11 electrons ( Z = l l ) and
uranium has 92 (Z = 92).
The most important questions in the theory of a complex atom
concern the electron density distribution and the energy spectrum.
In studying these questions it is necessary to consider the mutual
interaction between all the electrons, in addition to the attraction
between the electrons and the nucleus. Owing to the interelectronic
interaction, the energy values of the electrons will be sm aller (in
absolute value) than those given by the well-known express ion

E= - ™ 1 I
r i-

Just as in the hydrogen atom, each electron in a complex atom


is characterized by four quantum numbers. For Russell-Saunders
coupling (the case where the spins and orbital angular momenta
of the individual electrons in a given shell are added separately),
these quantum numbers are as follows:
(1) the principal quantum number n = 1, 2, 3, 4, . . . ,
(2 ) the orbital angular momentum quantum number I = 0 , 1 , 2 ,
. . . , (/i — 1 ) .
(3) the magnetic quantum number m = 0 , ± 1 , . . . , ± /, and
(4) the spin quantum number m = ± 1 /2 (characterizing the
projection of the spin on the z axis).
In the case of jj coupling, the four quantum numbers are as
follows:
( 1 ) the principal quantum number n ,
( 2 ) the orbital angular momentum quantum number / ,
(3) the total angular momentum quantum number / = | / zb */s | , and
398 R E L A T IV IS T IC QUANTUM M EC H A N IC S

(4) mj = — j, — j-\- 1 . . . / — 1, / (characterizing the component


of the total angular momentum along the z axis).
As we have already said, Russell-SaUnders coupling (or LS
coupling) is characteristic of light elem ents, while jj coupling is
characteristic of heavy elem ents. Both types of coupling give the
same number of levels.
A group of closely spaced energy levels, which is separated
from other energy levels by an appreciable gap, forms a shell
made up of several subshells. Just as for the hydrogen atom, the
classification of the shells is based, as a rule, on the principal
quantum number ti. The relationship between the principal quantum
number n and the letters which are used to denote the shells in
x-ray spectroscopy is as follows:

n Shell symbol
1 K
2 L
3 M
4 N
5 0
6 P
7 Q

Within the shells, electrons with values of the orbital quantum


number / equal to 0, 1, 2, 3, . . . form the s, p, d, f, . . . sub­
sh ells. To determine how the shells and subshells are filled, one
should take into account the Pauli exclusion principle, according
to which there can be only one electron in each quantum state
characterized by four quantum numbers. In a state with fixed
values of n, I, m, there can be at most two electrons, differing in
spin direction. Since the quantum number m, which varies from
- / to +/, can take 2 1 + 1 values, we obtain the following expression
for the maximum number of electrons in a given subshell:

Nnl = Nl = 2(2l - f 1). (25.1)1

1T h e s a m e v a l u e fo r N [ i s f o u n d fo r j j c o u p l i n g . F o r g i v e n v a l u e s o f t h e t h r e e q u a n t u m
n u m b e rs n , I, j , t h e f o u r t h q u a n t u m n u m b e r n i j m a y t a k e t h e v a l u e s —j ............. —Vi , V i j,
c o r r e s p o n d i n g lo 2j + 1 s t a t e s . H e n c e w c o b t a i n t h e s a m e v a l u e fo r N /

N t f b = 2( 2/ f 1) .

w here
O P T IC A L SPECTRA OF ALKALI METALS 399

It follows that the maximum number of electrons in the subshells


s(/ = 0 ), p(l — 1 ), d(l — 2 ) and f(l = 3) is as follows:

Ns = 2, Np — 6 , Nd= 10, N , = 14.

Subshells with a larger value of / are not encountered in unexcited


atoms, and therefore we shall not discuss them here.
We can now find the maximum number of electrons which can
occupy a given shell
ri —I
yv„ = 2 N, = 2(1 + 3 + ... + (2n — l)) = 2n '■+ 2 ”~ 1= 2 n\ (25.2)
1= 0

Thus there can be at most two electrons in the K shell, eight


electrons in the L shell, eighteen electrons in the M shell, 32
electrons in the N shell, and so on. It should be mentioned that
these relations between the quantum number and the number of
states in the various shells are true, generally speaking, only for
hydrogen-like atoms.
To establish the order in which the shells and subshells are
filled in complex atoms, it is necessary to consider the mutual
interaction of the electrons. As we have emphasized several
tim es, this problem cannot be solved exactly. The application of
quantum mechanics to multielectron system s, however, has led to
the discovery of various important properties of such system s,
including, in particular, the exchange interaction. Moreover, the
development of approximation methods has made it possible to ob­
tain highly satisfactory quantitative results for complex atoms.
As indicated in Chapter 23, the simplest approximation methods
are the variational methods developed by Ritz, Hylleraas, and
others; these give comparatively good results for the light atoms
(up to potassium). A more complete analysis of the structure of
an atom can be made by means of the self-consistent field method
developed by Hartree and Fock. This method has been used to
determine the energy and the electron distribution in heavy and
light atoms (see Chapter 23). This method also gives the shell
structure of complex atoms . 2
Unfortunately, the use of the self-consistent field involves some
very complicated computations, which can be carried out only by
computers. As a result, the eigenfunctions characterizing the
electron distribution are obtained in the form of numerical tables
rather than analytical expressions.

2
F o r m ore d e t a i l s s e e H a r t r e e , D. R. The Calculation o f Atom ic Stru c tu r e s, N e w York:
J o h n Wiley & S o n s , I n c . , 1957.
400 R E L A T IV IS T IC QUANTUM M EC H A N IC S

B. THE THOMAS-FERMI STATISTICAL METHOD

Besides the approximation methods, ’there are statistical


methods that have been introduced on the basis of principles
developed by Thomas and Fermi. These methods apply mainly to
heavy atoms.
In the statistical treatment, the electrons of an atom are re­
garded as a degenerate gas with T = 0, just as in the theory of
metals (see Chapter 6 ). The Thomas-Fermi statistical method is,
of course, le ss accurate than the Hartree-Fock self-consistent
field method, because many features of the behavior of the in­
dividual electrons are neglected. In spite of this general short­
coming, the Thomas-Fermi method provides a fairly simple
explanation of many important properties which are characteristic
for the average behavior of electrons in an atom.
The Thomas-Fermi method does not enable us to find the
shell structure of an atom, but it does explain several important
features of the filling of the electron shells. This method can be
used fdr calculating the total bonding energy of electrons in an
atom and the radii of atoms and ions. It can also be used to deter­
mine the influence of screening electron shells on the scattering
of fast electrons, bremsstrahlung, and the creation of electron-
positron pairs due to absorption of x-ray photons in the nuclear
field.
Presently statistical methods are being successfully used to
construct a theory of heavy nuclei and a theory of matter at high
pressures (for example, inside stars).

In heavy atoms the positively charged nucleus is surrounded by a cloud of negatively


charged electrons, which partially screen the electric charge of the nucleus. The po­
tential of an ionized atom at distances g rea ter than Its radius Is given In the first
approximation by the following expression:

(Z — N) e„ (25.3)

where Z is the atomic number, and N is the number of electrons. F or a neutral atom
.V = Z and, therefore, <J>oo= 0 ; that is, the electrons completely screen the charge
of the nucleus.
In constructing a statistical theory, three forms of energy should be taken into
account.
1. The electrostatic energy of attraction between the electrons and the nucleus.
This energy is related to the electron density p0 (the number of electrons per unit
volume) by the expression

Fn- ca ^ po'I'n d3 (25.4)

where e = — e0 is the charge of the electron, and >hn = ^?'- is the potential produced by

the nucleus.
O P T IC A L SPECTRA OF ALKALI METALS 401

2. The electrostatic energy of repulsion between the electrons

I'c c = 2 ^ P°'*’e ‘Px, (25.4a)

where

*]*e (r) = — e0
i r r'
A’1.

3. The kinetic energy of the electrons in the atom. Just as In the theory of solids at
absolute zero, the average kinetic energy of an Individual electron Is related to the
electron density p0 by the following expression. In accordance with Eqs. (6.33) and
(6.33a)’:
7'av = xPo2/B> (25.4b)

where

x (25.5)

Hence the kinetic energy of the electrons Is

T = x ^ p0* h d ‘x. (25.6)

The total energy of an electron gas in the field of a nucleus Is equal to the sum of the
potential energy, which consists of two parts [see (25.4) and (25.4a)], and the kinetic
energy [see (25.6)]. Thus the total energy Is

E = T + V n_e + V e _e =
Po (r) po (/•') d3x d3x' (25.7)
= x ^ Po!s/i d3x — e0 ^ p0fI>n d3x + y e% ^
If —r' I
The density of the electron gas must satisfy the condition

^ p0 d3x = N, (25.8)

where N is the number of electrons In the atom.


With the auxiliary condition (25.8), the variational principle can be formulated as
follows:

5 [E + e0<W } = 0. (25.9)
Using this principle, we can find a relationship between the total potential 3>= <J>n -(- $ e
and the density of electrons p0

P o - ^ - ( 2 / n o f o (4>-<I>o))V2, (25.10)

T h e s e f o r m u l a s w e r e o b t a i n e d o n t h e a s s u m p t i o n th a t a t m o s t two e l e c t r o n s c a n o c c u p y
e a c h quantum s t a t e c h a r a c te r iz e d by th re e quantum n u m b ers. T h u s th e T h o m a s -F e rm i
s t a t i s t i c a l th e o ry a u to m a tic a ll y in c o r p o r a te s th e P a u l i e x c l u s i o n p rin c ip le , w h ic h is of
f u n d a m e n t a l i m p o r t a n c e in t h e t h e o r y o f c o m p l e x a t o m s .
402 RELATIVISTIC QUANTUM MECHANICS
where the Lagrangian m ultiplier 'b0, which plays the role of a constant potential, must
be found from the boundary conditions. In the derivation of (25.10), we have used the
relations

t ^ ^ pos/«5p0 d ’x,

(25.11)
» £l f Po(r) PoO <Pxd}x' =
2 1 \r — r'
[6po (r) po (r’)+po (r) 6p0 (r1)] 3* '
Ir-r'l
d9*; sj f e d‘x = lN.

Substituting Eq. (25.10) for the electron density into Poisson’s equation for a spheri­
cally sym m etric electron distribution

Vs3> = ~ = 4ti^oPo (25.12)

and recalling that 4>„ = const, we obtain the T hom as-F erm l equation which form s the
basis of the statistical model of the atom
*4

1 jjL r (cp _ 4>0) = ( 2 ( « - <J>„)V2. (25.13)

When a specific problem is investigated, the solution of Eq. (25.13) must satisfy
certaly definite boundary conditions. For an atom the boundary conditions can be given
in the form

4> — $o = ^ r for r —« 0 (25.14)


and r

- (Z — N ) e , t (25.15)
for r = r 0.
r0
H ere r 0 Is determ ined from the condition that the electron density should be equal to
zero at r = r 0 , that Is, po (r0) = 0. Hence, In accordance with (25.10), we find

N ) e °-. (25.16)
fo
W'ith the help of Poisson's equation (25.12) and (25.13), the condition (25.8) may be
w ritten as

^r d 'r ^ d r = Ne0. (25.17)


0
It follows from (25.16) that <I>0 = 0 and r 0 = oo for a neutral atom (N = Z ) . Instead of
(25.17) we get, therefore,

\f r ^d ' r<<T> i hr = Zel>


7 .
o
This condition will hold If the second boundary condition Is satisfied

lim r<I>= 0. (25.18)


r~ * oo
O P T IC A L SPECTRA OF ALKALI METALS 403

We note that the Thom as-Ferm i equation (25.13) has one exact solution
8l7t!/)' 1
<J>—<I'o r* ’
(25.19)

as can be easily shown by substituting (25.19) Into (25.13).


This solution for a neutral atom (<I>0 = 0) will satisfy one of the boundary conditions
for r — o o [see (25.18)]. The second boundary condition for r —- 0 [see (25.14)], however,
will not be satisfied.
Unfortunately, solutions of the Thom as-Ferm l equation which satisfy both boundary
conditions cannot be expressed In analytic fo rm .4
Substituting (25.14) Into (25.10), we find the behavior of the density Po as r — 0,

p0= const (25.20)

The solution (25.19) for a neutral atom gives too high a value for <I> as r — co ,
The m ore exact H artree-Fock method shows that the electron density will decrease
exponentially as r — oo .
Since we are Interested only In the basic principles of the statistical method, we shall
construct an approximate statistical theory of the atom with the help of the variational
method. This will enable us to formulate a solution of the problem In analytic form with
only sm all quantitative deviations which are of no Importance to us.

C. SOLUTION OF THE THOMAS-FERMI PROBLEM


BY THE RITZ VARIATIONAL METHOD

Solving a problem by the Rltz variational method, one can propose any number of
te st functions which depend on the different values of the variational param eter X.
We shall choose the test function on the basis of the following considerations. The
function should agree roughly with the solution of the Thom as-Ferm l equation for

We m a y n o t e t h a t t h e n u m e r i c a l i n t e g r a t i o n o f t h i s e q u a t i o n p r e s e n t s t w o a d v a n t a g e s
o v er th e n u m e ric a l in te g ra tio n of th e H a r tr e e - F o c k e q u a tio n s . F ir s t, th e T h o m a s -F e rm i
eq u atio n i s m uch s im p le r th an th e H a r tr e e - F o c k e q u a t io n s . S eco n d , th is e q u a t io n a n d the
b o u n d a r y c o n d i t i o n s ( f o r e x a m p l e , <I> = 0 f o r a n e u t r a l a t o m w i t h Z = N) c a n b e t r a n s f o r m e d
i n t o a u n i v e r s a l for m w h i c h i s i n d e p e n d e n t o f Z . T o d o t h i s , w e m u s t r e p l a c e O ( r ) b y a
new fu nction

With t h e f u n c t i o n f ( x ) , E q . ( 2 5 . 1 3 ) b e c o m e s

\/* - ~ Y = , (25.13a)
ax

and the b oundary c o n d itio n s (25 .1 4 ) a n d (2 5 .1 8 ) b eco m e

f (x) = l for x -> 0 , f (. x) = 0 for x -> 0 0 . (25.14a)

T h e s e e q u a t i o n s a r e o f a u n i v e r s a l c h a r a c t e r , th a t i s , th e y do n o t n e c e s s a r i l y d e p e n d on
the q u a n tity Z . T h e r e f o r e , a f t e r n u m e r ic a lly i n t e g r a t i n g th e T h o m a s - F e r m i e q u a t io n , w e
c a n c h a n g e t h e v a r i a b l e ( w h i c h d e p e n d s o n Z ) a n d u s e t h e e q u a t i o n to i n v e s t i g a t e a n y
h e a v y atom . T h i s c a n n o t b e d o n e w ith th e H a r t r e e - F o c k e q u a t io n .
404 R E L A T IV IS T IC QUANTUM M EC H A N IC S

r —» 0 (this region Is the m ost im portant with reg ard to the solution of the problem as a
whole), and It should have a comparatively simple form , so that It can be exactly in­
tegrated in the calculation of the total energy. One tes.t function which satisfied these
requirem ents is

Po =
c- / g (25.21)
1Gw 3/3

This function Is already norm alized to the total number of electrons


00

^ p0d 3x = ^ ^ ^ y~ r e ~ ^ Xrdr = N, (25.21a)

and therefore the auxiliary condition (25.8) will be satisfied automatically.


F or r — 0, the te st function (25.21) changes in the same way (p0 ~ r ~ 8/2) as the
solution of the T hom as-Ferm l equation [see (25.20)]. As we shall see la te r, dlls ex­
plains the good quantitative agreem ent between resu lts obtained by means of the test
function (25.21) and those obtained by means of the potential satisfying the Thom as-
Ferm l equation.
The potential due to the electrons in the atom is

<I>e = ~ — (1 - Vxr - Y l r e~ VXr). (25.22)

This can be easily shown by substituting Eqs. (25.21) and (25.22) for p0 and <J>e into the
equation

V 2O e = 4jce0p0.

Ze 0
Using also the expression 4>n we can see that the total potential satisfies the
r '
boundary condition (25.15) for r = r0—>oo, when the charge density vanishes together

with the exponential term e~ ' .


We can also find an expression for the kinetic energy T in term s of the variational
p aram eter X. From Eqs. (25.6) and (25.21) we have

g_5/aYlr
T = 4 t . v. (25.23)
■s V~r

We have the following expressions for the potential energy due to the interaction
between the nucleus and the electrons [see (25.4)] and fo r the potential due to the mutual
interaction of the electrons [see (25.4a)]:

' e- Y T r ZNe-k
ZNe- X’Va (25.24)
Vn - e
8
—7 =- dr ’
Vr 2

Fe-e I’ dr e~ VXr{ \ - e - Vlr- ykre~V^) =


8 oJ ^ r -
= /V^X (25.25)
16 ‘
O P T IC A L SPECTRA OF ALKALI METALS 405

Adding Eqs. (25.23)-(25.25), we find that the total energy (25.7) of the electron cloud Is

/; = /U 2— UK, (25.26)

where

— A W * * * (*-?-> l&27)
The variational param eter X, corresponding to the reciprocal of the effective radius
of the atom, can be found from the condition for the minimum of the total energy E of

the atom, that is, ^ = 0. Hence we find


OK

9 (S rA V a _ A ^ >
R e([ ~ X 100 \ 2 J (Z'_N\ (25.28)

Jl l 25
9 '31 r.JV/ 3 a0
^ 7^ Z
E= (25.29)
4/1

For a neutral atom (N = Z), we have

* e ff
(25.30)
E 2515 (2l \ 2 /3 £ i z 7/3= —0.758 ... —z 7 /a .
9 64 \3ji/ flo a«

It Is worth noting that numerical integration of the Thom as-Ferm i equation leads to a
value for the energy of the atom which Is very close to that given by (25.30)

E T"F = —0.769 .. .ii- Z7/» = —20.94 Z ?/s ev. (25.30a)


flo
The expression (25.30a) (with the sign reversed) represents the total bonding energy
(or ionization energy) W of a neutral atom, that is, the energy required to remove all
the electrons from the atom.
The theoretical values obtained for W from (25.30a) a re quite reasonable even In the
case of the hydrogen atom, but they are somewhat higher than the corresponding experi­
mental values. The relative e r r o r decreases with increasing Z (see Table 25.1).

Table 25.1

Theoretical and experimental values of the total ionization


energy W of atoms (in units of t?02/« o )

Element U7 theor U7exper

H 0.769 0.5
Li 9.982 7.5
Na 206.9 162
Hg 21207 18130
406 R E L A T IV IST IC Q U ANT UM M E C H A N IC S

Concluding this section, let us compare the curves which are obtained for the charge
distribution In neutral argon ( Z = 18) on the basis of the T hom as-Ferm l statistical
theory and on the basis of H artree’s method of the self-consistent field (see Fig, 25,1).
It can be seen from the graph that the curve of p0 computed according to the H artree
method (Curve B) has characteristic maxima and minima corresponding to the electron
shells, whereas the curve calculated according to the Thom as-Ferm l statistical theory
(Curve A) describes only the average behavior of the electron density, and therefore has
no relative maxima. F or large values of r there Is also a marked lack of agreement
between the two curves: the H artree-F ock method gives functions which decrease
exponentially with Increasing r, whereas In the statistical theory the decrease Is pro­
portional to r~ 4 [see (25.19)].5

F i g . 2 5 .1 . C o m p a r i s o n o f th e e l e c t r o n d e n s i t y d i s t r i ­
b u t i o n s in a r g o n ( Z = 18) o b t a i n e d on t h e b a s i s of t h e
T h o m a s - F e r m i m e t h o d ( C u r v e A) a n d t h e H a r t r e e - F o c k
m e th o d ( C u r v e B). A p s e u d o l o g a r i t h m i c s c a l e i s u s e d
for t h e o r d i n a t e a x i s (w e h a v e p l o t t e d t h e q u a n t i t y
Z n ( 1 + Dag), w h e r e D = 4 r rr 2 Pg). T h e r e f o r e , t h e g rap h
w i l l b e l i n e a r for s m a l l Da0, a n d l o g a r i t h m i c for l a r g e
Da0. T h e c h o i c e o f t h i s s c a l e e n a b l e s u s to fo llo w
th e v a r i a t i o n o f Pg a t l a r g e v a l u e s o f r, a s w e l l a s for
r < a Q.

D. ENERGY LEVELS OF ALKALI METAL ATOMS

In studying spectral lines in complex atoms, it is necessary to


distinguish between outer and inner shells.
In the hydrogen atom there is only an outer shell (the K shell),
which contains one electron. In helium (Z — 2), the K shell is
completely filled (a noble or inert gas). In litnium (an alkali metal
in Group I of the periodic table, Z — 3), the inner shell (K shell) is
completely filled, but the outer L shell has only one electron.
The filling of the L shell is completed in N e(Z =10). In sodium
(an alkali metal, (Z = 11), the inner K and L shells are completely

5 A d e t a i l e d e x p o s i t i o n o f th e s t a t i s t i c a l t h e o r y o f an ato m i s g i v e n in P . G o m b as , Die
sLnt.ist7sch(> T/ieorit* des A to m s und Hire A nw vndungvn, V i e n n a : S p r i n g e r - V e r l a g , 1949.
O PT IC A L SPECTRA OF ALKALI METALS 407

filled, but there is one electron in the outer M shell. The filling of
the shells in these atoms is illustrated in Fig. 25.2.
It should be noted that the bonding energy of an electron in an
inner shell is much greater that that of an electron in an outer
shell. An indication of this is provided by the ionization energy,
which is more than 2 0 ev for inertgases, whereas for alkali metals
it is only slightly more than 5 ev. The removal of the first valence
electron in lithium requires the expenditure of 5.39 ev, but re­
moval of the second and third electrons from the inner shells re­
quires the expenditure of 76 and 122 ev, respectively.

Fig. 25.2. D iagram of th e fillin g o f e le c tr o n


s h e l l s in d iff e r e n t a to m s . On th e rig h t, a to m s
w ith c o m p le te ly f ille d s h e l l s (in e rt g a s e s ) . T h e
dark s p o ts r e p r e s e n t e le c tr o n s a n d th e lig h t
s p o t s ( w i t h a p l u s s i g n in t h e m i d d l e ) r e p r e s e n t
nuclei.

The atoms in Group I of the periodic table (Li, Na, K, Rb, Cs,
etc.) are known as alkali metals; each has an outer shell containing
one electron, just like the hydrogen atom. Their optical and
chemical properties, therefore, should be essentially the same as
those of the hydrogen atom (for example, all of these elements are
monovalent and they all exhibit doublet splitting of the spectral
term s).
The optical spectrum is caused by the transition of a valence
electron (that is, an electron in the outer shell) from an excited
state to a lower state. The excitation of electrons in the inner
shells generally requires considerably more energy than does that
of electrons in the outer shells; therefore, downward transitions from
excited states to the ground states of the inner shell are accompanied
408 R E L A T IV IS T IC quantum m e c h a n ic s

by the em ission of x-rays (see Chapter 26). The atomic nucleus


and the electrons in the inner shells together form what is known
as the atomic core. Thus the charge of the atomic core is equal
to Z„ = Z — N, where N is the number of electrons in the inner
shells. For alkali metals (Li, Na, etc.) N = Z — 1, and thus the
charge of the atomic core is equal to unity (Z„ = 1). Therefore,
the main part of the potential energy which retains the outer
electron in the metal is the same as for hydrogen, namely.

V, = - - Z a = - - . (25.31)

The analysis of the spectra of alkali metals can thus be based on


the corresponding expression for the energy which was obtained
in Chapter 13

£ • = — ££. (25.32)

Similarly, we can use the hydrogen wave functions as the zero-


order approximation of the wave functions

(25.33)

In alkali m etals, however, we cannot confine ourselves to the


Coulomb energy in treating the interaction between the valence
electron and the atomic core, and we must also take into account
the polarization forces and the “ smearing” of the atomic core
over a finite volume. This yields various corrections and removes
the I degeneracy which occurs in hydrogen.
In Bohr’s sem iclassical theory, the orbits of valence electrons
were rigorously divided into orbits which penetrate the atomic
core, and those which do not. In the case of “ nonpenetrating”
orbits (orbits with nearly circular trajectories) we need to con­
sider only the polarization forces, since the potential outside the
atomic core (that is, outside the inner shells) is completely
independent of the radial distribution of a spherically symmetric
charge. The radial distribution is very important only for “ pen­
etrating” orbits (elongated ellipses) (see Fig. 25.3).
In quantum theory the concept of a trajectory is no longer
meaningful. In order to classify orbits as “ nonpenetrating” or
“ penetrating,” we must introduce a new convention; namely, if
the wave function describing the behavior of the valence electron
can be set equal to zero inside the atomic core, the orbit is
“ nonpenetrating,” and if on the contrary the wave function can­
not be set equal to zero inside the atomic core, the orbit is
“penetrating.”
O P T IC A L SPEC TR A OF ALKALI M ETA LS 409

It should be noted in this connection that the s* orbit of a complex


atom is always penetrating, since its wave function differs from
zero inside the atomic core and, indeed, even inside the nucleus

<25-34)

We shall now calculate the polarization forces that arise between


an outer electron and the atomic core. The outer electron will
obviously repel electrons in the inner shells and attract the nucleus.
As a consequence, the atomic core is polarized, and a polarization
force arises between it and the outer electron

1 1 I 2el (Z— \)x


(25.35)
Fv = - ( Z - \ ) e \ r- (r + x)-\ r3

The quantity eu(Z— 1)x — p represents the polarization of the


atomic core. N onpenetrating
If we regard the atomic core as o rb it

an elastic dipole, we can set

/>= ?&. (25.36)

where p is the polarizability of the


atom, and

&= p (25.37)
F ig . 2 5 .3 . “N o n p e n e tr a t in g * a n d " p e n e ­
t r a t i n g ” o r b i t s in a l k a l i m e t a l a t o m s .
is the electric field produced by the
outer electron at the center of the
atomic core. Thus we can obtain the following expression for the
potential energy associated with the polarization6:
ou ou

I/. ' = $ V ' = - S $r-$, ur


d r = __ 2 r1 (25.38)

Since the polarization can be regarded as a perturbation in this


problem, we have the following expression for the polarization
energy:

A£p = J tflmVp ifnlmcPx= - (1 ). (25.39)

T he co efficien t of p o larizatio n i s u s u a l l y d e t e r m i n e d fr om s e m i e m p i r i c a l f o r m u l a s .
I t s n u m e r i c a l v a l u e s f o r a l k a l i m e t a l i o n s ( c o r e s ) a r e a s f o l l o w s ( i n u n i t s o f 10
■24 3s
c m ):

0 . 0 3 ( L i + ); 0 . 1 9 ( N a + ); 0 . 8 9 ( K + ); 1 . 5 0 ( R b + ); 2 .6 0 (C s +) .
410 R E L A T IV IS T IC QUANTUM M EC H A N IC S

Since, according to (13.29a),


/(/+ !)
3/i*

«' (/-*/.)/(/+«/.)(/ + D ( / + |

relation (25.39) may be reduced to

AEP = __2a a n—
3' (25.40)

where

1 n*’
3P

(25.41)
S - W + U b
64 — 3 n* 6l’

We can, therefore, find the total energy, which in this case depends
upon both I and n (for the time being, we are neglecting spin
corrections)

— __ * ! L A - \ p
cP n/-- „2 ^ a cp-

Substituting expression (25.40) for A£pand using the relation

Rh_ el
n2 2a0n2 ’

we obtain

_ e l ___ el_ 26 ^ _______ el


2a0 n- 2aa /i3 ~ 2a0 (n — 6)2 ’
(25.42)

since

l _ 1 1 .2 6
(„ —6 )a n* ii J ~ n2 "T" n 3‘

Introducing the “ effective principal quantum number” ne{( = n — 8 ,


we get
O P T IC A L SPECTRA OF ALKALI METALS 411

It should be noted that Eq. (25.41) cannot be used for s states,


since the coefficient 8 , becomes infinite when 1= 0. This happens
because polarization forces can be introduced meaningfully only
in the case where the outer electron is at a sufficiently great
distance from the atomic core. For s terms the wave function does
not vanish even when r = 0 [see (25.34)].
The influence of the inner electrons on the s orbits (penetrating)
is due mainly to the “ smearing” of the electronic cloud of the atomic
core. In general, the additional energy due to the “ smearing” of
the electrons over the volume of the atomic core is given by

A£V01 = I 'M '012 K-Old'x, (2 5 .4 4 )

where FVoi is the difference between the potential energy produced


by the electrons of the atomic core, taking into account their
distribution over a finite volume, and the potential energy pro­
duced by an equivalent charge concentrated at the center.
To estimate the order of magnitude of the correction 8 for s
terms, let us assume that the Z — 1 electrons of the inner shells
fill uniformly a volume of radius R. We then have

+ (25.45)

Replacing the wave function for the s states by its value at the origin
[see (25.34)], we find the following approximate expression for the
additional energy of the s term s7:

2 ZelR- e% 28
A£vol ^ 5 ajjrc3 2 n8 ’ (25.46)
where
2 Z/?2
5 ai (25.47)
This expression no longer diverges.
7A s a r u l e , t h e c o r r e c t i o n S f o r p e n e t r a t i n g o r b i t s i s t a k e n i n t o a c c o u n t in t h e f o l l o w i n g
m a n n e r . It i s a s s u m e d t h a t t h e o r b i t c o n s i s t s o f t w o p a r t s : a n o u t e r p a r t a n d a n i n n e r o n e .
In t h e o u t e r p a r t o f t h e o r b i t t h e e l e c t r o n i s a c t e d u p o n b y a p o i n t c h a r g e c q Z b ( i n a l k a l i
m e t a l s Z a = 1); i n t h e i n n e r o r b i t , t h e e l e c t r o n e x p e r i e n c e s t h e e f f e c t o f p o i n t c h a r g e CqZ,-
w hich s h o u ld be g re a te r th a n C q Z b b e c a u s e th e s c r e e n i n g e ff e c t of th e in n e r e l e c tr o n s is
r e d u c e d . It is d if f ic u lt to d e te rm in e th e c h a r g e Z j , th e o r e tic a l ly , a n d it is u s u a l l y r e g a rd e d
a s an e m p iric a l p aram eter.
We t n u s o b t a i n t h e f o l l o w i n g e q u a t i o n f o r 8:
1
— Z; (/ + 1 ), (2 5 .4 3 )
a° ' V2^ Zf-'(/+ ) 1

w h e r e a i s t h e m a x i m u m d i s t a n c e fr om t h e n u c l e u s to t h e e l e c t r o n i n t h e i n n e r e l l i p s o i d
( t h a t i s , t h e r a d i u s o f t h e a t o m i c c o r e ) . I t i s e a s i l y s h o w n t h a t t h e q u a n t i t y 8; d e c r e a s e s
r a p i d l y w i t h i n c r e a s i n g I. A p a r t i c u l a r l y g o o d v a l u e o f i s o b t a i n e d for $ te rm s .
412 R E L A T IV IS T IC QUANTUM M EC H A N IC S

We may note here that according to the Thomas-Fermi model,


the radius of an atom [see (25.28)] is

R= ^ 7 ’ (25.48)
where the coefficient 7 , which characterizes the distribution of the
charge inside the atom, is of the order of unity.
Consequently, for the total energy of an electron in the case of
“ penetrating” s orbits, we again obtain an equation of the same
form as (25.42)
Rti ____ el
(n— h f ~ 2ao” eff (25.49)

where neii = n — 8 , but now 8 is given by (25.47).


In order to analyze the difference between the corrections
8 for “penetrating” and “ nonpenetrating” orbits, we shall take as
an example the Li atom.
In this atom the p orbit ( / = 1)is nonpenetrating. Equation (25.41)
gives the value 8o^0.04 for the lowest state (n = 2). According
to (25.47), the corresponding expression for 8 5 for penetrating s
orbits must be one order of magnitude greater.
It should be noted that for increasing n a n d / = const, the
eccentricity approaches unity; that is, the elliptical “ orbits”
become more and more elongated:
e*= i _ J£H) (25.50)
ns
In heavy nuclei, therefore, there will also be penetrating orbits
with 1 = 0 as well as those with larger values of I. This is re­
flected in the corrections 8 for the alkali metal spectra, whose
values are given in Table 25.2.

Table 25.2

Corrections s for the spectra of alkali metals

Element K
1 11 0 .0 0 0 0 .0 0 0 0 .0 0 0 0,000
3 Li 0.412* 0 .011 0.002 0 .0 0 0
11 Na 1.373* 0.883* 0 .010 0 .001
19 K 2.230* 1.770* 0.140* 0.007
37 Rb 3.195* 2.711* 1.233* 0.012
55 Cs 4.131* 3.649* 2.448* 0.022

T h e v a l u e s o f 8 for p e n e t r a t i n g o r b i t s are i n d i c a t e d by an a s t e r i s k .
O P T IC A L SPECTRA OF ALKALI METALS 413

It follows from Eq. (25.49) and also from the Table 25.2 that,
for a given n , the largest downward shift (that is, in the direction
of decreasing energy) associated with the “ smearing” of the inner
electrons over a finite volume occurs in states with the lowest /.
In other words, the largest shift occurs for s term s . 9
The hydrogen atom alone has no penetrating orbits. For the Li
atom (Z = 3; that is, the next element after hydrogen in the first
group of the periodic table), the only penetrating orbit is the outer
s orbit. In the next alkali metal, namely, the Na atom (Z = ll), the
s and p orbits are penetrating.

E. FUNDAMENTAL SERIES

The energy levels of the hydrogen atom without relativistic


corrections are given by the well-known relation

From this we can find the values of the spectral terms

(1 s) = ^ = R,

(2s) = (2p) = -§ = £ , (25.52)

(3s) = (3/>) = (3d) = -£ = 4 -

It follows that the states of hydrogen are degenerate with respect


to both I and m. A schematic diagram of the energy levels in the
hydrogen atom is given in Fig. 25.4.
In the Li atom, the energy levels of the K shell (n— 1) are filled
(see Fig. 25.2), and therefore it is the L shell which is the outer one.
Table 25.2 shows that the K shell exerts the strongest influence on
s term s, and for the corresponding terms we have
R R
(nS) = (n — 0,412)2 = (n— 1 +0.588)2'

This shift is so large that it was difficult to determine experi­


mentally whether it belonged to n states or n — 1 states. In order

^In C h a p t e r 21, w h e n t h e f i n i t e s i z e o f t h e n u c l e u s w a s t a k e n i n t o a c c o u n t , i t w a s a l s o
f o u n d t h a t t h e l a r g e s t s h i f t o c c u r s f o r s t e r m s . In t h a t c a s e , h o w e v e r , i t w a s t h e p o s i t i v e
c h arg e t h a t w a s s p r e a d o v e r a f in ite volum e, and th e re fo re th e te rm s w e re s h if te d u p w a rd s
and not dow nw ards.
414 R E L A T IV IS T IC QUANTUM M EC H A N IC S

to make the term notation resemble that of hydrogen, spectros-


copists have originally attributed the shift to the n — 1 sta te 1 0

(ns) = (n*s) = = {n/ + s y , (25.52a)

where n * = n — 1 and s = 1— 8 , = 0.588 . We shall use an asterisk


for the original notation (n*s) in order to distinguish it from the
correct one (ns) .

F ig. 25 .4 . E n e r g y - l e v e l d iag ram o f m o n o v a le n t


a t o m s . T h e p o t e n t i a l i s u s u a l l y m e a s u r e d (in
ev) fr o m t h e l o w e s t l e v e l u p w a r d s . H e r e ,
h o w e v e r , w e w i s h to c o m p a r e t h e e n e r g y
l e v e l s of d iff e r e n t a to m s and, th e re fo re , w e
have ta k e n th e p o te n tia l a t in fin ity as th e
zero level.

The shift of the other term s of lithium (I—1, 2) relative to the


corresponding term s of hydrogen is negligible
(np) = (n*p): R
(n + p)- ’
(nd) = (n*d) = - - ^ d f ,

1 0 If t h e p r i n c i p a l q u a n t u m n u m b e r n i n l i t h i u m a s s u m e s t h e v a l u e s n = 2, 3, 4 ( t h e t e r m
n = 1 i s o c c u p i e d by tw o e l e c t r o n s an d form s an in n e r s h e ll ) , th an th e q u an tu m n u m b e r n *
t a k e s t h e v a l u e s rt * = 1, 2, 3 - • • •
O PT IC A L SPECTRA OF ALKALI METALS 415

where

p = — = — 0.041,
d = — 8rf= —0.002.

The shell to which a shift belonged could be, therefore, uniquely


determined and in the old notation the p, d , and other terms were
placed in shells which were later shown to be theoretically correct
(that is, n* = n ; see Fig, 25.4),
In the next alkali metal, Na (Z = 11), the inner (filled) shells are
the K and L shells (see Fig, 25.2). As can be seen from Table 25.2,
the inner shells of Na have a pronounced influence on both the s
terms and the p term s. The original notation which was used for
the s terms was
R R
(■ns) = (ri*s)= 3 7 3 )2 = („* _|_0.627)2 ’(25.53)

where n * = n — 2, and s = —8^ 4-2= 0.627. Thus, spectroscopists


had originally assumed values of the principal quantum number for
the s terms of sodium which were low by two units.11
The original notation that was used for the p terms was

(nP) = (n*P)= —0.883) = (n* + p)a • (25.54)

where n * = n — 1, p = 1— 8p= 0.117. Thus, the principal quantum


number for the p terms was reduced by one unit. The corrections
for the states d, f, . . . were negligible, and these term s had been
assigned to the shells that were later obtained from the theory.
The energy-level diagram of Na is given in Fig. 25.4.
The spectral series of alkali metals are as follows.
1. The principal series. The variable term is the p (principal)
term. The spectral frequencies in this series are given by

m= (l*s) — (n*p).

which means

for H: (Is) — (up),


for Li :(2s) — (np), (25.55)
for Na:(3s) — (np).

11 At p r e s e n t , s p e c t r o s c o p i s t s h a v e a l s o a d o p t e d a n o t a t i o n w h i c h f o l l o w s fr o m t h e o r e t i -
c a l c a l c u l a t i o n s . T h i s i s t h e n o t a t i o n u s e d in F i g . 2 5 . 4 .
416 R E L A T IV IS T IC QUANTUM M EC H A N ICS

2. The sharp series. The variable term is the 5 (sharp) term.


The spectral frequencies are given by

u>— (2 *p) — (n*s),

which means

for H:(2p) — (ns),


for Li:(2p) — (ns), (25.56)
for Na:(3p) — (ns).

3. The diffuse series. The variable term is the d (diffuse) term.


The spectral frequencies are given by

“ = (2*p) — (n*d). (25.57)

4. The fundamental series.


cd= (3*d) — (n*f). (25.58)

The variable term is the / (fundamental) term.


These series take into account the selection rule

M = ± 1.
The names of the series partly reflect the character of their
multiplet structure.

F. MULTIPLET STRUCTURE OF THE SPECTRAL LINES

Just as in the hydrogen atom, the multiplet structure of the


spectral lines of alkali metals is due to the spin and relativistic
effects. To find the splitting of the term s, let us use a relation
including both the relativistic and spin-orbit corrections for a
hydrogen-like atom [see (20.18)]

\F.n!j (___ n _ ______ 3\ (25.59)


1i n' \j+ 'h A)’

where a = e’//zc= 1 137 is the fine-structure constant.


To account for the effect of electrons from the inner shells of al­
kali m etals, we simply replace Z by a certain effective value Zeff <^Z.

R a­ n (25.60)
h i l ' Z'ct
n 7+ ‘ 4
OPTICAL SPECTRA OF AL K A L I METALS 417

For “ nonpenetrating” orbits, we may set — 1, because all


the Z —l electrons will screen the positive charge of the nucleus.
For “ penetrating” orbits the best value of Zcff is chosen from a
comparison with experiment.

Aujt zr ‘•'Pl/2 4 s
'•Put 3?
3 P 3 /2
2 s
?P,/2
?PV2 Aivjl ?Pyt
Aui, ~T~ A tv,] AiVt |
2 P;/t
2 Put

i
/ s

Atv2 > A lv3 >A(Vt Aiv2 = A(Vj = d w .


Principal se rie s, con­ Sharp se rie s, equi­
verging doublets distant doublets

F i g . 2 5 .5 . S p e c t r a o f a l k a l i m e t a l s .

Since the total angular momentum quantum number / assumes


the values

1 for i = o
2

and

/ = f=t '/a fo r/^ 0 ,

we may conclude that all spectral terms of alkali metals should be


doublets, except for the s term, where there is no splitting.
In order to find the magnitude of the splitting, let us calculate
the value of the spectral term s for two cases: first, when the spin
and the orbital angular momentum are parallel

A£; = ' + >/; ^ ^ eff ( n _____ 3 \ (25.61)


ft n* \l + 1 4) ’

and, second, when they are antiparallel


*a,Zeff /« 3\ (25.62)
ft ~~ n4 \ I 4)•
418 R E L A T IV IS T IC QUANTUM M EC H A N ICS

The splitting of the term s is equal to the difference between (25.62)


and (25.61)

Au) «'/(/+!)• (25.63)

We shall use this equation to explain the doublet splitting of the


principal ser ie s, that is, transitions originating from the p levels.
Setting I — 1 we find

R°?Z'ei{ (25.64)
Ao>„
2nz

It is evident now that the decrease in splitting will be inversely pro­


portional to the cube of the principal quantum number n (see Fig.
25.5); that is, the spectral of the main series are narrowing doublets.
In the sharp ser ie s, the initial
level is a doublet and the final level
(with variable s) is a singlet (see
Fig. 25.5). The distance between the
doublets in the sharp series is con­
stant (equidistant doublets)

Au).2 = Aa)3 = . . . = Au)„ =


__
~ 16

In particular, for the spectrum of


Li we have Zeff = 1 , and«* — 2 (the
p orbit is “ nonpenetrating” ), so that
this constant splitting is equal to
F ig . 25.6. D iag ram of th e sp ectral
term s of the sodium atom including
th e fin e s tru c tu re . Ato,2 = Af«8
16 *

A diagram of the multiplet structure of the principal and sharp


series of sodium (Z = 11) is given in Fig. 25.6.
The diffuse series does not obey such an explicit law for the
splitting of spectral lines. Each line will be split into three rather
than four levels (see Fig. 25.7), since in this case in addition to
the selection rules A/ = :±] , it is also necessary to take into

12
S e v e r a l o t h e r fo r m u la s h a v e a l s o b e e n p r o p o s e d for th e d o u b l e t s p littin g . F o r e x a m p le ,
on th e b a s i s of t h e q u a s i - c l a s s i c a l p i c t u r e of p e n e t r a t i n g o r b it s , L a n d e p r o p o s e d a form ula
r e p l a c i n g in ( 2 5 . 6 3 ) ri b y n c ( f n ~ $ a n d ^ e f ( by Z,- , w h e r e Z a i s t h e t o t a l c h a r g e o f
t h e i o n a n d Z , i s t h e e f f e c t i v e c h a r g e o f t h e n u c l e u s in t h e i n n e r r e g i o n [ s e e E q . ( 2 5 . 4 3 ) 1
i n t o w h i c h t h e o r b i t p e n e t r a t e s . F o r f u r t h e r d e t a i l s w e r e f e r t h e r e a d e r to E . C o n d o n a n d
G. S h o r t l e y , T h e o r y o f A t o m i c S p e c t r a , N e w Y o r k : C a m b r i d g e U n i v e r s i t y P r e s s , 19 5 8 .
O PTIC A L SPEC TR A OF ALKALI METALS 419

account the selection rule for the total angular momentum quantum
number

A/ = 0, ± 1.

The spectral lines of the fundamental series are also split into
three components.

j n * d 5 /2

I n
I
I *
1 ' I ?P)/2
--------------- 1----------------fp,/2
F ig. 25.7. S p litting of the d iffu se
series.

The multiplet splitting of the spectral lines of monovalent atoms


may be explained only by taking into account the spin properties of
electrons. We have already and repeatedly stated that only the
half-integral quantum numbers (which characterize the spin) can
lead to doublet splitting of the terms (as in the Stern-Gerlach
experiments).
Chapter 2 6

Mendeleyev's Peri odi c System of Elements

A. X-RAY SPECTRA OF ATOMS

X-ray spectra provide important information on the structure


of the inner shells of atoms and are therefore useful in studying
the sequence in which the shells are filled by electrons. We recall
that x-rays are emitted when a beam of fast electrons strikes the
plate of a cathode-ray tube (see Fig. 26.1). An analysis of the
emerging x-rays shows that they consist of two different types of
spectra—a continuous and a line spectrum. The continuous x-ray
spectrum arises as a result of the deceleration of electrons when
they strike the target. The continuous spectrum is therefore a
type of bremsstrahlung. If the deceleration of electrons is equal
to then according to classical electrodynamics,the energy
radiated by the electrons per unit time is given by the relation
dE_ 2
dt 3 c3 '
A characteristic feature of the continuous spectrum is that it
is cut off at short wavelengths. The wavelength )■min at which this
occurs decreases as the potential difference between the cathode
and the anode increases; the cutoff wavelength 1min can be deter­
mined from the law of conservation of energy

where V = is the electron energy before colliding with the


target, '■!> is the potential difference between the anode and the
cathode, llw is the energy of the emitted bremsstrahlung photon,
and is the kinetic energy of the electron after colliding with
the target.
Introducing the wavelength instead of the frequency u>.
we find that
I ch
M E N D E L E Y E V 'S PE R IO D IC SYSTEM 42

From the last equation it is evident that X can vary from oo (as
in the case when an electron does not lose any energy during the
collision with the target^-/Huu'1= v ) to a certain minimum value

y ___ ch (26.1)
m in ~y >

as in the case when an electron loses all its energy in the collision

F i g . 2 6 .1 . S c h e m a t i c d iag ram o f an x -ray tu b e.


C — c a th o d e ; T — targ et.

The dependence of the radiation intensity on a is plotted in


Fig. 26.2 for two values of the energy of the primary electrons (25
and 50 kev). It is clearly seen that an increase in $ corresponds
to a decrease of the cutoff wave­
length I min , in accordance with Eq.
(26.1). This equation was successfully
used in the determination of a more
accurate value of Planck’s constant
h than was given by the Wien and the
Stefan-Boltzmann laws (see Chapter
1 ).
When the energy of the electrons
incident on the target exceeds a cer­
tain critical value, determined by the
target material, a line spectrum,
which is known as the characteristic
spectrum, is superposed on the con­ F i g . 2 6 .2 . Sh o rt w a v e l e n g t h l i m ­
tinuous spectrum. The line spectrum i t s o f t h e c o n t i n u o u s x -ra y
characterizes material of which the spectrum .
target is made (or rather, the struc­
ture of the inner shells of the target material) in the same way as
the optical spectrum characterizes the structure of the outer shells
of the atom. For example, in the case of a rhodium target, a line
422 R E L A T IV IS T IC QUANTUM M EC H A N IC S

spectrum begins to be superposed on the continuous spectrum at


an energy of 31.8 kev.
The properties of the line spectrum are identical for all chem­
ical compounds of a given element. The characteristic x-ray
spectrum and the optical spectrum are different in this respect,
since the latter depends on whether the substance occurs in atomic
or molecular state (for example, the optical spectra of atomic
oxygen, the Oa molecule, and the HaO molecule are completely
different). This can be readily understood since only electrons of
the outer orbits participate in chemical bonding.
The spectral lines of the characteristic x-ray spectrum form
regular sequences or se r ie s, just like the optical lines of atoms.
These series are designated by the capital letters K, L, M, N, . . . \
the K series has the shortest wavelength, followed by the L series,
and so on.

B. CHARACTERISTIC SPECTRA OF ATOMS AND THE


STRUCTURE OF THEIR INNER SHELLS

The mechanism which is responsible for the characteristic


x-ray spectrum of an element was first explained by Kossel (1914).
Let us suppose that an electron which is incident on a target re­
moves an electron from, for example, the /< shell of the target
atom, and thus leaves a vacant site in the /< shell (see Fig. 26.3).
An electron may jump from the L,
M, N, . . . shells to this vacant site,
giving rise to x-ray lines (denoted
by /<„, K,, /< •,,...). The character­
istic x-ray spectrum is thus formed
as a result of transitions of electrons
from one inner shell to another. Since
the bonding energy of electrons mov­
ing in the inner orbits is much
greater than that of the outer elec­
trons. Electrons of much greater
F i g . 2 6 .3 . S c h e m a t i c d i a g r a m of
energy (several tens of kev) are re­
th e o r i g in o f a c h a r a c t e r i s t i c quired for production of the charac­
x -ra y s p e c t r u m a c c o r d i n g to K o s ­ teristic x-ray spectrum than for the
sel: e le c tro n s. T h e d a s h e d line excitation of optical spectra (where
r e p r e s e n t s t h e e j e c t i o n o f an several tens of ev are sufficient).
e l e c t r o n from t h e K s h e l l .
Two methods that can be used in
constructing a theory of the com­
plex atom, while taking into account the interaction of atomic
electrons. In the first method the main potential is taken to be
the potential of the nucleus when it is completely screened by
the inner electrons. We used this method in constructing the theory
of the optical spectra of alkali metals. The potential was determined
M E N D E L E Y E V 'S PE R IO D IC SY ST E M 423

by the nuclear charge (Ze0) and the charge of the electrons in the
inner orbitals [— (Z — l)r0]. The total potential was equal to

<I>= ^ (Z — (Z — 1)) = - . (26.2)

As the perturbation potential we selected an additional potential


which took into account the polarization and the space distribution
of the electron cloud. This method is particularly suitable for outer
electrons, as, for example, in atoms of alkali metals.
In studying the motion of electrons in the inner shells, it is con­
venient to use the potential of the nucleus

®= (26.3)

as the main term in the expression for the potential, and to regard
the additional potential produced by the electron shell as a correc­
tion. In this case, the presence of electron shells will result in a
screening (an effective decrease) of the nuclear charge Ze0 by the
amount S„e0, and the total potential will be

Oz=(Z~ ^ ’i) e°. (26.4)

For example, in the investigation of helium-like atoms, it was


shown that the interaction of the electrons in the /( shell reduces
the effective nuclear charge [see (25.35)], which then becomes
Z' Z — —. Thus the quantity S„ is equal to “/16 in this case.
The screening constant S„ must be a function of both n and I .
It becomes larger as n increases since there is a greater number
of electrons screening the nucleus. It also becomes larger (but
more slowly) as I increases, since the orbits become less and less
penetrating and the effective charge decreases somewhat on the
average. In the first approximation it may be assumed that the
screening constant is independent of 1.
The potential (26.4) gives the same formula for the spectral
terms as was obtained for a hydrogen-like atom, except that the
quantity Z is replaced by Z — Sn:

En = - (— (26.5)

From (26.5) we can obtain an expression for the frequency of the


Ky line

(Z — S,)2 (Z — S2)2
n l2 22
(26.6)
-«[
424 R E L A T IV IS T IC QUANTUM M EC H A N IC S

It follows that the frequency of the x-ray spectral lines increases


monotonically as a function of the atomic number Z. This was
first deduced by Mosley (1914), from an analysis of empirical data;
he wrote this relationship in a somewhat different form

= (26.7)

This formula can be obtained from (26.6) if it is assumed that the


screening constant for K and L shells is the sam e, i.e ., S1= S.2= S.
We know, however, that this is not quite correct, and, therefore, in
studying x-ray spectra, just as for optical spectra, we should
express the frequencies as differences of spectral term s. In
accordance with (26.5), the spectral terms may be represented in
the form

’\Vf —
r =1 ^ ■ Z - S n
(26.8)
Vf — rh

This relation is called Moseley’s law; it is usually analyzed


graphically. Ascribing different values to the principal quantum
number n, we get (see Fig. 26.4):
for K terms (n — 1)
Z —Si (26.9a)
Y r
for L terms (n = 2)
1f 1 ? __ Z — S-2 . (26.9b)
V R2 ’
for M terms (n = 3)
1Y n _ Z - S 3
(26.9c)
V R ~ 3

Investigations of the experimental


curves have given the
F i g . 2 6 . -I. M o s e l e y d ia g r a m .
following average values for the
screening constants: S, = 1, S., = 3.5, S;, = 10.5.! It has also been
established the x-ray spectra change monotonically with increas­
ing Z and that no periodic regularities are observed. This repre­
sents a further difference between x-ray spectra and optical spectra,
in which periodicity exists (see Section C).

1S e e A. Som m erfeld, Alom huu und Spcclriillinicn I, V icw cg., H raunschw eip;, 1951,
C h a p t e r s 4 a n d 5.
M E N D E L E Y E V 'S P E R IO D IC S Y S T E M 425

Thus, periodic properties appear only in valence electrons, and


not in inner electrons. The study of x-ray spectra made it possible
to prove definitely that the atomic number Z, introduced by
Mendeleyev, is determined by the charge of the nucleus, and not by
its m ass. It turned out that Mendeleyev had correctly arranged the
elements Co—Ni, Ar—K, Te—I in a sequence which was not the
same as the order in which their atomic weight increased.
There were also doubts concerning the correct sequence of the
rare earths (elements from Z = 58 to 71), whose chemical properties
are very sim ilar. Moseley’s law made it possible to verify
their arrangement in the periodic system. In addition, the study
of x-ray spectra made it possible to determine the filling of
the inner shells of the ferromagnetic metals and lanthanides, where
the Moseley curve is discontinuous.

C. MULTIPLET STRUCTURE OF X-RAY SPECTRA

X-ray spectral terms are primarily determined by the quantum


state characterized by quantum numbers n, / and /, from which the
electron has been removed, leaving a “ hole.”
As a rule, LS coupling occurs for outer electrons, whereas jj
coupling occurs for inner electrons (in sufficiently heavy atoms).
In the heaviest atoms jj coupling begins to play an important role
for outer electrons as well.
Since the inner electrons are relatively close to the nucleus,
they are mainly under the influence of the field of the nucleus, and
therefore, their energy states are close to those of a hydrogen-like
atom. We may, therefore, take the following formula as a starting
point:

(26.10)

where the fine structure constant is

In order to account for the screening of neighboring electrons,


we must make the substitution Z->-Z — Sn( in (26.10). We assume
in this approximation that the screening constant depends not only
on n, but also on I. Then for the x-ray spectra we obtain

___^fj_ __(Z —Snl)' ( z - s n,y a-


R Rh n‘J
426 R E L A T IV IS T IC QUANTUM M E C H A N IC S

Taking the approximate square root we find a generalization of


M oseley’s law to the case that includes relativistic and spin
effects

Y Tjw = 4- i (z-~ y ga - -j. (26 .il)

This formula shows that in M oseley’s curves there appears a


term ~ Z a in addition to the term which is proportional to Z. The
influence of this additional term becomes marked only at large
values of Z. This conclusion is in agreement with experimental
data.
In addition, Eq. (26.11) explains the multiplet structure of x-ray
term s. We note, first of all, that there is no splitting of the K
term, since only one state (lsi/2) is possible (/i = l, 1 = 0 , j =
For L term s, we have three components: L\ ^ 2si /i(n = 2, I — 0,
y= 7s). f .n ~ 2 p./3(n = 2 , ( = 1 , j = 'U) and L,„ ~ 2p,,„ (n = 2, l = U
/=*/*)*
To make Eq. (26.11) agree with the experimental data for
screening constants, we must set

S is - 3, S ip = 4.

We obtain the following equations for the corresponding terms:

|f f (Ll terms);

\ f = + (Z ~ 4)3 \ ( U \ t e r m s ) ; (26.12)

I /^ 2P 3/„ Z —4 . 1 ..3 1
]/ — = — 2“ + J 6 (2 — 4) t e r m s ) .

These relations are represented graphically in Fig. 26.5.


In exactly the same way, it is easily shown that the M terms
contain five components

Mi (3si/s), M m(3pi/,,),
Mm (3/73/j), M[v(3d3/2), Mv(3ds/,,),

where the screening constants are S3s = 8.5, S3p= 10, S:w= 1 3 .
There are seven components for the N terms.
The parallel doublet L\ and Lu [see (26.12)], which is due to
the screening of the nucleus by the electrons is known as an irreg­
ular, doublet, whereas the diverging doublet L\ and LM is known
as a regular doublet. The reasons for the adoption of this terminology
M E N D E L E Y E V 'S PER IO D IC SYSTEM OF ELEMENTS 427

go back to the first stage of the theory of multiplet splitting of


x-ray spectra. It may be recalled that a theory of the fine structure
was first constructed by Sommerfeld, starting with a relativistic
generalization of the Bohr theory. Sommerfeld’s formula gave a
correct value for the splitting of the spectral terms (see Chapter 2),
but neglected the spin properties. For x-ray spectra this formula is
T ^ _ ( Z - S nnJ _ , ( Z - S nnJ ' ( n 3 ^
R ~ n -r 2 n‘ 4 )'
where nv — I -f- 1. Applying this equa­
tion to the analysis of the L term s,
it can be shown that these terms split
into only two components which corre­
spond to the diverging doublet U and
Lm in our notation. Thus, only this
doublet was explained in Sommer-
feld’s relativistic theory. Accord­
ingly, it was called the regular
doublet. The doublet L\ and Lu , for
which no theoretical explanation was
given for a long time, was called the
irregular doublet. With the advent of
the Dirac theory, the irregular doublet
F ig . 2 6 .5 . M u ltip let s tru c tu re of
also found an explanation. Therefore,
th e x-ray L term s.
the reasons for the adoption of this
terminology are entirely historical (just as for the “ normal” and
“ anomalous” Zeeman effect). The correct theory of the multiplet
structure of x-ray spectra (and of the anomalous Zeeman effect)
was developed much later, when the spin properties of electrons
were taken into account.

D. THE DISCOVERY OF MENDELEYEV’S PERIODIC LAW

Upon arranging the known elements in order of increasing


atomic weight, Mendeleyev discovered that various chemical
properties tend to recur quasi-periodically. For example, the
chemical properties of sodium, potassium and other alkali metals
are sim ilar to those of lithium, and the chemical properties of
chlorine, bromine, iodine and so on, are sim ilar to those of
fluorine.
Mendeleyev ascribed to each element a number 1 (the atomic
number) giving its position in the periodic system . Although the
increase of Z is for the most part parallel to the increase of the
mass number of the elem ents, there are several exceptions—for
instance, (, 7 CO—2 8 Ni), ( 1 9 Ar—1 3 K), (3 2 Te—3 JI)—where the element
with the sm aller atomic number has a larger atomic weight.
428 R E L A T IV IS T IC QUANTUM M E C H A N IC S

Moreover, as we now know, there exists a large number of isotopes,


that is, atoms having the same Z, but different m asses (for ex­
ample, iH\ ,D2, ,T').
The periodic law has acquired particular significance in con­
nection with recent discoveries concerning atomic and nuclear
structure. In particular, the study of x-ray spectra and experi­
ments on the scattering of a particles by atoms have established
that the atomic number Z characterizes the charge of the nucleus
(and the number of electrons in the neutral atom).
Sixty-three elements were known when Mendeleyev discovered
the periodic law (1869). He predicted the existence of ten more
elem ents, and even described the basic chemical and physical
properties of three elements which were subsequently discovered,
namely, scandium (ojSc), gallium (^Ga) and germanium (;>iGe). The
inert (noble) gases were discovered at the end of the 19th century.
In Mendeleyev’s tim e, only three elements from the rare earth
group (lanthanides) were known: cerium, didymium (a mixture of
praseodymium and neodymium) and erbium. At the present time
the properties of all fourteen rare-earth elements have been
investigated.
By 1937, ninety-two elements were known, but four of these had
not yet been observed. It was later found that these four elements
were radioactive and virtually nonexistent in nature. They were
produced in the laboratory as a result of nuclear reactions.
In 1937, E. Segre produced an element with Z = 43 , called tech­
netium, by neutron bombardment of molybdenum. The half-life of
its most stable isotope, 4 3 Tc” , was found to equal 2.6 x 10“ years.
In 1938, it was first reported that an isotope of the last rare-
earth element with Z = 61 had been produced as a result of deuteron
bombardment of neodymium. The half-life of the most stable
isotope of this element, ciPm143, is about 2 0 years.
In 1940, E. Segre discovered an element with Z = 85 by irradi­
ating bismuth with a particles. He called this element astatine. The
half-life of its most stable isotope, ssAt'210, is 8.3 hours.
A short-lived element with Z = 87, called francium, was dis­
covered in 1939 by a Frenchwoman, Mile. Perey. This element is
produced in the a decay of 8 3 Ac'2-7. The half-life of its most stable
isotope, S7 Fr--:i, is equal to 22 minutes.
And finally, we must mention that with the development of
nuclear physics, it has become possible to produce transuranium
elem ents. These range from neptunium (Z = 93 ) to lawrencium
(Z = 1 0 3 ). 2 Thus,the periodic system now consists of 103 elements
without any intervening gaps.

T h e l a r g e s t n u m b e r o f t r a n s u r a n i u m e l e m e n t s h a s b e e n d i s c o v e r e d by G. S e a b o r g a n d
h is stu d en ts. The disco v ery of la w re n ciu m (L w ) w ith Z 10 3 w a s r e c e n t l y a n n o u n c e d .
M E N D E L E Y E V 'S P ER IO D IC SYSTEM OF ELEMENTS 429

E. FILLING OF THE ELECTRON SHELLS

In quantum mechanics, the levels of the electron shells are


filled in accordance with the following rules.
(a) There can be at most one electron in each quantum state
(Pauli’s exclusion principle), and therefore the maximum number of
electrons with a given / is equal to 2 (2 /4 -0 (see Chapter 25).
Thus the s, p, cl and f subshells can contain no more than 2, 6 , 10
and 14 electrons, respectively.
(b) Electrons tend to occupy the lowest energy levels. There­
fore, the shells w ith n = 1 will be filled first, then those with n = 2 ,
n = 3, and so on.
The shells would be filled in this way in an ideal scheme in
which the wave function of an atomic electron could be calculated
on the basis of the assumption that the charge of the nucleus and
the charges of the Z — f electrons are all located at the center. In
this case the energy levels of the remaining electron would be the
same as in the hydrogen atom, and therefore they would be degen­
erate with respect to /. As was shown in the investigation of alkali
metals, however, the distribution of the electrons over a finite
volume removes the / degeneracy, so that terms with a fixed value
of the principal quantum number n (that is, the terms in a specific
shell) are arranged in order of increasing I. The s term is there­
fore filled first, then the p term, and finally, the d term.
Moreover, the 4s subshell is located below the 3d subshell (and
5s is below 4d), while the 6 s subshell is below both the 5d subshell
and the 4/ subshell (sim ilarly, 7s is below 5/). It turns out that the
outer shell (in an unexcited atom) can consist only of s and p sub­
shells. The d and / subshells can be filled when they lie in the
first or second inner shell, respectively (the first inner shell is
taken to be the shell directly adjacent to the outer shell)? We
shall make an attempt to substantiate this by an investigation of
the ground-state configuration of the electrons in individual atoms
(see Fig. 26.6).

T h e o r d e r in w h i c h t h e e l e c t r o n s u b s h e l l s a r e f o r m e d c a n b e r e m e m b e r e d m o s t s i m p l y
w i t h t h e h e l p o f t h e f o l l o w i n g e m p i r i c a l r u l e : t h e l e v e l s a r e f i l l e d in t h e o r d e r in w h i c h t h e
s u m o f t h e p r i n c i p a l a n d o r b i t a l q u a n t u m n u m b e r s , n + I, i n c r e a s e s , an d l e v e ls w ith th e
s a m e v a l u e o f t h i s s u m a r e f i l l e d i n o r d e r o f i n c r e a s i n g n. S i n c e I t a k e s t h e v a l u e s
0, 1, 2, n — 1, w e c a n f i n d t h e r u l e f o r f i l l i n g t h e t e r m s in a n y s h e l l . F o r e x a m p l e , t h e
fourth p e rio d ( s e e b e lo w ) w ill b e f i l l e d in th e o rd e r

4 s (n + / = 4); 3d(n + 1= 5); 4 p ( n 4 / - 5)

a n d t h e s i x t h p e r i o d in t h e o r d e r

6 s ( n 4 / = 6); 4f ( n + I = 7);

5d { n 4 1 = 7); 6 p ( n + I = 7).
430 R E L A T IV IST IC QUANTUM M E C H A N IC S

Within the first and second periods of Mendeleyev’s system ,


the order in which the levels are filled conforms to the sequence
of levels in the hydrogen atom (the ideal scheme). If this ideal

112-118
-7p
104*-112*
rtTh■ i No,103
-6 a
oAc
—5f Q (32)
-6 a
i Fr - s»Pa
-7s
, T l -..Em
-6p
7?Nf -joHg F ig . 26.6. Diagram of th e fillin g of energy
-5a l e v e l s in t h e p e r i o d i c s y s t e m o f e l e m e n t s .
a CP 7# I—Li
-Uf(») P (32) O n ly s a n d p s u b s h e l l s c a n b e p r e s e n t in th e
7La
-5a o u t e r s h e l l of an a to m . A d s u b s h e l l c a n b e
f i l l e d o n ly b e g i n n i n g w ith t h e f i r s t i n n e r
•6s s h e l l . An f s u b s h e l l c a n b e f i l l e d o n l y b e ­
g in c.X 6
g i n n i n g w ith t h e s e c o n d i n n e r s h e l l . T h e
-5 p f i l l i n g o f t h e 3c? s u b s h e l l g i v e s t h e f e rr o ­
-Ua 0 (18) m a g n e t i c e l e m e n t s ( F e , C o , N i). T h e fil­
l i n g o f t h e 4f s u b s h e l l g i v e s t h e l a n t h a ­
-5s n id e s or rare e a r th s (5 8 C e — 7 iL u ) . T h e
36^r f i l l i n g o f t h e 5f s u b s h e l l g i v e s t h e a c t i n i d e s
-4p ( 9 0T h — 103). T h e a s t e r i s k s d e n o t e t h e a to m ­
ic num bers of e le m e n ts w hich h a v e n o t y e t
-3a(t(/)\ N (18) b e e n d i s c o v e r e d . T h e m ax im u m n u m b e r of
-Us ) e l e c t r o n s in a g i v e n s h e l l o r s u b s h e l l i s i n ­
d i c a t e d in p a r e n t h e s e s . E m a n a t i o n (E m ) ( th e
,Al ... Ar
e l e m e n t w i t h Z = 8 6) i s a l s o c a l l e d r a d o n (Rn).
/iNo - ,?Mg ~3P ) M (8)
-3s J
56 - «7Ne
-77 V {s>
,n - 2He
-1(2)}K (2)

scheme were applicable to complex atoms, the 'id. subshell would


start to be filled beginning with potassium ( Z = 19). According to
the table given in Chapter 25, however, 8 rf= 0.14(>, and 8 ^= 2,23, for
potassium, and therefore the energy of the electrons in the id and
4s states will be

Rfi Rh
(3 — 0.146)- 2.854“"’
(26.13)
Rh Rh _
(4 - 2.23)- 1.77- •

It can be seen that Z;M^>E4s, and hence the 4s level will be filled
before the id level. Consequently, the third period will contain only
seven elements (,,Na — ,sAr), just like the second period.
MENDELEYEV'S PERIODIC SYSTEM OF ELEMENTS 4 31

After the 4s subsheli is filled in Ca (Z = 20), one might expect


that the filling of the subshell would begin with scandium (Z = 21).
Spectroscopic data show, however, that in elements 2 |Sc— .2BNi the
3d subshell is filled first. This subshell becomes filled at the ex­
pense of electrons from .2!)Cu to aoZn of the 4s subshell, which then
must be refilled after all available states in the 3d subshell are oc­
cupied. Only after that can the refilling of the Ap subshell start.
Thus the fourth period contains 18 elements and consists of the 4s,
3d and Ap subshells (see Fig. 26.6). The fifth period repeats the
fourth period (the 5s, Ad, 5 p subshells are filled), and thus it also
contains 18 elements (mRb — BiXe).
The sixth period contains 32 elements (BBCs— 8 GRn), because
besides the outer shell, consisting of 6 s and 6 p states, the first
inner subshell (ten electrons) and the second inner subshell 4/ (the
14 electrons of the lanthanide or rare-earth elements) will be filled.
In exactly the same way the seventh period should repeat the
sixth period; that is, it should contain 32 elements (the 7s, 5f, 6 d.
Ip subshells). So far, however, only 17 elements of this period
have been discovered. The so-called actinides, in which the second
inner subshell 5 /is filled ( 9 0 TH—element 103) should have properties
similar to those of the lanthanides.
The first period therefore contains two elements, the second and
third periods eight elements each, the fourth and fifth periods 18 ele­
ments each, and the sixth and seventh period 32 elements each
(except that the seventh period is incomplete). The order in which
the states are filled is illustrated in Fig. 26.6.

F. APPLICATION OF THE THOMAS-FERMI METHOD TO THE


THEORY OF THE PERIODIC SYSTEM OF ELEMENTS

We shall now attempt to treat the ground-state configuration of the elements m ore
rigorously.
In a paper devoted to the statistical theory of the atom (1928), Ferm i proposed a
method, now known as the Thom as-Ferm l method (see Chapter 25), to explain the periodic
system of elements. With this method, he obtained the minimum values of Z for which
s , p, d and / states are filled in atoms. He obtained these values by starting from the
following quasi-classical ideas.
In classical theory the angular momentum of a particle L is related to the momentum
p by the expression
L = r x p.
Consequently,

where pn is the component of momentum perpendicular to the position vector r.


Obviously, the square p \ of this component of momentum cannot be g rea ter than the
square of the maximum momentum, which we shall denote by P. Therefore for a given P
and r the possible values of the angular momentum L must satisfy the inequality
432 RELATIVISTIC QUANTUM MECHANICS
It was shown in Chapter 13 that, in the quasi-classlcal treatm ent of the atom, the
square of the angular momentum m ust be [see (13,62)]

Z* = »*(/ + ! ) ' . (26.15)

This relation represents a com prom ise between the Bohr relation L2 = 1i2 (I + 1)2 and
the quantum mechanical relation L2 = Ti2l (I + 1), B
The maximum momentum P is related to the density of the electron gas (electrons in
the atom) p0by the expression (6.32)

P 2 = n 2(3n2f 0f ' 2 (26.16)

The electron density po may be found from the T hom as-Ferm i equation (see Chapter 25),
which, as we have already indicated, can be solved only by approximate or numerical
methods. A good approximation for p0, as follows from a solution of the Thom as-Ferm i
equation, is provided by the expression [see (25.21)]

(26.17)
16w3/2
where the coefficient X is found by the Ritz variational method.
Substituting these values for P 2and L2 into the inequality (26.14), we get

■t eZy/ i X , (l + lh ) 2
(26.18)
16 J r r2

Introducing the new variable lr = x t we find

3 * D (26.19)
e > —
x

where
1 16 \ a/a
D= (26.20)
faZ)
From the inequality (26.19) It Is evident that the right-hand side of (26.19) becomes
g re a te r than the left-hand side as x —- 0 (r — 0) and as x — co . The electrons in the atom
therefore can have a given value of / if x lies In the range Xi < x < x 2 for which the
Inequality (26.19) is satisified. Here x t and x» are roots of the equation

e- T /F = £ (26.21)
x ‘
The condition for the appearance of states with a given value of / is the equality of
both roots

X t

In this case we should equate not only the two functions them selves, but also their de­
rivatives. Then, in addition to Eq. (26.21), we will have

_1_ O~ I« Vx D_
(26.22)
3 Vx x 2’
M E N D E L E Y E V 'S PERIO DIC SY STEM OF ELEM EN TS 433

These two relations will be satisfied for

Vx = 3,
that is, when

D = 9e-».

Substituting the value for D from (26.20), we find the value of Z at which electrons with a
given / will first appear

2 = | f l (2/ + 1)3 = 7 (2/ + 1)'. (26.23)

where e = 2.718... Is the base of the natural logarithms and the coefficient 7 Is equal to
0.158.
A numerical solution of the Thomas-Fermi equation gives a very similar value for 7

7 X-F — 0.155.

This again Is a convincing demonstration that the density (26.17) represents a good
approximation to the density which Is given by a numerical solution of the Thomas-
Fermi equation.
Equation (26.23) enables us to calculate the Z values at which the s, p, d, and / states
begin to be filled. The results of this calculation are given in Table 26.1. The first row
gives fractional values of Z computed from formula (26.23) with 7 p_p = 0 .155. The
values of Z, calculated with 7 ^. p = 0.158, for which s, p, d, a n d /sta tes first appear
are practically identical with those calculated with 7 -p_p = 0.155. The second row gives
the nearest greater integral value of Z. The last row of the table gives the empirical
values of Z at which the states first appear, and also the symbol of the corresponding
elements.
Table 26.1
Atomic numbers at which the s, p, d, and /sta te s first appear for a given I

I s P d /
0 1 2 3

Theoretical value 7 J 0.15 4.2 19.4 53.2


(Thomas-Fermi) M l 5 20 54
Empirical value Z 1 (H) 5(B) 21 (Sc) 58 (Ce)

From this table it can be seen that this approximate theory is In good agreement
with the experimental data. We may note that complete agreement Is obtained if the co­
efficient 7 is taken to be 0.169 instead of 0.155.
It is well known that in light elements (Z = 1, 2, 3, 4) only s states are filled. The
filling of the p states begins with boron (Z = 5); this is in complete agreement with
theoretical data. Table 26.1 shows (in spite of the crudeness of the statistical model)
that the filling of the 3d subshell does not begin, as might be expected, in potassium
(Z = 19), but in scandium (Z = 21); that is to say, it does not begin until the 4s subshell
Is completely filled. Similarly, the Thomas-Fermi model explains the "delay” in the
filling of the 4/ subshell, which might be expected to begin in Ag (Z = 47). According to
the theory, however, the filling of the 4 / subshell should be shifted, and should begin only
in cerium (Z = 58). It follows from (26.3) that the filling of the 5g subshell (I = 4) would
begin in the element with Z = 124.
The Thomas-Fermi model accounts for a very important feature of the ground-state
configuration of atoms and explains the departure of the filling of the levels from the
ideal scheme (the hydrogen scheme) in terms of the "smearing” of the electron cloud.
434 R E L A T IV IS T IC QUANTUM M E C H A N IC S

G. PERIODICITY IN THE PROPERTIES


OF ELEMENTS

The periodicity in the properties of the elem ents can be ex­


plained quite naturally in quantum mechanics. It is connected with
the periodic nature of the filling of the outer shell, which contains
at most eight electrons (s and p states) and which determines the
chemical and optical properties of atoms. All elements, therefore,
can be divided into eight groups, depending on the number of elec­
trons in the outer shell.
The elem ents in Group I (hydrogen and alkali metals) have an
outer shell containing a single electron. As a result, the optical
terms (except the s term) are doublets and the elements are
monovalent, as will be shown below. The elements of Group n —the
alkaline earth metals (beryllium, magnesium, calcium, etc.)-have
two valence electrons; their spectral terms must therefore be
singlets and triplets, and the valence is equal to two. The elements
of Group III have an outer shell containing three electrons, and
therefore the maximum splitting of their optical term s must be
equal to four (quartets); their maximum valence is three.
On the contrary, the elem ents of Group VII—the halogens
(fluorine, chlorine, etc.)—lack just one electron to fill the outer
shell completely. Therefore, in addition to the maximum (positive)
valence of seven, they may have a negative valence of — 1 (the
number of electrons required to obtain a stable configuration).
They exhibit this valence in the so-called ionic compounds (see
Chapter 27).
Finally, in the inert gas group (neon, argon, krypton, etc.) the
outer shell is completely filled. We may even say that there is no
outer shell in these elem ents, because the energy bonding these
electrons in the outer shell is larger than that bonding all other
shells in the molecule and thus it would be more correct to ascribe
it to the inner shell. Thus, these elem ents may be assigned to the
“ zeroth” group. The elem ents of the zeroth group do not as a
rule enter into any chemical reactions, and thus these elements
are said to be chemically inactive.
There are, however, a number of exceptions to the rule that
there are eight elements in each period. The first exception is
constituted by hydrogen (Z — 1) and helium (Z = 2), which form the
first period. In this period there are only two elem ents, and not
eight. This is due to the fact that the K shell does not include p
states. Consequently, the properties of these elem ents are of a
dual nature. Because there is only one electron in the outer shell,
hydrogen should have the same chemical and optical properties as
the alkali m etals. Indeed, just like in these elements, the maximum
splitting of the spectral terms of hydrogen is two, and its valence
is ooe. Hydrogen, however, also resem bles the halogen group in
that it lacks just one electron for a complete outer shell, and it can
M E N D E L E Y E V 'S PE R IO D IC SYSTEM OF ELEM ENTS 435

therefore acquire a second electron forming, in the same way as


the halogens, a negatively charged ion.
Helium resembles the alkaline earth metals of the second group
in that it also has two electrons in the outer shell. The spectral
terms of both helium and the alkaline earth elements are either
singlets (spin 0), or triplets (spin 1). However, in its chemical
properties, helium is a typical representative of the inert gases,
because its outer /( shell is completely filled; hence it does not
participate in normal chemical reactions.
The maximum valence of elements is determined, as a rule, by
the number of electrons in the outer shell; that is, the valence of
atoms varies from unity (for atoms of the first group) to seven
(halogen group). There are, however, certain exceptions among the
elements in which the inner shells are filled after the outer shells.
It can be seen from the periodic table that there will be two
electrons in the outer shell of all elements from scandium (Z — 21)
to nickel (Z = 28), with the exception of chromium, where there is
only one. Owing to transitions of electrons from 3d states to 4p
states, however, the maximum valence of scandium (Z = 2 1 ) is equal
to three, and that of manganese (Z = 25) is equal to seven. Conse­
quently, it was necessary to place these elements in groups corre­
sponding to their maximum valences (this was correctly done by
Mendeleyev). Mendeleyev placed iron (Z = 26), cobalt (Z = 27) and
nickel (Z = 28) in a special group (Group VIII). The introduction of
this group is justified from the point of view of modern quantum
mechanics, since at most eight electrons can occupy the outer
shell. In general, however, iron behaves either as a bivalent or
trivalent element. All of these elements have sim ilar properties.
In particular, they have distinctive ferromagnetic properties caused
by uncompensated spins of the 3d electrons in the inner shell. The
presence of these states is due to the fact that from the energy
standpoint, the 3d state is more favorable during formation of the
crystal lattice than the other states in which the spins of the elec­
trons can be compensated.
In the elements following the ferromagnetic, the 3d subshell is
the first to be completely filled; the filling of the levels then con­
tinues in the 4s and then the 4p subshells. Krypton completes the
structure of the M shell (n = 4); therefore its optical and chemical
properties will be characteristic of the inert gases.
As we have already mentioned, the fifth period, which extends
from the alkali metal rubidium (Z = 37) to the inert gas xenon
(Z = 54), is a repetition of the fourth period and exhibits no new
features.
436 R E L A T IV IST IC QUANTUM M E CH A N IC S

Quantum theory also explains the characteristic properties of


the elements in the lanthanide series (the rare-earth elements),
which comes immediately after lanthanum and extends from cerium
(Z = 58) to lutetium (Z — 71). The elements in this series are
formed by consecutive addition of electrons to the deeper 4/ sub­
shell (the second inner N shell), even though the first inner shell
(0) and the outer shell (P) are still incompletely filled. Since the
chemical properties are determined mainly by the electrons of the
outer sh ells, all 14 rare-earth elements are much closer with re­
gard to chemical properties, than are the elements in which the
first inner d subshell is filled.
For a long time hafnium (Z = 72) was also included in the
lanthanide series. A theoretical analysis performed by Bohr
showed, however, that there can be at most fourteen elements in
this group (the possible number of /-states) and that, therefore,
hafnium must be a chemical analog of zirconium. Careful exper­
iments have confirmed this theoretical conclusion.
The actinide series in the seventh and last period is analogous
to the'lanthanide group. Beginning with thorium (Z = 90), the ele­
ments of this series are formed by the consecutive addition of
electrons to the deep-lying 5/ subshell of the 0 shell, while the 6 s,
6 p and 7s subshells remain completely filled and the 6 d subshell is

partially filled. The actinides include protactinium (Z = 91),uranium


(Z = 92) and also the following artificially produced transuranium
elem ents: neptunium (Z = 93), plutonium (Z = 94), americium (Z = 95),
curium (Z = 97), californium (Z = 98),,einsteinium (Z = 99), fermium
(Z = 100), mendelevium (Z=101), nobelium (Z=102) and the recently
discovered element 103 (lawrencium)f
The question of how many elements can be produced by artificial
means and experimentally detected, and the question of where the
periodic system ends, have not yet been finally answered. It is
clear, however, that the periodic system ends because of he in­
stability of nuclei (due mainly to their spontaneous fission).

STUc d i s c o v e r y a n d p r o p e r t i e s of th e n ew e l e m e n t s a r e d i s c u s s e d in G. T. S cab o rg ,
Thr Transuranium bllr m o n ts , R e a d i n g , M a s s . , A d d i s o n - W e s l e y , 1958.
Chapter 2 7

The Theory of Simple Molecules

A. BASIC FORMS OF THE CHEMICAL BOND

The chemical properties and the optical spectrum of an


element are determined mainly by the outer electrons of the
atom. Therefore, the regularities in the structure of the outer
shell, which account for the optical periodicities, also provide
a basis for the construction of a theory of the periodically re­
current chemical properties of the elements. It should be noted
that the chemical properties, unlike the optical properties, are
not exhibited by isolated atoms, but appear only in the presence
of other atoms, with which the atom forms chemical compounds.
The inner electrons have almost no influence on chemical
processes, since they are much more strongly bound to the
nucleus than the outer electrons. Chemical reactions therefore
liberate much le ss bonding than the energy of the inner elec­
trons.
In discussing the chemical properties of an atom, we must
distinguish between two main types of chemical bonds: ionic (or
heteropolar) and atomic (or homopolar). We shall consider both
of these types in greater detail.

B. HETEROPOLAR MOLECULES

Inorganic salts consist of positive and negative ions held to­


gether by an electrostatic (Coulomb) attraction to form a mole­
cule. Compounds of this type are called ionic, and their molecules
are said to be heteropolar. The ions may be either positive or
negative. The sign of the charge on the ion depends, on the one
hand, upon the ionization potential, that is, the energy that must
be expended in order to remove an electron from the outer
shell; and, on the other hand, on the electron affinity, that is,
the energy which the atom must acquire to hold an additional
electron in the outer shell.
Let us assume that a neutral atom with atomic number Z
contains N electrons in the inner orbitals and Z0 = Z — N elec­
trons in the outer orbitals. The electrons in the inner orbitals
438 R E L A T IV IS T IC QUANTUM M E C H A N IC S

will completely screen a corresponding fraction of the nuclear


charge, but will do so only in the region outside this inner shell
(starting at the outer shell). Thus, the Coulomb potential energy
holding the electrons in the outer shell is

r '

However, inside the atomic core the charge will be Z,|>Zn; that
is, the screening of the nuclear charge will not be complete (see
Chapter 25). In exactly the same way, the outer electrons will
completely screen the remaining part of the nuclear charge (Zae0)
only in the region outside the outer shell (that is, only in the case
of excited states). While in this case there appear polarization
forces proportional to r*, they are not able to hold an additional
electron. In the outer shell itself the charge will be incompletely
compensated; for this reason (but provided there are unfilled
states in the outer shell), the incompletely screened part of the
nuclear charge will hold additional electrons in this shell, thus
forming a negative ion. The rule is that the le ss electrons there
are in the outer shell of a neutral atom, the larger the total
screening of the nuclear charge in this shell. Therefore, an alkali
metal will lose the one electron in its outer shell more readily
than it will acquire additional electrons.
A curve showing the dependence of the ionization potential on
Z is plotted in Fig. 27.1. It shows a minimum for alkali metals and
a maximum for inert gases. This curve reproduces rather faith­
fully the periodicity exhibited by the number of electrons in the
outer shell.
In inert gases the ionization potential reaches its largest
value; the removal of an electron from the outer shell and its
transfer to another atom require a very large expenditure of
energy. In addition, no further electrons can be held in the outer
shell, which is completely filled. Therefore, inert gases do not
participate in ordinary heteropolar compounds (we shall also see
that they do not form homopolar bonds), and hence, as a rule,
they exist as unassociated atoms.
Atoms of alkali and alkaline earth metals readily give up their
valence electrons to another atom (the ionization potential is at

*F o r example, in sodium (Z 11), Ihe ten electrons in the inner shell completely
screen ten units of n u c le a r charge, which l e a v e s only th e e l e v e n th unit of n u c l e a r c h a rg e
to h e (partially) screened by the outer electron. In c h l o r i n e ( Z - 17) t h e t e n inner elec­
trons completely screen only ten units of n u c l e a r c h a rg e , so that the seven e l e c t r o n s of
the outer s h e ll m u st s c r e e n the r em ain in g ch a rg e , w hich they c a n a c c o m p li s h only p a rtially .
Therefore, a c h l o r i n e at o m i s a b l e to h o l d an a d d i t i o n a l e l e c t r o n m o r e e a s i l y t h a n s o d i u m ,
and is thus converted into the n e g a tiv e i o n Cl * . O n the other hand, a sodium atom g iv e s
up i t s o u t e r e l e c t r o n mo re r e a d i l y , a n d in t h i s w ay f o rm s a p o s i t i v e ion N a .
THE THEORY OF S IM P L E MOLECULES 439

its minimum here), and thus convert to positive ions (for example,
a Na+ ion).

F ig . 27. 1. T h e d e p e n d e n c e of th e i o n i z a t i o n p o ­
t e n t i a l o f a n e u t r a l atom on t h e a t o m i c num ber.

On the contrary, atoms in Group VI (including oxygen) and in


Group VII (halogens), and also hydrogen (which resem bles Group
VII with regard to the number of missing electrons), have a
higher electron affinity than the other elements (see Table 27.1).
The electron affinity of sodium is practically equal to zero, like
that of inert gases.
Table 27.1

Electron Affinity

Electron affinity
Element
(ev)

H 0.71
F 4.13
Cl 3.72
O 3.07

The first successful attempt to construct a theory of the ionic


bond was due to Kossel (1916), who made use of the Bohr theory
of the atom.
K ossel’s theory was based on the fact that the eight-electron
shells of the inert gases are closed, so that these atoms have
zero valence. Positive valence (or valence with respect to hydro­
gen) is determined by the number of electrons in the outer shell
in atoms in which these electrons are readily lost (atoms of
Groups I or II). Negative valence (valence with respect to fluorine
or twice the valence with respect to oxygen) is determined by the
number of electrons which the atom can acquire, that is, the
number of vacant states in the outer shell (see Chapter 26).
440

Negative valence is particularly proununrfd fia e le a e ita of Cb^


VI and VII, although berth types of valence may be ^
given elem en t. For exam ple, in the typical heteropotar e m p

Fi^. 27.2- Twt neutral inrtrppartr ir


TSs and C-.i_ Tbe Wari dots indicate
electrons: fee dot mdicaies a stats
afeick can be occupied br an e k e tm
o r a l to fee electron «^a of deif! oil,

HCL, chlorine has a valence of -1 , althongh other compounds;


p o ssh le in which chlorine has a valence of -r7. An example
the latter is Cl^Ch. We do not intend here to develop a compl
theory of the chem ical band, and sh all restrict ourselves ta
exam ination of one typical ionic m olecule, nam ely, the hetei
polar m olecule XaCL2 The energy bonding an outer electroi
atom ic sodium is 5.1 ev. When the valence electron of sodium
transferred to the outer sh ell of the chlorine [that is . Air
form ation of Na” and Cl- ions (see F ig. 27.2)], it carries son

what le s s bonding sin ce its affinity for chlorine ( I q = 3.7 (

is somewhat low er than the ionization energy of sodium (F

5.1 err). However, in the form ation of the m olecule, th is dcficiei


is com pensated by the Coulomb energy of attraction between
Xa~ and Cl” ions (see F ig. 27.3).
The total energy bonding these atom s in the XaCl molecule
given by the expression

— F i? 1*.

This energy has been very carefully determ ined from experim
and found to be equal to: = 4-2 ev. Hence for the Coulo

See G_ Herrberg, A z o m ic S p e c t m and A za m ic S s n x z z r a , $ e v Y ork- IW 4_


3The boedz-xg eaergr of aa electron in an atom (or molecule) is eqa»I tc tin energy
s a s i be expended in order to remove an electron. It is. therefore, et^xal to negnti1
fee eoetcj bolding fee electron in fee ccnpocnd (W= —I'); feat is, it will b e a P05
THEORY OF SIM PL E M OLECULES 441

bonding energy between the ions we find

U/C°ul= _ rf.ff _|_ U?!°n -|- 5.G ev:

Since l/Coul——ij/Coul—_ j j t we obtain a perfectly reasonable value


for the interatomic distance in the NaCl molecule: R = 2.5 • 10 ~ 8
cm.

F i g . 27 .3 . T h e f o rm atio n o f an N a C l m o l e c u l e
from Na"*” a n d C l i o n s . T h e i o n i z a t i o n p o t e n ­
t i a l o f s o d iu m (5.1 ev ) a n d th e af fi n i t y of the
c h l o r i n e ato m for an e l e c t r o n (3 .7 ev) a r e i n d i ­
c a t e d in p a r e n t h e s e s . T h e C o u lo m b b o n d i n g
e n e r g y b e t w e e n t h e i o n s in t h e m o l e c u l e i s
5 . 6 ev.

It should be noted that we have not considered here all the in­
teractions that occur in a heteropolar molecule. In addition to the
Coulomb forces of attraction, there will also be repulsive forces;
these exceed the Coulomb forces at small distances and prevent
the two atoms from approaching closer than the distance R. In
any case, this elementary discussion explains the principal physical
processes involved in the formation of heteropolar molecules; it
also explains, however qualitatively, the ionic structure of their
crystal lattice and the dissociation of these m olecules into indi­
vidual ions, a process which occurs in solutions.

C. THE MOLECULAR HYDROGEN ION

Aside from ionic compounds, there exist m olecules formed


directly from the neutral atoms, rather than from ions. The simplest
representatives of these m olecules are H2, 0 2 and N2. These are
called homopolar m olecules.
The formation of homopolar molecules cannot be explained
on the basis of classical theory or Bohr’s sem iclassical theory.
These theories are useful only for compounds held together by
electrostatic forces such as, for example, ionic compounds.
Before discussing the formation of homopolar molecules, let us
442 R E L A T IV IS T IC QUANTUM M E CH A N IC S

consider a very simple case, namely, the molecular hydrogen ion


H j, which consists of two hydrogen nuclei and a single electron.
This analysis is important for methodological reasons, because it
enables us to express in comparatively simple mathematical form
the features of the bond that arises between two hydrogen nuclei
owing to the exchange of an electron (exchange forces). The same
forces also appear in the homopolar hydrogen molecule Ha-
Let us denote the distance between the two hydrogen nuclei a
and a' by R and assume that R changes adiabatically; that is, R
changes so slowly that it cam be regarded as a constant in solving
the Schrodinger equation. In a more exact treatment, it is neces­
sary to take into account the vibrations of the nuclei about the
equilibrium position (the vibrational spectrum), and the rotation
of the nuclei about the center of mass (the rotational spectrum).
These questions have been treated in detail in Chapter 12.
Suppose r and r' represent the distances between the electron
and the nuclei a and a' , respectively. The Schrodinger equation
for the ionized hydrogen molecule can then be written as
■•V
(£ — H)«j» = 0. (27.1)
Here the Hamiltonian is

H= T——
r----el r , H (27.2)

and the kinetic energy operator has the form

where \ x = y'x (yx = ~ , y; = — ) since

r' = r — /?, (27.4)

and R may be regarded as a constant in the problem under con­


sideration.
We shall restrict ourselves to an investigation of the ground
state, and carry out our calculations with the help of perturbation
theory. The spin effects can be neglected in the case of a single
electron.
In the zeroth approximation we assume that the electron is
under the influence of either nucleus a or nucleus a' (see Fig. 27.4).
The Schrodinger equations describing these two possible unper­
turbed states are

(£a - T + ^ = 0 ,
(27.5)

(* •- t + f )‘
THE THEORY O F S I M P L E MOLECULES 443

Both eigenvalues and eigenfunctions are identical and correspond


to the Is state of the hydrogen atom. Since one of the wave
functions is associated with nucleus «, -and the other with nucleus
a’, we may write (« = 1 , l = m — 0 )

Ea = Ea, = Ex= - R h , /
(27.6)
to = tino (r). t o ' = tioo (/■'). r
r
where tn/m = tn»o is the wave function of
R a'
the ground state of the hydrogen atom Electron 1 Is near
[see (13.32)], namely, nucleus a i

t.o o ^ ) — y - , e a°. (27.7) R a'


Electron 1 is near
nucleus a'
The total energy of the system and the
zero-order eigenfunction are as follows: F i g . 2 7 .4 . D iag ram o f t h e i n t e r ­
a c t i o n o f p a r t i c l e s in t h e H 2
E» = EX= — Rh, (27.8)
m o l e c u l a r ion. T h e s o l i d a r r o w s
r e p r e s e n t the in te r a c tio n of the
t° — Crta “1“ tr.2to'- p a r t i c l e s in t h e z e r o t h a p p r o x i ­
m atio n . T h e a r r o w s d e p i c t e d by
The uncertainty in 0° is due to the fact a d a s h e d line r e p re s e n t the in­
that the presence of two nuclei leads to a t e r a c t i o n c o r r e s p o n d i n g to th e
degenerate state of the system . In solving perturbation.
(27.1)by perturbation theory, we must set
£ = £° + £' + .
(27.9)

Substituting (27.9) into (27.1) and restricting ourselves to the first-


order quantities, we find

(27.10)

For the solution | 0 (the electron near nucleus a), es represents the
main interaction, and - ^ the perturbation. On the contrary, for
the solution (the electron near a'), the main interaction i s - —,
and the perturbation is - —.
From Eq. (27.10) we can find the additional energy £', and
also a relationship between the coefficients C, and C., in the wave
function This can be done because the perturbation energy
removes the degeneracy (just as in the case of the helium atom).
444 RELATIVISTIC QUANTUM MECHANICS

To solve this problem, we can make use of the theorem which


states that the solution of the homogeneous equation [that is, one
of the solutions of Eqs. (27.5)] must be orthogonal to the right-hand
side of the same equation, that is, Eq. (27.10). Assuming that the
electron is near the nucleus a, we can neglect the perturbation
e~
energy — on the left-hand side of the equation, since it gives us
a second-order term when multiplied by </. Then the solution of
the homogeneous equation will be the function and according to
the theorem we have just stated, it must be orthogonal to the right-
hand side of Eq. (27.10)

I) d?x -!- Ci J <^a ( f + f - 1 ) *a.<Px = 0. (27.11)

Here we have used the fact that the wave function of the ground
state of the hydrogen atom is real.
In exactly the same way, assuming that the electron is near the
nucleus a', we may neglect the perturbation energy on the left-
hand side of Eq. (27.10) (in this case the perturbation energy is -^).
Since the solution of the homogeneous equation will now be the
function tya„ we obtain a second equation for the unknown quantities

-;-C, J = (27.12)

When r is replaced by r', becomes whereas in the reverse


substitution <fa, becomes In both ca ses, the volume element
cPx remains unchanged (d'x = d'x’) . Therefore, we can reduce
(27.12) to a form that is in agreement with (27.11), but with the
coefficients and C2 interchanged

C* + +
+ C, J ' F - \ - ' ; - - f j * a.<Px = 0. (27.13)

In further transforming Eq. (27.11), we must not forget that the


function is normalized to unity

J i]£<Fa: = 1. (27.14)

but is not orthogonal to the function <]v

j) M a - d :>x = S(R). (27.15)


THE THEORY OF S IM P L E MOLECULES 445

This follows from the fact that, although the expression S (R)
vanishes for R -*• oo (there will be no points where the two functions
<|i„ and both differ from zero), it becomes equal to unity for

Let us introduce the notation

and

(27.16)

where K is the Coulomb energy of interaction between the hydrogen


atom and the (atomic) hydrogen ion, and A is the exchange energy,
which has no classical analog. This energy arises because the
electron can be in both states and simultaneously (that is,
exchange occurs between states a and a'). Formally, the existence
of the exchange energy is reflected in the fact that the expression
for contains fya,, as well as <!>„. As we shall see in what follows,
it is this exchange energy which gives rise to an attractive force
between the nuclei. At certain values of the internuclear separation,
this force exceeds the force of repulsion, and as a result the H2
molecular hydrogen ion is formed. It is worth noting that this
mixed state cannot arise in the Bohr theory, and therefore the
existence of an ionized hydrogen molecule can be explained only
in terms of quantum mechanics.
With the help of (27.14)-(27.16), we can reduce Eqs. (27.11)
and (27.13) to the form
C, ( £ ' - / ( ) + C2 ( £ 'S -A ) = 0,
(27.17)
C i ( E ' S - A ) + Ci (E’ - K ) = 0,

obtaining two solutions: a symmetric one (Ct — C.2) and an antisym­


metric one (C! = — C2). The symmetric solution is

T /'2(I+S)
('i'a + 'lV).
(27.18)
, A — SK
E ' s = V s {R) = K ' 1+S ’

and the antisymmetric solution is

----- ------- f.'i —><> \


/ 2 ‘0 ^ S) 'a)’
(27.19)
A — SK
E'* = V 1 (R) = K
1 —S '
446 RELATIVISTIC QUANTUM MECHANICS

The factor , 2 ( *l - ± S = - is the normalization coefficient. We mayJ note


y )
here that the symmetric and antisymmetric solutions are already
mutually orthogonal

^ tjja 6 s d'x = 0 .

The quantity E may be regarded as an additional interaction V (R),


which binds the H atom and the H+ ion into a stable hydrogen
molecule. In order to find the specific form of this interaction as
a function of R, we must calculate the values of S, K and A. For
g- V
this purpose we use the well-known expansion of the function ——
into a triple Fourier integral

e ~ k°r 1 f elkr d ’k
r '-2 7 1 s ) k* + *§ (27.20)

After differentiating this expression with respect to the parameter


k], we can easily write the wave function for the ground state of the
hydrogen atom in the form of a Fourier integral

e i hr
'Woo ( r ) = — —
y no]
e
(k- + w d'k, (27.21)

where kn= —.
In accordance with (27.15), the expression for S becomes

S=^ {f) ^mo (f — R)d'x. (27.22)

We may replace ^mn(^) and ^ o o ^ — R ) by their expressions in


term s of the Fourier integral (27.21). Integrating with respect to
volume, we obtain

^e ‘^ + k)rd^x = 8r^(k-j-k’), (27.2.';)

and thus we find the following value for S:

ei>R
Ik'1 + k-y d*k
(27.24)
THE THEORY OF S I M P L E MOLECULES 447

The integral in this equation can be calculated with the aid of Eq.
(27.20), which should be differentiated three times with respect to
the parameter ArJ.
In order to calculate the additional Coulomb interaction, we
shall use the relation

where we must set /e„ = —.


Oq
Then, putting ka= 0 in (27.20), we obtain

(27.25)

In exactly the same way, we may represent the quantity l/R in the
form of the integral (27.25). Substituting these quantities into
formula (27.16) and integrating over the volume d'x with aid of
(27.23), we find

(27.26)

2
Hence, using (27.20) and setting k0= —,
Qq
we obtain an expression
for the additional Coulomb energy

(27.27)

In calculating the exchange interaction

(27.28)

we may use (27.21) for (^*— R) and (27.20) for

where 6 0 = —. Hence

(27.29)
448 R E L A T IV IST IC QUANTUM M E C H A N IC S

Substituting the values found for S, K and A into the expression


for the interaction energy [see (27.18) and (27.19)], we obtain the
following expression (R'a„ — 't):

ya,s el +
R I 1 \ - (27.30)
1 + ( l + E+

Here the upper signs (-) refer to the antisymmetric solution V3,
while the lower signs (+) refer to the symmetric solution. For
sm all values of R (/?<Ja0), we have

V3 — Vs — when R<^a0, (27.31)

and thus the energy, as we would expect,


is determined by the Coulomb energy of
repulsion between the two nuclei. For
large distances (R^>a0) we have

y a ,s _ - f- 2 el B-e~%
'"J Go G q
(27.32)

that is, the antisymmetric solution (+)


gives a repulsion, whereas the sym­
R metric solution (-) gives an attraction.
I’ ip. 2 7 .5 . C u r v e s of t h e i n t e r a c t i o n
The general nature of the varia­
e n e r g y in t h e m o l e c u l a r h y d r o g e n tion of V3 and Ks as a function of R is
io n a s a f u n c t i o n o f t h e d i s t a n c e R depicted in Fig. 27.5 which also shows
b e t w e e n t h e n u c l e i (in u n i t s o f a 0) the experimental data on the inter­
for t h e s y m m e t r i c ( V s ) a n d a n t i ­ action energy. It can be seen from
sym m etric (V a ) s t a te s .
this figure that only the symmetric
state is realized in practice.
Theoretical values obtained from the graph give the equilibrium
distance as Ru 2.50a0= 1.32 A, and therefore the ionization
energy is

/A---- — 1/(/?„) = -)- 0.0640 = 1.76 ev. (27.33)


a0

The corresponding experimental values are

/\'e*P -- LOGA, /;exp ^ 2.79 ev (27.34)

(the zero-point energy of oscillations is not included in the


theoretical and experimental values given here).
THE THEORY OF S IM P L E MOLECULES 449

The discrepancy between the theoretical and the experimental


data Is due to the fact that here, just as for the helium atom, the
perturbation energy is commensurable with the energy of the
zeroth approximation. Solving this problem by the variational
method, using a test function of the form
‘/a . Z ' r
Ta e "o (27.35)
, n"il
where Z', the effective charge of the nucleus, is taken as the
variational parameter, we can obtain values for and D which are
in considerably better agreement with experiment

flvar= 1 0 6 A, Dvar= 2.25 ev.

If several parameters are introduced, the variational method gives


results that are in practically complete agreement with the
experimental data.
It can be seen that the formation of the molecular hydrogen
ion Is essentially due to the quantum-mechanical exchange forces,
which in the symmetric state give rise to a stable molecule.
From the physical point of view, this can be explained as follows.
The probability of the electron being in the symmetric state is

pos = (^s) ‘2 = 2(1 *+ &) (27.36)

whereas the probability for the antisymmetric state is

p: = ( r r = 2 (1 [_ S) (fa + r* - 2 * ,u - (27-3?)

If we plot the curves of constant probability density of the electron


(see Fig. 27.6), we see that the electron tends to be located at the
midpoint of the line joining the two nuclei in the case of the sym ­
metric solution, whereas in the case of the antisymmetric solution
the position probability vanishes at this point. Since the electron
binds the two nuclei most strongly when it is halfway between them,
it is natural to expect that the first solution, and not the second,
will lead to the formation of a molecule. Moreover, in the case of
the symmetric solution, the curves showing the electron distribution
about the nuclei tend to merge when the nuclei approach one another;
this provides a graphical characterization of the homopolar bond. 4

4 F o r m ore d e t a i l s s e e P . G o m b a s , Thcorie und L o s u n g sm c th o d c n des Mchrtcilchen-


problems der Wollenmechanik, B a s e l : B i r k h a u s e r - V e r l a g , 1950.
450 R E L A T IV IST IC QUANTUM M E CH A N IC S

Two hydrogen nuclei, or a hydrogen and a deuterium nucleus, can


also be linked by other particles besides electrons. We may
mention in this connection the ^-m esic molecule (HD) , in which the
bonding between the hydrogen and deuterium nuclei is brought
about by a negative u meson. Alvarez produced a m esic molecule
of this type by passing negative n mesons through a bubble chamber.
The radius of such a molecule, as calculated from the equation
R — 2.5 will be 1/200 the radius of the molecular hydrogen
j.^ 0

state state

F ig. 27.6. E le c tro n d e n s ity d istr ib u tio n in th e m o le c u la r


h y d r o g e n ion.

ion, since the m ass of the u meson is approximately 1 / 2 0 0 as


large as the electron m ass. Thus, when the nucleus of the hydro­
gen atom approaches the nucleus of the deuterium atom, they
form a common nucleus, namely that of the .He:l molecule

As a result, an energy of 5.4 Mev is released and carried away by


the u meson. Thus the n meson acts almost as a catalyst of the
nuclear reaction.

D. HOMOPOLAR ATOMIC MOLECULES

The first successful attempt to give a theoretical explanation of


the homopolar molecule was made by Heitler and London (1927)
with the help of quantum mechanics. In a homopolar molecule,
exchange forces play a fundamental role. In their treatment,
Heitler and London used perturbation theory, which does not give
completely accurate quantitative results. Although more accurate
quantitative results can be obtained by means of the variational
method, the Heitler-London theory enables us to bring out in a
very simple way the physical features of the homopolar bond.
Let r, and r.t denote the position vectors of the first and second
electrons relative to nucleus a, and r\ and t\ the position vectors
THE THEORY OF S IM P L E MOLECULES 451

of the electrons relative to nucleus a' (see Fig. 27.2). Then


r.) = r.j - /?. In the zeroth approximation we obtain two
wave functions which are products of the ground-state hydrogen
wave functions

fyac = tya (r l) 'r'o- ir'i) — 'Wuo( / " l ) 'I'lOO ( r 2 R)> os >


'I 'n 'n ~ ' i V ( r l ) '^a ( r ‘i ) = 't’ lOO ( f " l — R ) ih o o

The first solution o(1(I. corresponds to the case when the first electron
is near the nucleus a (and the second electron near the nucleus a'),
while the second solution ']>a,a corresponds to the case when the
first electron is near the nucleus a', and the second electron is
near the nucleus a. Both these possibilities are depicted in Fig.
27.7, where the solid arrows show the atomic bonds, and the
dashed lines show the molecular bonds. When the distance between
the nuclei tends to infinity ( £ - > - 0 0 ) , all molecular bonds vanish.

F i g . 2 7 .7 . D ia g r a m o f t h e i n t e r a c t i o n s
in t h e H 2 m o l e c u l e . T h e s o l i d l i n e s
join the p a rtic le s w hose in teractio n is
t a k e n in to a c c o u n t in t h e z e r o t h a p ­
p r o x i m a ti o n . T h e d a s h e d l i n e s d e n o t e
i n t e r a c t i o n s w h i c h are r e g a r d e d a s
p ertu rb atio n s; a and a are the n u c le i
of t h e h y d r o g e n a to m s; 1 a n d 2 a r e
electrons.

Just as in the problem of the molecular hydrogen ion, the main


solutions (27.38) that give rise to an additional degeneracy of the
system will not be orthogonal5

s = $^ a a ^ a d ^ x . ^ J * 100 (/■) i 100 (r - R) ePx }a= S \ (27.39)

SA b a r o v e r a sy m b o l d e n o t e s q u a n t i t i e s r e f e r r i n g to t h e n e u t r a l m o l e c u l e .
452 R E L A T IV IST IC QUANTUM M E C H A N IC S

where S for the H2 ion is given by the expression (27.34). The


Coulomb energy of interaction of the two atoms is given by

K= J <]£„• { f + jrt - 7[ - 7 7 }■
d3*i -(27.40)
where the first and second parenthetic term s in the integrand cor­
respond to the potential energy of repulsion between the two nuclei
and the two electrons, and the third and fourth terms correspond to
the potential energy of attraction between the first electron and the
nucleus a' and between the second electron and the nucleus a.
In exactly the same way, we obtain the exchange energy

A e?i el p j d 'xi d 'x.y. (27.41)


ria r\

The expressions for K and A can be computed aproximately by


the same method as in the theory of the H| ion. For K we obtain
a comparatively simple result, and for A a more complicated
result,, since A is expressed in terms of an integral logarithm (as
shown by Sigura). The general character of the solution, however,
remains the same as in the theory of the HI ion. In particular,
the main forces which hold the two neutral atoms in the molecule
are the exchange forces. These forces have a minus sign at
comparatively large interatomic distances and correspond to the
mutual attraction of the atoms. Just as in the case of the molecular
hydrogen ion, we have two solutions. The first solution is sym­
metric

V3 — / — 1---------- r - (Vna' H Ya ’a) <


V 2(1 +S)
(27.42)
VS(R)-- K+A,
1 +S

and the second is antisymmetric

$ a = ,, ■ •- (<v. - <?«■«).
V 2(1 - S )
(27.43)
K—A
F a (/?) =
1 —S '

The general form of the curves of V a~s for a neutral hydrogen mole­
cule is approximately the same as for the H.2 molecular ion; there­
fore, only symmetric solutions will give stable m olecules. For the
radius corresponding to the equilibrium position [that is, the
minimum of the potential energy Vs (R) for the symmetric solution],
we obtain Rlt — 1.51 a„ — 0.80 A. The corresponding value for the
dissociation energy is

D — — Vs (Ru) = 0.11”) -“ 0"=3. 2 ev. (27.44)


THE THEORY OF S IM P L E MOLECULES 453

The experimental values of these quantities are

tfexp_o.74A, Dexp = 4.73ev. (27.44a)

We have omitted here the zero-point energy of oscillations, 0.27 ev


(see Chapter 12), from both the theoretical and the experimental
values.

E. SPIN AND THE SYMMETRY OF STATES

In the 1I2 ion and the H atom, there is only one electron, and its
spin leads only to insignificant spin-orbit interactions. On the
other hand, there are two electrons in the H.2 configuration, and the
spin plays an important role in the theory of this molecule, even
though the spin-orbit and the spin-spin interactions give only
small corrections. In the hydrogen molecule, just as in the helium
atom, the mutual orientation of the spins of the two electrons
determines the type of symmetry of the spatial part of the wave
function; this is of primary significance in connection with the
stability of a molecule. We shall therefore consider more fully
the question of the relation of the spin to the symmetry properties
of the molecule.
The total wave function T must contain a spin part in addition
to the spatial part. When the potential energy of the spin-orbit
interaction can be neglected, then, just as in the case of R ussell-
Saunders coupling, the total wave function can be represented by
a product of a spatial part and a spin part. For electrons (which
obey Fermi statistics) the total wave function must change sign
when coordinates and spins are interchanged (that is, the solution
must be antisymmetric). We therefore have two possibilities

T, = Ca (s„ i',)^s(r„ n), (27.45)

W2= CMsi. s, ) ^ {ru r4). (27.46)


It has been shown in Chapter 24 that the antisymmetric spin
function Ca describes two electrons with antiparallel spins, and
therefore function^, which is symmetric in the position coordinates,
corresponds to a state with total spin 0. In exactly the same way
the symmetric spin function Cs , as well as the antisymmetric
spatial function, describe a state with total spin 1 (the spins of
both electrons are parallel). In the case of the hydrogen molecule,
the only solutions which results in attraction is that corresponding
to 0 s ; thus, a stable molecule is obtained only in the case in which
the electron spins are antiparallel.
We shall now proceed to a general analysis of the states of a
molecule, using the symmetry properties. In this connection, we
454 R E L A T IV IS T IC QUANTUM M E CH A N IC S

note that in diatomic molecules the field of force p ossesses axial


symmetry with respect to the line passing through the nuclei
(the symmetry axis of the molecule). The absolute value of the
component of the total orbital angular momentum along this axis
of symmetry (which, incidentally, must be conserved) is denoted
by A. States corresponding to different A are denoted by the
following letters: X ( A = 0); II ( A = 1); A ( A = 2); etc.
In addition, each electronic state must be characterized by the
total spin S of all the electrons in the molecule. For a given value
of S, 'i = 2S - f - 1 states are possible. The quantity v, as in the
case of an atom, determines the multiplicity of the energy level. In
the case where the total spin is equal to zero (S = 0), we have
v = 1. For states with S = l , the multiplicity v = 3 , etc. The
total spin of electrons In a molecule can therefore be characterized
by the multiplicity v, and the corresponding term can be denoted
by VA.
In this notation, the symmetric solution for the spatial part of
the wave function (that is, the solution os) corresponds to the term
*2 (that is, A = 0, S = 0, v = 1 ), while the antisymmetric solution
(<^a) corresponds to the term 3v (a = 0, S = 1, v = 3). It is obvious
that the 3 Y] term corresponds to three states: in two of the states
the spin is directed along the symmetry axis of the molecule (in
a parallel or antiparallel direction), while in the third state it is
perpendicular to the symmetry axis.
It should be noted that symmetry plays a very important role
in the theory of molecules (particularly in the case of complex
m olecules). If, for example, we reflect the wave function in a
plane passing through the symmetry axis of the molecule (which
we take as the z axis), the energy of the molecule must remain
unaltered . 6 At the same time, if the component of the orbital
angular momentum or of the spin along the symmetry axis differs
from zero (A ^ 0 or S ^ O ) , the rotations which are associated
with these angular momenta will be reversed as a result of this
reflection . 7
For sim plicity, we restrict our treatment to the states in
which the orbital angular momentum is zero, that is, A = 0 (V
term s). In the case where the total spin of the electrons also
vanishes, that is, S = 0 , no change of states will occur in the
mirror reflection.

6 ]n th e c a s e of r e f l e c t i o n in t h e x z p l a n e , mirror r e f l e c t i o n a m o u n t s to a r e p l a c e m e n t of
y by - y.
7A s is w e l l k n o w n , th e a n g u l a r m om entum /, r x p is an a x i a l v e c t o r w h o s e d i r e c t i o n
is a m a t t e r of c o n v e n t i o n (it h a s o n e d i r e c t i o n in a r i g h t - h a n d e d s y s t e m of c o o r d i n a t e s ,
a n d th e o p p o s i t e d i r e c t i o n in a l e f t - h a n d e d s y s t e m ) . T h e d i r e c t i o n o f th e c o n t o u r b o u n d i n g
the a r e a a n d c o n s t r u c t e d from t h e v e c t o r s r a n d p r e m a i n s , h o w e v e r , u n a l t e r e d in b o th th e
rig h t- a n d l e f t - h a n d e d c o o r d i n a t e s y s t e m s .
THE THEORY OF S IM P L E MOLECULES 455

If, however, the spins of both electrons are parallel (5 = 1),


the following cases are possible.
(a) The component of spin along the'symmetry axis is equal to
zero ( S , = 0). In this case, the rotation characterizing the spin
remains unaltered as a result of mirror reflection (see Fig. 27.8,
where the initial and reflected spins are characterized by a
rotation denoted by II and II'). The corresponding terms are
designated by the symbol ^ .
(b) The component of spin along the axis of symmetry z differs
from zero (S^ = :L 1), In this case, the rotation which we associate
with the spin is reversed as a result of mirror reflection (see Fig.
27.8, where the initial spin is characterized by the rotation I and
the reflected spin by the rotation /').

B B'

F i g . 27 .8 . C h a n g e o f t h e a n g u l a r m om entum on r e f l e c t i o n
in t h e A A ' B B p l a n e , w h i c h p a s s e s th r o u g h t h e a x i s o f s y m ­
m etry z. If t h e i n i t i a l r o t a t i o n c h a r a c t e r i z i n g t h e a n g u l a r
m om entum o c c u r s in a p l a n e p e r p e n d i c u l a r to A A B B ( s e e /),
the d irectio n of th is ro tatio n w ill be re v e r s e d a fte r reflec tio n
( s e e I'). If, h o w e v e r , t h e r o t a t i o n t a k e s p l a c e in t h e p l a n e o f
r e f l e c t i o n , it w i l l b e u n a l t e r e d by r e f l e c t i o n (// = / / ) .

The terms whose spin changes on reflection are designated by


the symbol £~.
Therefore, the following terms of the ground state of the hydro­
gen molecule are possible:
’v+(A = 0, S = 0),
av +(A = 0 , S = 1 , S, = 0), (27.47)
3v - ( a = 0, S = 1, S, = -' 1),

where the last term is obviously twofold degenerate.


If the molecule consists of two identical atoms, there will be an
additional symmetry property. A diatomic molecule with identical
nuclei must have a center of symmetry, in addition to a plane of
symmetry. This center of symmetry is the mid-point on the line
joining the nuclei. In Fig. 27.8, it is located at the origin of the
coordinate system , that is, at the point z = 0. In this symmetry
transformation we must change the sign of the coordinates of all
456 R E L A T IV IS T IC QUANTUM M E CH A N IC S

the electrons. In particular, under this symmetry transformation


the positions of electron 1 and electron 2 will be interchanged in
the hydrogen molecule (the coordinates of the nuclei are left un­
changed). The symmetric wave function will remain unaltered;
that is, it is even (this is denoted by the subscript g). The anti­
symmetric function <>a changes its sign; that is, it is odd (this
is denoted by the subscript u). The main possible states of the
hydrogen molecule, taking into account both symmetry properties,
can be therefore denoted as follows:
1V + 3 V + 3 V “

and so forth.
The importance of symmetry with regard to the formation of a
molecule follows also from the fact that the ground state of most
diatomic molecules is a state in which the wave function is invariant
under all symmetry transformations . 8 Thus, is the main term
of the hydrogen molecule. The question of molecular symmetry,
however, lies outside the scope of this book.
It should be noted that in stable states of the hydrogen molecule
the spins of the two electrons are always oppositely directed. At
the same time, there are two types of hydrogen m olecules—para-
hydrogen and orthohydrogen. These names refer to the orientation
of the nuclear spins, and not to the orientation of electron spins.
In parahydrogen the spins of the nuclei are antiparallel, while in
orthohydrogen they are parallel. Since the number of possible
states for two particles with parallel spins is three tim es larger
than in the case of particles with antiparallel spins, ordinary hydro­
gen at room temperature will consist of an equilibrium mixture of
25% parahydrogen and 75% orthohydrogen. As the temperature is
lowered in the presence of a catalyst (for example, charcoal), the
percentage of parahydrogen in the equilibrium mixture increases,
and is practically 100% at 0°K. Parahydrogen produced at low
temperatures is extremely stable and can be preserved in such
an equilibrium system for a period of several weeks at room
temperature. Orthohydrogen has not yet been obtained in pure
form. The difference in the thermal conductivities at low tempera­
tures (the thermal conductivity of parahydrogen is larger) is used
for determining the composition of the mixture. Similarly, para­
hydrogen and orthohydrogen have somewhat different dissociation
energies and optical properties.

F. THE VALENCE THEORY

We shall now explain the concept of chemical valence in terms


of quantum mechanics. By chemical valence we mean the ability
a n e x c e p t i o n to t h i s r u l e , l e t u s m e n t i o n , fo r e x a m p l e , t h e O 2 m o l e c u l e , fo r w h i c h
3V
THE THEORY OF S IM P L E MOLECULES 457

of an atom to com bine with a sp e c ific number of other atom s. As


already m entioned, the fir s t s u c c e s s of quantum theory in con­
nection with the ch em ical p rop erties of atom s w as the explanation
of hetcropolar ch em ical compounds (K o sse l’s theory); th ese com ­
pounds are form ed as a resu lt of the red istribu tion of e le c tr o n s
in the outer s h e lls of the participating atom s. According to this
theory, the num erical value of the valen ce is determ ined by the
number of e le c tr o n s which an atom g iv e s up to another atom (posi­
tive ionic valen ce) or acq u ires from another atom (negative ionic
valen ce). In the form ation of a m olecu le, the e le c tr o n s in the
outer s h e lls of atom s are redistribu ted so that the v a len ce s of
the atom s are saturated.
Further p r o g r e ss in the investigation of the form ation of a
m olecu le w as m ade with the H eitler-L ondon theory. T his theory
succeeded in explaining the form ation of the sim p le st hom opolar
m olecule H2, which s e r v e s a s the b a sis of our p resen t concept of
the covalent bond. A ccording to the H eitler-L ondon theory, the
spins of the valen ce e le c tr o n s are m utually com pensated in the
hom opolar hydrogen m o lecu le. G eneralizing th ese r e su lts , it is
p o ssib le to conclude that the form ation of hom opolar m o le c u les
occu rs under the condition of mutual com pensation of the spins
of the valen ce e le c tr o n s. A ccordingly, th is type of valen ce is
a lso so m etim es called the spin v a len ce.
Since the saturation of valen ce bonds am ounts to a com pensation
of the sp ins of the valen ce e le c tr o n s, the ch em ical valen ce of atom s
is given by the number of e le c tr o n s with an uncom pensated spin
presen t in the outer sh e ll.
To illu str a te th ese gen eral p r in cip les, let us con sid er som e
sp ecific ex a m p les. Figure 27.9 g iv e s the grou n d -state configurations
of se v er a l e lem en ts of the p eriod ic sy ste m . The electro n sta te s
are shown a s b o x es, w hile the e le c tr o n s are denoted by arrow s
whose d irectio n s correspond to their spin orien ta tio n s. It is c lea r
from th is figu re that the configuration of the outer sh e ll of the
hydrogen atom (l.<;1) iS corresp on d s to a sin gle valen ce bond. The
valen ce of hydrogen, which is equal to one, is sm a lle r by a factor
of one than the m ultiplicity of its ter m s, which is two (the m ulti­
plicity is designated by the su p erscrip t on the left-hand side of
the term sym bol S ) .
S im ilarly, the ground state of the helium atom has the con­
figuration (2s‘J). It is evident that the m ultiplicity equals one (‘S),
w hile the valen ce is equal to zero.
The boron atom ( Z — 5) has the ground state (ls 22s'‘2p'), c o r r e ­
sponding to the doublet (i P ) , and consequently the va len ce equals
one. The excited state (ls'2s'2/P), corresponding to the quartet
(SP) is a lso p ossib le; in this state the valen ce of boron is equal to
3. Thus the p resen ce of se v er a l different v a len ce s in the elem en ts
of variou s groups in the period ic sy ste m can be explained in
com paratively sim ple form (se e Table 27.2).
458 R E L A T IV IS T IC QUANTUM M EC H A N IC S

Although accord ing to exp erim en t the e le m en ts of oxygen and


halogen groups can have se v e r a l differen t v a le n c e s, the O and
F atom s th e m se lv e s show only the principal v a len ce. T his is
due to the fact that th e ir m u ltip licity can be in c re a se d only if
an e le c tr o n is tra n sferred to a sh e ll with a la rg e r value of the
principal quantum num ber. T h is p r o c e ss is unfavorable from
the energy standpoint (the d su b sh ell is absent in O and F ). On
the con trary, for other e le m en ts of th ese groups there i s a pos­
sib ility of tran sition betw een sta te s of the sam e sh e ll, but having
differen t v a lu e s of I.

Is 2s 2p
H [T J Us')
He n 1 Us2)
t I I Us22s22p<)
lP m (ls22s’2p2)
t t i (ls22s72p3)
m i l {ls22s22p3)
N 11 t t t i Us22s'2p3)
0 LI M i l ) I Us22s22p;‘)
F I I I I I I I Us22s22p5)
I I I I I I I I [ls22s22ps)

F i g . 2 7 .9 . D ia g r a m s h o w i n g t h e f i l l i n g
of the e le c tr o n s h e l ls o f s e v e r a l atom s
w i t h t h e s p i n t a k e n i n t o a c c o u n t . Homo-
p o l a r v a l e n c e o f a t o m s i s d e n o t e d by a
dot, a n d i o n i c v a l e n c e i s d e n o t e d by
a I ( p o s i t i v e ) o r —( n e g a t i v e ) sign.

Table 27.2
M ultiplicity and hom opolar valen ce

Group of the
periodic sys­ 1 II III IV V VI VII
tem

Multiplicity 2 1 1, 3 2, 4 1. 3, 5 2, 4, 6 1, 3, 5, 7 2, 4, G, 8
Valence* 1 j 0, 2 I, 3 0, 2, 4 1, 3, 5 0, 2, 4, 0 1, 3, 5, 7
|

* T h e b o ld -f a c e ty p e in d i c a t e s th e p rin c ip a l v a le n c e .
THE THEORY OF S IM P L E MOLECULES 459

According to the configuration given in Fig. 27.9, nitrogen in


the ground state (ls^s'^/T) is trivalen t (the three e le c tr o n s in the
2p sh ell have p a ra llel sp in s). However, it can a lso be univalent
(antiparallel sp ins of the two e le c tr o n s in the 2 p subshell) and
even pentavalent (1 .sa2 s12 p:’) (the four spin v a le n c e s, a sso cia ted
with the p a ra llel spins of the e le c tr o n s in the 2 s and 2 p su b sh ells,
are augm ented by a fifth ionic valen ce a sso cia ted with the rem oval
of a second ele c tr o n from the 2s su b sh ell). In this connection we
note that the ionic valen ce of oxygen and fluorine is the sam e as
the spin valen ce (covalence).
It should be em phasized that, in gen era l, it is im p o ssib le to
divide rigorou sly the ch em ical bonds into hom opolar and h etero -
polar on es. The two types of bonds correspond to the lim iting
c a s e s of the ele c tr o n density distribution in the incom plete s h e lls .
The lim itin g c a se of asym m etry in the distribution of the electro n
density betw een the atom s corresp on d s to a heteropolar m o lecu le.
Such a m olecu le has a dipole m om ent and can be regarded a s an
ionic stru ctu re. The c a se of a hom opolar bond corresp on d s to
identical ele c tr o n density d istributions in the atom s of the m ole­
cu le. A hom opolar m olecu le has no dipole m om ent, and it can be
con sid ered a s a structure form ed from two neutral atom s.
Quantum theory p rovides a gen eral m ethod for the explanation
of valen ce fo r c e s and d ea ls with both types of bonds (homopolar
and heteropolar) in a sin g le sch em e. One of the ch ief m e r its of
the H eitler-L ondon quantum -m echanical theory of the H.2 m olecu le
is that it exp lain s the satu ration of hom opolar bonds in te r m s of
the saturation of the sp ins of the e le c tr o n s h e lls when e le c tr o n s
com bine into p a irs with antip arallel sp in s. When a hydrogen atom
approaches a H., m olecu le no additional p a irs with com pensated
spins are form ed, and hence there is no gain in en erg y . The H3
m olecule th erefore cannot e x is t.
It m ust be em p h asized , how ever, that the H eitler-L ondon theory
w as developed only for m o le c u les co n sistin g of atom s in the s
state, and th erefore an exten sion of its con clu sion s to m ore com ­
plex atom s m ust be of a som ew hat qualitative nature. Further
developm ent of the theory of hom opolar bonds has shown that in
the c a se of com plex atom s quantum law s alone are not su fficien t.
In th is c a se we m ust a lso include the influence of the sp e c ific
p rop erties c h a r a c te r istic of the ch em ical com pounds.

G. MASERS AND LASERS

The electromagnetic waves radiated by conventional radio transmitters have a com­


paratively wide frequency band. Everyone who has used a radio receiver knows that
transmitting stations which have nearly equal frequencies overlap one another. This is
due to the fact that conventional transmitters have insufficient stability and often ''drift”
into another frequency band.
460 R E L A T IV IS T IC QUANTUM M EC H A N IC S

Thus, the stability of even the best existing quartz oscillators is inadequate in a
number of cases (for example, high-stability oscillators are necessary for accurate
determination of distances by means of radar). For this reason, one of the great achieve­
ments of recent times was the development of m asers9by Townes and coworkers, and
independently by Basov and Prokhorov. In this device, quantum transitions between
discrete energy levels in a molecule are used as microwave generators.
We note first of all that the process of spontaneous em ission, which depends on the
Einstein coefficient A, has no essential significance in the case of molecular emission,
since its intensity is proportional to u* and is extremely small in the radiofrequency
region (that is, in the range of frequencies that are low compared with light frequencies ji
As far as induced transitions, which are proportional to the Einstein coefficient B, are
concerned, the probability of upward transitions (resonance absorption) and the probability
of downward transitions (induced emission) are identical (see Chapter 9); they are pro­
portional to the energy of the field. Therefore, a system (molecule) with two levels will
undergo a transition from one level to another under the Influence of a sufficiently strong
external field containing the resonant frequency, the transitions being accompanied by the
em ission and absorption of quanta.

F i g . 2 7 . 1 0 . S t r u c t u r e o f t h e am m o n ia m o l e c u l e
s h o w i n g tw o m i r r o r - s y m m e t r i c s t a t e s a a n d b
of th e sa m e energy.

If external radiation is passed through a substance, it will interact with its molecules
and cause resonant absorption as well as induced emission. In accordance with the
Boltzmann distribution (the number of particles in an equilibrium system with energy E
is proportional to e ~ E/kT), there must be fewer particles in the higher energy states
than in the lower states; therefore, absorption dominates induced emission in the thermal
equilibrium. The excess of absorbed energy is completely converted into the energy of
thermal motion of the molecules, raising the temperature of the gas.
In order for the system to amplify and not absorb the radiation incident on it, it is
necessary to disturb this thermal equilibrium in such a manner as to produce greater
occupancy of the higher energy levels than of the lower o n es.10 When this is done,
such a system will generate electromagnetic waves with extremely small line width
under the action of resonant radiation. The first masers were produced with ammonia
(NHa) molecules. The ammonia molecule consists of one nitrogen atom and three
hydrogen atoms and forms a right pyramid. However, from the laws of symmetry, the
stable state of the ammonia molecule corresponds not only to the case in which the
nitrogen atom is located above the triangle composed of the hydrogen atoms (Fig. 27.10a),
but also to the case in which the nitrogen atom is located at the same distance under the
triangle (Fig. 27.10b). The stable states a and Zihave equal energies and correspond to the

9T h e Icrm “m a s e r ” i s an ac r o n y m form ed from t h e f i r s t l e t t e r s o f m i c r o w a v e a m p l i f i ­


c a t i o n by .s t i m u l a t e d e m i s s i o n o f r a d i a t i o n .
I0Such a s t a t e c a n a l s o be d e s c r i b e d by t h e B o l t z m a n n d i s t r i b u t i o n f u n c t i o n , if it is
a s s u m e d t h a t th e t e m p e r a t u r e 7', w h i c h in t h i s c a s e p l a y s th e r o le of a p a r a m e t e r , a s s u m e s
n eg a tiv e values.
THE THEORY OF S IM P L E MOLECULES 461

minima of the potential energy; they are thus separated by a potential barrier, so that
classical theory does not allow transitions from one to the other. In quantum theory,
however, the probability of penetration across the potential barrier Is different from
zero. Consequently, the nitrogen atom, which Is above the triangle, may reposition It­
self below the triangle In the absence of any external action, and may then return to Its
original position. In molecular physics this phenomenon is called inversion. The process
of Inversion can be explained In terms of the existence of two perfectly Identical potential
wells, separated by a potential barrier of finite width. It is well known that this leads to
a splitting of the spectral lines, in spite of the fact that the energy states in the two po­
tential wells are completely identical to one another. For the ammonia molecule this
splitting, expressed In wavelengths. Is 1.27 cm for the most intense line; this corre­
sponds to a radiofrequency wave.
Another remarkable property of the ammonia molecule is that, with the help of an
electric field. It Is fairly easy to separate the molecules In the upper and lower energy
levels that have been formed as a result of Inversion. It Is found that when an ammonia
molecule is placed In an electric field, the two levels are shifted in different directions,
the upper Inversion level being shifted upwards (the energy increases) and the lower
inversion level downwards.
Since any system tends to a state In which its potential energy is minimum, we first
pass a beam of ammonia molecules through a carefully evacuated vessel containing an
electrostatic field which is produced by a quadrupole capacitor; this field decreases
towards the symmetry axis (see Fig. 27.11).
In this separating system the molecules in the upper level will tend to move to a
region where this field Is minimum (since the field increases their energy); that is,
they will be focused about the axis of the capacitor. The molecules that are in the lower
level, however, tend to move to the region of maximum field (that is, to the periphery,
where their energy will be minimum), and thus they will be ejected from the beam. After
passing through the above separating system, the beam of molecules, which contains a
predominance of excited molecules, enters a cavity resonator which, among its rather
wide range of radio frequencies (produced by conventional radio frequency methods) also
contains the resonant frequency of the molecular transitions.
This external radiation causes molecular transitions which are associated with a
line of small width (•%. 10 s e c -') atafrequency of 2.4 x 1010se c - ' (that is , with a relative
error no greater than 10~g), corresponding to the wavelength of 1.27 cm. This wavelength
is determined mainly by the lime of traverse of the molecules through the cavity resonator.
A molecular generator produced In such a way exhibits extremely stable frequency.
The frequency stability is so high that molecular clocks constructed on this principle
have an accuracy of approximately 1 sec per 300 years of continuous operation.11
On the basis of this principle, several types of radio receivers working in the micro-
wave region have been constructed. They all use paramagnetic crystals cooled to very
low temperatures. These devices, known as quantum paramagnetic amplifiers, greatly
increased the sensitivity of radio astronomical and radar equipment.
Recently, atoms and molecules have been used as generators and receivers in the
visible spectrum. In this case, we speak of lasers (light amplification by stimulated
emission of radiation).
What is the difference between an ordinary source of light and a laser? In an ordi­
nary source of radiation, for example, the Sun, the spectrum consists of a broad fre­
quency band and the individual incoherent quanta have arbitrary phases and directions.
Optical lasers enable us to obtain a monochromatic beam of light of high intensity (when
it is focused we obtain a radiation density a thousand times greater than that obtainable

1^ h e s t a b i l i t y o f t h e f r e q u e n c y in m a s e r s m a d e it p o s s i b l e to c o n s t r u c t tw o s t a n d a r d
i n d e p e n d e n t l y o p e r a t i n g , s y n c h r o n i z e d c l o c k s . T h e s e c l o c k s w e r e u s e d to m e a s u r e t h e
v e l o c i t y of l i g h t in o n e d i r e c t i o n u n d e r t e r r e s t r i a l c o n d i t i o n s a n d t h e r e f o r e to c h e c k the
fundam ental c o n c lu s io n s of the s p e c ia l theory of r e la tiv ity reg ard in g first-order e f fe c ts
( p ro p o r tio n a l to t h e v e l o c i t y v — cB).
We r e c a l l t h a t a il i n t e r f e r e n c e e x p e r i m e n t s s i m i l a r to t h e M i c h e l s o n e x p e r i m e n t i n v o l v e ,
in e f f e c t , o n ly o n e s t a n d a r d c l o c k a n d a r e u s e d to m e a s u r e th e v e l o c i t y of l i g h t o v e r a
c l o s e d p ath . T h u s , o n ly s e c o n d - o r d e r e f f e c t s ( p ro p o r t i o n a l to /32) c o u l d b e d e t e c t e d in
th e se experim ents.
462 R E L A T IV IS T IC QUANTUM M EC H A N ICS

by focusing sunlight), great coherence and extreme sharpness (with telescopic apparatus
a beam of light reaching the Moon would have a diameter of the order of 3 km). The
first laser produced by Maiman was a three-level ruby laser. Chromium atoms, which
are present in ruby (aluminum oxide) as a slight impurity, absorb light from a power
klystron over a wide band in the green and yellow regions. Initially, these atoms give
up (without emitting radiation) part of their energy to the crystal lattice and make a
transition to a metastable state from which dipole transitions are forbidden, and hence
they can be maintained in that state for a comparatively long time (of the order of
several milliseconds).

Fig- 2 7 . 1 1 . E l e c t r o s t a t i c f i e l d o f t h e
c a p a c i t o r in th e s e p a r a t i n g s y s t e m .

At a "negative” temperature (when the number of metastable atoms is greater than


the number of unexcited atoms and under the action of stimulated em ission, such a
system is capable of generating almost Instantaneously monochromatic waves with a
wavelength of 6943 A (red region). These transitions are Induced by the first emitted
quanta, which are retained in the ruby (the ruby has the shape of a circular rod, bounded
by silvered parallel ends, one of which is semitransparent). Photons moving parallel to
the axis induce the emission of photons having the same frequency and moving in the
same direction. This chain process Intensifies until a coherent ray finally passes through
the semitransparent mirror (after multiple reflection from the ends).
The prospects for practical application of lasers are enormous. These devices can
be used for the simultaneous transmission of various types of information, the establish­
ment of cosmic communication, the control of chemical reactions induced by thermal
excitation and so forth.
P a r t III

S o m e A p p l i c a t i o n s to N u c l e a r P h y s i c s
Chapter 2 8

Elastic Scattering of Particles

A. TIM E-DEPENDENT PERTURBATION THEORY

In the c a se when the H amiltonian is an exp licit function of tim e,


it is not, as a ru le, p o ssib le to obtain an exact solu tion of the
Schrodinger equation. If the tim e-dependent part of the Hamiltonian
V ' ( t ) is sm a ll in com p arison with the tim e-independent part of the
Hamiltonian Htf, tim e-dependent perturbation theory can be used
to so lv e the problem .
It is obvious that tim e-dependent perturbation theory can a lso
be applied when the perturbation energy is independent of tim e. In
this c a se we obtain the sam e solu tions as in the stationary p e r ­
turbation theory of Chapter 14.
Including the perturbation V ’ (t), we w rite the Schrodinger equa­
tion as

~ T = [ H# + V"(0 ] m (28.1)

The solution of this equation in the zeroth approxim ation (V"(f) = 0)


is

= (28.2)

where the wave function <Ji„ is a solu tion of the unperturbed tim e -
independent wave equation
E n<fn = W i f n, (28.3)

and s a tis fie s the orthonorm ality conditions

= l n.n. (28.4)

We sh all look for a solution of Eq. (28.1) in the form of Eq.


(28.2), assu m in g that the co efficien ts C n depend on tim e: C„->Cn(f).
T his approxim ate m ethod of solution w as proposed by D irac and is
known as the m ethod of variation of con stan ts.
466 FU N D A M EN TS OF NUCLEAR P H Y SIC S

Substituting Eq. (28.2) into Eq. (28.1) and using Eq. (28.3), we
obtain the follow ing equation for the unknown c o efficien ts C„:

~ n En (28.5)

—-E it
M ultiplying Eq. (28.5) by type* , integrating the resu ltin g
e x p r e ssio n over a ll sp a c e , and usin g the orthonorm ality condition
(28.4), we obtain a sy ste m of equations for the unknown c o efficien ts
C n>

— 4 Cn‘
I
^ C„'-e"°Vn" V'n'n" (f), (28.6)

w here

(0n'n" — (28.7)

and the m atrix elem en t

V'n'n" ( 0 = S W ( t ) y n " d * X .

D ira c’s sy ste m of equations (28.6), taken for a ll values of r i , is


com p letely equivalent to the origin al wave equation.
The approxim ation used in perturbation theory c o n sists in e x ­
panding the solu tion in the form

C n’ = Cn’ C n>-{ - ..., (28.8)

w here the z e r o -o r d e r c o e ffic ien ts are independent of V' . The


co e ffic ien ts for the fir s t and higher ord ers w ill be proportional
to V', V"\ and so forth.
Substituting Eq. (28.8) into Eq. (28.6) and retaining only the
fir s t-o r d e r te r m s , we find the follow ing sy ste m of equations for
the co e ffic ien ts C n’\

C£- = 0 (zeroth approxim ation),


(28.9)
—j C ’ni = ^ C n " e ita‘n'n"V'n'nii(t) (fir st approxim ation) and so on.
n"
The fir s t of the equations (28.9) show s that the unknown z e r o -
order co e ffic ien ts m ust be independent of tim e, that is ,

Ch' = const. (28.10)


ELASTIC SCATTERING OF PARTICLES 467

Their values are given by the initial conditions and ch a ra cterize


the state of the electron before the perturbation is applied.
Let us assu m e that at / = /0the electro n is in the state n . Then
we may set

Ch" = (28.11)

This ex p r e ssio n sp e c ifie s the initial conditions of the problem .


Substituting Eq. (28.11) into Eq. (28.9), we find

cn. (t) = — - J d te “ - « - n V (/). (28.12)

In quantum m echan ics one gen erally calcu lates the transition
probability w per unit tim e. Since the probability of finding a
p article in the state n’ is given by | C„< |3, we obtain the follow ing
ex p ressio n for the tran sition probability:

w = T t l i ^Cn' ^ (28.13)
n'

Equations (28.12) and (28.13) are the b a sis for the investigation
of many quantum -m echanical prob lem s in the fir s t-o r d e r tim e -
dependent perturbation theory.

B. THE CROSS SECTION FOR ELASTIC SCATTERING

We sh all now apply the r e su lts obtained above to the study of


the e la s tic sc a tte rin g of an electron .
We assu m e that at the initial tim e /O= 0 the p a rticle is free;
that i s , it m oves uniform ly with a m om entum p = Hk and an energy

*»-*?)•
At the instant of tim e q>= 0,
the p a rticle com es within the
range of interactions; that is ,
it m oves in a potential V ( r ) .
The p a rticle now has a finite
probability of m aking a tran­
sitio n to the state with the
m omentum p ' = hk' and energy
E'= ctlK! — that is, F ig . 2 8 .1 . S c a tte r in g o f a p a r t ic l e by a c e n ­
ter of force: Hk is th e m o m en tu m o f th e
the p a rticle is sca ttered as in c id en t p article; ftk is th e m om entum of
a resu lt of interaction (Fig. th e scattered p article; t? is th e angle of
28.1). scatterin g ; 0 d e n o te s th e s c a t te r in g ce n ter.
468 F U N D A M E N T A L S OF N U C L E A R P H Y S I C S

The wave functions of the initial and final s ta te s , d escrib in g the


fre e m otion (in the zeroth approxim ation), are [s e e (4.62)]
^ (k ) — Z,—Vig-icOT+ i kr j
(28.14)
(k ') = L - I / s e - i c t f 7 + i* 'r

w here L is the period, and the m om entum com ponents £,• and
k ’i (Z= 1 , 2, 3) are rela ted to the in teg ers m and n\ by m ean s of
the e x p r e ssio n s

2nn, 2-r.n',
. (28.15)

Substituting the wave functions (28.14) into Eq. (28.12), we find for
the co efficien t C„-

1 _ e M ( K' - K)
Cn'(t) = - p V H (28.16)
cn(K‘ — K) '

w here the m atrix elem en t of the perturbation energy is [(]/' = ]/ ( r ) ]

Vx — j e ‘xrV ( r ) d ' x , v. — k — k ’.

H ence, the tran sition probability is

d V 11/ 122 sin ct (K' — K)


dt Z , 1 x| cti3(K1— K) ’ (28.17)

We note that the function

s in ct (K1— K)
ji (K‘ — K)
(28.18)

has a sharp m axim um for su fficien tly large valu es of t and


K' — K ->■ 0. T his m ean s that in p r a c tic e we can r e s tr ic t o u r se lv e s
to only those changes of K ' for which the follow ing condition is
sa tisfie d :

c(\K ' — K \ ~ 2 v .

Since the quantity t — t — t0= M is the tim e ela p sed from the
initial instant, and the quantity c/2 (/<"' — /<^) = AE is the energy
spread resu ltin g from sc a tte rin g , we find a relation sh ip between
th ese quantities

A/| AC |~ A . (28.19)
E L A STIC S C A T T E R IN G OF PA R T IC L E S 469

This relation can be consid ered as a fourth uncertainty relation;


it is usually obtained from the theory of tran sition p r o c e ss e s .
The uncertainty in the energy is c h a r a c te r istic for any wave
p r o c e ss; its optical analog is the fam iliar e x p r e ssio n for the
broadening of sp ectral lin e s, resu ltin g from the finite duration of
the em issio n .
For su fficien tly large values of t { t - > oo) the uncertainty in
energy tends to zero and Eq. (28.18) b ecom es a statem ent of the
law of conservation of energy

K' = K.

This explains why this type of sca tterin g is said to be “ e la s t ic .” 1


M athem atically, this follow s from the fact that the function (28.18)
is a 8 function at t = oo. Integration of the 5 function lead s to the
replacem ent of K ' by K ■ To show th is, let us con sid er the integral

CO

/= ^ — F (K") d K ' . (28.20)


o

Introducing a new variable

ct ( K ' — K ) = t .

we get

eK.
71
— ctK

If the function F has no sin g u la r itie s, we obtain as f->-oo

/ = F(/C)-^+J s^ d l F (IQ-
— oo

On the other hand, from the definition of the 8 function it follow s


that

J H K ' — K )F {K ')d K ' = F{K). (28.21)


0

An e x a m p l e o f i n e l a s t i c s c a t t e r i n g i s p r o v i d e d by b r e m s s t r a h l u n g —a s c a t t e r i n g p r o c e s s
in w h i c h a n e l e c t r o n e m i t s a p h o t o n , s o t h a t K < K .
470 FUNDAM ENTALS OF NUCLEAR PH Y SIC S

Hence it is c le a r that e x p r e ssio n (28.18) b ecom es a 8 function as


t - + co, and, consequently, Eq. (28.17) for the tran sition probability
can be w ritten as

w = l j k * y . \ V * \n (K ’ ~ *)• (28.22)
h'

R eplacing the sum m ation (28.22) by integration we u se , in accord ­


ance with Eq. (28.15), the follow ing relation:

( - J = k ,,ld k ’d Q = k 0k ’d K ’dQ. (28.23)

The sc a tte rin g p r o c e ss is usu ally c h a ra cterized by a c r o s s s e c ­


tion, which is equal to the ratio of the probability w to the number of
p a r tic le s N incident per unit tim e on a unit su rfa ce S perpendicular
to the incident beam . O bviously the p a r tic le s that str ik e this su r ­
face per unit tim e are th ose located at a d istan ce not exceeding
the v elo city of the p a r tic le s v , that i s , the p a r tic le s contained in
the volum e v S = v. The number N is equal to the number of p arti­
c le s per unit volum e p0 = Zr3 m ultiplied by a volum e which is
n u m erically equal to the v e lo c ity of the p a rticle

N = h = i ' l <28-24>
With the aid of Eqs. (28.22) - (28.24), we find the follow ing
e x p r e ssio n for the sc a tte rin g c r o s s se ctio n :

o = J = | o ( » , <p)d2. (28.25)

The integrand ch a ra cterizin g the num ber of sc a tte re d p a r tic les


incident per so lid angle d l l ( d l l = sin &dO d y , w here & and 9 are the
sp h e r ic a l sc a tte rin g a n g les), known as the d ifferen tial c r o s s s e c ­
tion, is equal to

o ( 0 , ?) = ( ^ T) V j 2. (28.26)

In p articu lar, when the sc a tte rin g cen ter is sp h e r ic a lly sy m m e tr ic ,


we have

Vx = V(r)r*dr [ e ^d W,
0

w here d Q ’ is the so lid angle a sso cia ted with the vector r , w hereas
in Eq. (28.25) d l l is the so lid angle a sso cia ted with the vector A'.
EL A STIC S C A T T E R IN G OF PA R T IC L E S 471

Integrating the la st ex p ressio n over the solid angle d (2 ’, we


find
oo
V x— ^ ^ r sin y.r V (r) dr.
o

From this it is clea r that the d ifferen tial c r o ss sectio n of the


e la s tic sca tterin g is equal to

« ( » ) = ! / ( » ) I*. (2 8 -2 7 )
where

x= \k — k'\ = 2 k s \ n \ , (28.27a)

and the quantity


OO
f(H) = — ^ r sin v.r • V (r) d r (28.28)
o
is called the sca tterin g am plitude.
Equation (28.27), d escrib in g the e la s tic sca tterin g of the p ar­
tic le s by a center o f force V (r) in fir s t-o r d e r perturbation theory,
was origin ally developed by Born; it is th erefore called the Born
approxim ation.
We note that this problem can be a lso solved in the tim e -
independent perturbation theory, sin c e the potential energy of in­
teraction is tim e-independent. To obtain the sca tterin g c r o ss
sectio n we have used, how ever, the tim e-dependent perturbation
theory, the m athem atical apparatus of which is com p aratively
sim p le but is much m ore gen eral. In p articu lar, it can be used to
so lv e many p roblem s in m odern quantum electro d y n a m ics, taking
account of the interaction of ele c tr o n s with the doubly quantized
electrom agn etic field (se e Chapter 29).
The ex p ressio n for a(d), obtained from the perturbation theory,
is applicable only within som e definite lim its. In the c a s e of sh o r t-
range fo rc e s (nuclear fo r c e s , neutral atom , im penetrable sp h e r e s,
and so forth), which can be neglected for r R, w here R is a c e r ­
tain e ffectiv e radius, the magnitude of the c r o s s sectio n m ust be
either le s s or of the sam e order as their geom etric c r o s s sectio n
r.R* (even if th ese fo rc e s create an absolutely im penetrable b arrier).
For sh ort-ran ge fo r c e s , th erefo re, the range of applicab ility of
the perturbation m ethod is given by

o < -/? -. (28.29)

Equation (28. 27) is notapplicable to lon g-ran ge fo r c e s (Coulomb


fo rc e s) if the scatterin g angle is sm a ll. T his question req u ires
a m ore detailed an alysis (see below).
472 FU N D A M EN TA LS OF NUCLEAR PH Y SIC S

C. SCATTERING BY THE YUKAWA CENTER OF FORCE

The form of the potential energy for the Yukawa in teraction is


a s follow s:
kar
V = — A (28.30)
r ’

w here A is a constant related to the charge and R =J- is the e ffe c -


r?o
tive range of th e se fo r c e s. The interaction (28.30) has so m e im ­
portant applications in the theory of nuclear fo rc e s and, in p a r tic ­
u lar, in m eso n theory.
For nuclear fo r c e s , the quantity A is equal to g - , w here g!
d eterm in es the stren gth of the potential. The range of nuclear
fo rc e s is equal to the Compton w avelength of a pion

R = - - ~
m T_c
10“ 13 cm . (28.31)' '

In the c a s e of fa s t-e le c tr o n sca tterin g ( o r a - p a r t ic le sc a tte r ­


ing) by neutral atom s, the in teraction potential given by the T h om as-
F erm i m odel can be approxim ated by the e x p r e ssio n (2 8 .30).2 In
th is c a s e the quantity A = Z e \ , w here Z is the atom ic num ber, and
the effe c tiv e radius R of the atom in the T h o m a s-F e rm i m odel is
equal to
R = (28.32)

w here f is a num erical factor o f the order of unity.


F in ally, settin g R — oo, we obtain the Coulom b potential of the
nuclear field which, consequently, can be con sid ered as a lim itin g
c a se of Eq. (28.30).
Substituting Eq. (28.30) into (28.27) and using the relation
OO OO
^ r sin * r V (r) d r = — A ^ sin w e ~ k«r d r = —A ^ ft, ,
o o

we obtain the follow ing e x p r e ssio n for the d ifferen tial c r o s s sectio n
of e la s tic sca tterin g :
/A\ _ 4/zi^2/?1 (28.33)
v ' n< p -V A + u

2A s h a s b e e n m e n t i o n e d in C h a p t e r s 2 5 a n d 2 7 , a m o r e a c c u r a t e a p p r o x i m a t i o n o f t h e
T h o m a s - K e r m i p o t e n t i a l i s g i v e n by t h e e x p r e s s i o n ( 2 5 . 2 2 ) . T h e r e s u l t s o f t h e tw o a p p r o x ­
i m a t i o n s , h o w e v e r , d o n o l d i f f e r g r e a t l y fr om o n e a n o t h e r ( w h i c h i s a c o n s e q u e n c e o f t h e
short-rJinge c h a r a c t e r of th e forces). The a p p ro x im atio n (2 8 .3 0 ) is m ore convenient in
c a lc u la tio n s of sca tte rin g .
EL A STIC S C A T T E R IN G OF PA R T IC L E S 473

H ere, according to Eq. (28.27a),

x4 = 4£1sin s ~ 2 = ^ frj s i n

where p is the momentum of the p a rticle.


Two c a s e s should be distinguished in the a n a ly sis of Eq. (28.33).
1. Scattering of rela tiv ely slow p a r tic le s , when v.R<* 1 for all
sca tterin g an gles. In this c a se Eq. (28.33) shows that a (0) is inde­
pendent of the angle i> and becom es equal to

im-AOR*
«(») = n1 (28.34)

The independence of the sca tterin g c r o s s sectio n of the angle &


(isotropy) is a c h a r a c te r istic feature of sca tterin g of rela tiv ely
low -en ergy p a r tic le s by a cen ter of sh o rt-ra n g e fo r c e s .
2. In the c a se of rela tiv ely fa st-p a r tic le sca tterin g , the d iffer­
ential c r o ss sectio n b ecom es independent of R (the e ffectiv e range
of force) for all sca tterin g angles sa tisfy in g the condition /.R ;> 1.
Equation (28.33) in this c a se reduces to

o(d) = i g f - . (28.35)

It is evident that for angles sa tisfy in g this condition the scatterin g


by the Yukawa potential is the sam e as the sca tterin g by a Coulomb
cen ter. T h erefore, in the c a se of fa st e le c tr o n s or a -p a r tic le
sca tterin g by a neutral atom through com p aratively large an gles,
the influence o f atom ic ele c tr o n s is not important and the s c a tte r ­
ing is determ ined only by the potential of the nucleus.
Setting A = Ze\ and x = -~ -s in -2 in Eq. (28.33), we obtain the
fam iliar Rutherford equation
Z !e*m‘0
0(H)
(28.36)
4p* sin4y

which was obtained by the c la s s ic a l method in Chapter 2. Equation


(28.36) show s that for lon g-ran ge fo r c e s th ere is a strong de­
pendence of the scatterin g c r o s s se ctio n on the scatterin g
angle &.
H owever, for any large v alu es of the wave v e c to r s k — ~:V /e

can alw ays find sm a ll angles & such that the follow ing inequality
is true:
sin - < 1. (28.37)
474 FU NDA M ENTALS OF NUCLEAR PH Y SICS

In p articu lar, as &— 0, Rutherford’s form ula g iv es a divergent


value for a(&); in this c a se we m ust take into account the sh o r t-
range ch aracter of the fo r c e s r esu ltin g from the sc re e n in g action
of the ele c tr o n sh e ll. The condition (28.37) now d eterm in es the
region of inapplicability of the Rutherford form ula.
F or &= 0, that i s , for forw ard scatterin g — we find from
E q s. (28.32) and (28.33) the follow ing e x p r e ssio n for the d ifferen tial
c r o s s section:

= ~ a\Z 'h . (28.38)


9 —0

The total c r o s s se ctio n can be obtained from Eq. (28.33)

8nmlA‘R* (* sin
J [1 + 2k*R‘ (1 — cos 8)]2
o
_ 16tt/ngA2/?4 1
n* 4PR* + 1• (28.39)

F in ally, with the aid of Eq. (28.29), we can find the range of
applicab ility of the perturbation m ethod for our problem in the two
lim itin g c a s e s con sid ered above

A < lB ) for w < > ’ (28.40)


4 < - ! £ t o r « > l.

Problem 28.1. Represent the cross section for scattering of particles by a spherically
symmetric potential as a sum of partial cross sections (the sum of the cross sections
for waves with a well-defined value of the orbital quantum number I ). Obtain the scatter­
ing cross section for the general case and for the Born approximation.
Solution. Suppose the incident particle has a momentum p = fik and velocity
v = hh!m0, directed along the z axis.
The wave function of the incident particle is of the form of a plane wave

A plane wave can be expanded in spherical harmonics (see Problem 21.5). Then, using
the asymptotic expression for the Bessel function as r —'-co

T.I
t t sin kr —
(kry-
i+- Y \ Ykr

the incident wave can be represented as

plkZ V si" (kr- T-pj (28.41)


= / j i‘ (2/+ 1)--------- “ P>(cos0)-
EL A STIC S C A T T E R IN G OF PA R T IC L E S 475

In the presence of the potential energy V(r), the asymptotic expression for the wave func­
tion of the particle In a centrally symmetric field, In accordance with Eqs. (13,75) and
(13.78), should be chosen in the form

sin hr — +
CiPt (c os 3)
*as= 1 hr

where the phase shift 6, can be determined from the asymptotic solution of the Schrodinger
equation for the radial function in the presence of the potential V (r)

bh[*
Clearly the scattered wave is

i v i
^scat 't'as ^ inc:
/= 0

X[C(e,s; — il (21 + 1 ) ] — e 2 '[Ct*''8/ - *'(2/ + Dll ‘

The unknown coefficients Cj can be determined from the condition that the function 'J'scat
must be a diverging spherical wave. Thus the coefficient of the converging wave
-iU r-4 )
e ' 2 ' must be equal to zero. Then
/(« ) £tkr
^scat

The function /(9 ) is the scattering amplitude [see (28.28)], which, according to the
exact theory, is equal to

/(» ) = 4 k 2 (2/ + !) (e2ii‘ - 0 p i ( cos »)• (28.43)


1=0

The differential cross section characterizing the scattering of particles through an


angle 9 is equal to the ratio of the probability of scattered particles passing per unit
time through an element of the spherical surface dS = r~dQ:

= v l/<8) I2 dQ
to the number of particles incident per unit time on a unit surface perpendicular to their
velocity, that is, perpendicular to the z axis

^ i n c = u4'*inc4'in c = w-
From this we find the differential cross section

dz = i ^ s c M = |/( 8 ) jt 2.z sinW8 (28.44)


ltTnc
Here, assuming the scattering field to be axially symmetric, we set the solid angle
equal to

dQ. = 2- sin 9d9.


476 FU N D A M EN TA LS OF NUCLEAR PH Y SIC S

Substituting the value obtained for the scattering amplitude and using the orthonormality
property of Legendre polynomials, after integrating over the angles we get
Tt
j Pi ( cos ») Pr ( cos 9) sin 94 9 = )

which gives us the following expression for the total cross section:
oo

o= ^ 2 (2/+ i) s,n,#*- (28*45)


t=o
This expression for o is the desired sum of partial cross sections.
Let us compare the expression for the scattering amplitude found in the Born approx­
imation [see (28.28)] with the exact expression (28.43). The comparison shows that the
exact expression gives the same result as the Born approximation for small values of
the scattering phase angles 8;. Indeed, when5; 1, expression (28.43) becomes
oo

/(» ) = j 2 (2/ + 1} l ‘P‘ ( C0S 3) • (28.45a)


1=0
Solving Bq. (28.42) for a given I (for a partial wave) by perturbation methods, we can
show that
00
8/ = - | V (r) r J ^ i j k r ) d r . (28.46)

0
Next, using the expansion
oo

— = 2f r 2 (2^ ^ ; + ^ ) ^ (cos8).
IMJ ‘ 2
where

= 2li sin ,

the scattering amplitude (28.45a) can be reduced to the form (28.28) found In the Born
approximation.

Problem 28.2. Determine the cross section for scattering of particles by a spherically
symmetric potential barrier of height K0> 0 and radius a

V— I for r < a,
I0 for r > a,

when the radius a is much less than the de Broglie wavelength of the scattered particles,
that Is, when ka I.
Show that in this case the s wave (wave with / = 0) is the main contributor to the
scattering process.
Compare the exact solutions with those obtained by perturbation methods.
Solution. It may be seen from Eq. (28.46) that at ka I, the s wave (/ = 0) is the main
contributor.
Solving the problem by perturbation methods (Born approximation), that is, using
Eq. (28.28), we find the scattering amplitude
E L A STIC S C A T T E R IN G OF PA R T IC L E S 477

Consequently, the cross section in the Born approximation is equal to

«U = 2* J s i n « | / ( 0 ) | * d # = - ? (28.47)
h'
o'
Let us determine the scattering phase shift in this simple problem. We restrict
ourselves to a determination of the phase shift for 1 = 0 and low energies E, when we can
set ka < I.
When I = 0, the wave equations have the form
7.J + **/„ = 0 for r > a,
(28.48)
7.J — *,2/.0 = 0 for r < a,
where

Zo = * o r , *= = 2 " 'a° £•, ,.’2 = 2' ^ ( V <>- E ) = S - k \

Using the boundary conditions 7.o(0) = 0, the solution of Eq. (28.48) can be represented
as
y _I A sinhx'r for r < a ,
0 — 1 sin (kr + 60) for r > a.

Equating the wave functions and their derivatives at the boundary of the region r = a , we
find In our case of small /; (E <<; K0)

60 = arc tan | 4- tanh x'a j — k a i= k a — 1 J ,(28.49)

where
2ntoVt
ft2
Hence, according to Eq, (28.45), we have
/tanhxa
o = 4na2 - - 1 (28.49a)
\ xa
In the case
xa < 1 (28.50)
we can set

tanh xa r
xa : 1- 3 (™)2
Then Eq. (28.49) gives an expression for o corresponding to the Born approximation
[see (28.47)].
When xa 1(that is, when V0 —■oo) the cross section reaches Its maximum value

o —: 4na2. (28.51)

We n o t e that th e c o n d itio n (28.50), w hich in t h i s p ro b lem c a n also be w ritten a s

2
&& <K rra ,

i s e q u i v a l e n t to t h e c o n d i t i o n ( 2 8 . 2 9 ) fo r t h e a p p l i c a b i l i t y o f p e r t u r b a t i o n m e t h o d .
470 FU N D A M E N TA LS OF NUCLEAR P H Y S IC S

The last expression Is four times greater than the corresponding cross section for
elastic scattering by an impenetrable sphere, calculated according to classical mechanics
when o is determined simply by the geometric cross'section of the sphere tea2. This
discrepancy is due to the appearance of wave properties (diffraction) of the scattered
particles.

Problem 28.3. Determine the cross section for scattering of slow particles (ka<^ 1)
by a spherically symmetric potential well:

V — V„ for r < a ,
0 for r >. a. (28.52)

Indicate what distinguishes the cross section for scattering of particles by a potential
well from the cross section for scattering by a potential barrier.
Solution. In the exact solution of the problem, Eq. (28.48) should be replaced by

Xo’ + A 2X0 = 0 for r > a ,


X0" + *'2X„ = 0 for r c a ,
where

k'1 = ^ ( E + V 0) = k2 + xK

The phase shift is given by the following expression [instead of Eq. (28.49)]:
k_
B0 = arc tan tan k ka. (28.54)
k' '*)
When k'a 1 and £ < Vo, we again obtain Eq. (28.47), which was derived in first-
order perturbation theory. In this case the cross sections for scattering by a potential
well and by a potential barrier are identical.
The difference appears when the quandtyxa lx 1 = ^ b e c o m e s comparable to or
• / tanh x a
greater than unity. Thus, in the case of a potential barrier, the quantity -------- - monoton-
x'a
lcally approaches zero as V0 increases, whereas in the case of a potential well the
.tank'd I
corresponding quantity varies periodically over the range from 0 to oo as
Vq increases.
In particular, if the quantity k'a approaches - y , we find the following expression for
k'a 1
the cross section in the region ------- <g|ka:
tan k'a
_a2
(28.55)
°res k2a2'

Because of the smallness of the quantity fc2a2 = ~ ^ a2, the expression for o is much
greater than the maximum value for the cross section In the case of scattering by a
potential barrier (°rnax = 4tta2 when Vo— oo [see (28.51)]). Since the relation k'a —
= Y'*-2a2 -j- k2a2 = — for ka 1 is actually equivalent to the condition of appearance
of the first level In the spherically symmetric potential w ell,x a £ s — , the cross section
(28.55) corresponds to the case of "resonant” scattering. Subsequent resonance maxima
of the cross section occur when xu : and so forth.
Note. Ih e scattering phase shift, together with the expression for the cross section,
can be accurately determined for a very limited class of problems (scattering by a
spherically symmetric potential barrier or potential well, scattering by a potential in­
versely proportional to the square of the distance, and so forth). However, in the general
case we must use approximate methods. For intermediate values of the potential energy l/0
(for instance. In the case^of scattering of charged particles by a Coulomb field the constant
EL A STIC S C A T T E R IN G OF PA R T IC L E S 479

2e*
characterizing the potential energy must satisfy the condition —— 1) perturbation
nv
theory (that Is, the Bora approximation) gives good results. In cases when the Bora
approximation Is no longer applicable, the phase can be determined by other approximate
methods, for example, the WKB method [see Eq. (13.78)], the variational method and
so forth. In view of the special nature of these methods we shall not discuss them In this
general treatment.4 It should be noted that many qualitative features, which are char­
acteristic of the scattering of particles by various potentials, are very well illustrated
in the scattering by a potential barrier or a potential well.

4
T h e t h e o r y o f s c a t t e r i n g i s g i v e n a m o r e e x t e n d e d t r e a t m e n t in N. M o tt a n d G. M a s s e y ,
The T h e o r y o f A t o m i c C o l l i s i o n s , N e w York: O x fo rd , 1949; L. Schiff, Q u a n t u m M e c h a n i c s ,
N e w Y o r k : M c G r a w - H i l l , 1955*
Chapter 2 9

Second Quantization

A. SECOND QUANTIZATION OF THE SCHRODINGER


EQUATION

As an exam p le of second quantization, we sh a ll con sid er the


n o n rela tiv istic Schrodinger equation, and then g e n e r a lize the r e ­
su lts to the c a se of the M axw ell and D irac r e la tiv is tic equations.
In Chapter 5, it was shown that the solu tion of the tim e-dependent
Schrodinger equation in the g en era l c a se can be rep resen ted in the
form

(29.1)
= 2 Cn(/)^ -
n n

w here the c o efficien t C n (t ) , which c h a r a c te r iz e s the probability


that a p a r tic le is in the state rep resen ted by<^n> includes the tim e -
dependent part of the wave function

C„(/) = C ne \ (29.2)

H ere the quantities E n are the energy eig en v a lu es, and the eig en ­
functions of the tim e-independent Schrodinger equation sa tisfy
the orthonorm ality condition

\ = 8 „„.. (29.3)

In tran sferrin g from c la s s ic a l quantities to quantum -m echanical


on es, the position x , the m om entum p x , and the rem aining quantities
are replaced by th eir average valu es

,v = J ^ (l).^ (()d'x,
(29.4)
p .= ^ r(t)P A (t)c P x .
SECOND Q U A N TIZ A TIO N 461

The tim e variation of th ese average values cannot he determ ined


from the c la s s ic a l P oisson b rackets, which in this c a se coincide
with Hamilton’s canonical equations

dx cl 0/1
dt dp ci ’

and m ust be calculated from the com m utator relation

■t = (29.5)

Here the Hamiltonian is equal to

H= + (29*6)

The quantum equation of m otion [Eq. (29.5)] can be consid ered


as the fundamental equation d escrib in g the quantization of the
c la s s ic a l equations of m otion. This p r o c e ss is called fir s t quanti­
zation. In order to connect p and x by the usual relation

P_ (29.7)
W0

we m ust assum e that in Eq. (29.5) the operators p and x do not


com m ute with each other and obey the com m utation relation

px — x p = y . (29.8)

Thus the com m utation relation (29.8), which is b a sic to the


Schrodinger theory, can be regarded as a consequence of the
quantum equation of m otion (29.5), and the tran sition from the
c la s s ic a l equation of m otion to the Schrodinger equation is equiva­
lent to a tran sition from corpuscular concepts to the wave con­
cepts.
In order to include in the theory the corp u scu lar p rop erties
of the de B roglie w aves, it is n e c e ssa r y to introduce a number
of additional hypotheses (for exam p le, a hypothesis concerning
the prob ab ilistic nature of the <]/ w aves and the m eaning of the
coefficien ts | Cn | 2 as the probability of an electro n being in state
n , and so forth).
The p r o c e ss of second quantization enables us to take into
account both the corpuscular and wave p rop erties of the p a r tic le s.
The name “ second quantization” originated from the fact
that in this ca se we quantize the equation which has already been
quantized as a resu lt of the fir st quantization. We note that as a
482 FUNDAMENTALS OF NUCLEAR PH Y SIC S

r e su lt of secon d quantization, the c o efficien ts C„ becom e operators


( q num bers), w h ereas in the Schrodinger theory they rem ained
ordinary constan ts (c num bers).
In order to ca rry out the second quantization, it is n e c e ssa r y
to find the average value of the energy operator

H = J <|>* (0 (/) d 3x. (29.9)

Substituting (29.1) in p la ce of <^(i) and r e ca llin g that = £ n^n , we


find

= 2 E nC X ' ( t ) C n ( t )
rt, n'

Taking into account the orthonorm ality condition Eq. (29.3), we


obtain

H = 2 E nC * ( t ) C n (t). (29.10)
n

We note that the la st e x p r e ssio n is independent of t, sin c e it


is apparent from the equations

iEt
(29.11)
C n (t) = C ne C* (/) = C*e

that the tim e factor in the product C£ (/) • C„ (f) is sim p ly equal to
unity
C * ( t ) C n (t) = C nCn.

E x p ressio n (29.10) is not an operator in the Schrodinger theory,


sin c e the c o e ffic ien ts Cn are ordinary num bers ( c num bers). In the
theory of secon d quantization, how ever, th ese quantities and the
H am iltonian (29.10) should be regarded as o p era to rs, that is , as
q num bers.
To find the com m utation relation for the c o e ffic ien ts C„ , we
m ust substitute C n (t) and C*n (t) in place of x and p in the quantum
equation of motion_ (29.5), and the operator H m ust be replaced by
its average value H . We get

d- ^ i ! l = L ( H C n( ( ) - C n( t)H). (29.12)

A ccording to Eq. (29.11),

C,'(/) = - y C , / ' ‘ ‘ ,
SECOND Q U A N TIZ A TIO N 403

and therefore from (29.12) we find the follow ing rela tio n , which is
a fundamental postulate of the second quantization:

_ Un€ n. = U C n. - C , , U . (29.13)

Substituting H from Eq. (29.10), we obtain

- E n€ n. = 2 E n (C;C„C„. - C n'C*nC n). (29.14)


n

The la st equation has two so lu tio n s, corresponding to the B ose


s ta tis tic s and F erm i s ta tis tic s , r e sp ec tiv e ly . The fir s t solution i s 1

C„.C„-C„C„. = 0,
(29.15)
Cn'C% — C nC n>= onn

This can be e a s ily verified by substituting the ex p ressio n s


C nC n' — C n'Cn,
c r* _r * c . 1 r (29.16)

into the fir st and second ter m s, r e sp e c tiv e ly , on the right-hand


sid e of Eq. (29.14). We then obtain the identity

From Eq. (29.15), it follow s that the co e ffic ien ts Cn and C* are
op erators. Setting

C*„Cn = N, (29.17)

where N is the number of p a r tic les in the state n , we find from


Eqs. (29.15)

C nC > = \ + N . (29.18)

It follow s, ‘h erefo re, that th ese op erators do not vanish even in the
c a se where there are no p a r tic les p resen t. Indeed, even though
C'nCn = 0 when N = 0 , we s till have in that c a se CnC % = l . This
nonzero value for the com bination of co efficien ts C g iv es r is e to
a relationship between the vacuum (the field of virtual p a rticles)
and the rea l p a r tic le s.

I t f o l l o w s fr om E q . ( 2 9 . 1 5 ) t h a t t h e w a v e f u n c t i o n s f) a n d t) com m ute, w h ile


\jj a n d Ip * do n o t c o m m u t e

t p ( r , t ) i p * ( r , I) - Ib * ( r , t ) i p ( r , t ) = E l p n * ( / ) 0 n (r) = 5 ( r ' - r).


404 FUNDAMENTALS OF NUCLEAR PH Y SIC S

It is c le a r from the above equations that the operator C% should


be regarded as the “ creation ” operator for p a r tic le s , and C n
a s the “ d estru ctio n ” op erator. If at the in itial tim e there are no
p a r tic le s p r e sen t, the condition C„C*= 1 sig n ifie s that a p a rticle
can fir s t appear (owing to the action of operator C * ), and then
disappear (owing to the action of C„). The condition C*nC n = o im ­
p lie s that in the absen ce of p a r tic le s the in v e rse p r o c e s s , in which
a p a rticle is fir s t absorbed and then em itted , cannot occu r.
In the solu tion s (29.17) and (29.18), the quantity N m ust be a
p o sitiv e in teger. It is n o t, r e str ic te d to any m axim um value.
T h erefo re, th e se com m utation relation s correspond to the B ose
s ta t is t ic s , which allow s any num ber of p a r tic le s to be p resen t in
one sta te.
The second solu tion of Eq. (29.14) can be rep resen ted as

C n'Cn + C nC n’ = 0 ,
r* r i c r * __*
“1“ K-'n'-'n' — °nn' »
v^i7«-LJ7/

which *is e a s ily v e r ifie d by the d ir ec t substitution of Eq. (29.19)


into Eq. (29.14). Setting

C%Cn = N, (29.20)

we find

C nC*n = \ ~ N. (29.21)

Noting that C*Cn and C nC* cannot be n egative, we find that the number
of p a r tic le s in the sta te n can assu m e only two values: N = Q and
N = l. C onsequently, the P au li e x c lu sio n prin cip le is already in­
cluded into th is solution and the p a r tic le s obey the F erm i s ta tis tic s .
In p articu lar, if there are no p a r tic le s p resen t in itia lly ( N = Q), then
ju st a s in the c a se of the B ose s ta tis tic s we have

C*nC n = 0, C nC*n = 1. (29.22)

The equation obtained in secon d quantization d e s c r ib e s , th ere­


fo re, a state with a variab le (integral) num ber of p a r tic le s . Con­
seq u en tly, ele c tr o n s w ill be s im ila r to photons not only because
their m otion is d escrib ed by a wave equation, but a lso because
e le c tr o n s, just like photons, can be created and destroyed .
Since the creation and annihilation of ele c tr o n s req u ires an
energy m ore than tw ice as great as the r e s t-m a s s energy of an
e le c tr o n (sin ce an ele c tr o n is alw ays created together with a
p ositron ), the secon d quantization of the n on rela tiv isticS ch ro d in g er
equation is of purely m ethodological sig n ifica n ce. In ord er to
con sid er rea l wave p r o c e s s e s a sso cia ted with the creation or
annihilation of p a r tic le s , one should extend the m ethod of second
SECOND Q U A N TIZ A TIO N 405

quantization to r e la tiv istic equations —nam ely, the wave equation


for photons (M axwell’s equations) and the r e la tiv istic wave equation
for electro n s and p ositrons (D irac’s equation).

B. QUANTIZATION OF MAXWELL’S EQUATIONS

It is w ell known that the photon field (electrom agn etic field) can
be d escrib ed by a vector potential which s a tis fie s the d’A lem berts
equation

= (29.23)

and is subjected to the follow ing condition:

V- i 4 = 0. (29.24)

Since the vector potential A is a real quantity, we can rep resen t


the solution of (29.23) as

A = Y - ^ ( a e ~iCltU* ' r + a *e i n t (29.25)

where
*= 1 * 1. and = («; = 0 , zb 1 , z b 2, z b 3 . . .).

The condition (29.24) m eans that the vector a is perpendicular


to vector % , that is ,

y-a = 0. (29.26)

The norm alization coefficien t — jrr 1 /


£ 3/2 f
2r'c n -
*
is determ ined from
the condition that the energy of the electrom agn etic fie ld

B-iJiw+(«n^-iJ{(T¥),+ (29 27)


-j-( v x .4)2| = l(a * -a)+ (flV )]
X

m ust be equal to the sum of the products obtained by m ultiplying the


en e r g ie s Chv. of the individual p a r tic les by the corresponding squared
am plitudes.
Since the Hamiltonian is proportional to the com binations of
co efficien ts o f the form C*C-f- CC* (the am plitudes of a in this c a se
play the role of the co efficien ts C), the quantum equation of m otion
486 FUNDAM ENTALS OF NUCLEAR PH Y SIC S

(29.13) for the field of photons p e r m its only a solu tion correspond ing
to the B ose s t a t is t ic s . 2 F u rtherm ore, taking into account the con­
dition (29.24), we obtain the follow ing com m utation relation s:

a ta f — afdi = A iV = 8 /,- — - (29.28)

w here i, i ' = 1, 2, 3 and the am plitudes a t and a*,'refer to the sam e


v ecto r x , which p lays the ro le of the quantum number n [se e Eqs.
(29.15)]. The c o e ffic ien t An- e x p r e s s e s the tr a n sv e r se ch aracter of
the photon field
= i4,-i' = 0. (29.29)

In p articu lar, if there are no photons p resen t at the initial


instant, only the follow ing b ilin ea r com bination of am plitudes w ill
be nonvanishing:

a,.a?, = 8 ,,.— L i (29.30)

C. SPONTANEOUS EMISSION

In Chapter 9 we found the E in stein c o e ffic ien ts A for the spon­


taneous e m is s io n by using the co rresp on d en ce p rin cip le. In such
treatm ent, the r e a so n s for tra n sitio n s of e le c tr o n s from higher to
low er le v e ls rem ained unexplained (se e Chapter 9). Quantum
electro d y n a m ics (the nam e given to the theory that includ es second
quantization of the electro m a g n etic field ) exp lain s the tra n sitio n s in
ter m s of the interaction of an e le c tr o n with the fie ld of vacuum
(virtual) photons.
As can be seen from Eq. (29.30), th ere are quadratic com ­
binations of the quantized am plitudes of the electro m a g n etic field
which d iffer from zero even in the absen ce of r e a l photons.
The tim e-depend en t Schrodinger equation, taking into account the
doubly quantized fie ld of photons, can be rep resen ted as

[ - 4 5 - ‘' - i t ' - 7 ' * ) > = »• (29' 31)

N eglectin g the se co n d -o r d e r term s proportional to A'1, and taking


into account the tra n sv e r se character of the electrom agn etic w aves

_____________ (pM )* = G4.p)*, (29.32)


2 If these am p litu d es occur in th e H am iltonian in th e com bination C’ * C — C C * (th is
h a p p e n s , fo r e x a m p l e , fo r D i r a c p a r t i c l e s ) , t h e n o n l y t h e F e r m i s t a t i s t i c s w o u l d h o l d for
the c o r r e s p o n d i n g a m p l i t u d e s . T h i s is e a s i l y v e r if i e d by s u b s t i t u t i n g in to th e H a m ilto n ia n
t h e d i f f e r e n t s o l u t i o n s f o r t h e D o s e s t a t i s t i c s [ s e e E q . ( 2 9 . 1 6 ) ] a n d fo r F e r m i s t a t i s t i c s
[ s e e E q . ( 2 9 . 1 9 ) |.
SECOND Q U A N TIZ A TIO N 487

sin ce
\-A = 0,
Eq. (29.31) can be reduced to

<•=<>. (2 9 *3 3 )

H ere the unperturbed Hamiltonian H° = V + p2 is tim e -


independent, and the operator of the perturbation energy is

e
V' { t )
cm,,
A(t) • p .

Let us assu m e that the electro n is in itially in the sta te n . Then


under the influence of virtual photons3

= p (29.34)
cm» L /2cm0 * \ li­

the electro n can jump into the state n ’ .


In accordance with Eq. (28.12), we obtain the follow ing e x ­
p r e ssio n for the co efficien t ch aracterizin g this transition:

C»-(0 V i / 2jT d*-Pnn, (29.35)


ZdV clI* / (o)Bn, — to)
X

where the m atrix elem en t

Pn'n = 5 ft-e-to'p$nd*x. (29.36)

and iu = cx is the frequency of the em itted photons.


The quantity x - r ^ y is sm all sin ce the w avelength of the em itted
light is 10~5* cm , and the dim en sion s of the atom are of the order
of 10 8 cm . T h erefore, in a f ir s t approxim ation, the exponential
factor in Eq. (29.36)
e~l*'r = 1 — i x •r - f - . ..

should be s e t equal to unity. Such tran sition s are called the dipole
tr a n sitio n s .4

3 T a k i n g i n l o a c c o u n t t h e c o m m u t a t i o n r e l a t i o n s in ( 2 9 . 3 0 ) in t h e e x p r e s s i o n f o r t h e
v e c t o r p o t e n t i a l A in t h e c a s e w h e n t h e r e a r e n o p h o t o n s , w e s h o u l d r e t a i n o n l y a m p l i t u d e s
p r o p o r t i o n a l t o o * , t h a t i s , o p e r a t o r s fo r t h e c r e a t i o n o f p a r t i c l e s .

4 I f w e i n c l u d e t h e n e x t t e r m in t h e e x p a n s i o n w e o b t a i n q u a d r u p o l e r a d i a t i o n . Q u a d r u ­
p l e ra d ia tio n is ( r / A ) ^ lim e s w e a k e r th a n d ip o le r a d ia tio n an d i s of im p o r ta n c e only w hen
d ip o le tr a n s itio n s a re fo rbidden.
488 FUNDAM ENTALS OF NUCLEAR PH Y SIC S

The probability of a spontaneous tran sition from the energy


le v e l n to the le v e l n ’ is equal to

A nn. = j t C*n. ( t ) C n. (t) =


(29.37)
sin 1 ( “ /in' — “ )
( a - p * n ’n ) { a * . P n ’n ) .

H ere instead of two su m s over the wave num bers of am plitudes a


we left only one, sin c e the only com binations of am plitudes a and a*

o fi* = 8*.-------.

d ifferen t from z ero are th ose which r e fe r to the sam e m om entum x.


U sing the la st com m utation r e la tio n s, we find
( a - p t ’n) ( a * . Pn'n) = | \ ^ P n ' n \ T-
•>
Let us rep la ce the sum over x by an in tegral in Eq. (29.37) [see
a lso Eq. (28.23)]

S d3x= w S mid(odQ <29*38>


X

and include the fact that for su ffic ie n tly large valu es o f tim e t , we
have, in accord ance with (28.20),
s in t(u>-
= 8 (to — « )„ „ -). (29.39)
7t (!) -- (I)n n '

Then in the c a se of spontaneous tra n sitio n s Eq. (29.39) red u ces to


the law of con servation of energy

U) — i&nn' (29.40)

sin c e from Eq. (29.40) it follow s that the energy of the em itted
photon hio is equal to the energy ( E n — f v ) lo st by the atom as a
r e su lt of tran sition . For the probability of spontaneous e m is s io n ,
we obtain
* ' « n n ' I P n ' n I*
Ann' 2rAc3m’i

Evaluating this in tegral and rem em b erin g that P n ’n = — i/Ho“/m'/V/i,


we obtain the final e x p r e ssio n for the probability of spontaneous
e m is s io n

Ann' ---
nn
(29.41)
7lc<r ‘ r„'n
SECOND Q U A N TIZ A TIO N 489

which was already derived in Chapter 9 with the aid of the se m i


c la s s ic a l correspond en ce principle [se e Eq. (9.20)].

D. BETA DECAY

As another exam ple of the application of second quantization


let us consid er the theory of decay. This phenomenon c o n sists
of the e m issio n of an electro n (positron) by a nucleus, leading to
an in crea se (d ecrease) of the nuclear charge by unity.
The theory of decay, which r e se m b le s in som e r e sp e c ts the
e m issio n of photons by atom s, w as constructed on the b a s is of
second quantization.
There are no photons in an atom . A photon is created from the
vacuum when an atom m akes a tran sition from one energy state to
another. S im ila rly , a nucleus does not contain e lectro n s; they are
created only in the p r o c e ss of j3 decay.
As a resu lt of experim en tal in vestigation of /3 decay it has been
estab lish ed that th ese e le c tr o n s have a continuous spectrum bounded
by a certain m axim um energy equal to the differen ce in the energy
of the nucleus b efore and after the decay. It w as a lso esta b lish ed
that in /3 decay the angular m om entum of the nucleus changes by
a m ultiple of h , w hereas the angular momentum carried away by
the electron equals ('/2) h.
The apparent violation of the law s of conservation of energy and
angular m om entum in )9 decay w as r e so lv e d in the h yp othesis of
P auli, who assu m ed that the e m issio n of an electro n is accom panied
by the e m issio n of another p a r tic le —a neutrino p o s s e ss in g half-
integral spin and a r e s t m a ss c lo s e to zero.
A ccording to the F erm i theory, constructed on the b a sis of this
h ypothesis, j3 decay should be con sid ered as the tran sform ation of
one of the nuclear neutrons (n ) into a proton (p ), an electro n (e ~),
and an antineutrino (v)
n —>p - f - e ' - f - v .

S im ilarly, the e m issio n of p ositron s in ]3 decay should be con­


sid ered as the transform ation of a nuclear proton 5 into a neutron,
a positron and a n eu trin o 6
p — n -{- e+ -|- v.

We n o t e t h a t s i n c e t h e r e s t m a s s o f a n e u t r o n i s g r e a t e r t h a n t h e t o t a l r e s t m a s s o f t h e
proton, e l e c t r o n , a n d a n tin e u tr in o , it f o ll o w s th a t th e d e c a y o f a f r e e n e u tr o n s h o u l d a l s o
b e o b s e r v e d . T h e d e c a y o f a f r e e p r o t o n a p p e a r s i m p o s s i b l e from t h e e n e r g y s t a n d p o i n t a n d
th e refo re, p o s itro n d e c a y c a n b e o b s e r v e d o n ly in a b o und proton, w h en th e re q u ire d e n erg y
c a n b e t a k e n u p fr o m t h e n u c l e u s .

6We s h a l l e x p l a i n t h e d i f f e r e n c e b e t w e e n t h e n e u t r i n o a n d a n t i n e u t r i n o a t t h e e n d o f t h e
p r e s e n t c h a p t e r in t h e d i s c u s s i o n o f t h e n o n c o n s e r v a t i o n o f p a r i t y .
490 FUNDAMENTALS OF NUCLEAR PH Y SIC S

F u rth erm ore, the capture of a bound ele c tr o n is a lso p ossib le; as
a r e su lt of e le c tr o n capture a proton is changed into a neutron and
e m its a neutrino ( p -)- e~ — n v ) . As a rule; an e le c tr o n from the K
sh e ll is absorbed in th is p r o c e s s , and th erefore th is phenomenon is
c a lle d K capture, /(ca p tu re is sim ila r in nature to positron /3 decay,
sin ce in both c a s e s the charge of the nucleus is reduced by unity.
We sh a ll not con sid er h ere the d etails of j3 decay; our task
w ill be to d e sc rib e in gen eral term s the creation of an ele c tr o n
and antineutrino follow ing the F erm i theory.
The energy of in teraction o f a neutron with the e le c tr o n -
antineutrino field can be w ritten as
V e7 = m h (29.42)
w here / is a coupling constant introduced by F erm i. The m agnitude
of f is v ery sm a ll ( / ^ 1.4 x 10“49e r g x cm 3 ) so that this in ter­
action is c a lle d a weak in teraction . The spontaneous decay of
p a r tic le s is caused m ainly by weak interaction s; th e re fo r e, the
life tim e o f elem en tary p a r tic le s or nuclei is com p aratively large
and v a r ie s from a fraction of a second to b illion s of y e a r s. N uclear
p r o c e s s e s , on the other hand, are cau sed by stron g in teraction s,
which so m e tim e s are a thousand tim e s la rg e r than the electro m a g ­
n etic in te r a c tio n s. The duration of the p r o c e s s e s caused by such
in tera ctio n s is v e r y short (of the ord er of 1 0 - 2 3 se c ).
N eglectin g spin e ffe c ts , the wave functions ipj and <p- can be
rep resen ted as
— L - a/ s a * e kK‘- ik ' r ,
(29.43)
(p i = z ,— a/a b * e ic-At-ix ■ r#

If there are no p a r tic le s p resen t at the in itial instant, the follow ing
rela tio n s hold for the am plitudes a* and b *: a * a = b * b = 0, a a * —
— bb* -- 1 .
Then, accord ing to Eq. (28.12), we have the follow ing e x p r e ssio n
for the c o efficien t C ( t ) :

C(t) = — V pna * b * J dte-^K n-K p-K -'h (29.44)


o

H ere E n = c h K n is the neutron en ergy and E p — c H Kp is the proton


en ergy. The m atrix elem en t of the interaction V pn is

V p n = \j y . p U n e - i{l'+ x ) r d 3x. (29.45)

w here is the wave function of the neutron, Xp is the wave function


of the proton, and 7 is the D irac m atrix which d eterm in es the
nature of the in teraction . Since the in teraction energy m ust be
a s c a la r quantity, an analogous m atrix should a lso rela te the spinor
SECOND Q U A N TIZ A TIO N 491

am plitudes of the wave functions of the eldotronand neutrino in Eq.


(29.42). However, the influence of the sp e c ific choice of the Dirac
m a tr ice s m a n ifests its e lf only in the sj)in e ffe c ts , which are ne­
glected in the presen t treatm ent.
Just as in the investigation of dipole radiation, the quantity in
the exponent in the m atrix elem en t (29.45) is much le s s than unity,
that is , inside the nucleus

(A + k)-/-~|(A : + x ) - / ? |< 1 ,

where R is the nuclear radius. The exponential, th erefo re, m ay be


expanded in a s e r ie s
e-.(*+x)r = l — i (k + x) • r + . ..

If the m atrix elem en t (29.45) does not vanish upon replacem ent
of the exponential by unity, the corresponding tran sition s are said
to be allow ed )3 tra n sitio n s.
We note that the allow ed tra n sitio n s, which correspond to the
dipole tra n sitio n s in the theory of photons, are a sso cia ted with
definite se lectio n r u le s . 7

7In th e c a s e o f v e c t o r i n t e r a c t i o n , t h e m a t r i x e l e m e n t

V p n = f X p X n d 3 x

(the fourth c o m p o n e n t o f t h e f o u r - d i m e n s i o n a l v e l o c i t y ; th e f i r s t t h r e e c o m p o n e n t s for t h e


n u c l e o n a t r e s t v a n i s h ) w ill d i f fe r from z e r o ( a l l o w e d t r a n s i t i o n s ) i f t h e s p i n o f the
n u c l e u s ( t h a t i s , t h e t o t a l a n g u l a r m om entum ) r e m a i n s u n c h a n g e d in th e /3 d e c a y ( A J = 0,
F erm i t r a n s i t i o n s ) . T h i s F e r m i s e l e c t i o n rule i s s a t i s f i e d for t h e m a j o r it y o f n u c l e i .
T h e r e are, h o w e v e r , so m e c a s e s o f a l l o w e d d e c a y , for e x a m p l e ,

2 H e 6 -> 3L i 6 + e ~ + V

in w h i c h th e s p i n o f t h e n u c l e u s c h a n g e s by u n i t y ( A J = 1). T h e s p i n o f t h e nucleus
i s e q u a l to z e r o (o n e a l p h a p a r t i c l e p l u s two n e u t r o n s w ith a n t i p a r a l l e l s p i n s ) , w h i l e t h e
s p i n o f t h e 3 L i 6 n u c l e u s i s e q u a l to u n i t y (one a l p h a p a r t i c l e p l u s o n e p ro to n a n d o n e
n e u t r o n w ith p a r a l l e l s p i n s , j u s t a s in d e u t e r i u m ) .
In o r d e r to e x p l a i n t h e s e s e l e c t i o n r u l e s , Gam ow a n d T e l l e r p o i n t e d o u t t h a t w h e n w e
form th e i n t e r a c t i o n e n e r g y o p e r a t o r , w h i c h i s a s c a l a r (V = Vp n V el/), o n e may t a k e a
p r o d u c t o f two v e c t o r s , a s w e l l a s o t h e r r e l a t i v i s t i c a l l y i n v a r i a n t c o m b i n a t i o n s o f t h e four
w a v e f u n c t i o n s . F o r e x a m p l e , w e may t a k e a p r o d u c t o f two p s e u d o v e c t o r s , w h i c h i s a l s o a
re la tiv is tic invariant. T he p se u d o v e c to r interaction

^pn ~ x

l e a d s to th e s e l e c t i o n r u l e s A J = 0, i l . It s h o u l d b e e m p h a s i z e d t h a t for a n u c l e o n at
r e s t , th e f i r s t t h r e e c o m p o n e n t s w ill b e d i f f e r e n t from zero .
In v iew of th e f a c t t h a t th e p s e u d o v e c t o r i n t e r a c t i o n f o r b i d s 0 -> 0 t r a n s i t i o n s , w h i c h
w ere n e v e r t h e l e s s o b s e r v e d e x p e r i m e n t a l l y , a s w e l l a s for s e v e r a l o t h e r r e a s o n s , p r e s e n t -
d ay t h e o r y u s e s a c o m b i n a t i o n o f v e c t o r a n d p s e u d o v e c l o r i n t e r a c t i o n s ( a s in th e F e y n m a n
a n d G ell-M ann v e r s i o n o f t h e th e o r y o f /3 d e c a y ) . T h i s v e r s i o n e n a b l e s u s to e x p l a i n the
b a s i c e x p e r i m e n t a l d a t a o b t a i n e d in t h e i n v e s t i g a t i o n of fi d e c a y .
492 FUNDAMENTALS OF NUCLEAR PHYSICS

In the subsequent d iscu ssio n , we sh a ll r e str ic t o u r se lv e s to the


treatm en t of allow ed tra n sitio n s; in th is c a se it is su fficien t to make
the exponential term in the m atrix elem en t (29.35) equal to unity.
We then obtain the follow ing e x p r e ssio n for the coefficien t C :

C' = — V a+6 + (29.46)


n l® v P"u 0 c ( K + * - K np) ’

w here E 0 = c h K „ p = c h ( K n — K p) is the en ergy c a r rie d away by the


light p a r tic le s (electro n , neutrino or antineutrino) in |3 decay.

F i g . 2 9 .1 . E n e r g y d i s t r i b u t i o n o f e l e c t r o n s in
fi d e c a y ( a c c o r d i n g t o t h e F e r m i t h e o r y ).
E q i s t h e m ax im u m e n e r g y o f t h e /3 sp e c t r u m .
T h e o r ig in of t h e c o o r d i n a t e s y s t e m c o r r e s p o n d s
2
to t h e e n e r g y m g c .

U sing Eq. (29.39) and changing the sum m ation over the m om enta
of e le c tr o n and neutrino to an integration [se e (29.38)], we obtain
an e x p r e ssio n for the probability of j3 decay

w= i J d ,* d > k H K + * - K n P). (29.47)


k. X

Integrating th is e x p r e ssio n o v er a ll p o ssib le a n gles at which the


e le c tr o n and antineutrino e m erg e from the nucleus and a lso over
the energy of the antineutrino, we obtain an equation for the energy
distribution (E = cHK) of the e le c tr o n s (the 0 spectrum )

j w{E)dE, (29.48)
moc3

w here

w ( E) - C W ; C V ~ P - m * c ' (E 0 - Ef. (29.49)


SECOND QUANTIZATION 493

The en ergy-d istrib u tion curve of the j3 e le c tr o n s, obtained in the


F erm i theory, is plotted in Fig. 29.1." From this curve it is clear
that the energy of the em itted ele c tr o n s lie s between
and £ max= E 0 .
A ccording to the P a u li-F e r m i theory, there is no violation of
the law of conservation of energy, sin ce the total energy of the
em itted antineutrino and electro n m ust alw ays be equal to the total
energy lo st by the nucleus during /? decay . 9

E. NONCONSERVATION OF PARITY IN THE DECAY


OF PARTICLES

One of the fundamental discoveries In the theory of weak Interactions was the dis­
covery of the nonconservation of parity by Lee and Yang (1956). This phenomenon gives
ris e to a spatial asymmetry in the spontaneous decay of elem entary particles and, in
particu lar, in nuclear 0 decay.
The nonconservation of parity can be observed experimentally In the following two
phenomena.
1. The asymmetry of the angular distribution of electrons in the 0 decay of nuclei
with an oriented spin (the number of 0 electrons emerging along the direction of nuclear
spin does not equal the number of electrons emerging in the opposite direction).
2. The existence of circu lar polarization (helicity) in the particles formed during
decay (for example, electrons formed in 0 decay or p mesons formed in the decay of
it mesons), even in the case when the decaying system has zero spin.
The phenomenon of nonconservation of parity in 0 decay or in the decay of a n meson
was explained with the help of the theory that assigned a definite circu lar polarization
(helicity) to the neutrino. At one time it was thought that the neutrino (which is formed
in positron 0 decay) and the antineutrino (which is formed in electron 0 decay) were
identical particles (Majorana’s hypothesis). It was suggested that this hypothesis could
be tested in double 0 decay.
If the neutrino and antineutrino were identical, one would expect comparatively
large values for the probability of double 0 decay without the em ission of a neutrino (one
neutron of the nucleus em its an electron and a neutrino, while another neutron em its
an electron and absorbs this neutrino), that is, with the em ission of only two electrons.
If, however, double 0 decay consisted simply of two successive identical 0 decays with
the em ission of two electrons and two antineutrinos, the probability of decay should be
much less. Experiment has confirmed the correctness of the second hypothesis and
clearly demonstrated that the neutrino must be different from the antineutrino.
We note, incidentally, that both particles a re neutral and have a spin 1/2. Physicists,
however, were able to establish a difference between the p articles from phenomena
associated with nonconservation of parity. It has been dem onstrated experimentally that
the asymmetry observed during positron 0 decay of a nucleus with an oriented spin,
when a neutrino is emitted together with a positron, is the rev erse of the asymmetry
that is observed during electron 0 decay, in which an antineutrino is emitted together
with an electron. It was assumed, therefore, that the neutrino differs from the anti-
neutrino by the type of circular polarization. In order to explain the experimental data,
it was necessary to postulate that a neutrino resem bles a photon with left-hand circu lar
polarization, while an antineutrino resem bles a photon with right-hand polarization. The

It s h o u l d b e n o t e d t h a t t h e maxim um o f t h i s c u r v e i s s l i g h t l y s h i f t e d t o w a r d s sm a ll
e n e r g i e s . T h i s a s y m m e tr y i s d u e to t h e f a c t t h a t t h e a n t i n e u t r i n o m a s s i s e q u a l to zero ,
w h i l e th e e l e c t r o n m a s s i s d i f f e r e n t from z e r o . If t h e m a s s of t h e a n t i n e u t r i n o w e r e e q u a l
to t h e e l e c t r o n m a s s , t h i s c u r v e ( n e g l e c t i n g t h e C o u lo m b a t t r a c t i o n of t h e e l e c t r o n a n d th e
n u c l e u s ) w o u ld be s y m m e t r i c a l ; t h a t i s , t h e maxim um w o u ld o c c u r a t t h e p o i n t E / 2 .
g
F o r m o r e d e t a i l s , s e e H. B e th e a n d P . M orrison, E l e m e n t a r y N u c l e a r T h e o r y , N ew York:
J o h n Wiley & Sons, In c., 1958.
494 FUNDAMENTALS OF NUCLEAR PHYSICS

only difference is that the spin of the photon is 1 (in units of H), whereas the spin of
the neutrino is 1/2.

F i g . 29 .2 . H e l i c i t y o f t h e n e u ­ F i g . 2 9 .3 . H e l i c i t y of a le f t-
tr in o and an tin eu trin o . The h a n d e d n e u t r i n o in r i g h t - h a n d
neutrino h a s a left-h an d circ u la r and left-hand coordinate
p o la r iz a tio n and the an tin eu trin o system s.
h a s a rig h t-h an d c ir c u la r p o la r i­
zation.

The circu lar (longitudinal) polarization of the neutrino is generally called the helicity.
The neutrino has left-hand circ u lar polarization, or negative helicity. This meaiis
that if a left-handed screw rotates along the direction of polarization, it moves in the
direction of the momentum. The antineutrino, however, has right-hand circ u lar polari­
zation, or positive helicity (see Fig. 29.2). In order to conserve helicity when changing
from one Lorentz fram e of reference to another, so that it can be adopted as a character­
istic of the neutrino, it is necessary that the re s t m ass of the neutrino be exactly equal
to zero. 10
Several authors describe polarization with the aid of an axial vector, which is per­
pendicular to the plane of rotation and has a different direction in the right-hand and
left-hand coordinate system s (see Fig. 29.3). It should be noted, however, that, although
the axial vector J and the polar vector p have different mutual orientations in the right-
hand and left-hand coordinate system s, the helicity is nevertheless conserved; that is,
the helicity of the neutrino is still negative, and only the method of description has
changed.11
Starting with the polarization properties of the neutrino, we shall make an attempt
to give a qualitative explanation of the nonconservation of parity during the spontaneous
decay of particles.
Let us consider, for example, the 0 decay of nuclei with oriented spin. We note that
the spin of the nucleus (its longitudinal polarization) is m ore naturally described by a
rotation, since the direction of the axial spin vector is arb itrary . The spatial asym­
m etry which should be observed in the phenomena characterized by the nonconservation
of parity is associated with the fact that in electron 0 decay, right-handed antineutrinos
a re em itted upwards and downwards with spins oriented, respectively, parallel and
antiparallel to the spin of the nucleus. The electrons will be formed predominantly with
a helicity opposite to that of an antineutrino (that is, with a negative helicity). This pro­
duces a spatial asym m etry due to which the number of electrons emitted in the direction
of nuclear spin does not equal the number of electrons emitted in the opposite direction
(see Fig. 29.4). In general, we obtain the following equation for the number of emitted
electrons as a function of the angle 0 between the direction of electron momentum and the
upward direction, which two directions form a right-handed system with the polarization
of a nucleus: cn,
wc_(9) = w 0_ ( l ■— a cos 9), (29.50)
where a is positive and is equal to approximately 0.4.
1 ^If th e r e s t m a s s of a p a r t i c l e d i f f e r s from z e r o , t h e p a r a l l e l s p i n a n d m o m entum v e c t o r s
may be d i r e c t e d a t an a n g l e a f t e r t r a n s i t i o n from o n e L o r e n t z fram e o f r e f e r e n c e to an o th e r .
^ S i n c e t h e d i r e c t i o n of t h e ( a x i a l ) s p i n v e c t o r * r e l a t i v e to t h e p o l a r v e c t o r o f t h e m o­
m en tu m /> i s d i f f e r e n t in th e r i g h t - h a n d a n d l e f t - h a n d c o o r d i n a t e s y s t e m s , L e e , Y an g , L a n ­
d au \ Z h u r n a l E k s p e r i m c n t a V n o y i T e o r e t i c h c s k o y F i z i k i , 32, 405 (1957)1, a n d o t h e r s a s s u m e ,
on t h e c o n t r a r y , t h a t in t h i s c a s e t h e n e g a t i v e h e l i c i t y of a n e u t r i n o c h a n g e s to a p o s i t i v e
h e l i e r t y . In o t h e r w o r d s , t h e y a s s u m e t h a t t h e m e th o d of g e o m e t r i c d e s c r i p t i o n of t h e p a r t i ­
cle can c h a n g e its internal p ro p e rtie s (helicity).
SECOND QUANTIZ ATION 495

A sym m etry was detected In the elec tro n 0 decay of Co 60 nuclei with o rien ted spins
and also In the angular d istribution of ele c tro n s In the 0 decay of fre e , p o larized neutrons
(for which u «= 0.1).
Since a neutrino with a negative hellclty em erges during positron decay, the asym­
m etry pattern will be opposite to the one described above. The number of em itted posi­
trons Is related to the angle 9 by the equation

,.(*>) = K-0+ (> + a cos 9); (29.51)

that Is, the positrons that are formed have mainly a positive hellclty and are emitted
prim arily upwards. An asymmetry opposite to that of electron decay was observed ex­
perim entally In positron 0 decay of Co58 nuclei with oriented spins.

Electron Positron
0 decay 0 decay
F i g . 2 9 .4 . S c h e m a t i c d i a g r a m o f t h e /3 d e c a y
o f n u c l e i w ith o r i e n t e d s p i n s . T h e d i r e c t i o n
o f r o t a t i o n c h a r a c t e r i z e s t h e d i r e c t i o n of t h e
spin of th e p a r tic le s ; p is the p a r tic le
mom entum.
Longitudinal polarization was observed particularly clearly in the spontaneous decay
of pions into muons and a neutrino. Let us choose a coordinate system In which the pion
is at re st and consider the negative plon which decays Into a negative muon and an anti-
neutrino. Since the antineutrino has a positive helicity and the momenta of the muon and
the antineutrino m ust be equal and opposite, we find that the muon also m ust have a
positive helicity. Indeed, only In this case will the total spin of the muon-antineutrino sys­
tem be equal to the Initial spin, that is, zero. In the decay of positive pions Into a neutrino
and positive muons, the muons, however, will obviously have a negative helicity (see
Fig. 29.5).

F ig . 29.5. D e c a y o f a p i o n at r e s t in to a muon and a neutrino.


496 FUNDAMENTALS OF NUCLEAR PHYSICS

The total longitudinal polarization of the created muons has also been confirmed experi­
mentally.
A m ore detailed discussion of the nonconservation of parity lies outside the scope
of this book, and we m ust re fe r the re a d e r to the special literatu re on this subject. 1 2

12S e e t h e p a p e r s in T. D. L e e a n d C. N. Y a n g , N o b e l L e c t u r e s , P h y s i c s , N ew York:
E l s e v i e r P u b l i s h i n g C o m p an y , 1964.
A|>|MMl(lix A

Hilbert Space and Transformation Theory

There is an elab orate form al stru ctu re of quantum m echanics


that is im portant for se v er a l reason s: the wave m echan ics and
the m atrix m echan ics can be unified in one coherent sch em e. The
conceptual structure has a deep intuitive appeal that g iv e s the
theory a se n se of com p leten ess and solidity,, The physical content
can be em bedded in an ex ten siv e and rigorou s m athem atical
fram ework. And, m ost im portant for the validity of the physical
theory, it is a powerful and flexib le phenom enological tool that
em b races a wide variety of em p irica l knowledge. The conceptual
structure and s ta tistic a l foundations of quantum m echan ics are
form ulated m ost fully in the fram ew ork of “ abstract H ilbert
sp a c e .” It is , how ever, not n e c e ssa r y to go into the m athem atical
tech n ica lities to achieve an accurate physical grasp of the theory.
For a com p lete treatm ent of the m athem atical and sta tistic a l
foundations one should con sult the original lite r a tu r e . 1 In this
appendix we sh all outline som e definitions and notational apparatus
around three broad topics: v ecto r sp a c e s, op erators and the inner
product.

V ector S p a c e s

An a b s t r a c t (com plex) v e c t o r s p a c e is a se t of ab stract e le ­


m ents called v e c to r s (or points in the space) which together with
com plex num bers obey the follow ing axiom s:
(1) If Vq and *1'b are v e c to r s, >pa + 'J'j, = ^ + »pa is a lso a v e c to r .
(2) <Pa + ( V b + V c ) = ( V a + V b) + V c .
(3) If x and y are com p lex num bers and f is a v ecto r, then x'P
is a vecto r and x(y'i’) = (xy) V.
(4) (x + y) 4* = xV + y'P and xiM^ + ^b) = xVq + x'i'j,.
(5) There is a null vector Vnun = 0 such that 'I1 + Vnuii = V and
0 ¥ = Vnuii for all V.

1 P .A .M . D i r a c , The P rin c ip le s of Quantum M echanics, fo u rth e d i t i o n , C l a r e n d o n P r e s s ,


Oxford, 1958. W. H e i s e n b e r g , The P h y s ic a l P rin cip les of Quantum Theory , t r a n s l a t e d by
C. E c k a r t a n d C. H o y t, U n i v e r s i t y of C h i c a g o P r e s s , C h i c a g o , 1930. ( D o v e r P u b l i c a t i o n s ,
N ew Yor k, 19 49). J . von N e u m a n n , Mathem atical F oundations of Quantum M echanics,
t r a n s l a t e d by R. T. B e y e r, P r i n c e t o n U n i v e r s i t y P r e s s , P r i n c e t o n , 1955.
498 APPENDIX A

The stru ctu re of a v ecto r sp ace is appropriate for introducing


the notion of l i n e a r i n d e p e n d e n c e : the s e t of v e c to r s *Pi, ^ 2 , . . . ,
*Pk are lin ea rly independent if the only s e t of com p lex num bers z \ ,
22. • • • , z k satisfy in g the relation ei'i'i + z<^2 + • • • + zk^k = 'Pnull
is the triv ia l s e t z \ = 2 2 = ■ ■ ■ = zk = 0. If it is p o ssib le to sp ecify
n lin ea rly independent v e c to r s but not n + 1 , the sp ace is said to be
n -d im en sio n a l. If there is no lim it to the num ber of lin ea rly inde­
pendent v e c to r s that may be sp e c ifie d , then the sp ace is “ infinite
d im e n sio n a l.” In an rc-dim ensional sp ace a se t of n lin ea rly inde­
pendent v e c to r s *Pi, ^ 2 , . . . , Wn form s a b a s i s that s p a n s the space
in the se n se that any elem en t 'Pa of the sp ace can be w ritten a s a
lin ea r com bination of the b a s is v e c to r s i = 1 , 2 , . . . . n,

n
'Pa = E
i=1
a.-Vf

The c o e ffic ie n ts a,, i = 1, 2, . . . , n are com p lex num bers that char­
a c te r iz e the v e c to r *PQ in the b a s is *Pi, ^ 2 , . . . , 'Pn. If a,- and 6 ,-,
i = 1, *2, . . . , n are the c o m p o n e n t s of the v e c to r s 'Pa = Sa,-*Pf and
*Pb = then c,- = a, + 6 ; are the com ponents of the v ecto r *PC =
S c /'P , = 'Pq + 'Pb and za; are the com ponents of the v e c to r z V a,
w here 2 is a com p lex num ber.
The g eo m etric concept which is the ^ -d im en sion al g e n e r a liza ­
tion of the notion of “ lin e s and plan es p assin g through the o r ig in ,”
can be developed with the follow ing definition:
A su b set 911 of a v e c to r sp ace is ca lled a l i n e a r m a n i f o l d if it
contains a ll the lin ea r com binations yi'Pi + >>2 ^ 2 + • • • + y ^ k along
with any fe(fe = 1 , 2 , . . . ) of its e le m en ts 'Pi, ¥ 2 , . . . , *P*. A lterna­
tiv ely , if 21 is an arb itrary se t of v e c to r s containing the d istin ct
v e c to r s 'Pi, *P2 , . . . , 'Pft, then the s e t of a ll lin ea r com binations
xiV i + X2 ^ 2 + • • • + xft'Pfc (with arb itrary com p lex num bers xi, X2 ,
. . . , xk) is a lin ear m anifold 911. 2JI is c a lled the “ lin ear m anifold
spanned by 21.” If the v e c to r s 'Pi, ^ 2 , . . . . *P* are lin ea rly inde­
pendent, the m anifold is fe-dim ensional.
The utility of the concept of a lin ea r m anifold is that it is the
dom ain of definition of a lin ea r op erator. In g en eral an operator
is a mapping o r co rresp on d en ce from a dom ain c o n sistin g of c e r ­
tain points in the v e c to r space into a range c o n sistin g of certa in
other points, denoted sy m b o lica lly by 'P -> R(*P). In quantum m ech­
a n ics we are concern ed with l i n e a r o p e r a t o r s for which by defini­
tion

R(«P + 0 ) = R(W + R (<I>) and R(a<P) = aR('P).

That is , the dom ain of definition of the operator R is a lin ear


m anifold. Note that a lin ea r operator need not be defined for all
v e c to r s in the sp a ce. For exam p le, if the space is the “ function
sp a c e ” c o n sistin g of square integrable functions on the interval
H I L U E R T S P A C E AND T R A N S F O R M A T I O N T H E O R Y 499

-™ < x < then there are v e c to r s (functions) </» for which x</> or
dt/j/Ox are not square integrablc.
There is a “ projection property” a sso cia ted with a linear
manifold: any vector 'I' may be reso lv ed into the sum of two v e c ­
tors 'I* 'I'll i M'i, w here lI'|| is in the manifold ©2 (i.e ., there are
com plex num bers cq, c 2, • • • ><'k such that 'I' |) - c i ’l'i + c 2 'I' 2 * • • • i
*'*ll,*» where 'IV ll'2, . . . . M'* span the ©2 ) and'I'j is en tirely outside
©2. As a notational device we may define the lin ear operator
as follow s: *1' \\ = for all 'I'. The domain of definition of Pgjj is
the entire sp ace, while the range c o n s is ts of ©land the null vecto r.
If i lie s outside ©I, = Vnuii- If yV2 lie s wholy within©!, P ^ ^ =
'J'2. If *•' Is an arbitrary v ector, lie s en tirely within © 2 and
Pjjj2 V = PiW(Pgn'I') = Pgji1!'. Since this relation holds for all we may
a s s e r t the operator equation

An (Hermitian) operator sa tisfyin g this relation is called idem potent


or a projection operator. It has eigen valu es 0 and 1.

O perators

Let SI stand for a physical ob servable which upon sharp m ea s­


urem ent y ie ld s any one of a sequence of valu es SI{, Sl2, • ■• which
are c h a r a c te r istic of the ob servab le. The s e t of valu es Sti, Sl2, . . .
is ca lled the s p e c t r u m of 81. An algebra of ob serv a b les is s e t up
as follow s: le t z be a com p lex num ber, then zSI is an observable
with c h a r a c te r istic valu es z Sl{, zSl2, . . . and if a m easurem en t of 81
y ie ld s 81*, the m easurem en t of zSI y ie ld s zSI*. The observab le 8 l 2 =
8181 y ie ld s the m easured value (SI* ) 2 when 81 y ie ld s 81*. If 81 and 83
are sim ultan eou sly m easurable then the sum 81 + © is defined to be
the observable that y ie ld s SI* + S3; when 81 y ie ld s 81* and© y ield s
©i. F inally, the product 81© may be defined by the a r tific e

SI© = -(8 1 + ©>2 - -(8 1 - © )2


4 4

if SI and © are sim ultan eou sly m easu rab le. T h ese definitions pro­
vide a “ p h y sica l” construction of polynom ials of se v e r a l sim u l­
taneously m easurable ob servab les SI, ©, e tc .
In quantum m echan ics the m athem atical idealization of an ob­
servab le is a lin ear operator on a H ilbert sp a ce. Let us fir s t out­
line the purely algeb raic a sp ects of op erators which can be defined
without r e fe r en ce to the space on which they act. An a b s t r a c t o p ­
e r a t o r a l g e b r a is a co llectio n of elem en ts called op erators which
together with com p lex num bers are endowed with the stru ctu re of
a v ecto r space (in a techn ical s e n s e , not to be confused with the
H ilbert space) and in addition an operator product. E xp licitly,
500 A PP E N D IX A

(1) If M and N are op era to rs, then M + N' = N + M is a lso .


(2) M + (N + 0) = (M + N) + 0.
(3) M ultiplication by a com p lex num ber is allowed; i.e ., xM is
an operator and x(yM) = (xy)M.
(4) (x ± y)M = xM + yM and x(M ± N) = xM ± x N .
(5) There is a null operator 0 such that 0 + M = M and 0M = 0
for all M.
F u rtherm ore, there is a com p osition law (the “ operator prod­
u ct” ) which a ssig n s to each ord ered pair of op erators (M, N ) another
operator which is denoted by MN . The com p osition law obeys the
follow ing axiom s (we now denote op erators by Ti, T2 , T 3 ):
(a) T 1 T2 is a operator if T \ andT 2 are; i.e ., the operator algebra
is c lo s e d with r e sp e c t to the product.
(b) T 1 (T2 T3 ) = (T 1 T2 )T 3 (a sso c ia tiv e ru le).
(c) T 1 (aT2 + i>T3 > = a T \ T 2 + 6 T 1 T3 and (aT2 + b T ^ T i = a T 2 T 1 +
bTsT1 .
(d) There is a unit operator T unj t = 1 such that I T = Tl = T for
all op erators T .
Once a con crete identification of the op erators is made and a
labeling sch em e adapted a “ m ultiplication tab le” can be s e t up.
The m ultiplication table can be form ulated with the help of the
notion of lin early independent o p era to rs. A ssum ing there are only
a finite num ber n of lin ea rly independent op erators Ti, T2 , . ■ ■ , T n ,
we may e x p r e ss any operator a s a lin ear com bination of th e se . In
particular, as the operator algebra is c lo se d ,

T iT j = £ C*Tk ,
k - 1

w here the com p lex co e ffic ien ts C t are the s t r u c t u r e c o n s t a n t s of


the algeb ra.
The algeb raic p rop erties defined by operator axiom s (l)-(5 )a n d
(a)-(d) can be r e a lize d co n cretely in term s of lin ea r transform a­
tions on a v ecto r sp ace. The operator Ti is id entified with the
transform ation which m aps V into TxV. The transform ation is
lin ear

Ti(alP1 + bV2) = aTiVi + b T 1V 2 ■

Sum and differen ce Tj ± T2 is identified with (Tx + T2 )V = TiV ± T2 ,P.


while z T 1 corresp on d s to the transform ation (2 Ti)'P = 2 (Ti'P), these
relation s being valid for all 4* for which Ti4» and T2 V are defined.
If the range of T 2 includes the dom ain o fT i, the operator product
T 1 T2 is defined by (TiT 2 )'!' = T i(T 2 4 *) for a ll 4» in the dom ain o fT 2 .
A transform ation is n o n s i n g u l a r if it tran sform s d istin ct v e c ­
to rs into distin ct v ectors; that is , if Vx ? 4»2, then T4<x / TV2. An
alternative definition: T is nonsingular if to each v ecto r Ox = TVx
there is a unique solution Vx. In p articular, ifTVx = Vnun, then
H IL B E R T S P A C E AND T R A N S F O R M A T IO N TH EO RY 501

Vi = 'Pnuii. The unique solution ¥j may be e x p r e sse d in term s of


<J>1 by an operator S defined by the relation ¥ j = S $i. It then fol­
low s that ¥ 1 = ST¥i. If this la tter equation holds for a ll ¥ i, we may
a s s e r t the operator equation ST = 1; S is c a lled the left in v erse
of T. If TS = 1, then S is the right in v erse of T. If ST = 1 = TS,
then S is the in v erse of T and is given the notation T” 1 a lso , T = S-1.
If A and B have in v e r se s, then the in verse of A B is (AB) - 1 = B- 1 A- 1
even if A and B do not com m ute. F in ally, if the v e c to r s ¥ i, ¥ 2 ,
. . . , ¥*, are lin early independent and span a m anifold 2)2, and if V
is a nonsingular transform ation, th en F ¥i, FW2 , . . . , F¥*, are lin early
independent and span the tran sform ed m anifold VSW.

Inner P r o d u c t

An H erm itian inner product is a mapping that a ssig n s to each


ordered pair of v e c to r s ¥, $ a com p lex number designated by (¥, 0 ).
The number (¥, $) is called the inner product of ¥ and $ . An inner
product has the follow ing p rop erties:

(¥, <t>) = (0, V)* (H erm itian sym m etry)


(¥ ,a $ + Ml) = a (¥ ,$ ) + 6 (¥,fi)
(¥ ,¥ ) > 0 a n d (¥ ,¥ ) = 0 im p lies V = ¥ niiU.

The n o r m or “ length” of a v ecto r ¥ is ||¥ || = V(¥, ¥) and the d is­


tance betw een ¥ and <1> is || V - <t>||. If (¥, $) = 0 the v e c to r s are
orthogonal. A se t of v e c to r s ¥ j, ^ 2 , • • • form s an orthonorm al
b a sis if (¥,-, ¥ p = 8 ,*; 1 , k = 1,2, . . . . Some im portant th eorem s are
the Schwartz inequality |(¥ , <i>) | < ||¥ || • ||$ || and the triangle in­
equality || VH + || $|| > || ¥ + 0 1|. For each lin ear m anifold 9)1 there is
an orthonorm al b a sis which spans SDl.
The projection operator Pgjj of a m anifold 311 spanned by the
v ecto rs ¥ i , ¥ 2 , . . . , ¥ * , can be w ritten in term s of the inner prod­
uct as

V = E IW -.V )
i= 1

The projection operator onto the ray or one dim ensional manifold
defined by ¥; is P ,¥ = ¥ ,( ¥ ,,¥ ) . If ¥ i , ¥ 2 , . . . is an orthonorm al set
the projection operators are related to the v e c to r s by the equations
P,¥*. = S i k ^ k - The orthonorm ality property is P/P^ = S^P^ and the
co m p leten ess relation is 2 P,- = 1 , the unit operator.
Let ¥ j , ¥ 2 , . . . be the eigenfunctions of an operator M c o r r e ­
sponding to the eigenvalues , . . . . For sim p licity , let us
assum e the eigen valu es are d istin ct, M] j M'k for i -/ k; then (¥,-, ¥ ft) = 0
and the v e c to r s may be n orm alized so they form an orthonorm al
502 A P P E N D IX A

b a s is . The relation of the projection op erators P j of this ortho-


norm al b a s is to the operator M is the follow ing: MP j = P j M = M j P j .
The spectral of M is M = E m ] P j . Let F(A)be an arb i-
resolu tion
f= l
trary function of a rea l or com p lex variab le A. The operator func­
tion F(M) is defined by
CD
F ( IW) = E F M j)P j
i i=
In the m athem atical rea liza tio n of an ob servab le SI in ter m s of
an operator A , the eigen valu es A \ , A 2 of the operator A are identi­
fied with the c h a r a c te r istic v a lu es 211 , 8 I 2 of the o b serv a b le. The
con stru ction of polynom ials of o b serv a b les is identified with the
con stru ction of functions of an operator given ju st above.
T here is a p a rticu larly im portant notation, the D irac notation,
in which a k e t | > is placed around the sym bol <P to denote a v ecto r.
| *P > is c a lle d a k e t v e c t o r . The inner product betw een v e c to r s *Pa
and *1>b is denoted by < *Pa | 'P*, >. The e x p r e ssio n < *Pa | is regarded
as a v e c to r in its own right; it is c a lle d a b r a v e c t o r or sim p ly a
b r a . The p rim ary distin ction betw een bra and ket v e c to r s is that
if the ab stract v ecto r *P tra n sfo rm s as V -» U*P under a unitary
tran sform ation U, the b ra |*P> -* I7|'P> = |U'P>, w hile the ket < *P|
tra n sfo rm s (contragrediently) as <*P| -» <U*P| = <*P|U+ = <*P|I7"1.
Under th ese tran sform ation p ro p erties the inner product <'P|<1>> =
< | U<t> > rem ain s invariant. M atrix elem en ts of an operator M
are w ritten as <*P|M|<1>> = (*P,M$). In the Dirac notation, the de­
fining equation for a p rojection operator reads

P g j ll ^ = E I % ><%• I'P>
i= 1
for an arb itrary ket | *P >. Since | *P > is arb itrary it is a p e r m is­
sib le notational d ev ice to om it w riting the | *P >. Then the e x p r e s­
sion for a p rojection operator in the new notation is

Pm = E l^ x ^ l
i= )

w here the sum runs over the v e c to r s *Pi, V2 , . . . , *P„ which span
the m anifold 912. An arbitrary operator A may be e x p r e sse d in
ter m s of its m atrix elem en ts A ik = (<!>,-,AO*) in a com p lete ortho-
norm al b a sis . . . by the notation

A = E
i,k
A ik I't’i X l ' t l

T here is a notational sim p lifica tio n p o ssib le when working with


a p articu lar b a s is (I>],<1>2 , . . . in that the ubiquious <!’ is a redundancy
H I L B E R T S P A C E AND T R A N S F O R M A T IO N T H E O R Y 503

in the notation: the ket vector | <l>, > can be abbreviated to read | i >.
The Hilbert space for a sp ecific physical problem is built on the
cononical coordin ates q v q 2 , ■ ■ ■ Q>t f ° r a sYs tem of fc-degrees of
freedom . The abstract v e c to r s have no num erical sign ifican ce,
rather the q ’s se r v e to label what w ill be con sid ered a com plete
orthonorm al se t, yVqv q 2 .......... Qk • The ket w i l 1 be denoted by \<iv
q 2 , . . . , q k >. In the (im proper) 8 -function norm alization schem e
the orthonorm ality relation is

< q\ , q'2. . . . . . . . . . . . q'k I q ^ q 2' ■ • • > Qk > = S ( q Y - q \ ) S ( q 2 - q2) ■ ■ ■ 8 ( q k - q'k)

An arbitrary vector *i*a may be expanded in the Vq b a sis:

I Va> = I ■ I ta^qv q2....... <?A) l 9 1 .<72 ......... Q k > d (l \ d(i 2 ■ ■ ■ d (l k

where the expansion co efficien t \pa ( q v q 2 , . . . , q k ) is the probability


amplitude for finding p article 1 at p osition <jlt etc.; i.e ., ypa ( q v q 2 ,
. . . , q k) is the Schrodinger wave function in the coordinate rep re­
sentation. Taking the inner product of | Va > with the bra < q v . . . q k \
and using the orthonorm ality relation , one may verify that

4>a (cI \ ' cl 2 .......... q k ) = < q V q 2 ............

In quantum m echanics a p article may have a d isc r e te internal


degree of freedom , spin, for exam ple, w here the d isc r e te variab le
a takes on eith er of two v alu es, t for spin up or i for spin down.
For a p article with spin and one spatial d egree of freedom x , an
arbitrary state 'I'a may be expanded as

w here <Jv(x) is the probability am plitude for finding the p article at


x with spin a (se e Chapter 16).
Treating now a p article on the interval - » < x < ~ with no spin,
we w rite the inner product of two v e c to r s Wq and Vj, by fir s t ex­
panding n't, in term s of the se t W*

ao

then taking the inner product using the distributive rule under the
integral. The r e su lt is
504 A PPE N D IX A

the ordinary function sp ace inner product. The projection operator


Pab onto the in terval a < x < b is

Pab*V Vx (VXlV )dx ,

or in the D irac notation

ab — r \ v * > dx < I .
•'a
An arb itrary operator M is e x p r e sse d in term s of its x -sp a ce
m atrix e le m en ts <x | M| y > = by the form ula
y* CD 00

M = / / <x|M|y> l ^ x Vyldxtfy
• /-0 0 fc /-C 0

the action of an operator Mon a (ket) state v e c to r | V > is w ritten as

/
* 00 y * CD

Ml I I ’P ^ x x I M l y X y l ' P > dxdy ,


QD —CD

w here it is now appropriate to c a ll < x | M| y > the kernel of the


integral operator M. The average value M, or what is the sam e
thing, the expectation value of M in the state <t>, is

y«00 00

M = (<I>, M<t>) = / / <<J>| xXx| M| yXy| <D>dx(iy



CD —00

An operator V is c a lle d lo ca l if its m atrix ele m en ts have the


sp e c ia l form < x | M | y > = 5(x - y ) V ( x ) , w here S(x - y) is the Dirac
8 -fun ction and V(x) is an ordinary function. The expectation value
of a lo ca l operator is

V < $ | x > V (x) < x | 4> > dx .

The kinetic energy operator T has the m atrix elem en t < x | T | y > =
2
_ y )t which is a lso regarded as a lo ca l op erator. With
2m
the help of in teg ra tio n -b y -p a rts the expectation value of T can be
w ritten

2 r™
T / </>(x)* V ^(£(x)r/x ,
2m -m

w here </>(x) ^ < x | <l> >.


App end ix II

The Statistical Assertions of Quantum


Mechanics

We con sid er a cononical sy ste m of k d eg rees of freedom , em ­


ploying the coordin ates q x . . . q k to sp ecify its configuration and
the cononically conjugate m omenta to sp ecify its condition of
m otion. In the wave m echanical mode of descrip tion , in the co­
ordinate rep resentation, everything that can be said about the
“ state of the sy ste m at one tim e ” (its configuration, condition of
m otion and the valu es of all its physical quantities) m ust be de­
rived from the wavefunction <f>(q^ . . . q k ). The functions adm itted
as wave functions are those that are square integrable (norm aliz­
able, ||</>|| finite) and furtherm ore n orm alized to unity,

0<<?P • ■• , qk) I dqx dq. 1 ,


—CD J -C D

although in applications to continuous sp ectra this requirem ent


may be relaxed . There are three prim ary sta tistic a l a sser tio n s:
(1) the probability of finding the sy ste m within a volum e V of con­
figuration space is

q k )\ d q j • • • d q h

(2 ) if the energy of the sy ste m has the operator // with eigenvalues


Ei , E 2 , . . . and eigenfunctions ip2 , . . . , then the probability that
the sy ste m has the energy value E j in the state 4> is

2
'A/?i • • • q k )* • • • ,q k > d qd1q h

and (3) the average value of a physical ob servable 81 to which the


operator A corresp on d s is
/ *00 y-»0
d
, q k)* Qk >
nn J — CD dq: dqk

4>(qv •••, q k)d(*i • • • dqk


506 A P P E N D IX B

in the quantum state 4>. The in terrelation of th ese three a s se r tio n s


w ill now be exam ined.
In ord er to p resen t a sse r tio n (1) in a g en eral form , we intro­
duce the projection op erators P j d j h j = 1 , • • • , k , a sso c ia te d with
the intervalue I j defined by q'j < q j < q j . The p rojection operator is
defined by

$ (? !. • ■ • . q k ) for qj < qj < qj


P j U j X M q i,
0 otherwise .

The p rojection operator a sso cia ted with the (rectangular) volum e
V is f 3 1 (/ 1 ) P 2 (/2) • • • Pfe(/ft) and the in tegral sp ecifyin g the prob­
ability of finding the p a rticle d escrib ed by 0 in V may be reduced
to the e x p r e ssio n

| | P 1 ( / 1) P 2 ( / 2 ) • • ■ P k < / * ) * II2

F or a sse r tio n s (2), le t Pe„ denote the projection operator a s s o c i­


ated wl*th the eigenvalue E n of H, that is ,

Pg <Am = xPn for m = n, and = 0 for m / n.

The probability of finding the value E n upon a sharp m easurem en t


of the energy of a sy ste m in the state $ can then be put in the form

II PEn«>||2 .

The probability of finding the sy ste m within an energy interval


is
E ' < E <. E"

II P U e )^ \\2 ,

w here P (/g) is the p rojection op erator a sso cia ted with the interval

P (/e) = E
E '< E n <E"
p En -

A sse r tio n s (1) and (2) may now be unified in what is c a lled “ the
m ost g en era l probability a sse r tio n p o s s ib le ” :

S ta tistic a l P ostulate: The probability that in the state <I>


the p h ysical quantities with the op erators / \] ,/\ 2, . . . ,/\mtake
on valu es from the, r e sp ec tiv e in terv a ls / 1 , / 2, . . . , I m is

IV ||P 1 (/ 1 )P 2(/2) ■ • ■Pm ^m ^ll 2 ,

w here P i , P 2, . . . Pm are the projection op erators belonging


to the op erators Ai . . . A m .
TH E S T A T IS T IC A L A S S E R T IO N S O F QUANTUM M EC H A N ICS 507

A ssertion (1) has m h, with A| r/i, A 2 , A>t (U>\ while


a sser tio n (2) has m 1 , with A 1 II. In order to insure that this
postulate is a coherent sta tistic a l statem ent the following prop­
e r tie s m ust be varified:
(a) Since the order of the operators is arbitrary in form ulating
the sta tistica l question, the order of the projection operators in
the sta tistic a l a sser tio n m ust be im m aterial. This im p lies that
the projection op erators m ust com m ute for arbitrary in tervals
Z1 . l 2 . - - - . f m. and this in turn im p lies that the operators A ]t A 2 ,
. . . , A,„ m ust com m ute among th e m se lv e s.
(b) Vacuous propositions may be in serted at w ill, v is . if the
interval /' is contained within the interval 1, P i l ) P ( l ' ) = P ( l ' ) , or
the projection operator for obtaining any value w hatever is the
identity.
(c) If the interval l j lie s outside the spectrum of A/, for som e } ,
the corresponding projection operator is the null operator and the
probability is zero .
(d) P rob ab ilities are additive; i.e ., if an interval / is reso lv ed
into two disjoint subintervals / = !' + /', with operators P d ) and
P d " ) corresponding to I' and l", r e sp ec tiv e ly , then

||P(/)<i> | | 2 = || P d ' ) (j> | | 2 + || P d ")$ ||2 .

(e) The total probability W ranges over the valu es 0 < W < 1
for norm alized O.
A ssertio n (3) at the beginning of th is appendix, can be e x p r e sse d
in term s of the inner product. If we introduce the notation Exp 181,01
to stand for the statem ent “ the expectation value of the physical
observable 81 in the state 0 ,” a sse r tio n (3) may be stated

Exp 181,0 I = A = (0, AO),

w here A is the operator corresponding to 2f. Let F(A)be any func­


tion of the r e a l param eter A. There is a theorem to the effect that
if the ob servable 81 has the operator A, then the ob servable F ( 8 I)
has the operator F(A). A ssertion (3) may then be g en era lized to
read

Exp 1F (21), 0 | = F(A) = (0, F(A) 0) (3')

A ssertio n (37) can be derived from the sta tistic a l postulate as


follow s: subdivide the interval ~) into a sequence of subinter­
va ls l n = |A„, An+il, w here - ~ < A_„ < • • • < + A-i < Ao < Ai < • ■ • > An <
+ ~ . Let An be s om e number in the interval A;- < Aj < A,+i. The
average value F(A) in this m esh is
______ n
F(A) = X) F (Aj)(O, P (/_,-) 0)
508 A PPE N D IX B

If we introduce the m onotonic in crea sin g projection operation E(A)


defined by

E(A) = E p Uj)
\j< X
(co n v ersely , P ( l j ) = E(AJ+i) - E(A;)) and le t the n -►~ and the m esh
s iz e vanish, the sum approaches the S tieltjes integral
/ •ao
E(A)(<ME(A)<D) ,
,O
D
which by definition of a function of an operator is (0, F(A)Q>). So
far we have indicated that a s s e r tio n s (3) and (3/ ) follow from the
S ta tistica l P ostu late. What is rem arkable is that the c o n v e r se is
a lso true: the S ta tistica l P ostu late fo llo w s from a s s e r tio n (3). The
proof of th is statem en t is obtained by a techn ical application of the
follow ing theorem : Let A \ , A 2 , . . . , A m be a s e t of m utually com ­
muting op erators; there e x is t s an operator R and functions E^A),
E2 (A), . . . ,E m(A) such that Ai = Ei(R ) , / \ 2 = E2 (R), . . . , F m = F m ( R ) .
In 'the p reced ing d isc u ssio n we have assu m ed that the state is
a pure sta te, that is , it is d e sc rib e d by sin g le v e c to r 0 . In gen era l,
in the p h ysical preparation of a state som e of the v a ria b les are
left uncontrolled and the state is not com p letely sp e c ifie d . This
situation can be form ulated in term s of a c la s s ic a l probability
distribution that is su p erim p osed on top of the quantum m echanical
uncertainty. L et us suppose that the sy ste m is in one of the sta te s
'Pa., a = 1,2,. . . , but we don’t know p r e c is e ly which one. L et Wa be
the c la s s ic a l probability that the sy ste m is in the state *i'a. The
b a sic sta tistic a l postulate can now be reform ulated as

W = E lU P l ( / l ) P 2 </2) • • • P m (/m) Va ||2


a

and a sse r tio n (3) can be g e n e r a lize d to read

Exp I 81; tV0, V a l = E Wa (Va, A'VJ


a

The s ta te s M'a are an arb itrary s e t of sta tes; they need not be
m utually orthogonal. F or the Wa ’s it is required that Wa > 0 and
1.
The s ta tis tic a l a s s e r tio n s can a lso be form ulated in term s of
the tra c e . The trace of an operator A is

lr A E ( ' l ’n ^ > ,

w here the sum runs over a com p lete orthonorm al b a s is . .. •


The trace is independent of which b a sis is used to define it. The
TH E S T A T IS T IC A L A S S E R T IO N S O F QUANTUM M EC H A N ICS 509

sta tistic a l a sse r tio n s are

IV lrp /V /iH V /2 > • • • I’m d m )

and

Kxpl SI; lVa, 'l'a I tr p A

w here p is the density m atrix (operator)

P = £ wa l \ = E l ^ ^<¥*1
a a

The density m atrix is useful for making a gen eral statem en t about
the tim e evolution of the sy ste m . Expectation values change with
tim e according to the gen eral rule

— Expt a ; Wa , V J = tr \ p A + PA I.
dt

In the Schrodinger picture the burden of change is put on p; nam ely,


4 = 0 and +i ~ h p = Hp - pH, w here H is the H am iltonian. In the
H eisenberg picture p = 0 and - i H A = HA - AH, (se e P roblem 8.4).
The Hamiltonian con trols the evolution of an isolated sy ste m . When
the sy ste m is subjected to a m easurem en t of one of its o b serv ­
ables R and the eigenvalue R ' obtained the uncontrollable disturb­
ance of the m easuring p r o c e ss fo r c e s the sy ste m discontinuously
into the state d escrib ed by the eigen vector Vr' corresponding to
the eigenvalue R' , If the m easurem en t of R is repeated (before the
evolution generated by the Ham iltonian m oves the sy ste m into
another state) the sy ste m is already in an e i g e n s t a t e o f Rand
thus y ie ld s the value R ' with certain ty.
Problems

C hapter 1

1.1 E xp ress the space and tim e dependence of E, M and A in


term s of am plitudes and phase angles for a plane wave
moving in the p ositive x d irection . What are the conditions
for plane or c ir c u la rly p olarized light? What is the initial
data at t = 0 for solution of M axwell’s equations or the wave
equation for A? E xp ress the solution in term s of a power
s e r ie s in tim e. E xp ress the solution for t < 0 in term s of a
solution for t > 0 with appropriate initial data.
1.2 D eterm ine the m agnitudes of E and H for unpolarized sun­
light ch aracterized by the so la r constant (Poynting vector)
Sx = 2 caE 'cm ^ m in -1. What is the ratio of e le c tr ic to mag­
netic fo rc e on an electron moving with the sp eed of an e le c ­
tron in the fir s t Bohr orbit?
1.3 Com pare the wave and photon d escrip tion s of norm al reflection
of sunlight from a m irro r. What is the energy d en sity, p r e s­
su re against the m irro r and the density of photons? Take
A = 5500A. What is the e le c tr ic field in ten sity of a photon
absorbed in a 1 cc d etector?
1.4 Find an ex p ressio n for the angular momentum density of an
electrom agn etic field . Show that a photon c a r r ie s an angular
m omentum of m agnitude H. C alculate the torque on a quarter
wave plate by a norm ally incident le ft-c ir c u la r ly (right-
screw ) p olarized light.
1.5 D eterm ine the sp ectra l density of radiation under the a s ­
sum ption that the o s c illa to r rep resen tin g the behavior of the
w alls for angular frequency a> can assu m e the energy value
E = 0 or any one of a continuum of valu es E > Sw. Show that
the gap in the energy spectrum is related to the behavior of
p a in the quantum region k T ^ Tico •

A nsw er. The partition function Z = l e ~ aEn = 1 + e ~ aQ>/ a u > .

C hapter 2

2.1 C onsider the scatterin g of 4 MeV alpha p a r tic les on gold


atom s (Z = 79). Show that the distance of c lo s e s t approach is
r m in = (ZZ'<?^/2E0) (l + C S C | d) and evaluate it for scatterin g
512 PROBLEM S

an gles 5 °, 20° and 80°. At what angles w ill the scatterin g


deviate from the Rutherford form ula?
2.2 C onsider the sca tterin g of alpha p a r tic les from gold fo il. A
radium so u rce y ie ld s 4.8 MeV a -p a r tic le s at a rate of 3.7 x
10 1 0 p a r tic le s per second per gram of Ra. The target has
5.9 x 10 2 2 a to m s -c m '3, a th ick n ess of 4 x 1 0 ' 5 cm , and a
c r o s s sectio n a l area of 8 m m 2. The target is situated 1 cm
from the so u r c e. C onsider a detecting sc r e e n of 2 m m 2 area
5 cm from the target. What amount of radium is required for
a counting rate of 30 per m inute at sca tterin g an gles 10° and
120 ° ?
2.3 D erive (2.46) for a g en era l on e-d im en sion al potential by
evaluating d l / d E d ir e c tly . Reduce the validity of the Bohr
quantization rule for la rg e n by equating the c la s s ic a l ex­
p r e ssio n for the frequency of the e x p r e ssio n in te r m s of the
en ergy le v e l sp acin g. (T his application of the corresp on d ­
ence prin cip le show s that (2.46) and (2.47) g iv e a co n sisten t
c la s s ic a l lim it if the sam e constant t appears in both equa­
tions.)
2.4 Exam ine the c la s s ic a l and quantum d escrip tio n s of the e m is ­
sion of ligh t from a harm onic o s c illa to r . On the b a s is of the
corresp on d en ce p rin cip le, argue that the % appearing in the
Bohr quantization rule is the sam e P lanck’s constant ch ar­
a c terizin g the corp u scu lar nature of light. Deduce the energy
le v e l spacin g and infer the le v e ls betw een which e le c tr ic
dipole tra n sitio n s are p o ssib le (se le ctio n r u le s).
2.5 U se the co rresp on d en ce p rin cip le to infer the energy le v e ls
of a rigid rotator c o n sistin g of a m a ss m held at d istan ce b
from a fixed ax is of rotation. Note that the angular momentum
is independent of the m echan ical p a ra m eters.
2.6 Show that for c ir c u la r m otion in a gen eral c en tra lly sym ­
m etric field , the angular m om entum is quantized in step s
of t .
2.7 Find the energy le v e ls of c ir c u la r orb its in the Bohr plane­
tary m odel of the hydrogen atom using the co rresp on d en ce
p rinciple d ir ec tly . Note that as usual n is left undeterm ined
up to an additive constant of integration. Infer the se le c tio n
r u les for e le c tr ic dipole tran sition s betw een c ir c u la r orbits
and find the accom panying angular m om entum change.
2.8 E stim ate the life tim e of a stationary Bohr orbit by computing
the tim e required for the correspond ing c la s s ic a l m otion to
radiate away an amount of energy equal to the quantum energy
le v e l sp acin g. Compute the cum ulative tim e for s e v e r a l su c­
c e s s iv e tran sition s betw een c ir c u la r orb its and com pare it
with the tim e taken to radiate the sam e amount of energy
c la s s ic a lly .
2.9 .U s e the Bohr quantization conditions to determ ine the energy
le v e ls of an isotrop ic th ree-d im en sio n a l harm onic o sc illa to r .
PROBLEM S 513

Let the potential energy be } m a 2 r2 and use polar coordinates


in the plane of the c la s s ic a l motion.
2.10 Find the energy le v e ls of the r e la tiv istic lin ear harm onic
o sc illa to r according to the B ohr-Som erfeld quantization rule.
Evaluate the low est order correctio n to the n o n -r e la tiv istic
e n e r g ie s. The potential may be introduced via the sc a la r
potential c>‘l> = b / « 2 or by replacing the m a ss invariant by

M 0 c 2 ► J /e x 2 .

C hapter 3
3.1 C alculate the d eB roglie wave lengths of the follow ing par­
tic le s each with a kinetic energy 500 keV: photon, electron ,
proton, and alpha p a r tic le s. A lso of therm al oxygen atom s at
300°K.
3.2 What is the d isp ersion law = co(k) if the group v elo city is
in v ersely proportional to the phase v e lo city ? How is this
c a se r e a lize d physically?
3.3 Com pare the reflectio n of a p article and a wave from a
moving su rfa ce. Show that AE/Aco = A p / A k , w here AE(Ap) is
the change in the energy (momentum) of the p article and
Aai(Ak) is the change in frequency (wave number) of the w ave.
Show this independently of the d isp ersion law of the wave and
the energy-m om entum relation of the p a rticle. How can this
r e su lt be gen eralized and what is its sign ifican ce?
3.4 C alculate the d eflection of a charged p article by a thin slab of
m agnetic field , ex p ressin g the change of m omentum in term s
of the v ecto r potential. How is the difflection explained in
term s of the wave picture? What is the relation betw een mo­
mentum and wave vecto r in the p resen ce of a m agnetic field?
A nsw er. SK = 0 a c r o ss the slab and p + Ae / c = £k.
3.5 C onsider a p article bouncing back and forth in a rigid box of
lin ear dim ension L. What is the m inim um m easurable ki­
netic energy of the p article? How much energy is required
to con strain an electro n to rem ain within a volum e of nu­
c le a r s iz e ?
3.6 D eterm ine the m axim um tim e a free p article w ill rem ain
within a volum e of radius R by con sid erin g the lim itation s on
the sp ecifica tio n of the initial data in the c la s s ic a l d escrip ­
tion of m otion.
3.7 Find an uncertainty relation connecting angular m omentum
and angular orientation. C onsider a rigid rotator. What is
the m inim um uncertainty in L?
3.8 One b illia rd ball bounces on another with a c e n te r -to -c e n te r
height of ten tim es the radius. What is the optim al horizontal
514 PROBLEM S

lo ca liza tio n to m axim ize the num ber of bounces? What is the
m axim um num ber of bounces?
A nsw er, n ~ 50.
3.9 What is the optim al lo ca liz a tio n of an ideal pendulum to
m axim ize the tim e it w ill rem ain balanced in an inverted
p osition and what is the m axim um tim e?
A n sw er. About s ix tim e s the period for sm a ll o sc illa tio n s.

C hapter 4

4.1 Under what cir c u m sta n c e s can a narrow potential be approx­


im ated by a 5-function? What is the effect of a 5-fun ction
potential on the continuity property of a solu tion of Schrod-
in g e r ’s equation?
4 .2 Show that the potential V(x) = - g S ( x 2 - a 2) has a d iscr e te
an itsym m etric eigenfunction for g > t 2/ m and not one for
g'< t /m.
4 .3 C on sider the potential V(x) = <» for |x | > L and V(x) = -V 0 5(x)
for | x | < L. Show that for Vo > 0 there is one eigenvalue E < 0
and that it rem ain s d isc r e te as L approaches infinity.
4.4 Find a com p lete s e t of orthonorm al eigenfunctions for the
potential V = - | Vo | 5(x) on the in terval < x < <*>. Check the
c o m p le ten ess relation (4.67).
4.5 (S tu rm -L iou ville theorem ) Let i/'l and ip 2 be solu tion s with
e n e r g ie s Ei and E 2 of two Schrodinger equations with po­
ten tials Vi and V2, r e sp e c tiv e ly . Show that

\p*2 4>\ ~ '/'2 ^ l ] ! = <2m/7i2) J 0 * [(E2 - V2) - (Ei - V ^ ^ d x

for any in terval a < x < b. Show that if ip 1 and \p2 are solu ­
tions for the sam e potential and if E 2 > E i, then \p2 has at
le a s t one node betw een each pair of con secu tive nodes of
Over the en tire in terval, then, tp2 has at le a s t one m ore node
than ipy. (The eigenfunction s are ordered in energy according
to the num ber of n od es. A s e t of eigenfunctions is com p lete
if th ere is one for each in tegral num ber of nodes.)
4.6 (C om parison potential for d is c r e te sta tes) C onsider the in­
terval 0 < x < 00 with the boundary condition IKO) = 0 . Find
the potential for which U = x n is a zero energy solu tion of the
Schrodinger equation. Show that the potential obtained has no
d isc r e te eigenfunctions for n r e a l. If n is com p lex, n =i +
i f } , show that the potential has a rb itrarily many bound (i.e .,
- d iscr e te) sta te s . The solu tion may be taken to be rea l U =
Vx cos ft lnx.
PROBLEMS 51 5

4.7 Let V0 be the com parison potential obtained in problem 4.6.


Show the following:
(a) If V > V() for x > x(), there is no d iscr e te eigenfunctions
of V with a node located beyond xo.
(b) If V < V0 for x > xo, there is a d iscr e te eigenfunction
with arb itrarily many nodes beyond xo. There are in­
finitely many d iscr e te sta te s of V.
(c) If V > V0 for 0 < x < xo, there is no solu tion for the po­
tential V with E < 0 that has a node in the interval 0 < x < xo.
(d) If V < Vo for x < x0 (i.e ., if V , m ore rapidly than V0),
then there are eigenfunctions with arb itrarily many nodes
in the interval.
4.8 Scale the function in problem 4.6 to find som e eigenfunctions
of the potential V = ~ for x < a and x > b, V = - ^ ( l / x 2 -
l/b 2 )a 2 6 2 / ( 6 2 - a2) for 0 < a < x < b . Is there a r e str ic tio n
among the param eters V0 , a, and b for this m ethod to work?
Can one find a com plete se t this way?

C hapter 5

5.1 Let a p article be prepared in the state ^(x,0) = \/2a0(xo - x)


explfpx + a(x - xo)l at t = 0 , w here a > 0 , xo < 0 and d ( x) =
- ( 1 + signx). A ssum e that no fo rc e s act on the p a rticle for
t > 0. Expand ^(x,0) in term s of energy-m om entum eigen ­
functions and show that for t > 0 ,

^ ^ _ \/8afi exp(ipx - t/3p2 + i y2/ 4/3) e~L dL


77 (2a/3 + iy) J q 1 _ i z L2
-2
where /3 = 1it/2m , y = x - xo - 2)3p, and z = (1 + iy/2a/3) . In­
terp ret tp for t -» oo#
5.2 R econsider the initial wave function sp ecified in problem 5.1.
A ssum e the p article is in a potential field V = - | V o | 8 (x).
Expand ^(x,0) in term s of the eigenfunctions of problem 4.4
and follow the m otion in the d isp e r sio n -fr e e approxim ation
(3.20). Compute the probability of the p article being re flec te d
by the potential. What is the probability of finding the par­
ticle in the ground state?
5.3 The initial value problem of the tim e-dependent Schrodinger
equation can be solved in the form of an integral operator
with the G reen’s function kernel G((x,x') = ^ 'Pn (x'> 'Pn t' x '>* e ~la>nt
n
w here \pn is a com plete s e t of orthonorm al energy eigen ­
functions.
(a) Find the differen tial equation and boundary conditions for
G t in the c a se of a gen eral on e-d im en sion al potential.
516 PROBLEM S

(b) Show that the fo r c e -fr e e m otion of a n o n -r e la tiv istic par­


tic le in an s-d im e n sio n a l E uclidian sp ace is given by
Gt(r,r') = ( m / 2 n i m S/2 exp Ifro (r - r')2 /2*M.
(cf. P roblem 3.3 in the text. How can Gt s e r v e a s an approx­
im ation to the S-function?)
5.4 Find the angular dependence and total cu rren t of charged
p a r tic les flow ing from a so u r c e at the o rig in . The wave
function far from the so u rce is cos 6 e lkr/ r in sp h erica l co­
o rd in ates.
5.5 A p a rticle is in a uniform gravitational field in a region
bounded from below by a p erfectly reflectin g su r fa c e . Adapt
the WKB m ethod to the c a s e of an im penitrable w all and show
that the energy le v e ls go as (n + 3/4 ) 2 / 3 for la rg e n . Show that
E n - E n- i = for la rg e n, w here o> is the c la s s ic a l angular
frequency.

C hapter 6

6.1 Find the probability for a p a rticle of charge + Z'eo and speed
v to penetrate through the Coulomb b a r r ie r to a nucleus of
charge Zeo. (Gamow factor)
6.2 D erive (6.35), using in tegration by parts to pick out term s of
ord er T2 in e!T„.
dv
6.3 Find the p r e ssu r e exerted by the w a lls to contain a p article
in a v e r y deep potential w ell of volum e V at absolute zero
tem p era tu res.
6.4 Find a rela tio n betw een the volum e and p r e ssu r e of an e le c ­
tron g a s at 0 °K.
6.5 Show that (6 . 6 6 ) holds for an arbitrary period ic potential
(B lock ’s th eorem ).

C hapter 7

7.1 E x p ress l / r as an integral operator in the m om entum r e p r e ­


sentation. _______
7.2 Find the k ern els for the e x p r e ssio n of p2 and Vp2 + m \c2 as
in tegral op erators in the coordin ate rep resen tation .
7.3 Evaluate the com m utator p x V ( r) - V ( r ) p x .
7.4 Let R(r,p)be a polynom ial in the com ponents of r and p. Show
that

— = i- (Rx - xR)
dpx h

and indicate e x p lic itly the m eaning of the partial d erivative.


7.5 .(a ) Find an e x p r e ssio n for the operator (A - z B) ~l in pow ers
of 2 , w here A and B are non-com m uting o p era to rs.
PROBLEM S 517

(b) Let A and B be N x N m a trices and let z \ be the root of


the equation dot (A - zB) 0 which is s m a lle s t in absolute
value. Prove that the expansion of part (a) con verges
w henever \ z \ < | z i | and d iv erg es w henever | z | > | z i |.
7.6 Show that in general any quantum m echanical operator can be
ex p r e sse d as an integral operator.
7.7 Find an ex p r e ssio n for the tran slation operator T a , defined
by Ta i//(x) = ip ( x i a ) .
7.8 The solution of the initial value problem of the tim e-dependent
Schrodinger equation can be ex p r e sse d in term s of an oper­
ator U ( t ) that tran sform s the wave function at tim e t - 0 into
the wave function at tim e t; that is , ip i t ) = U ( t ) i/AO).
(a) Upon what p rop erties of the Schrodinger equation does
this depend?
(b) E xp ress U U ) as an integral operator in term s of a com ­
plete se t of energy eigenfunctions.
(c) Evaluate the kernel of the integral operator U i t ) for the
m otion of a fre e p a rticle.
7.9 The concept of the quantum m echanical average value has its
natural gen eralization in the notion of the “ inner product” in
Idle rep resen tation space (se e Appendix). The average value
M = (4*,MV) is the inner product of 'I' with the v ecto r M'F.
D em onstrate the following:
(a) Let 4',-, i =
1 . . . 0 O ,be a com plete s e t of b a s is v ecto rs
which is orthonorm al with r e sp e c t to the inner product,
(¥,■, Tfe) = Sik. Show that the action of an operator M is
com p letely determ ined by its m atrix elem en ts M,■* = (4*,-,
M4»*).
(b) An operator H is H erm itian if ('F, HO) = (HV, 0) for all
v e c to r s ¥ and <1>. Show that the eigen valu es of H are rea l,
the eig e n v e c to r s corresponding to d istin ct eigen valu es are
orthogonal, and there e x is ts a b a s is in which H is diagonal.
Find the conditions on the m atrix elem en ts H,-* such that
H is H erm itian.
(c) If (4*, CW = 0 for all v e c to r s V and if © is H erm itian, then
<0 , ©'t') = 0 for all 4* and 0 . One may then a s s e r t the oper­
ator equation Cl = 0 .
(d) If (4*, A4>) = (B'F, <l>) for a ll 4* and $, then B is said to be the
H erm itian conjugate of A and is given the sp e c ia l notation
B = A+. Find the relation betw een the m atrix elem en ts of
A and A+. Show that A is the H erm itian conjugate of A+
and that a H erm itian m atrix is self-co n ju g a te, H = H+.
E xp ress the H erm itian conjugate of the operator product
AC in term s A+ and C+.
(e) Show that if the inner product is p ositive definite, i.e .,
(4>, <I>) > 0 for all $ but the null v ecto r, then the eigen valu es
of AA+ are non-negative.
(f) An operator U is unitary if (UV, I/O) = (41, 0) for all 4* and <J>.
518 PROBLEM S

If W,, i = I. . . *) , is an orthonorm al b a s is, then W; = (TP; is


a lso . Find the conditions on the m atrix e lem en ts Uik such
that U is unitary. Show that U is n on -sin gu lar.
(g) Show that the op erators defined in prob lem s 7.7 and 7.8
are unitary.
7.10 If A and B are N x N m a tr ic e s and if A B = I, show that BA = I,
/ being the N x N unit m atrix. C onstruct a counter exam ple
to show this r e su lt is not true in g en eral for “ infinite
m a tr ic e s ,” i.e ., N

C hapter 8

8.1 Show that the com m utation rela tio n s betw een the op erators
for the v e lo c ity com ponents of a charged p a rticle in a m ag­
n etic field are v x vy - vy v x = ^ - H z and c y c lic perm utations.
ml c
8.2 Show that if V,- and are energy eigenfunctions and if M does
not depend ex p lic itly on tim e, then the tim e dependence of o ff-
diagonal m atrix e le m en ts is given b y — ('Pi, M'F*) = faji*(^t-,MW*)
dl
w here fay,-* = E,- - E j .
8.3 Let U ( t ) be the operator that g e n e r a te s the solu tion *P(f) of the
tim e-depend en t Schrodinger equation (with H am iltonian H)
from arb itrary in itial data 'P(O) in the form WU) = U ( t ) ¥(0).
See problem 7.8.
(a) Show that 1/(0 s a tis fie s the operator equation i H U U ) =
H U it), with in itial data 1/( 0 ) = 1 .
(b) Deduce from the d ifferen tia l equation that U U ) is unitary.
(c) Specify under what conditions and in what s e n s e U U ) may
be w ritten as U U ) = exp ( - i H t / H ) .
8.4 In the Schrodinger pictu re o b serv a b les are rep resen ted by
op erators that do not depend e x p lic itly on tim e, the change of
dynam ic v a r ia b le s with tim e being d escrib ed by the change of
state v e c to r '{'(/). In the H eisenberg picture the tim e depend­
ence is tra n sferred to the op erators th e m se lv e s by m eans of
the unitary operator in problem 8.2. The procedure is MU) =
(yi'(0 >M'l, (<)) = (Vq,)U(OV0), w here Vq = 'F(0 ) is the state vector
at / = 0 and w here 171(f) = U U ) ~ l M U U ) .
(a) Show that/7ilil= ftlW - Kft, w here U = U U ) ~ Y H U U ) . If H i s
independent of tim e H = H. T his is the H eisenberg equa­
tion of m otion.
(b) If AB - BA = iC, th en QB - I3Q = iC.
(c) D erive an e x p r e ssio n for the tim e d erivative of the op­
era to r product CL(/) B (/).
(d) Find the p osition operator in the H einsenberg picture for
„ the m otion of a fr e e p a r tic le. E x p ress the r e su lt in the
coordin ate rep resen tation .
PROBLEM S 519

(e) Using the position operator in part (d), find the tim e de­
pendence of the variance of the position of a fre e ly moving
p a rticle.
8.5 Show that the mean value of the kinetic energy in a state be­
longing to the d iscr e te spectrum is related to the mean value
of the potential energy by the relation 2 T = (r • W ). If V is a
hom ogeneous function of d egree a, V(Kr) = A“ V( r ) , then 2 T = a V
(virlal theorem ).
8 .6 V erify the follow ing “ sum r u le s .”
(a) E Ir, * | 2 = (?>* - (V*, r2 Vfc)
^
E
2

(b) <*ik I Tik I2 = —— (“F’-sum rule")


i Zmo
( c ) Z o > l \ r i k \2 =

(d) E ^ Ir.fcl2 = ^ - < V 2 V),

(e) E
I
V*k I Uk I2 = diverges
H ere Ticjik = E ; - E*. Note that &>,-* > 0 if k d esign ates the
ground sta te. Sum r u les are im portant becau se they provide
a m eans of testin g the p h ysical content of the theory even
though the m athem atical problem cannot be solved com ­
p letely.

C hapter 9

9.1 Form ulate the s e t of coupled differen tial equations governing


the population of a large num ber of quantum s ta te s . Show that
the total num ber of le v e ls occupied does not change with tim e.
Identify the conditions under which the occupation num bers
are p ositive d efinite. D erive the rela tio n s among the E instein
c o e ffic ien ts which sp ecify the approach to therm odynam ic
equilibrium .
9.2 Compute the curren t density of the n on -station ary state
which d e sc r ib e s the tran sition from the 2 p to the I s state of
hydrogen and calcu late the radiation field from this so u rce.
Identify the angular distribution and state of polarization for
each value of the m agnetic quantum number m. See Chapter 13.

C hapter 10

10.1 At what quantum le v e l can a wave packet in a harm onic


o s c illa to r be lo ca lized to 1 % of the total excu rsion ? U se the
uncertainty p rinciple.
10.2 Find the stationary sta te s of a lin ear harm onic o s c illa to r in
a uniform e le c tr ic field .
520 PROBLEM S

10.3 Suppose that at t = 0, a p a rticle in the potential field V =

^mcj2 x 2 is d escrib ed by the wave function ^(x.O) = co n st.

exp -jifeox - —a2 (x - xo)2 j , w here a 2 = m a/H . C alculate the


probability am plitude for the p article being in each of the
en ergy e ig e n s ta te s . Find the wave function for t > 0. D isc u ss
the spreading of the wave GD packet.
10.4 Show that e“ss+2st = ^ — Hn ( t ) is the generating function for
n=0 n\
T ch eb ych eff-H erm ite p olyn om ials. U se it to e sta b lish the
appropriate orthonorm ality re la tio n s.
v * CD

10.5 Show that / Hm ( x ) e ~ x2 e ,p*dx = yjn impme ' p2/4.


• / — CD

10.6 Infer the m atrix elem en ts of the position operator of a har­


m onic o s c illa to r from the sp ectru m , se le c tio n r u les and the
F -s u m r u le.
10.7 Solve the H eisenberg equations of m otion to find the tim e-
dependent H eisenberg op erators for the position and mo­
m entum of a harm onic o s c illa to r .
10.8 Find the tim e-depend en t G reen’s function for the harm onic
o s c illa to r . N orm alize by com paring it to the fre e p article
G reen’s function at a tim e cot « 1 .
10.9 Study the harm onic o sc illa to r using the operator p rop erties
of the v a ria b les

a = \J(ma>/2~h) x + \J(Ti/2mtS) d / d x ,

a+ = V(ma)/2 ^) x - \J(1i/2mco) d / d x .

P rove the follow ing statem en ts:


(a) The H am iltonian operator is H = a+a + 1 w here the energy
is m easu red in units of Hco.
(b) The com m utation rela tio n s
aa + - a +a = 1

Ha - aH = - a
Ha + - a + H = a

hold a s operator equations.


(c) The H erm itian conjugate (or adjoint) of a is a+. // is
H erm itian (self-a d jo in t).
(d) If U e is an energy eigenfunction with eigenvalue e, then
a\Jf is an eigenfunction with energy e - 1 and (a*)n U e is
an eigenfunction with eigenvalue e + n. T his g iv e s a s e r ie s
of equally spaced energy le v e ls .
PROBLEMS 521

(e) There is a finite low est eigenvalue.


(f) The s e r ie s m ust term inate at the low er end by arriving at
a “ ground s ta te ” (io that s a tis ife s the equation a U o 0
(= the null vector). Solve this differen tial equation for U o
and n orm alize.
(g) The “ excited s ta t e s ” are U n = (n\)~'h (a*)" U o with en erg ies
(^i i- 0 /ico.This g iv es a com plete s e t of orthonorm al eigen ­
functions.
(h) D eterm ine the m atrix elem en ts of a, a \ x and p by alge­
braic m ean s. Show that there are no finite dim ensional
m a trices which sa tisfy the sp ecified m ultiplication prop­
e r tie s .

Chapter 11
11.1 P rove the orthogonality and norm alization condition of L e­
gendre polynom ials using the generating function (1 - 2 rx +
r2)~'h = £ rn Pn (x).
n= 0
11.2 D erive the com m utation relation s betw een the com ponents of
angular m om entum and the com ponents of (a) the position
operator, and (b) the momentum operator,
11.3 Let A x , A y and A z sa tisfy the com m utation relation s Lx A y -
Ay L x = i~HAz , L y A x - A x L y = - i H A z , LXA X - A x L x = 0, and c y c lic
perm utations, with r e sp ec t to the angular m om entum oper­
a to rs. Such operators are ca lled “ vector o p e r a to r s” ; e x ­
am ples are r and p. P rove the follow ing relation s:
(a) L XA 2 - A 2 L x = 0
(b) L X(A • L) - (A • L) L x = 0
(c) L/2 A x — A X L 2 — i H ( A y L z + L/Z Ay — A z Liy — Ly A z )
(d) L2 (L 2 A x - A X L 2 ) - ( L 2 A* - AXL 2 ) L 2 =
2 ( L 2 A x + A X L 2 ) - 4 L X( A - L )
(e) M + M A )?Z = (L • * > - > ) £ ' .
11.4 Show that in a state ^ with a sharp value of L2, L z <p = m<p, the
m ean valu es of L x and L y are zero.
11.5 Suppose a sy ste m can be r e so lv e d into two weakly interacting
su b sy stem s 1 and 2 so that the total angular momentum L is
Li + L2 . If the su b sy stem s are in sta te s ch a ra cterized by
definite valu es of the quantum num bers / x, l l z and I2 , h x re­
sp ectiv ely , what j^re the p o ssib le valu es of L2 and what is the
average value of L2?
11.6 E xp ress the sp h erical harm onics for 1 = 0 , 1 and 2 as poly­
n om ials in x, y and 2 .
11.7 Find the transform ation rule of the sp h erical functions Yn>
Y1 0 a n d Y i- ifo r a rotation of the coordinate sy ste m through
Eulerian an gles a, fi and y.
522 PROBLEMS

Let L, = L x ± i L y . V erify the follow ing rela tio n s


(a) ( i ± y =
(b) L2 = +)

+
+
+ 2

1
(c) L z L± - l ±l z = ±~HL±
(d) L+L- - L . L+ = 2 H L Z
(e) L2 L± " L ±L \ = 0
(f) l 2 l 2 - L Z L 2 = 0
11.9 Study the way in which the m ultiplication p ro p erties in prob­
lem 1 1 . 8 can be rep resen ted in te r m s of m a tr ic e s, or what is
the sam e thing, lin e a r tran sform ation s on a fin ite dim ensional
v e c to r sp a c e . Let 'Pxm be a fin ite s e t of d egenerate eig en ­
functions of L2 with eigenvalue A. H ere n la b e ls the elem en ts
of the s e t. Show the follow ing:
(a) L z is an eigenfunction of L2.
(b) T here is som e lin ea r com bination of t h e n ’s which is an
eigenfunction of L z . Denote the eigenvalue by m t and the
p articu lar lin ea r com bination by ¥xm.
(c) L+^Xm is an eigenfunction of L2 with eigenvalue A.
(d) L±'i'Xm is an eigenfunction of L z with eigenvalue (m ± l ) t .
(e) F rom the condition that the norm of is nonnegative,
it fo llo w s (r e sp e c tiv e ly for +) that A > m( m ± l)7j2, the
equality holding if the v e c to r is null.
(f) The condition for the e x iste n c e of fin ite m u ltip letes is
that the s e r ie s gen erated by (L±)n term in a tes at both ends.
T h is im p lies that A has the form 1(1 + D U 2 , w here / may
have the v a lu es 0, 1 /2 , 1, 3 /2 , . . . , and that m runs over
range — /, — I + 1 , . . . , / — 1 , I.
(g) Find the m a tr ice s of order 2 1 + 1 that r e p r ese n t the action
of the op era to rs L z and L± on the v ecto r sp ace spanned by
, - I < m < I, for a fixed value of I, F or exam ple,
OP,/ m ,’ L,z V,Im ') = (L z )mm ' = m S mm S e lec t a rphase convention
com p atiable with ( 1 1 . 8 8 ).
(h) W rite out the e x p lic it m a tr ic e s for I = 1 /2 and v e r ify the
com m utation rela tio n s by d ir e c t m atrix m ultiplication.

Chapter 12
12.1 Find the en ergy le v e ls and eigenfunctions of an isotrop ic
th ree-d im en sio n a l harm onic o s c illa to r . Find energy eig en ­
functions that are a lso eigenfunctions of L2 and L z . What is
the d egen eracy of each lev e l? What part of the degeneracy
ste m s from isotrop y of the potential?
12.2 W rite out the e x p licit I = 1, m = 1, 0, - 1, eigenfunction s of
lo w est energy of a three dim ensional sp h e r ic a lly sym m etric
harm onic o s c illa to r . E xp ress th ese eigenfunction s a s lin ear
-co m b in a tio n s of the solu tion obtained by using sp earation of
v a r ia b le s in C artesian co o rd in a tes. If each trip let of wave
PROBLEMS 523

functions is orthonorm al, show that the m atrix of co e ffic ien ts


of the transform ation is a unitary m atrix.
12.3 For the isotrop ic potential V (r) ~ -#S(r - a), determ in e the
range of g for which there is an I = 0 d isc r e te state but no
/ = 1 d iscr e te state.
12.4 In the c a se of a continuous spectrum , the wave function far
from the scatterin g cen ter is ch aracterized by the “ phase
sh ift” S /( /d defined as follow s: rRkj(r) - c sin ^kr - ~

for kr » I (kr » 1 for / = 0), w here h k = \ / 2mEk. For a “ short


ran ge” potential, 5/ is independent of r (se e page 2 2 2 and
Chapter 28B). C alculate the I = 0 phase sh ift for the potential
in problem 12.3.
12.5 The com bination J(+%(p) = ;,(p) occu rs frequently and is
ca lled the “ sp h erical B e s s e l function.” The second solution,
irregu lar at the origin p = 0, is n/(p) = ~ ( - i )1 J l _,/2 (p), con­
siste n t with the definition of Neumann functions, N p = (Jp cos^p -
J - p)/sin?7p. V erify the follow ing prop erties:
(a) differen tial equation.

2 , / ,2 / ( / + 1)
nl + + I k - -----r — In = 0

(b) e x p lic it form ,


sin p cos p
Jo
P

sin p cos p cos p sin p


0
J —
p29 p * ^1 —
p2 9
p

(c) recu rren ce r ela tio n s,


f . 2/ + 1 . / + 1 .
+ Jl -1 = p Ji • )i — ]i + ^+i

(d) asym ptotic form (p » /, p » 1 ),

h (p) ^ 7 s i n (p - y )

nz(p) ~ - j c o s (p - y )
(e) behavior at p = 0 ,
524 PROBLEMS

(2/)! -I- 1
n {(p) = - p
211\

(f) in tegral rep resen tation (se e page 2 0 2 ),

1 / ' +1
i l j L(p) = — / P l ( x ) e l p x dx
2 y_!

(g) orthonorm ality,


^ CD
/ j l ( k r ) j l (k'T)r‘2‘dr = — S( k' k' )
J o 2kk

12.6 Find the conditions under w hich the sp h erica l potential w ell
V = - |V 0 | fo r 0 < r < a, and V = 0 for r > a , can support N
s -w ave bound s ta te s , but not N 4 1,
12.7 In the lim it of zero kinetic en ergy, an s-w a v e phase shift
b ehaves a s, S0 (fe) = - k a , w here a is the “ sc a tte rin g len gth .”
Find an approxim ate relation betw een sca tterin g length and
the en ergy of a lo o se ly bound sta te, Eg = 0 .

Chapter 13
13.1 Find the m om entum d istribution of an e le c tr o n in the Is, 2s
and 2 p sta te s of hydrogen.
13.2 C alculate the life tim e of a hydrogen atom in the 2p sta te.
13.3 Give an e x p r e ssio n for the tran sition r a te s betw een con secu ­
tive c ir c u la r orb its in a hydrogen -lik e atom .
-1 ® 1
13.4 Show that (1 - t) expixi/(l - t )I = ^!) 1 LJ^(x)/fe is the gen-
ft= 0
eratin g function for L egu erre polynom ials L ^ ( x ) = Q®( x) .
13.5 At what quantum le v e l is a m uonic atom the sam e s iz e as a
norm al hydrogen atom in its ground state?
13.6 What is the probability of finding an e le c tr o n with quantum
num bers n , I inside a nucleus of radius R n ?
13.7 Show that if the energy is r e g a r d e d a sa com p lex v ariab le, the
Coulomb “ sca tterin g am plitude” for a definite value of /,
/ 2(§ ® \
' - l J / 2 ik, has a sim p le pole sin gu larity at the energy
value corresp on d in g to each le v e l of the d is c r e te spectrum .

Chapter 14
14.1 Evaluate the shift in energy le v e ls of a harm onic o sc illa to r
produced by a perturbing 5-function potential that is cen trally
located . State the lim it of valid ity of the approxim ation.
1 4 .2 , C alculate the lo w est order effect on the sp ectru m of a lin ear
harm onic o s c illa to r due to the r e la tiv istic in c r e a se in m a ss
PROBLEMS 525

of the p a rticle. In what circu m stan ce is the r e la tiv istic e ffect


pronounced? When is the perturbation approxim ation valid?
Cf. problem 2 .in .
14.3 T reat the Stark effect of the n 3 lev e l of hydrogen. D eter­
mine the pattern of splitting and residual m ultiplicity by
sym m etry argum ents. How does the se cu la r equation factor
in the represen tation in which L? is diagonal? What linear
com binations of unperturbed eigenfunctions are energy eigen ­
functions in the p resen ce of the field?
14.4 Evaluate the static (&> == 0) electro n ic p olarizab ility of a hy­
drogen atom in its ground state.
14.5 For the continuous part of a spectrum , the sta te s may be
labeled by the eigenfunctions of the fre e p article Hamiltonian
Ho and an (exact) integral equation for the eigenfunctions
(r) of the H amiltonian Ho + V s e t up along the pattern of
perturbation theory. The integral form of the Schrodinger
equation is

^ ( r ) = expik • r + J 'Ge (r,r') V(r') 4^ (r') d3 r',

where Gg is the energy G reen’s function


exp ik I r - t I
Ge (r,r')
4n’| r - r' |
c h a r a c te r istic of the fre e p article H am iltonian. The G reen’s
function can be defined in term s of a com p lete s e t of energy
eigenfunctions <£„(r) of the (any) Hamiltonian Hq as follow s:
^ *„<')*„<«■ '>*
G e ( t, t ')
n E - E„
(a) F orm ulate a differen tial equation and boundary conditions
for the G reen’s function.
(b) Evaluate the ex p licit form of the fre e p a rticle G reen’s
function from its definition in term s of a com p lete s e t of
energy e ig e n sta te s.
(c) Find the relation between the energy G reen’s function and
the tim e G reen’s function G t . (See problem 5.3.)
(d) The e ffe c t of a “ short ran ge” potential on the wave func­
tion at la rg e d istan ces from the sca tterin g c en ter is de­
scrib ed by the “ scatterin g am plitude f i d ) , ” defined by the
asym ptotic form (for r oc)
i kr
e
V '> - e , k , r + fid)
r

(See Chapter 28, Section B) D erive an exact ex p r e ssio n for


in term s of the exact solution 4*k of the Schrodinger
f ( 6)
equation.
526 PROBLEMS

(e) F orm ulate a perturbation approxim ation for Vk and f ( 6 ) on


the b a s is of the above r e s u lts .
14.6 F or a definite value of the angular m om entum the integral
from of the Schrodinger equation for a sp h erica lly sym m etric
potential takes the form

w here

for I = 0 .
(a) Evaluate g Ei ( r , r ' ) for I > 0.
(b) What is the relation betw een the wave functions and b e­
tw een the G reen’s functions of the th ree-d im en sio n a l
(problem 14.5) and the partial w ave form u lation s. It is
n e c e s s a r y to d istin gu ish ingoing and outgoing wave bound­
ary conditions e ± l k r / r . The addition form ula for sp h erical
harm on ics is helpful,

w here cos© = c o s 0 c o s 0 ' + sin 6 sin Q' c o s(<£ - <£').


(c) An ex a ct e x p r e ssio n for the s-w a v e phase sh ift is

V erify this relation and g e n e r a liz e it for I > 0. What nor­


m alization condition is im plied for u*.? See problem 12.4.

Chapter 15
15.1 Identify the conditions under which the K lein-G ordan equation
red u ces to the Schrodinger. How does the in itial data prob­
lem for the secon d order d ifferen tial equation reduce to that
of the fir s t order equation?
15.2 Show that the fu n ction -sp ace inner product

is independent of tim e if tfj and <£ are solu tion s of the K lein-
-Gordan equation. Find the form of the inner product in mo­
m entum sp a ce. Is the Inner product p o sitiv e definite?
PROBLEMS 527

15.3 A general solution of the Klein-Gordan equation is a super­


position of a positive-frequency and a negative-frequency
part, with time dependence and e ' , OJt , respectively.
Show that under a Lorentz transformation that does not
reverse the sen se of tim e, a positive-frequency solution is
transformed into a positive-frequency solution, i.e ., that the
decomposition is Lorentz invariant.
15.4 Find a com plete set of positive-frequency solutions of the
Klein-Gordan equation that are orthonormal with respect to
the inner product in problem 15.2. Show that the com plete­
ness relation (for unequal tim es) is

X fjx) f j x ' f
1 f i k i x -x ) k
a (2T7-)3 J
where co = o > ( k ) = c \ / f e 2 + m 2 c t y f t 2 and where the Minkowski
space inner product is k( x - x ' ) = a >(< - l') - k • ( x - x ' ) .

Chapter 16
16.1 What is the value of the g factor of a particle described by
the Hamiltonian (l/2mo) [V • (p - ( e / c ) A)J ?
16.2 A particle of spin 1/2 and magnetic moment p m oves in a
precessin g magnetic field
Hx = U sin0 cos cot , Hy = U sin0 sinca<, Hz = U cos 6 .
At time t = 0, the spin is parallel to the 2 axis. What is the
probability of the spin being antiparallel to the 2 axis at som e
later time?
16.3 Find the (time-dependent) position operators in the Heisen­
berg representation of a particle of spin 1/2 and magnetic
moment p moving in a nonhomogeneous magnetic field

Hx = 0, Hy = - ky, Hz = H0 + kz .
16.4 Determine the energy spectrum and wave functions of a charged
particle moving in uniform electric and magnetic fields that
are perpendicular to one another.

Chapter 17
17.1 Show that the four m atrices d ' , a ' ) are Hermitian, linearly
independent and form a com plete b a sis for 2 x 2 m atrices.
17.2 How many m atrices are required to form a com plete set of
Hermitian, linearly independent N x N m atrices? How many
528 PROBLEMS
mutually commuting, Hermitian, linearly independent N x N
m atrices are there?
17.3 In the Dirac theory, a and p are mutually commuting linear
operators. Develop a notation for the linear vector space on
which the operators act. Find an expression for the inner
product in the com posite (direct product) space. Show that
the Dirac Hamiltonian is Hermitian.
17.4 U se the Heisenberg equation of motion to show
(a) .^-(x + n p3 a/2m c) = pg (p - (e/c) \)/m ,

(b) — (t + i h p 3/2 m c 2) = p 3 (H - e ^ l / m c 2 .
dt
17.5 What transform ation of the spinor wave function m ust be made
to resto re the form of the Dirac equation after a change of
gauge in the electrom agnetic potentials?
17.6 Show that the form of the Dirac equation rem ains unchanged
by Lorentz transform ation or spatial rotation.

Chapter 18
18.1 Suppose a system co n sists of two weakly interacting sub­
sy stem s each of spin 1 /2 . The total spin is S = si + sg. What
are the possible eigenvalues of S2 and S2? Compute the value
of si • S2 in the triplet (spins parallel) and singlet (spins
antiparallel) states of the com posite system . Find eigenfunc­
tions of S2 and S z as linear combinations of products of eigen­
functions of the subsystem s.
18.2 An electron moving in a central field of force is in a state
specified by the quantum numbers I j m j . What are the possible
values of the 2 components of orbital and spin angular mo­
mentum and what is the average value of each?
18.3 Is the parity operator / a linear operator? Is it Hermitian?
What are the commutation properties of I with the operators
r, p, L, S and J ?
18.4 An axial vector transform s as a vector under proper rota­
tions (i.e ., rotations without space inversion) but does not
change sign under inversion. C lassify the following as being
either vector or axial vector: E, II, A, S, L, p and u x v
where u and v are v ecto rs.
18.5 Show that if a system is in a state characterized by a sharp
value m j for the 2 component of the total angular momentum,
the mean value of the total angular momentum about an axis
2 " making an angle 0 with the 2 axis is costf.
18.6 Evaluate the particle flux of positive energy and negative
energy plane wave solutions of the Dirac equation. Also ca l­
culate the flux of the corresponding charge conjugate solutions.
18.7 .Can the charge conjugation transform ation be represented by
a linear operator?
PROOLEM S 529

Chapter 19
19.1 Evaluate p and ,j for a Dirac p article in an electrom agn etic
field. V erify that the Lorentz force is given c o r r e c tly .
19.2 C alculate the sp in -orb it and contact potentials for an e le c ­
tron outside a c lo se d atom ic core con sistin g of a hydrogen-
like atom .

Chapter 20
20.1 C alculate the energy lev e l splitting of a hydrogen atom in a
weak e le c tr ic field . N eglect the Lamb shift and assu m e the
Stark effe c t is sm a ll in com parison with the fine stru ctu re.
Account for the latter by using eigenfunctions of J2, J z andL 2
for the unperturbed s ta te s.
20.2 Evaluate the Stark e ffect for the n = 2, j = ^ le v e l of hydrogen
for the c a se w here the Stark effect is com parable with the
Lamb sh ift. R epresent the Lamb shift by a phenom enological
perturbation m atrix elem en t that touches only the s -s t a te .
20.3 Study the hydrogen Stark e ffect for the le v e l n = 2 in the
tran sition region where the Stark e ffect and the fine stru ctu re
are of the sam e order of m agnitude. P lot the energy le v e ls
as a function of S.
20.4 Compute the life tim e s of the 2pi, and 2pi, sta te s of hydrogen.
'2 '2

20.5 Find an e x p r e ssio n for the lifetim e of the m etastab le 2s-|


state of hydrogen in a very weak e le c tr ic field . At what field
strength w ill the lifetim e by 10 “ 3 se c ? Is the Stark le v e l shift
appreciable at this field strength? Hint: Evaluate the m atrix
elem en t of r using a perturbed 2 s^ wave function that con­

tains an adm ixture of the 2 p | state and take advantage of the


sm a lln e ss of the Lamb shift.
20.6 Find the m ean value of the operator u = L + g s S in a state
ch a ra cterized by the quantum num bers J, J2, L, S. (This
g iv e s a g en eralization of the Lande form ula in the c a se where
the g factor of the electro n is not exactly equal to two. See
page 346.) Hint: U se problem 11.3(e).

Chapter 21
21.1 D eterm ine the form of the contact interaction when the finite
nuclear s iz e is taken into account. C alculate the splitting of
the 2 s i - 2 p i le v e ls of hydrogen stem m ing from this effect.
It is about - 0.1 Me.
530 PROBLEMS

21.2 C alculate the hyperfine sp littin g of the l s i state of hydrogen


using a c la s s ic a l m odel in which the nucleus is rep resen ted
by a uniform ly m agnetized sp h ere of radius R/v. Give a
c la s s ic a l explanation o f the sig n of the sp littin g.
21.3 Find the m agnetic field at the nucleus of an s state electro n .
21.4 C onsider a hydrogen atom in the ground state in a uniform
m agnetic fie ld . Find the appropriate lin ea r com binations of
e lectro n -p ro to n spin wave functions that are energy eigen ­
functions in the c a se w here the in teraction with the external
fie ld is the sam e order of m agnitude as the electron -p roton
d ip o le-d ip o le in teraction.
21.5 C alculate the hyperfine sp littin g of a h yd rogen -lik e atom in a
state of n o n -z e ro orbital angular m om entum . Hint: U se
problem 11.3(e).

Chapter 22
22.1 Spfecify a com p lete se t of com m uting con stan ts of the m otion
for positronium .
22.2 C alculate the fine stru ctu re of positron iu m . Obtain the
H am iltonian by s e m ic la s s ic a l argum en ts. Account e sp e c ia lly
for “ hyp erfin e” sp littin g, knowing that the m agnetic m om ent
of the positron is equal in m agnitude and opposite in sign to
that of the e le c tr o n . P r e sen t the r e su lts in an energy le v e l
diagram for n = 1 , 2 .
22.3 Evaluate the Lamb sh ift for positronium .
22.4 C onstruct a theory for positronium using a D irac Ham iltonian
for each p a rticle and the Coulomb in teraction betw een them .
Separate the m otion of the cen ter of m a ss from the rela tiv e
m otion in the approxim ation of retaining only the low est order
r e la tiv is tic c o r r e c tio n s for the r e la tiv e m otion. D oes the
“ h yp erfin e” in teraction em e r g e autom atically? Hint: The
action of spin o p erators on a product wave function can be
e x p r e ss e d as follow s:

(<7( 1* ± cr<2)) (tf> X l/») = 11 <f> X t/f) ± ^ X <7<2) Ij? ) .

Chapter 23
23.1 C onstruct a com p lete s e t of orthonorm al tw o -p a rticle eigen ­
functions from a com p lete s e t of o n e -p a r tic le eigenfunction s.
23.2 D erive the H a rtree-F o ck equations for determ ining the b est
s in g le -p a r tic le functions to give an an tisym m etric (or sy m ­
m etric) tw o -p a rticle wave function of low est en ergy.
23.3 C alculate the n 1 and 2 le v e ls of a h yd rogen -lik e atom , in­
c lu d in g fine stru ctu re and the Lamb shift, using a variational
method.
PROBLEMS 531

23.4 Show how the variational method with trial functions 0 =


a,r can be reduced to a m atrix procedure if the 4 ’s are
regarded as the variational p aram eters. The a’s may be
se le c te d by intuition or by an itteration sch em e. Note how
this g iv e s a finite se t of orthonorm al v e c to r s.
23.5 Evaluate the ground state energy of helium using the follow ­
ing “ s e lf-c o n s is te n t” variational method: assu m e the fir st
electro n is in a hydrogenic I s state with Z = Z ' and calcu late
the screen ed field se e n by the second electro n . D escrib e the
second electro n by a hydrogenic I s function with Z = Z ".
Vary Z' to find the low est energy for a given Z \ Then im pose
the sym m etry by settin g Z ' = Z " .

Chapter 24
24.1 Give the p o ssib le valu es of the total angular momentum for
the follow ing sta tes (term s): 1 S, 2 S, 3 S, 2 P, 3 P, 2D and 4 D.
24.2 Which ter m s are p o ssib le for the follow ing tw o -electro n con­
figurations: (a) nsn's, (b) nsn'p, (c) nsn'd, and (d) np, n'p?
Which ter m s are c o n sisten t with the exclu sion principle if
n = n'?
24.3 Couple three unit angular m omenta /i = 1% = I3 = 1 to y ield a
resu ltan t eig en sta te L = 1, L2 = 0 of L2, w here L = 11 + I2 + >3 .
How many independent sta te s of this so r t are there?
24.4 E stim ate the low -lyin g excited state e n e r g ie s of helium in
the approxim ation of neglectin g exchange e ffe c ts . Do this
assum ing that one electro n is d escrib ed by the Is function
found in the ground state calcu lation and carryin g out the
variational procedure for a hydrogenic 2 s (and independently,
2p) wave function in the screen ed Coulomb field . The 2s state
m ust be taken orthogonal to som e appropriate Is sta te. The
root of the variational equation may be found by a rapidly
converging iteration m ethod.
24.5 C alculate the low -lyin g ex cited state e n e r g ie s of helium taking
into account exchange and spin e ffe c ts . U se appropriately
sy m m etrized product eigenfunctions for the configurations
(I s , 2 s) and ( I s , 2 p), w here the Is function is that found in
the ground state calcu lation . The required m atrix elem en ts
are given in problem 24.6 for r e fe r en ce .
24.6 C onsider the hydrogenic wave functions
Vls = 2 a 3 / 2 e ~ " Y 00

xV2s = Ml - Br)e~Pr Y 00

^ 2 p = cre'^ Y \ m
w here a = Z i s / a o , /3 = Z2 S/ 2 ao, and y = Z2 p/ 2 oo and w here fa, B
and c are determ ined by the orthogonality and norm alization
PROBLEMS 5 33

25.2 Using the v iria l theorem , show that in the T h om as-F erm i
m odel of a neutral atom the energy of e le c tr o sta tic repul­
sion between the electro n s is 1 /7 the magnitude of the e le c ­
trostatic attraction between the electro n s and the nucleons.
25.3 E stim ate the order of magnitude of the follow ing quantities
in a neutral atom according to the T h o m a s-F erm i model:
(a) the s iz e of the atom,
(b) the average e le c tr o sta tic repulsion between two e le c tr o n s,
(c) the average kinetic energy of one electron ,
(d) the average speed of an electron ,
(e) the average angular m omentum of an electron ,
(f) the mean radial quantum num ber.
25.4 Show that the m ean perturbation of all sta te s of a given term
is zero for the sp in -o rb it in teraction.
25.5 C alculate the L • S splitting of the 3P term of helium .
25.6 Evaluate the L • S splitting of a sin g le n p e le ctr o n in a sp h eri­
c a lly sym m etric potential. E xp ress the r e su lt in ter m s of an
arbitrary radial m atrix elem en t. E stim ate the radial m atrix
elem en t for the doublet splitting of sodium .
25.7 Two ele c tr o n s m ove in an ( np) 2 configuration in a sp h erically
sy m m etrica l potential. Regarding the e le c tr o sta tic repulsion
betw een the ele c tr o n s as a perturbation, evaluate the splitting
of term s in fir s t order approxim ation. N eglect the spin-
orbit interaction. U se qualitation con sid eration s to infer
ordering of the term s. Hint: To fa cilita te diagonalization of
the se cu la r equation, u se a rep resen tation in which M^and Ms
are diagonal and note that the sum of the roots of a se cu la r
equation is equal to the sum of the diagonal m atrix ele m en ts.

Chapter 26

26.1 Find the p o ssib le atom ic term s (a term is c h a ra cterized by


L and S) in a configuration of two equivalent d -e le c tr o n s.
Give the total number of sta te s and the num ber of sta te s in
each term . What valu es of J are p o ssib le to each term ?
26.2 In an atom ic configuration the term that has the low est
energy can be determ ined by Hund’s se m i-e m p ir ic a l rules:
(1) The ground state w ill have the la r g e st value of S con­
siste n t with the Pauli p rinciple,
(2 ) L w ill have the la r g e s t value co n sisten t with the value of
S determ ined in rule (1),
(3) The total angular momentum of the ground state is J =
| L - S | if the unfilled su bshell is half full or le s s than
half full; it is J = L + S if the su bshell is m ore than half
full.
Give a qualitative physical ju stification for each of th ese r u le s.
534 PROBLEMS

26.3 U sing Hund’s r u les (se e problem 26.2), find the ground state
of the configuration n p x for x = 1, 2, . . . . 6 . F or each value
of x, state an elem en t for which this c a s e is r e a lize d physi­
c a lly .
26.4 Do problem 26.3 for the configuration n d x for x = 1, 2 .............10.

Chapter 27
27.1 E stim ate the r ela tiv e freq u en cies and sep aration of energy
le v e ls for the e le c tr o n ic , vibrational and rotational m otions
of a diatom ic m o lecu le.
27.2 D erive the Schrodinger equation d escrib in g the m otion of the
nu clei of a diatom ic m olecu le in the approxim ation that the
n uclei m ove much m ore slo w ly than the e le c tr o n s and thus
e x p erien ce only an in teraction with the e le c tr o n s that is
averaged over many ele c tr o n revolu tion s. T his procedure
provides a sep aration of va ria b les betw een the e le c tr o n ic and
nhblear m otions. The approxim ation is c a lled the a d i a b a t i c
o r B orn-O ppenheim er approxim ation.
27.3 What are the p o ssib le sy m m etry sta te s of the diatom ic m ole­
c u les D2, N2, LiH form ed from the bonding of the two atom s
in their ground sta te s?
27.4 What spin sy m m e tr ie s are p o ssib le for the rotational sta tes
of the deuterium m olecu le D2 in the ele c tr o n ic ground state?
The deuterium nucleus has spin 1.
27.5 C alculate the energy^of a rigid e le c tr ic dipole in a uniform
e le c tr ic fie ld . U se second order perturbation theory.
27.6 Show that the fo rc e betw een two hydrogen atom s in their
ground sta te s v a r ie s as 1/R7 if the atom s are separated by a
la rg e d istan ce R.

Chapter 28
28.1 The rate for making tra n sitio n s from an in itial state i to the
final state f is

wit = — \U'<f\2 Pf(E) •

w here p f is the num ber of final sta te s per unit energy interval
(“ Golden Rule No. 2 ” ).
(a) Find the e x p r e ssio n for U'if in fir s t order perturbation
theory. What n orm alization convention is im plied for the
wave function?
j(b) D erive an e x p r e ssio n for in second order perturbation
theory.
PROBLEMS 535

(c) Evaluate the density of sta te s p ( for a final state con sistin g
of two free p articles of definite total energy and m om entum .
(d) Evaluate p f for a final state con sistin g of three equal m ass
(free) p a r tic le s. E xp ress the r e su lts in the cen ter of m ass
sy ste m . How is the partition of energy among the three
p a rticles accounted for by a probability distribution?
(e) C alculate the dependence of p{ on energy near the energy
threshold for an /V-particle final state.
28.2 C alculate the rate for induced tran sition s from state m to
state n of an atom ic sy ste m in an e le c tr ic field with sp ectral
density £(&>) at tran sition frequency a> = — (Em - E n). Identify
n
the E in stein c o efficien t B and thereby infer the rate for
spontaneous e m issio n .
28.3 C onsider a p article of m a ss m bound in a th ree-d im en sion al
harm onic o sc illa to r potential -fer2. The particle is irradiated
with a m ild pulse U' = e x 2 e ~ u/ r)2 for the tim e i n t e r v a l <
t < ~. D eterm ine which tran sition s are p o ssib le and calculate
their p rob ab ilities. Identify the lim itin g c a s e s of sudden and
adiabatic perturbations. State the lim its of validity of the
perturbation calculation.
28.4 At t = 0, a hydrogen atom in its ground state is irradiated
with a uniform periodic e le c tr ic fie ld . D eterm ine the m ini­
mum frequency of the field n e c e ssa r y to ionize the atom and
com pute the ionization probability per unit tim e. As an ap­
proxim ation, the electro n in the final state may be regarded
as fr e e . State the lim its of validity of the approxim ation.
28.5 Show that the scatterin g of slow p a r tic les in a sh ort range
potential is ch aracterized b y d ^ k ) ~ fe2i+1. Find the proportion­
ality constant in Born approxim ation and state the conditions
under which the approxim ation is valid.
28.7 Taking into account the sym m etry of the wave function, give
the d ifferen tial c r o s s se ctio n for e la s tic Coulomb scatterin g
of an electro n on an electro n and of an alpha p article on an
alpha p a rticle. D istinguish spin sta te s and a lso give the
form ula for the sca tterin g of unpolarized e le c tr o n s. Identify
quantum e ffe c ts and show how they disappear in the c la s s ic a l
lim it. F or r e fe r e n c e , the exact sca tterin g am plitude for a
fixed Coulomb potential is
j■ ( q) _____ y ^ . T d + iy) ' e ~2iyln sin 8/2
2 sin 2 | E(1 - iy)

w here y = e ^ / t v . Cf. Chapter 13, Section D.


28.8 Compute in Born approxim ation the differen tial scatterin g
c r o s s sectio n for the scatterin g of fa st neutrons by a Coulomb
field .
536 PROBLEMS

28.9 Set up coupled Schrodinger equations to d e sc rib e the “ two


channel” reaction and sca tterin g p r o c e s s e s
(11) a + b a + b,
(12) q + b -* c + d ,
(22) c + d -* c + d.

(21) c + d -> a + b.

A ssu m e the p a r tic les are s p in le s s and of unequal m a ss.


Separate out the m otion of the c en ter of m a ss.
(a) How m ust the p oten tials V n , V i 2 , V 2 1 . V 22 be rela ted in
ord er to d e sc rib e a sy ste m that is invarient with r e sp ec t
to r e v e r s a l of the s e n s e of tim e? How are the c r o s s s e c ­
tions for rea ctio n s (12) and (21) rela ted ? (detailed balance)
(b) Give e x p r e ssio n s for the e ffe c tiv e c r o s s se ctio n s for th ese
r ea ctio n s in Born approxim ation.
(c) F orm ulate an e x p r e ssio n for the reaction and sca tterin g
am plitudes in ter m s of partial wave am plitudes of definite
* angular m om entum . What relation am on g/'11,/'i 2 ,f 2 i . / 2 2 (for
a definite value of /) is im plied by con servation of prob­
ability?
Note: The r e su lts are p resen ted m ost sim p ly in term s of a
m atrix notation. The m any-channel g en era liza tio n of the
quantity e 2 ‘ ' is c a lle d the S -m a trix .

Chapter 29
29.1 V erify the anticom m utation rela tio n s
t p a ( r , t ) tftp ( r ' , t ) + tfra ( r , t ) = ta p S ( r - r)

t pa ( r , t ) ^ ( r ( < ) + \ j j p ( , T' , t )<Pa ( r , t ) = 0

s a tis fie d by the secon d quantized D irac w ave function. The a


and j3 indicate com ponents of the D irac spinor and l a^ is the
unit 4 x 4 m atrix. The p o sitiv e-freq u en cy parts of and i/V
d e sc r ib e the creation of p o sitro n s and e le c tr o n s , r e sp ec tiv e ly ,
w hile the n egative-freq u en cy parts d e sc rib e the annihilation of
e le c tr o n s and p o sitr o n s, r e sp e c tiv e ly . E lectron s and p ositron s
are d escrib ed by independent s e ts of m utually anticom m uting
c rea tio n and annihilation op erators
C(p >C(e ) C (C, ) C (P) = 0

c(p,c(p>)* + C (n1 )* C n(P) = 0.


29.2 Evaluate the com m utation rela tio n s betw een com ponents of
.th e cu rren t and charge d en sity of a quantized Dirac wave
field , i .e ., evalu ate j ( r , t ) j v (r',t) - j u (r't'>i^(T,i) at equal tim e s.
PROBLEMS 537

29.3 C alculate the Born approxim ation m atrix elem en t for the
production of an e lectro n -p o sitro n pair in an extern al e le c tr ic
field

<l>(r,() = 4>0 co s kx co s cot.

29.4 E xp ress the energy and momentum operators of a quantized


Dirac wave field in term s creation and distruction op era to rs.
29.5 Let M(x) be a m a ssiv e pseu d oscalar m eson field sa tisfyin g
the K lein-G ordan equation. C alculate the force betw een two
stationary Dirac “ n u cleo n s” in second order perturbation
theory, assum ing the interaction energy betw een the “ nu­
c le o n ” and m eson fie ld s is

H' = g I ip+(x) p3 p l i p ( x ) M ( x ) d 3 x .

You might also like